Sie sind auf Seite 1von 530

Financial Accounting 1

SUBJECT NO. 1
Study Pack

STRATHMORE
UNIVERSITY
DISTANCE LEARNING
CENTRE
P.O. Box 59857,
00200, Nairobi,
KENYA.
Tel:
Fax:

+254 (02) 606155


+254 (02) 607498

Email: dlc@strathmore.edu

Copyright
ALL RIGHTS RESERVED. No part of this publication may be reproduced, stored in a retrieval
system or transmitted in any form or by any means, electronic, mechanical, photocopying, recording
or otherwise without the prior written permission of the copyright owner. This publication may not
be lent, resold, hired or otherwise disposed of by any way of trade without the prior written consent
of the copyright owner.
THE REGISTERED TRUSTEES STRATHMORE EDUCATION TRUST 1992

Acknowledgement

ii

ACKNOWLEDGMENT

We gratefully acknowledge permission to quote from the past examination papers of the following
bodies: Kenya Accountants and Secretaries National Examination Board (IASNEB);
Chartered Institute of Management Accountants (CIMA); Chartered Association of Certified
Accountants (ACCA).

Acknowledgement

iii

INSTRUCTIONS FOR STUDENTS


This study guide is intended to assist distance-learning students in their independent
studies. In addition, it is only for the personal use of the purchaser, see copyright
clause. The course has been broken down into eight lessons each of which should be
considered as approximately one week of study for a full time student. Solve the
reinforcement problems verifying your answer with the suggested solution contained
at the back of the distance learning pack. When the lesson is completed, repeat the
same procedure for each of the following lessons.
At the end of lessons 2, 4, 6 and 8 there is a comprehensive assignment that you
should complete and submit for marking to the distance learning administrator.
SUBMISSION PROCEDURE
1.
2.
3.
4.

After you have completed a comprehensive assignment clearly identify each


question and number your pages.
If you do not understand a portion of the course content or an assignment
question indicate this in your answer so that your marker can respond to your
problem areas. Be as specific as possible.
Arrange the order of your pages by question number and fix them securely to the
data sheet provided. Adequate postage must be affixed to the envelope.
While waiting for your assignment to be marked and returned to you, continue to
work through the next two lessons and the corresponding reinforcement
problems and comprehensive assignment.

On the completion of the last comprehensive assignment a two-week period of


revision should be carried out of the whole course using the material in the revision
section of the study pack. At the completion of this period the final Mock
Examination paper should be completed under examination conditions. This should
be sent to the distance-learning administrator to arrive in Nairobi at least five weeks
before the date of your sitting the IASNEB Examinations. This paper will be marked
and posted back to you within two weeks of receipt by the Distance Learning
Administrator.

Acknowledgement

iv

Acknowledgement

CONTENTS
ACKNOWLEDGMENT.....................................................................................ii
INSTRUCTIONS FOR STUDENTS..................................................................iii
FINANCIAL ACCOUNTING I COURSE DESCRIPTION..................................vi
LESSON ONE...................................................................................................1
INTRODUTION TO ACCOUNTING...........................................................................1
LESSON TWO................................................................................................32
FINAL ACCOUNTS..................................................................................................32
LESSON THREE............................................................................................68
ACCOUNTING THEORY..........................................................................................68
LESSON FOUR..............................................................................................81
ADJUSTMENTS TO FINAL ACCOUNTS.................................................................81
LESSON FIVE..............................................................................................136
FURTHER ADJUSTMNETS TO ACCOUNTS.........................................................136
LESSON SIX................................................................................................176
OTHER ASPECTS OF FINAL ACCOUNTS............................................................176
LESSON SEVEN..........................................................................................231
PARTNERSHIPS.....................................................................................................231
LESSON EIGHT...........................................................................................287
COMPANY ACCOUNTS.........................................................................................287
LESSON NINE.............................................................................................334
REVISION AID.......................................................................................................334

Course Description

vi

FINANCIAL ACCOUNTING I COURSE DESCRIPTION

The subject gives a thorough and comprehensive introduction to double bookkeeping. It develops
the students understanding of the final; accounts of business and that of clubs and societies, and
the treatment of capital expenditure and the purchasing of stock.
Following this it deals with the cashbook and bank reconciliation preparation of accounts from
incomplete records. Its prime purpose is to pre[pare candidates for the Section One examination
of the CPA Kenya accountancy paper and is based on the materials used to prepare students at
Strathmore School of Accountancy.
Text Book: Business Accounting Volume 1 by Frank Wood

STRATHMORE UNIVERSITY STUDY PACK

Lesson One

LESSON ONE
INTRODUTION TO ACCOUNTING
a) NATURE OF ACCOUNTING
Accounting is defined as the process of identifying, measuring and reporting
economic information to the users of this information to permit informed
judgment
Many businesses carry out transactions. Some of these transactions have a
financial implication i.e. either cash is received or paid out. Examples of these
transactions include selling goods, buying goods, paying employees and so
many others.
Accounting is involved with identifying these transactions measuring (attaching
a value) and reporting on these transactions. If a firm employs a new staff
member then this may not be an accounting transaction. However when the
firm pays the employee salary, then this is related to accounting as cash
involved. This has an economic impact on the organization and will be recorded
for accounting purposes. A process is put in place to collect and record this
information; it is then classified and summarized so that it can be reported to
the interested parties.
b) USERS OF ACCOUNTING INFORMATION
Accounting information is produced in form of financial statement. These
financial statements provide information about an entity financial position,
performance and changes in financial position.
Financial position of a firm is what the resources the business has and how
much belongs to the owners and others.
The financial performance reflects how the business has performed, whether it
has made profits or losses. Changes in financial positions determine whether
the resources have increased or reduced.
The users of accounting information have an interest in the existence of the
firm. Therefore the information contained in the financial statements will affect
the decision making process.
The following are the users of accounting information:
i.
Owners:
They have invested in the business and examples of such owners include
sole traders, partners (partnerships) and shareholders (company). They
would like to have information on the financial performance, financial
position and changes in financial position.
This information will enable them to assess how the managers of the
business are performing whether the business is profitable or not and
whether to make drawings or put in additional capital.
FINANCIAL ACCOUNTING 1

ii.

Customers
Customers rely on the business for goods and services. They would like to
know how the business is performing and its financial position.
This information would enable them to assess whether they can rely on the
firm for future supplies.

Acknowledgement

Suppliers
They supply goods or services to the firm. The supplies are either for cash or
credit. The suppliers would like to have information on the financial performance
and position so as to assess whether the business would be able to pay up for the
goods and services provided as and when the payments falls due.
iii.

iv.

Managers
The managers are involved in the day-to-day activities of the business. They
would like to have information on the financial position, performance and
changes in financial position so as to determine whether the business is
operating as per the plans.
In case the plan is not achieved then the managers come up with
appropriate measures (controls) to ensure that the set plans are met.
The Lenders
They have provided loans and others sources of capital to the business. Such
lenders include banks and other financial institutions. They would like to
have information on the financial performance and position of the business
to assess whether the business is profitable enough to pay the interest on
loans and whether it has enough resources to pay back the principal amount
when it is due.

v.

The Government and its agencies


The Government is interested in the financial performance of the business to
be able to assess the tax to be collected in the case there are any profits
made by the business.
The other government agencies are interested with the financial position
and performance of the business to be able to come with National Statistics.
This statistics measure the average performance of the economy.

vi.

The Financial Analyst and Advisors


Financial analyst and advisors interpret the financial information. Examples
include stockbrokers who advise investors on shares to buy in the stock
market and other professional consultants like accountants. They are
interested with the financial position and performance of the firm so that
they can advise their clients on how much is the value their investment i.e.
whether it is profitable or not and what is the value.
Others advisors would include the press who will then pass the information
to other relevant users.

vii.

The Employees
They work for the business/entity. They would like to have information on
the financial position and performance so as to make decisions on their
terms of employment. This information would be important as they can use
it to negotiate for better terms including salaries, training and other
benefits.
They can also use it to assess whether the firm is financially sound and
therefore their jobs are secure.

viii.

The Public
FINANCIAL ACCOUNTING 1

Introduction to Accounting

Institutions and other welfare associations and groups represent the public.
They are interested with the financial performance of the firm. This
information will be important for them to assess how socially responsible is
the firm.
This responsibility is in form the employment opportunities the firm offers,
charitable activities and the effect of firms activities on the environment.

c) THE ACCOUNTING EQUATION


A business owns properties. These properties are called assets. The assets are
the business resources that enable it to trade and carry out trading. They are
financed or funded by the owners of the business who put in funds.
These funds, including assets that the owner may put is called capital. Other
persons who are not owners of the firm may also finance assets. Funds from
these sources are called liabilities.
The total assets must be equal to the total funding i.e. both from owners and
non-owners. This is expressed inform of accounting equation which is stated as
follows:
ASSETS = LIABILITIES + CAPITAL
Each item in this equation is briefly explained below.
Assets:
An asset is a resource controlled by a business entity/firm as a result of past
events for which economic benefits are expected to flow to the firm.
An example is if a business sells goods on credit then it has an asset called a
debtor. The past event is the sale on credit and the resource is a debtor. This
debtor is expected to pay so that economic benefits will flow towards the firm
i.e. in form of cash once the customers pays.
Assets are classified into two main types:
i) Non current assets (formerly called fixed assets).
ii) Current assets.
Non current assets are acquired by the business to assist in earning revenues
and not for resale. They are normally expected to be in business for a period of
more than one year.
Major examples include:
Land and buildings
Plant and machinery
Fixtures, furniture, fittings and equipment
Motor vehicles
Current assets are not expected to last for more than one year. They are in most
cases directly related to the trading activities of the firm. Examples include:
Stock of goods for purpose of selling.

Lesson One

Trade debtors/accounts receivables owe the business amounts as a


resort of trading.
Other debtors owe the firm amounts other than for trading.
Cash at bank.
Cash in hand.

Liabilities:
These are obligations of a business as a result of past events settlement of which
is expected to result to an economic outflow of amounts from the firm. An
example is when a business buys goods on credit, then the firm has a liability
called creditor. The past event is the credit purchase and the liability being the
creditor the firm will pay cash to the creditor and therefore there is an out flow
of cash from the business.
Liabilities are also classified into two main classes.
i)
ii)

Non-current liabilities (or long term liabilities)


Current liabilities.

Non-current liabilities are expected to last or be paid after one year. This
includes long-term loans from banks or other financial institutions. Current
liabilities last for a period of less than one year and therefore will be paid within
one year. Major examples:

Trade creditors/
or accounts payable owed amounts as a result of
business buying goods on credit.
Other creditors - owed amounts for services supplied to the firm
other than goods.
Bank overdraft - amounts advanced by the bank for a short-term
period.

Capital:
This is the residual amount on the owners interest in the firm after
deducting liabilities from the assets.
The Accounting equation can be expressed in a simple report called the
Balance Sheet. The basic format is as follows:
Name
Balance sheet as at 31.12.
Sh
Capital
Non Current Liabilities
Loan
xx

Sh

Sh
Non Current Assets
Land & Buildings
Plant & Machinery
xx
Fixtures, furniture & fittings
Motor vehicles

Current liabilities
xx

Sh

xx

xx
xx
xx

Introduction to Accounting

Overdraft
Creditors

xx
xx

xx

Capital and Liabilities

Current Assets
Stocks
Debtors
Cash at bank
Cash in hand

xx
xx
xx
xx

xx
xx

Total assets

xx

Lesson One

The above format of the balance sheet is the horizontal format however
currently the practice is to present the Balance Sheet using the vertical format
which is shown below.
Name
Balance sheet as at 31.12.
Non Current Assets
Sh
Land & Buildings
Plant & Machinery
Fixtures, furniture & fittings
Motors vehicles
Current Assets
Stocks/inventories
Debtors/ trade receivables
Cash at bank
Cash in hand
Current Liabilities
Bank Overdraft
xx
Creditors/trade payables
Net Current Assets
Net assets

Sh

Sh
xx
xx
xx
xx
xx

xx
xx
xx
xx

xx

Capital
Non Current Liabilities
Loan (from bank or other sources)

(xx)
xx
xx

xx
xx
xx

Please pay attention to the format. The Non Current assets are listed in order of
permanence as shown i.e. from Land and Buildings to motor vehicles. The
Current Assets are listed in order of liquidity i.e. which asset is far from being
converted into cash. Example ,stock is not yet sold, (i.e. not yet realised yet)
then when it is sold we either get cash or a debtor (if sold on credit). When the
debtor pays then the debtor may pay by cheque (cash has to be banked) or cash.
The Current Liabilities are listed in order of payment i.e. which is due for
payment first. Bank overdraft is payable on demand by the bank, then followed
by creditors.
Note that in the vertical format, current liabilities are deducted from current
assets to give net current assets. This is added to Non Current assets, which
give us net assets.
Net assets should be the same as the total of Capital and Non Current
Liabilities.

Introduction to Accounting

Example 1.1
B Kelly has a business that has been trading for some time. You are given the
following information as at 31.12.2002

Buildings
11,000
Furniture & Fittings
5,500
Motor Vehicles
5,800
Stocks
8,500
Debtor
5,600
Cash a bank
1,500
Cash in hand
400
Creditors
2,500
Capital
30,800
Loan
5,000
You are required to prepare a Balance Sheet as at 31 December 2001
B Kelly
Balance Sheet as at 31 December 2001
Non Current Assets
Buildings
Furniture & Fittings
Motor Vehicles
Current Liabilities
Stock
Debtors
Cash at bank
Cash in hand
Creditors
Net Current Assets
Net Assets
Capital
Non-Current Liabilities
Loan

11,000
5,500
5,800
22,300

8,500
5,600
1,500
400
16,000
(2,500)
13,500
35,800
30,800
5,000
35,800

Example 1.2
L Stokes sets up a new business. Before he actually sells anything he has
bought motor vehicles of 3,000, premises of 7,000, stock of goods 2,000. He
still owes 800 in respect of them. He had borrowed 4,000 from D Evans. After
the events just described and before trading starts, he had 300 cash in hand
and 600 cash at bank.

Lesson One

You are required to calculate the amount of his capital.


Solution:
Assets:

Motor Vehicle
Premises
Stock
Cash at bank
Cash in hand
Liabilities:
Creditors
Loan - D Evans

3,000
7,000
2,000
600
300
12,900

800
4,000

Capital

(4,800)
8,100
8,100

Remember the Accounting equation:


Assets = Liabilities + Capital.
To get capital we rearrange the equation as follows:
Capital = Assets - Liabilities
Total Assets = 12,900
Total Liabilities = 4,800
Capital = 12,900 - 4,800
= 8,100
Example 1.3
C Kings has the following items in his balance sheet as on 30 June 2002.
Capital 41,800, Creditors 3,200, Fixtures 7,000, Motor Vehicles 8,400,
Stock of goods 9,900, Debtors 6,500, Cash at bank 12,900 and Cash in hand
240.
During the first week of July 2002:
a. He bought extra stock of goods 1,540 on credit.
b. One of the debtors paid him 560 in cash.
c. He bought extra fixture by cheque 2,000.
You are to draw up a balance sheet as on 7 July 2002 after the above
transactions have been completed.

10

Introduction to Accounting

First we need to look at the effect of the above transactions on the assets and
liabilities of C Kings.
For
(a) Buying extra stock increases the level of stock by 1,540 and because
this is bought on credit the creditors increase by 1,540 also.
(b) Amount received from the debtor means that the level of debtors reduces
and cash increases by 560.
(c) Extra fixtures bought by cheque, will increase the fixtures and reduce
the cash at bank by 2,000.
This can be summarized as follows:

Capital
Creditors
Fixtures
Motor Vehicles
Stock
Debtors
Cash at bank
Cash in hand

Opening
Balances

41,800
3,200
7,000
8,400
9,900
6,560
12,900
240

Increase/(Decrease)
1,540
2,000
1,540
(560)
(2000)
560

Closing
Balances

41,800
4,740
9,000
8,400
11,440
6,000
10,900
800

Given these closing balances then the balance sheet can be drawn as follows:
C Kings
Balance sheet as at 7 July 2002.
Non Current Assets
Fixtures
Motor Vehicles
Current Assets
Stock
Debtors
Cash at bank
Cash at hand
Current Liabilities
Creditors
Net Current Assets
Net Assets
Capital

9,000
8,400
17,400

11,440
6,000
10,900
800
29,140
(4,740)
24,400
41,800
41,800

Lesson One

11

From the illustration remember that any change in the items of the balance
sheet will have a double effect on the accounting equation has a double effect
and therefore the equation will always balance.

12

Introduction to Accounting

Example 1.4
D Moody has the following assets and liabilities as on 31 April 2002:

Creditors
15,800
Equipment
46,000
Motor Vehicle
25,160
Stock
24,600
Debtors
23,080
Cash at bank
29,120
Cash in hand
160
During the first week of May 2002 Moody:
a. Bought extra equipment on credit for 5,520.
b. Bought extra stock by cheque 2,280.
c. Paid creditors by cheque 3,160.
d. Debtors paid 3,360 by cheque and 240 by cash.
e. Moody put in extra 1,000 cash as capital.
Required:
a. Determine the capital as at 1st May 2002.
b. Draw up a balance sheet after the above transactions have been
completed.
Solution:
(i) Using the accounting equation of Assets = Liabilities + Capital, then assets
and liabilities can be listed as follows.
Assets

Liabilities

Equipment
46,000
Creditors
15,800
Motor Vehicle
25,160
Stock
24,600
Debtors
23,080
Cash at bank
29,120
Cash in hand
160
148,120
Capital = Assets Liabilities
= 148,120 - 15,800 = 132,320
(ii) To draw up the balance sheet, we consider the effect of the above transactions
on the relevant balances:
a. Buying extra equipment means that the equipment balance will increase by
5,520 and the creditors will also increase by the same amount.
b. Buying extra stock by cheque means that the level of stock goes up by
2,280 and the balance at bank reduces by the same.
c. Paying creditors by cheque reduces the balance on the creditors account
and also reduce the amount at the bank.
d. Debtor paying the firm reduces the debtors balance by 3,600 and increases
the cash at bank and cash in hand by 3,360 and 240 respectively.

Lesson One

13

e. Additional cash of 1,000 increases the cash in hand balance by 1,000 and
the capital balances.

14

Introduction to Accounting

This is also summarized as follows:


Opening
Adjustment
Closing
Balance
Increase/Decrease
Balance
Assets/Liabilities

Equipment
46,000
+5,520
51,520
Motor Vehicle
25,160
25,160
Stock
24,600
+2,280
26,880
Debtors
23,080
-3,600
19,480
Cash at bank
29,120
(-2,280 3,160 + 3,360)
27,040
Cash in hand
160
(+240 + 1000)
1,400
Creditors
15,800
(+5,520 3,160)
18,160
Capital
132,320
+1,000
133,320
The balance sheet will therefore be prepared as follows:
D Moody
Balance sheet as at 7 May 2002
Non Current Assets
Equipment
Motor vehicle

Current Assets
Stock
Debtors
Cash at bank
Cash in hand

51,520
25,160
76,680

26,880
19,480
27,040
1,400
74,800

Current Liabilities
Creditors
Net Current Assets
Net Assets

(18,160)
56,640
133,320

Capital

133,320

Double Entry Aspects


The Accounting equation forms the basis of double entry and therefore it should
always be maintained. Any change in assets, liabilities or capital will have a
double effect such that assets will always be equal to liabilities plus capital. If the
owners put in additional capital then this will increase the cash at bank and the
capital amount therefore the equation is still maintained.
Name
Debit
Date

Detail

Folio

Amount

Credit
Date

Detail

Folio

Amount

Lesson One

15

In this account the date will show the opening period of the asset ,liability or
capital i.e. the balance brought forward. It will also show the date when a
transaction took place (i.e. either an asset was bought or liability incurred).
The detail column (also called the particulars column) shows the nature of the
transaction and reference to the corresponding account. The Folio Column for
purposes of detailed recording shows the reference number of the corresponding
account. The amount column shows the amount of the asset, liability or capital.
The left side of the account is called the debit side and the right side is called the
credit side. All assets are shown or recorded on the debit side while all the
liabilities and capital are recorded on the credit side. Each type of asset or liability
must have its own account whereby all transactions affecting them are recorded in
this account. Therefore there should be an account for Premises, Plant and
Machinery, Stock, Debtors, Creditors etc.
Under the accounting equation if all assets are represented by liabilities and
capital therefore all debits should be the same as credits.
For the double entry to be reflected in the accounts, every debit entry must have a
corresponding credit entry. The transactions affecting these accounts are posted
in the account as debit entry and credit entry to complete the double entry.
When we make a debit entry we are either:
i. Increasing the value of an asset.
ii. Reducing the value of a liability.
iii. Reducing the value of capital.
When we make a credit entry we are either:
i. Reducing the value of an asset.
ii. Increasing the value of a liability.
iii. Increasing the value of capital.
Example 1.5
H Jumps has the following assets and liabilities as on 30 November 2002:
Creditors 39,500; Equipment 115,000; Motor vehicle 62,900; Stock 61,500;
Debtors 57,700;Cash at bank 72,800 and Cash in hand 400.
Compute the balance on the capital account as at 30 November 2002.
During the first week of December 2002, Jump:

16

Introduction to Accounting

a.
b.
c.
d.
e.

Bought extra equipment on credit for 13,800.


Bought extra stock by cheque 5,700.
Paid creditors by cheque 7,900.
Received from debtors 8,400 by cheque and 600 by cash.
Put in an extra 2,500 cash as capital.

You are to draw up a balance sheet as on 7 December 2002 after the above
transactions have been completed.

Lesson One

17

Answer:
Capital = Assets Liabilities

Assets

Liabiliti
es
115,000 Creditor
s
62,900
61,500
57,700
72,800
400
371,300

Equipment
Motor vehicle
Stock
Debtors
Cash at bank
Cash in hand

39,50
0

Capital = 371,300 - 39,500 = 330,800

Creditors A/C
a/c
2002

Motor Vehicles

2002

2002

2002

Bank
7900
c/d 62,900
1.12 Bal c/d
31,600

1.12 Bal b/d

39,500

1.12

Bal b/d 62,900 1.12 Bal

62,900
62,900
39,500

2002

1.12 Bal b\d


Creditors

2002

1.12 Bal b\d


Bank

39,500
Equipment a/c
2002
115,000
13,800 7.12 Bal c\d
128,800
128,800
Stock a/c
2002
61,500
5700 7.12 Bal c\d
67,200

128,800

67,200
67,200

18

Introduction to Accounting

Debtors a/c
2002

2002

1.12 Bal b\d

57,700 Bank
Cash
570 7.12 Bal c\d
57,700

Bank

Cash at Bank a/c


2002

72,800 Stock
Creditors
8,400 7.12 Bal c\d
81,200

2002

1.12 Bal b\d


Debtors

2002

1.12
400
Debtors
Capital

2002

7.12 Bal b\d

2002

7.12 Bal b\d


H Jump

Bal

8,400
600
48,700
57,700

5,700
7,900
67,600
81,200

Cash in hand a/c


2002

b\d
600
2500 7.12 Bal c\d
3500
Capital
2002
1.12 Bal b\d
333300 Cash
128,800
Creditors Of Equipment
2002
13800 Equipment
13,800

3500
3500

330800
2500
128,800

13800
13,800

Lesson One

19

Balance sheet as at 7 December 2002


Non Current Assets
Equipment
Motor vehicles

Current Assets
Stock
Debtors
Cash at Bank
Cash in Hand
Current Liabilities
Creditors of equipment
Creditors
Net Current Assets
Net Assets
Capital

128,800
62,900
191,700

61,200
48,700
67,600
3,500
187,000
13,800
31,000

(45,400)
141,000
333,300
333,300

20

Introduction to Accounting

Example 1.6
Write up the asset, capital and liability accounts in the books of M Crash to record
the following transactions:
2002
June 1
Started business with 50,000 in the bank.

2
Bought motor van paying by cheque 12,000.

5
Bought Fixtures 4,000 on credit from Office Masters Ltd.

8
Bought a van on credit from Motor Cars Ltd 8,000.

12
Took 1,000 out of the bank and put it into the cash till.

15
Bought Fixtures paying by cash 600.

19
Paid Motor Cars Ltd by cheque 8000.

21
A loan of 10,000 cash is received from J Marcus.

25
Paid 8,000 of the cash in hand into the bank account.

30
Bought more Fixtures paying by cheque 3,000.
Capital a/c

Cash at

bank a/c
2002
2002
30/6 Bal c/f 50,000 1/6
Van
12,000

Bank

50,000

2002
1/6

Capital
12/6

12/6Cash

Cash

2002
50,000 2/6
8,000

1,000
19/6Motor

ltd 8,000
50,000

50,000

30/6 Fixtures

3,000
30/6 Bal c/f 34,000
58,000
58,000
2002
2/6
12,000
8/6
8,000

2002
5/6
4,000
15/6

Motor Van

Bank
Super

M 30/6
20,000
20000
Fixtures
2002
young
Cash

Bal

c/f
20000

Lesson One

600
30/6
3000

21

Bank

Bal c/f

7,600

7,600
2002
19/6
8000

7,600

Motor Car Ltd Creditors


2002
Bank 8/6 Van
8000

2002
30/6

2002
12/6
1,000
21/6 J. Marcus

2002
30/6

8000
8000

Office Masters Ltd - Creditor


2002
B\f 8/6 Fixtures
4000
4000

Cash in hand
2002
Cash 15/6
600
25/6 Bank
10000 30/6 Bal c/f
11000
J. Marcus - Loaner
2002
c\f 21/6 Cash
10000

4000
4000

Cash
800
2400
11000

10000

Note that the difference between the debit side and the credit side is the balancing
figure. Most assets will have a balance on the credit side and most liabilities and
capital accounts will have a balance on the debit side.
A simple balance sheet from these balances will be as follows:
M Crash
Balance Sheet as at 30th June 2002

Non Current Assets


Fixtures
Motor vehicles
Current Assets
Cash at bank
Cash in hand

34,000
2,400
36,400

7,600
20,000
27,600

22

Current Liabilities
Creditors others
Net Current Assets
Net Assets
Capital
Non Current Liabilities
Loan J Jarvis

Introduction to Accounting

(4,000)
32,400
60,000
50,000
10,000
60,000

Let us now consider other transactions that take place in a business and the
accounting entries to be made.
Accounting for sales, purchases, incomes and expenses.
Sales:
This is the sell of goods that were bought by a firm (the goods must have been
bought with the purpose of resale). Sales are divided into cash sales and credit
sales. When a cash sale is made, the following entries are to be made.
i.
Debit cash either at bank or in hand.
ii.
Credit sales account.
For a credit sale:
i.
Debit debtors/ Accounts receivable account.
ii.
Credit sales account.
A new account for sales is opened and credited with cash or credit sales.
Purchases:
Buying of goods meant for resale. Purchases can also be for cash or on credit. For
cash purchases:
i.
Debit purchases.
ii.
Credit cash at bank/cash in hand
For credit purchases, we:
i.
Debit purchases.
ii.
Credit creditors for goods.
A new account is also opened for purchases where both cash and credit purchases
are posted. NOTE: NO ENTRY IS MADE INTO THE STOCKS ACCOUNT.
Incomes:
A firm may have other incomes apart from that generated from trading (sales).
Such incomes include:
Rent
Bank interest
Discounts received.

Lesson One

23

When the firm receives cash, from these incomes, the following entries are
made:
Debit cash in hand/at bank.
Credit income account.
Each type of income should have its own account e.g. rent income, interest
income.
Incomes increase the value of capital and that is the reason why they are
posted on the credit side of their respective accounts.
Expenses:
These are amounts paid out for services rendered other than those paid for
purchases. Examples include:
Postage and stationery
Salaries and wages
Telephone bills
Motor vehicle running expenses.
Bank charges.
When a firm pays for an expense, we:
i.
Debit the expense account.
ii.
Credit cash at bank/in hand.
Each expense should also have its own account where the corresponding entry will
be posted. Expenses decrease the value of capital and thus the posting is made on
the debit side of their accounts.
The following diagram is a simple summary of the entries made for incomes and
expenses.

24

Introduction to Accounting

Debit cash book/bank/in hand

INCOME

Credit

INCOMES/EXPENSES

Debit Expense A/C

EXPENC
Credit cash book
/bank/in hand
Returns Inwards and Returns Outwards.
Returns Inwards: These are goods that have been returned by customers due to
various reasons e.g.
i.
They may be defective/damaged,
ii.
Being of the wrong type .
iii.
Excess goods being delivered.
Goods returned may relate to cash sales or credit sales. For the goods returned in
relation to cash sales and cash is refunded to the customer the following entries
are made:
i.
Debit returns inwards
ii.
Credit cashbook.
For goods returned that relate to credit sales; no cash has been given to customer,
the following entry is to be made.
i.
Debit returns inwards.
ii.
Credit debtors.
Returns Outwards: These are goods returned to suppliers/creditors. They may be
for cash purchases or for credit purchases. For cash purchases a cash refund
given to the firm by the supplier,
i.
Debit the cashbook (cash at bank/hand).
ii.
Credit returns outwards.
For credit purchases and no refund has been made:
i.
Debit creditors.
ii.
Credit returns outwards.

Lesson One

25

Diagrammatically shown as follows:


Debit returns inwards.
Cash
Credit cashbook.
Debit returns inwards

Inwards
Credit

Credit debtors
Debit cash
Returns

Cash
Outwards

Credit returns

outwards
Debit creditors
Credit
Credit returns
outwards
Now lets us take one example that includes most of the above transactions.
Example 1.8
You are to enter the following transactions, completing the double entry in the
books for the month of May 2002.
2002
May 1
Started business with 2,000 in the bank.

2
Purchased goods 175 on credit from M Rooks.

3
Bought furniture and fittings 150 paying by cheque.

5
Sold goods for cash 275.

6
Bought goods on credit 114 from P Scot.

10
Paid rent by cash 15.

12
Bought stationery 27, paying in cash.

18
Goods returned to M Rooks 23.

21
Let off part of the premises receiving rent by cheque 5.

23
Sold goods on credit to U Foot for 77.

24
Bought a motor van paying by cheque 300.

30
Paid the months wages by cash 117.

31
The proprietor took cash for himself 44.
Example
2002

1/5
2,000

Bank a/c
2002

Capital 3/5Furn&
150
24/5
300

fitting
Motor

vehicle

26

Introduction to Accounting

31/5
1,555
21/5
5

Bal

c/f

Rent
2,005

31/5
2,000

Bal

2,005

Capital a/c
c/f 1/5 Bank

2,000

Purchases a/c
2002

Rooks

2002

2/5M
175
6/5
114

Scot 31/5
289
289

Bal

289

Creditor M Rooks a/c


2002

Returns
in 2/5
175
Bal
c/f

2002

18/5
23
31/5
152

c/f

Purchases

175

175

Furniture & Fittings a/c


2002
2002

3/5 Bank
150
31/5 Bal c/f 150

Sales a/c
2002

2002

31/5 Bal c/f


Cash

352 5/5

275
150

150

23/5

U. Foot 77
352
352

Lesson One

27

Cash in hand a/c


2002
2002

5/5 Sales
275
10/5 Rent
15

12/5 Stationery 27
6/5Purchases 114
30/5 Wages
117
31/5 Bal c/f 116
275

P Scot a/c

2002

31/5

275

Bal c/f

2002
114

114

114
Expenses Rent a/c
2002
11/5 Bal c/f
27

2002
15 10/5 Cash

Expenses Stationery a/c

2002
15

12/5 Cash

27

2002

31/5Bal c/f
27

27

28

Introduction to Accounting

Returns Out a/c


2002

2002

31/5 Bal c/f


Bank
5

23

18/5

Income Rent a/c

2002

2002

M Rooks

Debtors U Foot a/c


2002
2002

23/5 Sales
77
31/5 Bal c/f
300

23

77

21/5

2002

117

31/5 Bal c/f

2002
117

31/5

Motor vehicle a/c


2002

2002

24/5 Bank 300 31/5 Bal c/f

Expenses
Drawings a/c
2002

30/5 Cash
c/f 44

Bal c/f

Wages

31/5 Cash

44

a/c
200

31/5

Bal

Accounting for drawings, discounts allowed and discounts received.


Drawings
The owner makes drawings from the firm in various ways:
i) Cash or bank withdrawals
When the owner withdraws money from the business we debit drawings and credit
cashbook (cash in hand or cash at bank).
ii) Taking goods for own use and
When the owner takes out some of the goods for his own use, we debit drawings
and credit purchases.
iii) Personal expenses, paid by the business
Here we debit the drawings and credit expense account

Lesson One

29

Taking some of the other assets from the business e.g. motor vehicles or using part
of the premises.
Sometimes the owner may take over some of the assets of the business e.g. vehicle
or converting business premises into living quarters or not paying into the
business cash collected personally from the customers. When this happens we
debit drawings and credit the relevant asset e.g. motor vehicles, premises or some
building or even debtors.

30

Introduction to Accounting

Discounts
Discounts received.
A discount received is an allowance by the creditors to the firm to encourage the
firm to pay the amount dues within the agreed time. It is an amount deducted
from the invoice price.
When a discount is given by the supplier then we debit creditors account and
credit discounts received e.g. A. Ltd sells some goods on credit to B Ltd. 1,000
under the terms of sale, B Ltd, will receive a discount of 5% if they pay the amount
due within one month. B decides to take up the offer and pays the amount within
the given time. B will record the transaction as follows.
Debit: Creditor A Ltd
Credit: Discounts Received
Creditor

A.

Ltd

a/c

Purchases a/c
2002

2002
2002

Bank
950
1,000
Discount received 50
1000

Purchases

Discounts Received a/c

2002

200

Bal c/f
50
Ltd
950

A Ltd

2002
1,000

A Ltd

1000

2002

50

Bank a/c
2002
A

Discounts Allowed
These are the allowances made by a firm on the amounts receivable from the
customers to encourage prompt payment. The amounts deducted from the sales
invoice. In the previous example when A Ltd issued the discount and was taken up
by B the entries will be:
i. Debit - discount allowed
ii. Credit - debtors - B Ltd.
2002
Sales

Debtors B Ltd a/c

2002
1,000 Bank

950

2002

Discount
Debtor 1,000

Sales a/c
2002

50

Lesson One

31

1,000

2002
Debtor

Discount allowed a/c


2002
50 Bal c/f

1,000

50

2002
Debtor

Bank a/c

950

TRIAL BALANCE
The trial balance is a simple report that shows the list of account balances
classified as per the debits and credits. The purpose of the trial balance is to show
the accuracy of the double entries made and to facilitate the preparation of final
accounts i.e. the trading, profit & loss account and a balance sheet.
The debits of the trial balance should be the same as the credits, if not then there
is an error in one or more of the accounts.
The trial balance in example 1.8 would be extracted as follows:
Name
Trial balance as at 31 May
2002
Debit Credi
t

Rent income
5
Debtor U Foot
7
Motor vehicle
300
Bank
1555
Purchases
289
Wages
117
Capital
2000
Creditor

M
152
Rooks
Furniture
&
150
Fittings
Sales
352
Cash in hand
72
Creditor P Scot
114
Expenses Rent
15
Expenses

27
Stationery
Returns
23
Outwards
Drawings
44
.
2464
2464
From the trial balance please note that assets and expenses are on the debit side.
Capital, liabilities and incomes are normally listed on the credit side.

32

Introduction to Accounting

The next example is a detailed one that shows extracting of trial balance once all
the postings have been made in the relevant accounts.
Example 1.9
Write up the following transactions in the books of S Pink:
2003
March
1
Started business with cash 1,000.

2
Bought goods on credit from A Cliks 296.

3
Paid rent by cash 28.

4
Paid 1,000 of the cash of the firm into a bank account.

5
Sold goods on credit to J Simpson 54.

7
Bought stationery 15 paying by cheque.

11
Cash sales 49.

14
Goods returned by us to A Cliks 17.

17
Sold goods on credit to P Lutz 29.

20
Paid for repairs to the building by cash 18.

22
J Simpson returned goods to us 14.

27
Paid A Cliks by cheque 279.

28
Cash purchases 125.

29
Bought a motor vehicle paying by cheque 395.

30
Paid motor expenses in cash 15.

31
Bought fixtures 120 on credit from R west.
Solutions
Capital a/c

Cash in hand

a/c
2003

2003

2003

31/3 Bal c/d 1,500 1/3 Cash 1,500


1/3 Capital
28
11/3 Sales
1,000

2003
1,500
49

3/3 Rent
4/3

20/3

Bank
Repairs

18
28/3
Purchases

125
30/3

exp.

Motor

15
31/3

Bal c/d

363
1,549
1,549
Purchases a/c

Lesson One

33

2003
2003

2/3 A Hanson 296 31/3 Bal c/d 421


28/3 Cash
125

Creditors A Cliks ac
2003

2003

421
Purchases

421

14/3 Returns out

17 2/3

296
27/3 Bank

279
296

296
Rent Expenses a/c
2003

3/3
15

2003
Cash

28

31/3

Bank a/c
2003

Bal c/d

28

2003

4/3 Cash

1,000

5/3 Stationery
27/3 A. Hanson

279
29/3 Motor van
395
31/3

Bal c/d

311
1,000
1,000
Debtor J Simpson a/c

Sales

a/c
2003

3/3
Sales
JSimpson
54

2003
54

22/3 Returns in
31/3 Bal c/d

2003
14

` 2002
31/3

40

Bal c/d

132

5/3

11/3 Sales

49
17/3 P Lutz
29
54

54
132

132

34

Introduction to Accounting

Stationery a/c
2003

15
31/3 Bal c/d

2003
7/3
Bank

Returns outwards a/c


15
2003

2003

31/3
Cliks

Bal c/d

17 14/3

17

P Lutz Debtor a

Building repairs -

expenses
2003

2003

17/3 Sales 29 21/3 Bal c/d


29
c/d 18

2003
20/3

Cash

2003

18

31/3 Bal

Returns - Inwards
2003
2003

22/3 J Simpson 14 31/3 Bal c/d


395

2003
14

29/3 Bank

R West Creditor (others)


2003
31/3 Bal c/d
15

2003
31/3 A. Webster

2003

120 31/3 Fixtures 120

Fixtures

2003

120 31/3 Bal c/d

Motor vehicle
2003

395 31/3 Bal c/d

Motor expenses
2003

120

2003
30/3 Cash
15

31/3 Bal c/d

Lesson One

35

36

Introduction to Accounting

S PINKS
TRIAL BALANCE AS AT 31 MARCH 2003
Debit
()
Capital
Purchases
Cash in hand
Bank
Rent expense
Sales
Fixtures
Debtor J Simpson
Debtor P Lutz
Motor vehicle
Creditors
Motor expenses
Returns inwards
Creditors others R
West
Stationery
Returns outwards
Building repairs

Credit
()
1500

421
363
311
28
132
120
40
29
395
15
14

120

15
18
1769

17
1769

Example 1.10
The following transactions took place during the month of May:
2003
May
1
Started firm with capital in cash of 250.

2
Bought goods on credit from the following persons: R Kelly 54;
Pcombs 87;
J Role 25; D Mobile 76; I. Sims 64.

4
Sold goods on credit to: C Blanes 43; B Long 62; F Skin 176.

6
Paid rent by cash 12.

9
C Blanes paid us his account by cheque 43.

10
F Skin paid us 150 by cheque.

12
We paid the following by cheque: J Role 25; R Kelley 54.

15
Paid carriage by cash 23.

18
Bought goods on credit from P Combs 43; Mobile 110.

21
Sold goods on credit to B Long 67.

31
Paid rent by cheque 18.

Lesson One

37

Answer
200
3
31/
5

200
3
12/
5

200
3
12/
5

Bal
c/d

Creditor R Kelly
200
3
Ban
54 2/5 Purcha
k
ses

Creditor J Role

Ban
k

200
3
31/5 Bal
c/d

200
3
4/5
21/
5

Capital

200
3
250
1/5 Cash

25 2/5

Purcha
ses

Creditor I Sims
200
3
64 2/5 Purcha
ses

Debtor B Long
200
3
Sale
62 31/ Bal c/d
s
5
Sale
67
s
129

250

54

25

64

129
.
129

200
3
1/5

200
3
31/
5

200
3
31/
5

200
3
4/5

200
3
4/5

Cash in Hand

200
3
Capit 250 6/5 Rent
al
15/ Carria
5
ge
. 31/ Bal c/d
5
250

12
23
21
5
25
0

Creditor P Combs
200
3
Bal c/d 130 2/5
Purcha
ses
. 18/5 Purcha
ses
130

Creditor D Mobile

200
3
Bal
186 2/5 Purcha
c/d
ses
. 18/ Purcha
5
ses
186
Debtor C. Blares

200
3
Sales 43
4/5 Bank

Debtor F Smith
200
3
Sales
176 10/ Bank
5
. 31/ Bal c/d
5
176

87
4
3
13
0

76
11
0
18
6

43

15
0
2
6
17
6

38

Introduction to Accounting

Sales
Purchases

200
3

200
3
2/5

R Kelly

2/5

P Combs

54 31/
5
87

2/5

J Role

2/5

200
3

45
9

31/
5

200
3

348

4/5

43

4/5

25

4/5

F Skin

D Mobile

76

4/5

B Long

17
6
67

2/5

L Sims

64

18/5

P Combs

43

18/5

D.
Mobile

10
0
45
9

10/5

C
Blanes
H
F
Skin

.
459

Cash
Bank

2003
15/5

J Role

25

15 12/5
0
31/5

R
Kelly
Rent

54

Bal
c/d

9
6
19
3

31/5

31/
5

Carriage Expenses

43 12/5

.
19
3

19x
6
6/5

62

.
34
8

Bank
200
3

200
3

Bal
c/f

C
Blanes
F Long

9/5

Bal
c/d

Rent
19x
6
12 31/
5
1
8

18

Bal
c/d

30
.

200
3
Cash

23 31/
5

Bal c/d

23

Lesson One

39

3
0

30

Trial Balance as at 31/5/2003


Capital
Cash
Creditor R Kelly
Creditor P Combs
Creditor J Role
Creditor D Mobile
Creditor L. Simms
Debtor C. Blanes
Purchases
Sales
Debtor- B. Long
Debtor- F Skin
Bank
Carriage
Rent

Debit
215

Credit
250
-

130

186
64
-

459

348

129
26

96

30

978

978

40

Introduction to Accounting

REINFORCEMENT QUESTIONS
Question One
Spark has been trading for a number of years as an electrical appliance retailer
and repairer in premises which he rents at an annual rate of $1,500 payable in
arrears. Balances appearing in his books at 1 January 19X1 were as follows:
$
Capital account
Motor van
Fixtures and fittings
Provision for depreciation on motor van (credit)
Provisions for depreciation on fixtures& fittings
(credit)
Inventory at cost
Receivables for credit sales:
Brown
Blue
Stripe
Cash at bank
Cash in hand
Payables for supplies:
Live
Negative
Earth
Amount owing for electricity
Local taxes paid in advance

$
1,808
1,200
806
720
250
366

160
40
20
220
672
5
143
80
73
296
45
100

Although Sparks has three credit customers the majority of his sales and services
are for cash, out of which he pays various expenses before banking the balance.
The following transactions took place during the first four months of
Januar Februa Marc
y
ry
h
$
$
$
Suppliers invoices:
Live
468
570
390
Negative
87
103
Earth
692
187
Capital introduced
500
Bankings of cash (from cash sales)
908
940
766
Expenditure out of cash sales before
banking:
Withdrawals on account
130
120
160

19X1
April
$
602
64
1,031
150

Lesson One

41

Stationery
Travelling
Petrol and van repairs
Sundry expenses
Postage
Cleaners wages
Goods invoiced to credit customers:
Brown
Blue
Stripe
Cheque payments (other than those to
suppliers):
Telephone
Electricity
Local taxes
Motor van (1 February 19X1)
Unbanked at the end of April

12
6
19
5
12
60

14
10
22
4
10
60

26
11
37
7
15
65

21
13
26
3
19
75

66
120

22
140

10
130

12
180

44

38

20

48

40
62
-

49
47
800
-

59
20
220
-

66
106
12

Spark pays for goods by cheque one month after receipt of invoice, and receives a
settlement discount of 15% from each supplier.
Credit customers also pay by cheque one month after receipt of invoice, and are
given a settlement discount of 10% of the invoice price.
Required:
Write up the ledger accounts of Spark for the four months to 30 April 19X1, and
extract a list of account balances after balancing off the accounts.

Question Two
Mary
Balance Sheet as at 31 December 2000
Non Current Assets
Premises

Plant

Current Assets:
Stock
Debtors
Cash at bank
Cash in hand
Current liabilities:
Creditors

25,000.0
0
12,000.0
0
37,000.0
0

11,000.00
10,000.00
5,000.00
3,000.00
29,000.00
(12,000.0

17,000.0

42

Introduction to Accounting

0)

Capital

0
54,000.0
0
34,000.0
0

Non Current Liabilities:


Loan from bank

20,000.0
0
54,000.00

During the year to 31 December 2001 the following total transactions occurred:

a) Mary withdrew a total of 10,000.00 in cash


b) Stock in trade was bought, all on credit, for 34,000.00
c) Sales were made totaling 60,000.00 of stock in trade which had cost
37,000.00. Of these sales 51,000.00 were on credit and 9,000.00 for cash.
d) A total of 16,000.00 was drawn from the bank in cash to the cash till.
e) Electricity for the year paid by cheque totaled 2,000.00
f) Rates for the year paid by cheque totaled 1,000.00
g) Wages for the year all paid cash totaled 10,000.00
h) Sundry expenses all paid in cash totaled 2,000.00
i) Creditors were paid a total of 36,000.00 all by cheque
j) Debtors paid a total of 54,000.00 all in cheques.
k) The bank charged interest on the loan deducting 3,000.00.
Required:
Prepare a revised balance sheet.
(20 marks)

Question Three
a) Explain the nature of accounting and the accounting equation
(8 marks)
b) Calculate the profit for the year ended 31 December 2001 from the following
information
(12 marks)
Non Current
Assets

01.01.200
1

31.12.200
1

Lesson One

Property
Machinery

43

20,000.00
6,000.00
26,000.00

Current Assets:
Debtors
Cash
Current
Liabilities:
Creditors
Overdraft
Net Current
Liabilities
Net Assets

20,000.00
9,000.00
29,000.00
8,000.00

4,000.00
1,000.00
5,000.00

1,500.00
9,500.00

5,000.00
6,000.00

3,000.00
9,000.00

11,000.00
(6,000.00)

12,000.00
(2,500.00)

20,000.00

26,500.00

Drawings during the year amounted to 4,500.00


Additional capital introduced by the owner 5,000.00

Question Four
Brian Barmouth is a sole trader. At 30 June 2000 the following balances have been
extracted from his books:
Sales
Purchases
Office expenses
Insurance
Wages
Rates
Heating and
Lighting
Telephone
Discounts allowed
Opening stock

47,600.
00
22,850.
00
1,900.0
0
700.00
7,900.0
0
2,800.0
0
1,200.0
0
650.00
1,150.0
0
500.00

44

Introduction to Accounting

Returns inwards
Returns outwards
Premises
Plant and
Machinery
Motor Vehicles
Debtors
Bank balance
Creditors
Loan-long term
loan
Capital
Drawings for the
year
Closing stock

200.00
150.00
40,000.
00
5,000.0
0
12,000.
00
12,500.
00
7,800.0
0
3,400.0
0
10,000.
00
60,000.
00
4,000.0
0
550.00

Required:
Construct a trial balance, from the above list of balances.
CHECK YOUR ANSWERS WITH THOSE GIVEN IN LESSON 9 OF THE
STUDY PACK

Acknowledgement

45

LESSON TWO
FINAL ACCOUNTS
FINAL ACCOUNTS FOR SOLE TRADERS
(a) TRADING ACCOUNT
The trading account summarises the trading activities (sale and purchase of
goods/stocks) of the
business and tries to determine the gross profit for the
relevant financial period.
The gross profit is then taken up in the profit and
loss account as part of the income.
Format for the trading account:
Name
Trading Account for the year ended 31 Dec.

Sales
Less: Returns Inwards

x
(x)
x

Less: Cost of Sales


Opening stock
Purchases
Add: Carriage Inwards

x
x
x
x

Less: Returns Outwards


Cost of stock available for sale
Less: Closing stock
Gross Profit

x
x
x

(x)
x

Example: 2.1
From the following details draw up the trading account of Springs for the year
ended 31 December 2002, which was his first year in business.

Carriage inwards
Returns outwards
Returns inwards
Sales
Purchases
Stock of goods: 31 December 19x7

6,700
4,950
8,900
387,420
333,330
74,890

FINANCIAL ACCOUNTING 1

46

Final Accounts

Springs
Trading Account for the year ended 31 Dec 2002

Sales
Less: Returns Inwards

387,420
8,900
378,520

Less cost of sales


Purchases
Add: Carriage Inwards

333,330
6,700
340,030
4,950
335,080
74,890
260,190

Less: Returns outwards


Less: Closing stock
Gross Profit
118,330
Example 2.2

The following details for the year ended 31 March 2003 are available. Draw up the
trading account of R Sings for that year.

Stocks: 1 April 2002


Returns inwards
Returns outwards
Purchases
Carriage inwards
Sales
Stocks: 31 March 2003

16,523
1,372
2,896
53,397
1,122
94,600
14323

Answer
R Sings
Trading Account for the year ended 31 Mar 19x8

Sales
Less: Returns Inwards

94,600
(1,372)
93,228

Less: Cost of sales


Opening Stock
Purchases
Add: Carriage Inwards

16,523
53,397
1,122
54,519

Lesson Two

Less: Returns Outwards


Cost of goods available for sale
Less: Closing stock
(49,642)
Gross Profit

47

2,896

51,623
68,146
18,504
43,586

(b) PROFIT AND LOSS ACCOUNT


It shows the net profit or net loss that the business has made from all the activities
during a financial period.
The net profit (or loss) is determined by deducting all the expenses from all the
incomes of the same financial period.
In practice, the trading account is combined together with the net profit and loss
account into one report so that the format is as shown below:
Name
Trading, Profit and Loss Account for the year ended 31/12/19xx

Sales
Less: Returns Inwards

x
x

Less: Cost of sales


Opening stock
Purchases
Add: Carriage Inwards

x
x
x
x

Less: Returns Outwards


Cost of goods available for sale
Less: Closing stock
Gross Profit
Discount received
Rent received
Interest received
Other incomes

x
x
x

(x)
x
x
x
x
x
x

Less: Expenses
Carriage Outwards
Discounts allowed
Postage & stationary
Salaries & wages
Rent paid
Insurance & rates
Bank charges
Other expenses
Net profit/ (loss)

x
x
x
x
x
x
x
x

(x)
x/(x)

48

Final Accounts

Lesson Two

49

Example 2.3
From the following trial balance of P Boones draw up a trading and profit and loss
account for the year ended 30 September 2002, and a balance sheet as at that
date.
Dr
Cr

Stock 1 October 19x8


23,680
Carriage outwards
2,000
Carriage inwards
3,100
Returns inwards
2,050
Returns outwards
3,220
Purchases
118,740
Sales
186,000
Salaries and wages
38,620
Rent
3,040
Insurance
780
Motor expenses
6,640
Office expenses
2,160
Lighting and heating expenses
1,660
General expenses
3,140
Premises
50,000
Motor vehicles
18,000
Fixtures and fittings
3,500
Debtors
38,960
Creditors
17,310
Cash at bank
4,820
Drawings
12,000
Capital
126,360
332,890

332,890

50

Final Accounts

Answer
P Boones
Trading, Profit and Loss Account as at 30 September 2003

186,000
(2,050)
183,950

Sales
Less: Returns Inwards
Less: Cost of sales
Opening stock
Purchases
Add: Carriage inwards

23,680
118,740
3,100
12,1840

Less: Returns Outwards


Cost of goods available for sale
Less: Closing stock
Gross Profit

3,220

118,620
142,300
29,460
(11,2840)
71,110

Less Expenses
Salaries & wages
Carriage outwards
Rent
Insurance
Motor expenses
Office expenses
Lighting & heating
General expenses
Net Profit

38,620
2,000
3,040
780
6,640
2,160
1,660
3,140

(58,040)
13,070

Lesson Two

51

(c) BALANCE SHEET


This is a simple report that shows the assets and liabilities of the business and the
capital of the owner as at a certain point in time. The format is at shown below:
Name
Balance sheet as at 31/Dec/19xx
Non Current Assets

Land & Buildings


Plant & Machinery
Fixtures, Furniture & Fittings
Motor vehicles

x
x
x
x
x

Current Assets
Stock/inventories
Debtors trade
Debtors others
Cash at bank
Cash at hand
Current Liabilities
Bank overdraft
Creditors trade
Creditors others
Net current assets
Net Assets

x
x
x
x
x
x
x
x
x

(x)
x
x

Capital
Add: Net profit

x
x
x

Less: Drawings

(x)
x

Non Current Liabilities


Loan (s)

x
x

The balance Sheet of P Boones in example 2.3 will be produced as follows:

52

Final Accounts

P Boones
Balance Sheet as at 30 Sept 2002

Non Current Assets


Premises
Fixtures & fittings
Motor vehicles

50,000
3,500
18,000
71,500

Current Assets
Stock
Debtors
Cash at bank

29,460
38,960
4,820
73,240

Current Liabilities
Creditors
Net Current Assets
Net Assets
Capital
Add: Net Profit
Less: Drawings

(17,310)
55,930
127,430
126,360
13,070
139,430
(12,000)
127,430

`
D) BOOKS OF PRIME ENTRY
The diagram below shows the components of an accounting system for a firm that
carries out trading activities from the source documents that record the evidence
of transactions, where the documents are recorded and the postings to made.

Lesson Two

53

Source
Final
Documents
Accounts
Sales
Invoice
The

Books of

The

Prime entry

Ledger Balances

Recorded Sales
Journal

List of the

Sales
Ledger

THE

Trading
Account
Purchas
e
Invoice

Recorded

Purchase
s
Journal
TRIAL

Credit
note

Recorded

The

Return
Inwards
Journal

Purchases
Ledger

Profit

Debit
Loss
Note
Account

Other
Correspo
Balance
ndence

BALANC
E

Recorded

Receipt
s
Cheque
s
Petty

&

Return
Outwards
journal

Loss

The
cashbook
&
Recorded

Petty
cashbook

General
Ledger

Recorded General
Journal
Balance Sheet

A brief description of each component is explained below.


SOURCE DOCUMENTS

54

Final Accounts

This shows the evidence transactions. They are collected, filed and posted in the
books of prime entry. Example, if a firm sells goods on credit, then an invoice is
raised. The source documents as shown in the above include:

Lesson Two

55

Sales invoice
Purchases invoice
Credit note
Debit note
Receipts, cheques and petty cash vouchers
Other correspondences.

(i) Sales Invoice


The sales invoice is raised by the firm and sent to the debtor/customer when the
firm makes a credit sale.
The sales invoice contains the following:
i. Name and address of the firm
ii. Name and address of the buying firm
iii. Date of making the sale invoice date.
iv. Invoice number
v. Amount due (net of trade discount)
vi. Description of goods sold
vii. Terms of sale
(ii) Purchases Invoice
A purchase invoice is raised by the creditor and sent to the firm when the firm
makes a credit purchase. It shows the following:
i. Name and the address of the creditor/seller
ii. Name and address of the firm
iii. Date of the purchase (invoice date)
iv. Invoice number
v. Amount due
vi. Description of goods sold
vii. Terms of sale
(iii) Credit note
A credit note is raised by the firm and issued to the debtor when the debtor
returns some goods back to the firm. Its contents include:
i. Name and address of the firm
ii. Name and address of the debtor
iii. Amount of credit
iv. Credit note number
v. Reason for credit e.g. if goods sent but of the wrong type.
The purpose of the credit note is to inform the debtor or customer that the
debtors account with the firm has been credited i.e. the amount due to the firm
has been reduced or cancelled.
The credit note may also be issued when the firm gives an allowance of the
amounts due from the debtors. From the context we can assume that all credit
notes are issued when goods are returned.
(iv) Debit note
This is raised by the creditor and issued to the firm when the firm returns some
goods to the creditor. It includes the following items:
i. Name and address of the firm
ii. Name and address of the creditor

56

Final Accounts

iii. Amount of debit


iv. Debit Note number
v. Reason for the debit
The purpose of the debit note is to inform the firm that the amount due to the
creditor has been reduced or cancelled.
The
Firm

Credit sales (sales invoice)

The
Debtor

Returns inwards (credit note)

Credit purchase (purchase invoice)


The
Firm

Returns outwards (debit note)

The
Creditor

(vi) Receipts
A receipt is raised by the firm and issued to customers or debtors when they
make payments in the form of cash or cheques. It shows:
i.
ii.
iii.
iv.

The name and address of the firm


The date of the receipt
Amount received (cash or cheque or other means of payment)
Receipt number.

Cheques
When a firm opens a current account with the bank, a chequebook containing
cheques issued. The cheques allow the firm to make payments against the account
with the bank. When a firm issues a cheque to its creditors for payments, it
authorizes the bank to honour payments against the firms account with the bank.
The cheque contains the following information:
i. Name and account number of the firm (account holder)
ii. The date of the cheque
iii. Name of the payee (creditor)
iv. Name of the firms bank
v. Amount payable in words and figures
vi. The cheque number
vii. The authorized signature(s)
Petty cash vouchers
A petty cash voucher is raised by a cashier to seek authority for payments
(payments of small value in the firm which require cash payments e.g. fuel, busfare, office snacks), which is approved by a senior manager and filed for record
purpose. It shows:

Lesson Two

57

i. Date of payment
ii. Amount paid
iii. Reason for payment
iv. Authorized signature(s):
v. Person approving
vi. Person receiving
The person receiving the money must then return a document supporting how
the money was utilized e.g. fuel receipt, bus ticket e.t.c.
(vii) Other correspondence
These include information received within or outside the firm that has a
financial implication in the accounts.
Examples are:
i. Letters from the firms lawyers about a debtors balance.
ii. Hire-purchase/credit sale or credit purchase agreements that relate to
non-current assets.
iii. Memorandum from a senior manager requiring changes to be made in
the accounts.
iv. Bank statement from the bank, e.g. bank charges.
BOOKS OF PRIME ENTRY
They record the source documents.
Sales Journal
It is also called a Sales Day Book. It records all the sales invoices issued by the
firm during a particular financial period. The format is as follows (with simple
records of invoice).

SALES JOURNAL
Date 19x8
1st March
3rd March
5th March

Total

Detail
S. Spikes
T. Binns
L.Thompson

Page 5
Folio

Amount

SL.10
SL.19

200.00
350.00
SL,8
150.00

700.00

The individual entries in the sales journal are posted to the debit side of the
debtors accounts in the sales ledger and the total is posted on the credit side of
the sales account in the general ledger.

58

Final Accounts

This is shown below:


Sales Ledger
19x
8
1/3

S Spikes

19x
8
Sale 200
s

General Ledger

19x
8

Sales Ledger
19x
8
3/3

19x
8
3/3

Sal
es

General Account
19x
8
5/3
Credit sales
for period

700

General Ledger

T Binus

19x
8
350

L Thompson

19x
8
Sal 150
es

Example 2.4
You are to enter up the sales journal from the following details. Post the items to
the relevant accounts in the sales ledger and then show the transfer to the sales
account in the general ledger.
2003
Mar

1
3
6
10
17
19
27
31

Credit
Credit
Credit
Credit
Credit
Credit
Credit
Credit

sales
sales
sales
sales
sales
sales
sales
sales

to
to
to
to
to
to
to
to

J Gordon
G Abrahams
V White
J Gordon
F Williams
U Richards
V Wood
L Simes

1,870
1,660
120
550
2,890
660
280
780

Lesson Two

59

Answer

SALES JOURNAL
Date (2003)
1/3
3/3
6/3
10/3
17/3
19/3
27/3
31/3

Page 10

Detail

Folio

J. Gordon
G. Abrahams
V. White
J. Gordon
F. Williams
U. Richards
V. Wood
L. Simes

Amount
1,870.00
1,660.00
120.00
550.00
2,890.00
660.00
280.00
780.00
8,810.00

Sales Ledger
J Gordon
20
03
157
0
550

20
03
1/3
10/
3

200
3
3/3

200
3
6/3

G Abrahams
200
3
Sal
1,6
es
60

Sal
es

U White
200
3
120

F Williams

20
03
19/
3

200
3
27/
3

200
3
31/
3

Sal
es

Sal
es

Sal
es

U Richards
20
03
660

V Wood
200
3
280

L Simes
200
3
750

Sales a/c

60

Final Accounts

200
3
200
3
17/
3

Sal
es

200
3

200
3

289
0

Cre
dit
Sale
s

Purchases Journal
Purchases journal is also called a purchases day-book. It records all the purchase
invoices received by the firm during a particular financial period. It has the
following format (including records of invoices).

Date 19x6
1/5
2/5

PURCHASES JOURNAL
Description/Detail
C. Kelly
L. Smailes

Page 15
Amount

Folio
PL. 10

400
PL. 20

TOTAL

350
750

The individual entries in the purchases journal are posted to the credit side of
the creditors accounts in the purchases ledger and the total is posted to the debit
side of purchases account of the general ledger. This is shown below:
19x
6

19x
6

C Kelly
19x
6
1/5 Purchas
es

L Smailes
19x
6
2/5 Purchas
es

Returns Inwards Journal

40
0

25
0

19x
6
31/
5

Purchases a/c
19x
6
Sundry
75
Credito
0
rs

Lesson Two

61

It is also called the returns inwards day-book. It records all the credit notes raised
by the firm and sent to customers during a particular financial period, it has the
following format.
RETURNS INWARDS JOURNAL
Date
1 March
2 March
5 March

Detail

Folio

S. Spikes
C. Kelly
T. Bills

SL. 22

TOTAL

Pg 10
Amount
20
SL. 18
SL. 9

18
15

53

Individual entries in a return inwards journal are posted to the credit of the
debtors accounts in the sales ledger and the total is posted to the debit side of the
return-inwards account of the general ledger.

Sales Ledger

General Ledger

S. Spikes a/c

1/
3

Returns
In

20

C Kelly a/c

2/
3

Returns
In

18

31/
3

Returns Inwards a/c

Sundry
53
Debtors

T. Bills a/c

5/
3

Retur
ns In

15

62

Final Accounts

Lesson Two

63

Returns Outwards Journal


It is also called the returns outwards daybook. It records all the debit notes
received by the firm from the creditors during a particular financial period. It has
the following format.
RETURNS OUTWARDS JOURNAL
DATE

DETAILS

FOLIO

2 May
3 May
4 May

L. Thompson
M. Hyatt
T. Bills

PL. 10

AMOUNT ()
PL. 15
12

PL. 7

TOTAL

14
19

35

Individual entries are posted on the debit side of the creditors account in the
purchases ledger and on the total to credit side of the returns outwards account in
the general ledger.
Purchases Ledger

General Ledger

L. Thompson a/c
Outwards a/c
`

2/5 Returns out


14
sundry
creditors

Returns

31/5

35

M. Hyatt a/c

3/5 Returns out 12

T. Bills a/c

4/5 Returns Out

19

64

The following example 2.5 shows how the four journals are used.

Final Accounts

Lesson Two

65

Example 2. (Frankwood adapted)


You are to enter the following items in the books, post to personal accounts, and
show transfers to the general ledger.
19x5
July 1

3
5

8
12
14
20
24
31
31

Credit purchases from: K Hill 3800; M Norman 500; N Senior 106.


Credit sales to: E Rigby 510; E Phillips 246; F Thompson 356.
Credit purchases from: R Morton 200; J Cook 180; D Edwards 410;
C
Davies 66.
Credit sales to: A Green 307; H George 250; J Ferguson 185.
Returns outwards to: M Norman 30; N Senior 16.
Returns inwards from: E Phillips 18; F Thompson 22.
Credit sales to: E Phillips 188; F Powell 310; E Lee 420.
Credit purchases from: Ferguson 550; K Ennevor 900.
Returns inwards from: E Phillips 27; E. Rigby 30.
Returns outwards to: J Cook 13; C Davies 11.

Study the solution provided:


SALES JOURNAL
DATE

DETAIL

AMOUNT ()

3 July
3 July
3 July
8 July
8 July
8 July
20 July
20 July
20 July

E. Rigby
E. Phillips
F. Thompson
A. Green
H. George
J. Ferguson
E. Phillips
F. Powell
E. Lee

510
246
356
307
250
185
188
310
420

TOTAL

2,772

Sales Ledger
19x
5
3/7

Sal
es

E Rigby
19x
5
51 3/7 Retur
0
ns
Inwar
ds

30

19x
5
3/7

20/
7

E Phillips
19x5

Sal
es

246 14/7

Returns

18

Sal
es

188 31/7

Retuns
in

27

66

Final Accounts

J. Ferguson
19x
5
3/7

F. Thompson
19x
5
Sal
35 14/ Retur
es
6 7
ns in

19x
5
8/7

Sal
es

Green
19x
5
30
7

Sal
es

H George
19x
5
25
0

19x
5
8/7

22

19x
5
8/7

19x
5
20/
7

19x
5
20/
7

19x5
Sal
es

185

F. Powell
19x5
Sal
es

310

Sal
es

E Lee
19x5

420

PURCHASES JOURNAL
DATE
1 July
1 July
1 July
5 July
5 July
5 July
5 July
24 July
24 July

DETAIL
K. Hill
M. Norman
N. Senior
R. Mortan
J. Cook
D. Edwards
C. Davies
C. Ferguson
K. Ennevor

Total

Purchases Ledger

AMOUNT ()
380
500
106
200
180
410
66
550
900
3,292

Lesson Two

67

N. Senior
1995

1995
12/7

Returns out
16

Returns out
30

Returns out
13

1/7
500

Purchases

5/7

Purchases
180

C. Davies
1995

1995
31/7

22

J. Cook
1995

1995
31/7

Purchases

M. Norman
1995

1995
30/7

1/7

Returns out
11

5/7

Purchases
60

K. Hill
1995

1995

1/7

Purchases
380

R. Morton
1995

1995

5/7

Purchases
200

D. Edwards
1995

1995

5/7

Purchases
410

C. Ferguson
1995

1995

27/7

Purchases
550

68

Final Accounts

K. Ennevor
1995

1995

24/7

Purchases
900

RETURNS INWARDS JOURNAL


DATE
14 July
14 July
31 July
31 July

DETAILS
E. Phillips
F. Thompson
E. Phillips
E. Rigby

AMOUNT
18
22
27
30
97

RETURNS OUTWARDS JOURNAL


12
12
31
31

July
July
July
July

M. Norman
N. Senior
J. Cook
C. Davies

30
16
13
11
70

General Ledger
Sales a/c
1995

1995

31/7 Sundry debtors


2772

1995

Purchases a/c
1995

31/7 Sundry creditors


3292

1995

31/7 Sundry debtors


97

Returns Inwards a/c


1995

Lesson Two

69

Returns Outwards a/c


1995

1995

31/7 Sundry creditors


70

CASH BOOKS
A cashbook records all the receipts (cash and cheques from customers and debtors
or other sources of income) and all the payments (to creditors or suppliers and
other expenses) for a particular financial period. The cashbook will also show us
the cash at bank and cash in hand position of the firm.
There are two types of cashbooks:
i.
ii.

Cash in hand cashbook, which records the cash transactions in the firm or
business.
Cash at bank cashbook, which records the transactions at/with, the bank.
The cashbook is the most important book of prime entry because it forms part of
the general ledger and records the source documents (receipts and cheques). The
cash at bank cashbook and cash in hand cashbook are combined together to get a
two-column cashbook. The format is as follows:
Two-column cashbook.
CASH BOOK
Date Details
Bank

Cash
()

Bank
()

Date

Details
()

Cash
()

Additional columns for discounts allowed and discounts received can be included
with the cash at bank columns to get a 3 column cashbook. The format is as
follows:

70

Final Accounts

Date Details
Bank
Cash

Discount Cash
Allowed

()

Bank

Date Details

()

Discounts

Received

()

The balance carried down (Bal c/d) for cash in hand and cash at bank will form
part of the ledger balances and the discounts allowed and discounts received
columns will be added and the totals posted to the respective discount accounts.
The discount allowed total will be posted to the debit side of the discount allowed
account in the general ledger and the total of the discount received will be posted
to the credit side of the discount-received account of the general ledger.
Cash at bank can have either a credit or debit balance. A debit balance means the
firm has some cash at the bank and a credit balance means that the account at the
bank is overdrawn. (the firm owes the bank some money).
Example 2.7
Write up a two-column cashbook from the following details, and balance off as at
the end of the month:
2003
May

1
2
3
4
5
7
9
11
15
16
19
22
26
30
31

Started business with capital in cash 1,000.


Paid rent by cash 100.
F Lake lent us 5,000, paid by cheque.
We paid B McKenzie by cheque 650.
Cash sales 980.
N Miller paid us by cheque 620.
We paid B Burton in cash 220.
Cash sales paid direct into the bank 530.
G Moores paid us in cash 650.
We took 500 out of the cash till and paid it into the bank account.
We repaid F Lake 1,000 by cheque.
Cash sales paid direct into the bank 660.
Paid motor expenses by cheque 120.
Withdrew 1,000 cash from the bank for business use.
Paid wages in cash 970.

Cash Book
Capital
F Lake

Cash
1000

Bank
5000

Cash

Bank

Lesson Two

(loan)
Sales
N Miller
Sales
G Moores
Cash C
Sales
Bank C

Capital
F. Lake
(Loan)
Sales
N Miller
Sales
G Moores

71

980
620
530
650

Cash
1000

650

Cash C
Sales
Bank C
363
0

Cash Book
Bank
Rent
5000 B McKenzie
980 B Burton
620 Bank C
530 F Lake
(loan)
Motor
Expenses
500 Cash C
660 Wages
1000 Balances c/d
7310

Cash
100

Bank
650

220
500
1000
120

100

970
1840
3630

4540
7310

Example 2.7(Frankwood adapted)


A three-column cashbook is to be written up from the following details, balanced
off, and the relevant discount accounts in the general ledger shown.
19x8
Mar 1
Balances brought forward: Cash 230; Bank 4,756.

2
The following paid their accounts by cheque, in each case deducting 5
percent
discounts: R Burton 140; E Taylor 220; R Harris 800.

4
Paid rent by cheque 120.

6
J Cotton lent us 1,000 paying by cheque.

8
We paid the following accounts by cheque in each case deducting a 2
per
cent cash discount: N Black 360; P Towers 480; C Rowse
300.

10
Paid motor expenses in cash 44.

72

Final Accounts

12

15
18

21
24
25

29
31

H Hankins pays his account of 77, by cheque 74, deducting 3 cash


discount.
Paid wages in cash 160.
The following paid their accounts by cheque, in each case deducting 5
per cent
cash discount: C Winston 260; R Wilson & Son 340; H
Winter 460.
Cash withdrawn from the bank 350 for business use.
Cash Drawings 120.
Paid T Briers his account of 140, by cash 133, having deducted 7
cash discount.
Bought fixtures paying by cheque 650.
Received commission by cheque 88.

Lesson Two

73

Answer
Disct
Bank
Bal b/d
R Burton
E Taylor
R Harris
J Cotton:
loan
H Hankins
C Winston
R Wison &
Son
H Winter
Bank
Commission

Cash
230

7
11
15
3
13
17
23
350
89

580

Cash Book
Bank

Disct

4756
133
209
285
1000

Rent
N Black
P Towers
C Rowse
Motor
expenses
74 Wages
247 Cash
323 Drawings

3/1

Sundry
Debtors

Bank
120
351
468
780

9
12
20
44
160

350
120

437 T Briers
Fixtures
88 Balances c/d
7552

Discounts Received
3/1 Sundry
Creditors

Cash

133

48

123
580

650
4833
7552

48

Discounts Allowed
89

Petty Cash Book and the imprest system of Accounting.


Petty Cash Book is a record of all the petty cash vouchers raised and kept by the
cashier. The petty cash vouchers will show summary expenses paid by the cashier
and this information is listed and classified in the petty cash book under the
headings of the relevant expenses such as:

Postage and stationery


Traveling
Cleaning expenses.

The format is as shown:


Petty Cash Book
Receipts

Date Detail Payments

Expenses

The

74

Final Accounts

Amount

Postage Stationery

Traveling

Ledger

The balance c/d of the petty cash book will signify the balance of cash in hand or
form part of cash in hand. The totals of the expenses are posted to the debit side
of the expense accounts. If a firm operates another cashbook in addition to the
petty cash book, then the totals of the expenses will also be posted on the credit
side of the cash in hand cashbook.
The Imprest system
This system of accounting operates on a simple principle that the cashier is
refunded the exact amount spent on the expenses during a particular financial
period. At the beginning of each period, a cash float is agreed upon and the
cashier is given this amount to start with. Once the cashier makes payments for
the period he will get a total of all the payments made against which he will claim
a reimbursement of the same amount that will bring back the amount to the cash
float at the beginning of the period.
This is demonstrated as follows:

Start with (float)


Expenses paid
Balance
Reimbursement
Cash float

1,000
(720)
280
720
1,000

Example 2.8
A cashier in a firm starts with 2,000 in the month of March (that is the cash float).
I n the following week, the following payments are made:
1st March bought stamps for
2nd March paid bus fare for
2nd March cleaning materials
3rd March bought fuel
3rd March cleaning wages
4th March bought stamps
4th March paid L. Thompson (creditor)

80
120
240
150
300
200
400

Lesson Two

5th March fuel costs


150
On the 5th of March the cashier requested for a refund of the cash spent and
this amount was reimbursed back.
Required:
Prepare a detailed petty cash book showing the balance to be carried forward to
the next period and the relevant expense accounts, as they would appear on the
General Ledger.

75

76

Final Accounts

Answer
Receipt
s

Date

()
2000 1/3
1/3
2/3
2/3
3/3
3/3
4/3
4/3
5/3
1640 5/3
5/3
3640
2000 6/3

Detail

Payment
s
Amount
()

Bal b/d
Stamps
Bus Fare
Cleaning
Materials
Fuel
Cleaning
wages
Stamps
L Thompson
Fuel
Bal c/d

Expenses
Postag
e ()

80
120
240

Cleani
ng ()

THE
LEDGE
R
Travel
()

()

80
120
240

150
300

150
300

200
400
150
1640

200
.
280

.
540

150
420

400
.
400

2000
3640

Bal b/d

The General Journal


It records information from other correspondence (information that is not recorded
in the above books of prime entry). It explains the type of entries that will be made
in the ledger accounts giving a reason for these entries.
The type of transactions recorded here are:
i. Writing off of assets from the accounts e.g. bad-debts.
ii. Drawings for goods or other assets from the business by the owner, not
cash drawings.
iii. Purchase or sale of non-current assets on credit.
The format is as shown:
The General Journal
GENERAL JOURNAL
Date
1/3

Detail
Account to be debited
Account to be credited
(Narrative)

Debit

Credit
x
x

Lesson Two

77

78

Final Accounts

Example 2.9
You are to show the journal entries necessary to record the following items:

2003 May 1 Bought a motor vehicle on credit from Motors Ltd for 6,790.
2003 May 3 A debt of 34 owing from N Smart was written off as a bad debt.
2003 May 8 Furniture bought by us for 490 was returned to the supplier
Wood
Offices, as it was unsuitable. Full allowance will be given us.
2003 May 12 we are owed 150 by W Hayes. He is declared bankrupt and
we received
39 in full settlement of the debt.
2003 May 14 we take 45 goods out of the business stock without paying for
them.
2003 May 28 Some time ago we paid an insurance bill thinking that it was
all in respect
of the business. We now discover that 76 of the amount paid was in fact
insurance of
our private house.
2003 May 28 Bought Machinery 980 on credit from Xerox Machines Ltd.

Lesson Two

79

a. Answer
GENERAL JOURNAL
Date (19x5)
1/5

Detail
Debit ()
Credit ()
Motor Vehicle
6,790
Motors Ltd
6,790
Motor vehicle bought on credit
from Motors Ltd
_________________________________________________________________________
3/5
Bad debts
34
N Smart - Debtors
34
Amount due from N Smart
written off as bad
_________________________________________________________________________
8/5

Wood offices
490
Furniture
490
Office Furniture returned to
Wood offices
_________________________________________________________________________
12/5
Bad debts
111
W. Hayes
111
Amount owed now written off
as bad debt.
________________________________________________________________________
14/5
Drawing for goods
45
Purchases
45
Goods taken from the
business for personal use.
_________________________________________________________________________
8/5
Drawings
76
Insurance Expenses
76
Insurance relating to private house
now transferred to drawings
_________________________________________________________________________
28/5
Machinery
980
Xerox Machines
980
Machinery bought from
Xerox Machines

80

Final Accounts

THE LEDGER
The ledger is simply the accounts. The Ledger is classified into 3 main classes.
1. Sales Ledger, which has the accounts of all the debtors.
2. Purchases Ledger, which has the accounts of all the creditors.
3. The General Ledger. Has all the other accounts i.e. other assets, liability,
incomes and expenses and capital.
The ledger accounts can also be classified as follows:
LEDGER
ACCOUNTS

IMPERSONA
L
ACCOUNTS

PERSONAL
ACCOUNS

REAL
ACCOUNTS
DEBTORS
(for
goods)

NORMAL
a/cs

CREDITORS
(For goods)
Other
Non-current
Liabilities
assets
Other
Inventories/
Assets
Stocks

Income
Expenses

Lesson Two

Capital

81

82

Final Accounts

REINFORCING QUESTIONS
QUESTION ONE
Mr J Ockey commenced trading as a wholesaler stationer on 1 May 2000 with a
capital of 5,000.00 with which he opened a bank account for his business.
During May the following transactions took place.
May 1 Bought shop fittings and fixtures from store fitments Ltd for 2,000.00
May 2 Purchased goods on credit from Abel 650.00
May 4 Sold goods on credit to Bruce 700.00
May 9 Purchased goods on credit from Green 300.00
May 11
Sold goods on credit to Hill 580.00
May 13
Cash sales paid into bank account 200.00
May 16
Received cheque from Bruce in settlement of his account
May 17
Purchased goods on credit from Kay 800.00
May 18
Sold goods on credit to Nailor 360.00
May 19
Sent Cheque to Abel in settlement of his account
May 20
Paid rent by cheque 200.00
May 21
Paid delivery expenses by cheque 50.00
May 24
Received from Hill 200.00 on account
May 30
Drew cheque for personal expenses 200.00 and assistant wages
320.00
May 31
Settled the account of Green.
Required
a)
b)
c)
d)

Record the transactions in the books of prime entry.


Post the entries in the ledger accounts
Balance the ledger accounts where necessary
Extract a trial balance as at 31 May 2000.

Lesson Two

83

QUESTION TWO
The following trial balance has been drawn up from the accounts of Endpages
bookshop.
Endpages Bookshop
Trial balance as at 31 December 2002
Dr

Sales
Purchases
Salaries and wages
Office expenses
Insurance
Electricity
Stationery
Advertising
Telephone
Rates
Discount allowed
Discount received
Rent received
Returns inwards
Returns outwards
Stock at 01 Jan 2001
Premises
Stock as at 31 Dec
2001
Fixtures and fittings
Debtors and Creditors
Cash in Hand
Cash in bank
Capital
Drawings
Stock as at 31 Dec
2001

Cr

151,500.00

103,500.00
18,700.00
2,500.00
1,100.00
600.00
2,400.00
3,500.00
800.00
3,000.00
100.00
200.00
2,000.00
1,500.00
3,500.00
46,000.00
80,000.00
41,000.00
5,000.00
4,800.00
200.00

7,500.00
12,000.00
11,000.00

14,000.00
________
328,700.00

41,000.00
328,700.00

Required
Prepare a Trading and profit and loss account for the year ended 31 December
2002 and a balance sheet as at that date.
(20 marks)

84

Final Accounts

QUESTION THREE
The following is the trial balance of KSmooth as at 31 March 2002. Draw up a set
of final accounts for the year ended 31 March 2002.

Stock 1 April 2001


Sales
Purchases
Carriage inwards
Carriage outwards
Returns outwards
Wages and salaries
Rent and rates
Communication
expenses
Commissions payable
Insurance
Sundry expenses
Buildings
Debtors
Creditors
Fixtures
Cash at bank
Cash in hand
Drawings
Capital

Dr

1,816,000

Cr

9,234,000

6,918,500
42,000
157,000
64,000
1,024,000
301,500
62,400
21,600
40,500
31,800
2,000,000
1,432,000
816,000
285,000
297,000
11,500
762,000
5,088,800
152,028

152,028

Lesson Two

85

QUESTION FOUR
Skates drew up the following trial balance as at 30 September 2002. You are to
draft the trading and profit and loss account for the year to end 30 September
2002 and a balance sheet as at that date.
Dr
Cr

3,095,500
Capital
842,000
Drawings
311,500
Cash at bank
29,500
Cash in hand
1,230,000
Debtors
937,000
Creditors
2,391,000
Stock 30 September
410,000
2001
625,000
Motor van
1,309,000
Office equipment
9,210,000
Sales
55,000
Purchases
21,500
Returns inwards
30,700
Carriage inwards
30,900
Returns outwards
163,000
Carriage outwards
297,000
Motor expenses
40,500
Rent
1,281,000
Telephone charges
49,200
Wages and salaries
137,700
Insurance
Office expenses
28,400
Sundry expenses
17,153,200

17,153,200

CHECK YOUR ANSWERS WITH THOSE GIVEN IN LESSON 9 OF THE


STUDY PACK

86

Final Accounts

COMPREHENSIVE ASSIGNMENT No.1


TO BE SUBMITTED AFTER LESSON 2
To be carried out under examination conditions and sent to the Distance Learning
Administrator for marking by the University.
EXAMINATION PAPER.
ANSWER ALL QUESTIONS

TIME ALLOWED: THREE HOURS.

QUESTION ONE
The books of Mr T, a trader in tea showed the following balances as at 31 March
1998:
Opening stock of tea
Purchases Tea
Salaries paid
Buildings
Cash in hand
Cash at bank
Rent, rates and council taxes
Insurance premium paid
Miscellaneous receipts
Sales
Discounts allowed
Bad debts
Building repairs
Miscellaneous expenses
Advertisement
Commission to sales manager
Furniture and fittings
Air conditioners
Sundry debtors
Sundry creditors
Loan on mortgage
Interest paid on the above
Prepaid expenses
Drawings
Bills payable (Current liability)
Bank charges
Legal charges
Motor vehicles
Travelling and conveyance
Capital

Shs.
100,000
400,000
80,000
95,000
2,000
135,000
15,000
3,000
10,000
720,000
4,750
3,250
2,900
8,700
20,000
32,400
35,000
30,000
100,000
80,000
70,000
3,000
4,000
18,000
30,000
2,000
6,000
80,000
10,000
280,000

Lesson Two

87

The following further information was obtained :


1. Closing stock was Shs.55,000.
2. Legal charges include Shs.5,000 for the cost of stamps and registration of a
new building acquired during the year.
3. Purchases include 4000 kg tea valued at Shs.20,000, which was found
totally spoilt. An insurance claim of Shs.15,000 has been accepted by the
insurance company.
4. Travelling and conveyancing include proprietors personal travelling for
which he is charged Shs.4, 800.
5. The sales manager is entitled to commission of 7.5% of the total sales.
However any bad debts incurred during the year are deductible from such
commission entitlements.
6. Debtors include:
7. Shs.10, 000 due from M & C0 (Creditors include Shs.18, 000 due to the
same party).
8. Shs.5, 000 due from the sale of furniture.
9. Further bad debts of Shs.2, 000
10.Provision for bad debts is to be created at 2% of net amount outstanding
from trade debtors.
11.Depreciation is chargeable as follows:
Buildings
2.5%
Furniture and Fittings
10%
Air conditioners
15%
Motor vehicles
20%
12.Miscellaneous receipts represent sales proceeds of furniture, whose written
down value was Shs.12, 000.
13.Prepaid expenses include insurance premiums for the next year.
Required:
Prepare a trading, profit and loss account for the year ended 31 st March 1998 and
a Balance Sheet as at that date.
QUESTION TWO
Hall Ltd., which makes up its accounts to 30th June each year, has a fleet of motor
lorries. Annual depreciation on motor lorries is calculated at a rate of 25% on the
reducing balances, with a full years depreciation being made in the year of
purchase, but no charge in the year of sale. An extract from the companys
balance sheet as on 30th June 1997 showed the following:
Shs
Motor Lorries at cost:
Less provision for depreciation:
Net book Value:

164,900
93,382
71,518

During the year ended 30th June 1998 purchases and sales of lorries were as
follows:

88

Final Accounts

Purchases:
1997
July 30th
Oct 1st
1998
Feb 25th
June 24th
Sales:

Reg.No
H11
7,000
H13

9,000

H14

5,900
Purchased on:

1,592

10th July 1994

H4

Cost (Shs)

14th May 1993

H1

2,560

9th March 1996

H6
H7

8,500

H12

Reg.No
Proceeds (Shs)

1997
July 30th
300
Oct 1st
1998
Mar 1st
4,600
June 25th

Cost (Shs)

850
8,000

21st Sept 1996

3,648

2,700

Required:
Write up the following accounts in the books of the company for the year ended
30th June 1998:
a) The Motor lories account
b) Motor lorries provision for depreciation account
c) Motor lorries disposal account.
QUESTION THREE
The following trial balance was extracted form the books of Rodney, a sole trader,
at 31st December 1997:
Drawings/Capital
Debtors/Creditors
Purchases/Sales
Rent and Rates
Light and heat
Salaries and wages
Bad debts
Provision for bad debts
Stock in trade 31st Dec 1996
Insurance
General Expenses

Shs
2,148
7,689
62,10
1
880
246
8,268
247

Shs.
20,27
1
5,462
81,74
2

326
9,274
172
933

Lesson Two

89

Bank balances
Motor van at cost/Provision for
depreciation
Proceeds on sale of van
Motor expenses
Freehold premises at cost

1,582
8,000

3,600
0
250

861
15,00
0

Rent received
Provision for depreciation on
buildings

750
5,000
117,4
01

117,4
01

The following matters are to be taken in to account:


Stock in trade at 31st December 1997 was Shs.9,884
Rates paid in advance at 31st December 1997, Shs.40
Rent receivable due at 31st December 1997, Shs.250
Lighting and heating due at 31st December 1997, sh.85
Provision for doubtful debts to be increased to Shs.388
Included in the amount for insurance Shs.172, is an item for Shs82 for motor
insurance and this amount should be transferred to motor expenses.
7. Depreciation has been and is to be charged on vans at an annual rate of 20%
on cost.
8. Depreciate buildings Shs.500
9. On 1st January 1997 a van which had been purchased for Shs.1,000 on 1 st
January 1994 was sold for Shs250. The only record of matter is the credit of
Shs.250 to Proceeds of sale on van account.
1.
2.
3.
4.
5.
6.

Required:
A Trading Profit and Loss account for the year ended 31 st December 1997 and a
Balance Sheet as at date using vertical format.
QUESTION FOUR
The Batley Print Shop rents a photocopying machine from a suppler for which it
makes quarterly payments as follows:
Three moths rental in advance;
A further charge of 2 pence per copy made during the quarter just ended.
The rental agreement began on 1st August 19X4, and the first six quarterly bills
were as follows
Bills and dates received
(Shs)
1 August 19X4
1 November 19X4

Rental (Shs) Cost of copies (shs)


2,100
2,100

0
1,500

Total cost
2,100

3,600

90

1
1
1
1

Final Accounts

February 19X5
May 19X5
August 19X5
November 19X5

2,100
2,100

1,400
1,800

2,700
2,700

3,500
3,900

1,650
1,950

4,350
4,650

Required:
Given that Batley Printing shop ends its accounting year on 31 August,
Calculate the charge for photocopying expenses for the year to 31 August, 19X5
and the amount of prepayments and / or accrued charges as at that date.
Show the entries in the ledger of the Batley Printing Shop.
QUESTION FIVE
The historical cost convention looks backwards but the going concern convention
looks forwards.
Required:
a) Explain clearly what is meant by:

The historical cost convention


The going concern convention.

b) Does traditional financial accounting, using the historical cost convention,


make the going concern convention unnecessary? Explain your answer fully.
c) Which do you think a shareholder is likely to find more useful a report on
the past or an estimate of the future? Why?
END OF COMPREHENSIVE ASSIGNMENT No.1
NOW SEND TO THE DISTANCE LEARNING CENTRE FOR MARKING

Acknowledgement

91

LESSON THREE
ACCOUNTING THEORY
(a) International Accounting Standards and International Financial
Reporting Standards.
The foreword to accounting standards defines Accounting Standards as
Authoritative statements of how particular types of transaction and other events
should be reflected in financial statements. Accounting Standards are developed
to achieve comparability of financial information between and among different
organizations. International Accounting Standards (IASs) and International
Financial Reporting Standards (IFRS) are meant to apply to most organizations in
the world. IASs and IFRSs are produced by the International Accounting
Standards Board (IASB) whose objectives are:
(a) To formulate and publish in the public interest accounting standards to be
observed in the presentation of financial statements and to promote their
worldwide acceptance; and
(b) To work generally for the improvement and harmonization of regulations,
accounting standards and procedures relating to the presentation of
financial statements.
The IASB is an affiliate of the International Federation of Accountants (IFAC)
established in 1977 which co-ordinates the Accounting profession worldwide.
Most accounting bodies of countries are members of IFAC.
The IASC develops IASs through an international process that involves the
worldwide accountancy profession, the preparers, users of financial statements
and national standard setting bodies and other interested parties.
The IASB sets up a steering committee to develop a statement of principles, an
Exposure Draft and ultimately an Accounting Standards once a new topic is
suggested. The process includes:
Identifying and reviewing of all the issues associated with the topic,
Studying national and regional accounting requirements and practice,
consultation with the member bodies standard setting bodies and other
interested groups,
Public Exposure of the draft Accounting Standard,
Evaluation by the steering committee and the board of the comments
received on exposure drafts.
Currently the IASB has developed about 40 IASs. Examples include:

FINANCIAL ACCOUNTING 1

92

Accounting Theory

IAS 1 Presentation of Financial Statements


IAS 2 Inventories
IAS 16 Property plant and equipment.

Previously new standards were called International Accounting Standards but from
2003 any new standards will be called International financial Reporting Standards.
However in the current practice is to refer to all standards as International
Financial Reporting Standard.
In Kenya, Accountants used to prepare the financial statements in accordance with
Kenya Accounting Standards (IASs), which were developed and published by
ICPAK (Institute of Certified Public Accountants of Kenya). This were later dropped
and International Accounting Standards adopted.
Reasons why Accountants should observe International Accounting Standards:
a) Use of IASs adds credibility to the financial statements as they can be
compared with others globally.
b) Facilitates communication within an enterprise that has foreign branches
or subsidiaries due to harmonized reporting by the separate entities in the
group.
c) Adds value to the financial statements incase an entity is sourcing for
foreign capital.
d) Incase an entity wishes to be quoted on the Stock Exchange Market more
so for companies.
(c)Accounting Concepts Bases and Policies
I) Concepts/conventions/principles
Accounting Concepts are broad basic assumptions that underlie the periodic
financial accounts of business enterprises. Examples of concepts include:
i)

ii)

The going concern concept: implies that the business will continue in
operational existence for the foreseeable future, and that there is no intention
to put the company into liquidation or to make drastic cutbacks to the scale of
operations.
Financial statements should be prepared under the going concern basis unless
the entity is being (or is going to be) liquidated or if it has ceased (or is about
to cease) trading. The directors of a company must also disclose any
significant doubts about the companys future if and when they arise.
The main significance of the going concern concept is that the assets of the
business should not be valued at their break-up value, which is the amount
that they would sell for it they were sold off piecemeal and the business were
thus broken up.
The accruals concept (or matching concept): states that revenue and
costs must be recognized as they are earned or incurred, not as money is

Lesson Three

93

received or paid. They must be matched with one another so far as their
relationship can be established or justifiably assumed, and dealt with in the
profit and loss account of the period to which they relate.
Assume that a firm makes a profit of 100 by matching the revenue (200)
earned from the sale of 20 units against the cost (100) of acquiring them.
If, however, the firm had only sold eighteen units, it would have been incorrect
to charge profit and loss account with the cost of twenty units; there is still two
units in stock. If the firm intends to sell them later, it is likely to make a profit
on the sale. Therefore, only the purchase cost of eighteen units (90) should
be matched with the sales revenue, leaving a profit of 90.

94

Accounting Theory

The balance sheet would therefore look like this:

Assets
Stock (at cost, i.e. 2 x 5)
Debtors (18 x 10)

10
180
190

Liabilities
Creditors

100
90

Capital (profit for the period)

90

If, however the firm had decided to give up selling units, then the going concern
concept would no longer apply and the value of the two units in the balance
sheet would be a break-up valuation rather than cost. Similarly, if the two
unsold units were now unlikely to be sold at more than their cost of 5 each
(say, because of damage or a fall in demand) then they should be recorded on
the balance sheet at their net realizable value (i.e. the likely eventual sales
price less any expenses incurred to make them saleable, e.g. paint) rather than
cost. This shows the application of the prudence concept. (See below).
In this example, the concepts of going concern and matching are linked.
Because the business is assumed to be a going concern it is possible to carry
forward the cost of the unsold units as a charge against profits of the next
period.
Essentially, the accruals concept states that, in computing profit, revenue
earned must be matched against the expenditure incurred in earning it.
iii)

The Prudence Concept: The prudence concept states that where


alternative procedures, or alternative valuations, are possible, the one selected
should be the one that gives the most cautious presentation of the businesss
financial position or results.
Therefore, revenue and profits are not anticipated but are recognized by
inclusion in the
profit and loss account only when realized in the form of
either cash or of other assets the ultimate cash realization of which can be
assessed with reasonable certainty: provision is made for all liabilities

Lesson Three

95

(expenses and losses) whether the amount of these is known with certainty or
is best estimate in the light of the information available.
Assets and profits should not be overstated, but a balance must be achieved to
prevent the material overstatement of liabilities or losses.
The other aspect of the prudence concept is that where a loss is foreseen, it
should be anticipated and taken into account immediately. If a business
purchases stock for 1,200 but because of a sudden slump in the market only
900 is likely to be realized when the stock is sold the prudence concept
dictates that the stock should be valued at 900. It is not enough to wait until
the stock is sold, and then recognize the 300 loss; it must be recognized as
soon as it is foreseen.
A profit can be considered to be a realized profit when it is in the form of:

Cash
Another asset that has a reasonably certain cash value. This includes
amounts owing from debtors, provided that there is a reasonable certainty
that the debtors will eventually pay up what they owe.

A company begins trading on 1 January 20X2 and sells goods worth 100,000
during the year to 31 December. At 31 December there are debts outstanding
of 15,000. Of these, the company is now doubtful whether 6,000 will ever be
paid.
The company should make a provision for doubtful debts of 6,000. Sales for
20x5 will be shown in the profit and loss account at their full value of 100,000,
but the provision for doubtful debts would be a charge of 6,000. Because
there is some uncertainty that the sales will be realized in the form of cash, the
prudence concept dictates that the 6,000 should not be included in the profit
for the year.
iv)

The consistency concept: The consistency concept states that in preparing


accounts consistency should be observed in two respects.
a) Similar items within a single set of accounts should be given similar
accounting treatment.
b) The same treatment should be applied from one period to another in
accounting for similar items. This enables valid comparisons to be made
from one period to the next.

v) The entity concept: The concept is that accountants regard a business as a


separate entity, distinct from its owners or managers. The concept applies
whether the business is a limited company (and so recognized in law as a
separate entity) or a sole proprietorship or partnership (in which case the
business is not separately recognized by the law.
vi) The money measurement concept: The money measurement concept states
that accounts will only deal with those items to which a monetary value can be
attributed.
For example, in the balance sheet of a business, monetary values can be
attributed to such assets as machinery (e.g. the original cost of the machinery;

96

Accounting Theory

or the amount it would cost to replace the machinery) and stocks of goods (e.g.
the original cost of goods, or, theoretically, the price at which the goods are
likely to be sold).
The monetary measurement concept introduces limitations to the subject
matter of accounts. A business may have intangible assets such as the flair of a
good manager or the loyalty of its workforce. These may be important enough
to give it a clear superiority over an otherwise identical business, but because
they cannot be evaluated in monetary terms they do not appear anywhere in the
accounts.

Lesson Three

vii)

97

The separate valuation principle: The separate valuation principle states


that, in determining the amount to be attributed to an asset or liability in the
balance sheet, each component item of the asset or liability must be
determined separately.
These separate valuations must then be aggregated to arrive at the balance
sheet figure. For example, if a companys stock comprises 50 separate items, a
valuation must (in theory) be arrived at for each item separately; the 50 figures
must then be aggregated and the total is the stock figure which should appear
in the balance sheet.

The materiality concept: An item is considered material if its omission or


misstatement will affect the decision making process of the users. Materiality
depends on the nature and size of the item. Only items material in amount or in
their nature will affect the true and fair view given by a set of accounts.
An error that is too trivial to affect anyones understanding of the accounts is
referred to as immaterial. In preparing accounts it is important to assess
what is material and what is not, so that time and money are not wasted in the
pursuit of excessive detail.
Determining whether or not an item is material is a very subjective exercise.
There is no absolute measure of materiality. It is common to apply a
convenient rule of thumb (for example to define material items as those with a
value greater than 5% of the net profit disclosed by the accounts). But some
items disclosed in accounts are regarded as particularly sensitive and even a
very small misstatement of such an item would be regarded as a material
error. An example in the accounts of a limited company might be the amount
of remuneration paid to directors of the company.
The assessment of an item as material or immaterial may affect its treatment
in the accounts. For example, the profit and loss account of a business will
show the expenses incurred by he business grouped under suitable captions
(heating and lighting expenses, rent and rates expenses etc); but in the case
of very small expenses it may be appropriate to lump them together under a
caption such as sundry expenses, because a more detailed breakdown would
be inappropriate for such immaterial amounts.
Example:

viii)

a) If a balance sheet shows fixed assets of 2 million and stocks of 30,000 an


error of 20,000 in the depreciation calculations might not be regarded as
material, whereas an error of 20,000 in the stock valuation probably would
be. In other words, the total of which the erroneous item forms part must be
considered.
b) If a business has a bank loan of 50,000 balance and a 55,000 balance on
bank deposit account, it might well be regarded as a material misstatement if
these two amounts were displayed on the balance sheet as cash at bank
5,000. In other words, incorrect presentation may amount to material
misstatement even if there is no monetary error.
ix)
The historical cost convention: A basic principle of accounting (some
writers include it in the list of fundamental accounting concepts) is that
resources are normally stated in accounts at historical cost, i.e. at the amount

98

Accounting Theory

that the business paid to acquire them. An important advantage of this


procedure is that the objectivity of accounts is maximized: there is usually
objective, documentary evidence to prove the amount paid to purchase an
asset or pay an expense. Historical cost means transactions are recorded at the
cost when they occurred.
In general, accountants prefer to deal with costs, rather than with values. This is
because valuations tend to be subjective and to vary according to what the
valuation is for. For example, suppose that a company acquires a machine to
manufacture its products. The machine has an expected useful life of four years.
At the end of two years the company is preparing a balance sheet and has decided
what monetary amount to attribute to the asset.
x)

Objectivity (neutrality):An accountant must show objectivity in his work.


This means he should try to strip his answers of any personal opinion or
prejudice and should be as precise and as detailed as the situation warrants.
The result of this should be that any number of accountants will give the same
answer independently of each other. Objectivity means that accountants must
be free from bias. They must adopt a neutral stance when analysing
accounting data. In practice objectivity is difficult. Two accountants faced with
the same accounting data may come to different conclusions as to the correct
treatment. It was to combat subjectivity that accounting standards were
developed.

xi)

The realization concept: Realization: Revenue and profits are recognized


when realized. The concept states that revenue and profits are not anticipated
but are recognized by inclusion in the income statement only when realized in
the form of either cash or of other assets the ultimate cash realization of which
can be assessed with reasonable certainty.

xii)
xiii)

Duality: Every transaction has two-fold effect in the accounts and is the
basis of double entry bookkeeping.
Substance over form: The principle that transactions and other events are
accounted for and presented in accordance with their substance and economic
reality and not merely their legal form e.g. a non current asset on Hire
purchase although is not legally owned by the enterprise until it is fully paid
for, it is reflected in the accounts as an asset and depreciation provided for in
the normal accounting way.
Example 3.1
It is generally agreed that sales revenue should only be realized and so
recognized in the trading, profit and loss account when:
a) The sale transaction is for a specific quantity of goods at a known price,
so that the sales value of the transaction is known for certain.
b) The sale transaction has been completed, or else it is certain that it will
be completed (e.g. in the case of long-term contract work, when the job is

Lesson Three

c)

99

well under way but not yet completed by the end of an accounting
period).
The critical event in the sale transaction has occurred. The critical event
is the event after which:
i) It becomes virtually certain that cash will eventually be received
from the customer.
ii) Cash is actually received.

100

Accounting Theory

Usually, revenue is recognized


(a) When a cash sale is made.
(b) The customer promises to pay on or before a specified future date, and the
debt is legally enforceable.
The prudence concept is applied here in the sense that revenue should not be
anticipated, and included in the trading, profit and loss account, before it is
reasonably certain to happen.
Required
Given that prudence is the main consideration, discuss under what circumstances,
if any, revenue might be recognized at the following stages of a sale.
(a) Goods have been acquired by the business, which it confidently expects to
resell very quickly.
(b) A customer places a firm order for goods.
(c) Goods are delivered to the customer.
(d) The customer is invoiced for goods.
(e) The customer pays for the goods.
(f) The customers cheque in payment for the goods has been cleared by the bank.
Answer
(a) A sale must never be recognized before a customer has even ordered the
goods. There is no certainty about the value of the sale, nor when it will take
place, even if it is virtually certain that goods will be sold.
(b) A sale must never be recognized when the customer places an order. Even
though the order will be for a specific quantity of goods at a specific price, it is
not yet certain that the sale transaction will go through. The customer may
cancel an order, the supplier might be unable to deliver the goods as ordered
or it may be decided that the customer is not a good credit risk.
(c) A sale will be recognized when delivery of the goods is made only when:
i) The sale is for cash, and so the cash is received at the same time.
ii) The sale is on credit and the customer accepts delivery (e.g. by signing a
delivery note).
(d) The critical event for a credit sale is usually the dispatch of an invoice to the
customer. There is then a legally enforceable debt payable on specified terms,
for a completed sale transaction.
(e) The critical event for a cash sale is when delivery takes place and when cash is
received, both take place at the same time. It would be too cautious or
prudent to await cash payment for a credit sale transaction before
recognizing the sale, unless the customer is a high credit risk and there is a
serious doubt about his ability or intention to pay.
(f) It would again be over-cautious to wait for clearance of the customers cheques
before recognizing sales revenue. Such a precaution would only be justified in

Lesson Three

cases where there is a very high risk of the bank refusing to honour the
cheque.

101

102

Accounting Theory

II) Bases
Bases are the methods that have been developed for expressing or applying
fundamental accounting concepts to financial transactions and items. Examples
include:

Depreciation of Non current Assets (e.g. by straight line or reducing balance


method)
Treatment and amortization of intangible assets (patents and trade marks)
Stocks and work in progress (FIFO, LIFO and AVCO)
III)
Policies
Accounting policies are the specific accounting bases judged by business
enterprises to be the most appropriate to their circumstances and adopted by them
for the purpose of preparing their financial accounts.
Qualities of Useful Financial Information
The four principal qualities of useful financial information are understandability,
relevance, reliability and comparability.
Understandability: an essential quality of the information provided in the
financial statements is that it is readily understandable by users. For these reason
users are assumed to have a reasonable knowledge of business and economic
activities and accounting.
Relevance: information has the quality of being relevant when it influences the
economic decisions of users by helping them evaluate past, present or future
events or confirming or correcting their past evaluations. The relevance of
information is affected by its nature and materiality.
Reliability: information is useful when it is free from material error and bias and
can be depended upon by users to represent faithfully that which it purports to
represent or could reasonably be expected to represent. To be reliable then the
information should:
a) Be represented faithfully,
b) Be accounted for and presented in accordance with their substance and
economic reality and not merely their legal form,
c) Be neutral i.e. free from bias,
d) Include some degree of caution especially where uncertainties surround some
events and transactions (prudence),
e) Be complete i.e. must be within the bounds of materiality and cost. An
omission can cause information to be false.
Comparability: users must be able to compare the financial statements of an
enterprise through time in order to identify trends in its financial position and
performance. Users must also be able to compare the financial statements of
different accounting policies, changes in the various policies and the effect of these

Lesson Three

103

changes in the accounts. Compliance with accounting standards also helps achieve
this comparability.
The Accounting Profession in Kenya
The Accountants Act Cap 531 (1977) establishes the Institute of Certified Public
Accountants of Kenya (ICPAK) and two boards, to be known as the Registration of
Kenya Accountants Board (RAB) and Kenya Accountants and Secretaries National
Examinations Board (IASNEB)

104

Accounting Theory

The following are the functions of ICPAK as outlined by the Act;


a) To promote standards of professional competence and practice amongst
members of the institute.
b) To promote research into the subjects of accountancy and finance, and
related matters, and the publication of books, periodicals, journals and
articles in connexion therewith;
c)

To promote the international recognition of the institute;

d) To advise the Examinations board on matters relating to examination


standards and policies;
e) To carry out any other functions prescribed for it under any of the
provisions of the Act or under any other written law; and
f)

To do anything incidental or conducive to the performance of any of the


preceding functions.

A council known as the Council of the institute governs the Institute, which
consists of the Chairman, nine members from the institute and one member
appointed by the Minister of finance.
The Registration of Accountants Board (RAB) functions include issuing out
practicing certificates and registration of qualified persons as members of the
institute.
The Act also outlines the following as the functions of IASNEB:
a) To prepare syllabuses for accountants and secretaries examinations, to
make rules with respect to examinations, to arrange and conduct
examinations and issue certificates to candidates who have satisfied
examination requirements;
b) To promote recognition of its examinations in foreign countries; and
c) To do anything incidental or conducive to the performance of any preceding
functions.
Example 3.2 PILOT PAPER OCTOBER 1991
Briefly explain the meaning and the significance of the following:
(i) Accounting concepts.
(ii) Accounting bases.
(iii)
Accounting policies.
(iv)
Accounting standards.
(Total: 20 Marks)
(Covered adequately in the text).

Lesson Three

105

Example 3.3 PILOT PAPER JULY 2000


(a) Define the following accounting concepts and for each explain their implication
in the preparation of financial statements.
(i)
(ii)
(iii)
(iv)

The Going concern concept


Business entity concept
Materiality
Realization

4 marks
4 marks
4 marks
4 marks

(b) Two accounting concepts or conventions could clash or there could be


inconsistency between them.
Give two examples of such situations and explain how the inconsistency should
be resolved.
4 marks
Solution:
(i) The Going Concern Concept
The concept of going concern is that an entity will continue trading into the
foreseeable future at a similar level as it does when the accounts are prepared.
Going concern has implications for the value of the entities assets and the way the
user may read the financial statements. If a business is to cease trading after the
period of account the financial statements should be prepared on a break up basis
as all liabilities will be due and assets will be valued at net realizable value.
(ii)
The Entity Concept
The organization preparing accounts is a distinct and separate entity. Financial
statements are prepared to reflect the activities of the entity. This concept
prevents any confusion between the owners private finances and those of the
entity, hence the option of drawings when a proprietor effectively reduces the
capital of the entity.
(iii) Materiality
Materiality relates to significant amounts and items in the financial statements. A
rough guide to what material amount is 5% of pre tax profits. However, this is only
a guide. If say, cash in hand is offset against the overdraft balance this is a
material misstatement.
Materiality prevents time being wasted on items which do not impact on the
results of the entity; it provides a focus on the significant items.
(iv) Realization
The realization concept involves recognizing amounts in the financial statements at
the point at which they crystallize. Profit should not be reflected in the profit and
loss account until it has been earned.
The realization concept means that the profit in the financial statements should be
reasonably stated.

106

Accounting Theory

(c) Clashes between accounting concepts


Accruals and prudence
The accruals concept requires future income (e.g. in relation to credit sales) to be
accrued. The prudence concept dictates that caution should be exercised, so that
if there is doubt about the subsequent receipt, no accrual should be made.

Consistency and prudence


If circumstances change, prudence may conflict with the consistency concept,
which requires the same treatment year after year.
In both situations, prudence must prevail.
Example 3.5 DECEMBER 1994 QUESTION FIVE
(a) Explain the nature of the Accounting Equation.
(5 marks)
(b) What are accounting standards and why are they important?
(5 marks)
(c) Describe the role of the Institute of Certified Public Accountants of Kenya.
(5 marks)
(d) In addition to the Kenya Accounting Standards, why is it important for an
Accountant to make use of International Accounting Standards?
(4
marks)
(Total: 19 marks)
(Covered adequately in the text)

Lesson Three

107

REINFORCEMENT QUESTIONS
Question One
Explain, with examples, each of the following terms:
Fundamental accounting concepts
Accounting bases
Accounting policies
Question Two
Accounting practice depends upon the guidance provided by a number of
accounting concepts, some of which are to be found in IAS 1 and/or in the
conceptional framework of the International Accounting Standards Committee.
Required:
(a)

Define and explain the relevance of the following accounting concepts.

(b)

Neutrality
Money measurement
Accruals
Substance over form
Consistency
(15 marks)

Give two examples of situations in which there is a clash or inconsistency


between two accounting concepts or conventions, and explain how the
inconsistency should be resolved. (In answering this part of the question, you
need not confine yourself to considering the concepts listed in part (a))
(5 marks)

marks)

(20

Question Three
If the information in financial statements is to be useful, regard must be had to the
following:

Materiality
Comparability
Prudence
Objectivity
Relevance

108

Accounting Theory

Required
Explain the meaning of each of these factors as they apply to financial accounting
including in your explanations one example of the application of each of them.
(Four marks for each of (a) to (e).)
(20 marks)

Lesson Three

109

Question Four
a)
b)

Explain what is meant by materiality in relation to financial statements and


state two factors affecting the assessment of materiality.
(4 marks)
Explain what makes information in financial statements relevant to users.
(5 marks)

c)
1. Two characteristics contributing to reliability are neutrality and prudence.
Explain the meaning of these two terms.
2. Explain how a possible conflict between them could arise and how that
conflict should be resolved.
(5 marks)
d)

One of the requirements of financial statements is that they should be free


from material error. Suggest three safeguards, which may exist, inside or
outside a company to ensure that the financial statements are free from
material error.
(6 marks)

CHECK YOUR ANSWERS WITH THOSE GIVEN IN LESSON 9 OF THE


STUDY PACK

Acknowledgement

110

LESSON FOUR
ADJUSTMENTS TO FINAL ACCOUNTS
a) ACCRUALS AND PREPAYMENTS
Revenue and costs must be recognized as they are earned or incurred, not as
money is received or paid. They must be matched with one another so far as
their relationship can be established or justifiably assumed, and dealt with in
the profit and loss account of the period to which they relate. Therefore all
incomes and expenses that relate to a particular financial period will be
matched together to determine the profit for the year.
ACCRUALS
Income:
Accrued Income
This is income that relates to the current year but cash has not yet been received.
An accrued income should be reported in the profit & loss account and the same
income will
be shown in the balance sheet as a current asset.
Example 4.1
A firm lets out part of its properties and receives rent of 2,000 per month,
assuming that this is the first year of renting and rent is received in arrears (rent 4
January is received early Feb).
The ledger accounts of the firm will be as follows:
Cashbook
Year 1
Feb (rent 4 Jan)
Mar (rent 4 Feb)
April (rent 4 Mar)
May (rent 4 Apr)
June (rent 4 May)
July (rent 4 Jun)
Aug (rent 4 July)
Sept (rent 4 Aug)
Oct (rent 4 Sept)
Nov (rent 4 Oct)
Dec (rent 4 Nov)

2,000
2,000
2,000
2,000
2,000
2,000
2,000
2,000
2,000
2,000
2,000
22,000

FINANCIAL ACCOUNTING 1

Lesson Four

111

Year 1

31/12 P&L 24,000

Rent Income
Year 1
Jan
C/B
Feb C/B
Mar C/B
April C/B
May C/B
Jun
C/B
July C/B
Aug C/B
Sept C/B
Oct C/B
Nov C/B
Dec Accrued c/f

24,000

2,000
2,000
2,000
2,000
2,000
2,000
2,000
2,000
2,000
2,000
2,000
2,000
24,000

Although the cashbook is showing that rent received amounts 22,000, the full
rental income of 24,000 will be reported in the Profit & Loss a/c as rent income
and the accrued rent for Dec of 2,000 will be reported in the balance sheet as a
current asset.
Expenses: Accrued Expenses
An accrued expense is an expense that is payable or due for payment but has not
yet been paid during that period.
An accrued expense should be charged in the P&L account and shown in the
balance sheet as a current liability.
Assume in the above example that the firm is meant to pay the rent, thus it
becomes an expense with the facts still the same i.e. 2,000 payable in arrears.
The ledger account will be as follows.
Year 1

Cashbook
Year 1
Feb (rent 4 Jan)
Mar (rent 4 Feb)
Apr (rent 4 Mar)
May (rent 4 Apr)
June (rent 4 May)
July (rent 4 June)
Aug (rent 4 July)
Sept (rent 4 Aug)
Oct (rent 4 Sept)
Nov (rent 4 Oct)
Dec (rent 4 Nov)

2,000
2,000
2,000
2,000
2,000
2,000
2,000
2,000
2,000
2,000
2,000

112

Adjustment to Final Accounts

Rent Expenses
Year 1
C/B Rent for
Rent for
Rent for
Rent for
Rent for
Rent for
Rent for
Rent for
Rent for
Rent for
Rent for
31/12 Bal c/d

Jan
Feb
Mar
Apr
May
June
July
Aug
Sept
Oct.
Nov

2,000
2,000

Year 1

2,000
2,000
2,000
2,000
2,000
2,000
2,000
2,000
2,000
2,000

31/12

P&L

24,000

24,000

24,000

The cashbook shows that the rent for the 11 months was paid for. However in
the P&L a/c we should report rent for the full year of 24,000 and the 2,000, rent
for Dec being the accrued expense will be shown in the balance sheet as a current
liability.
PREPAYMENTS
Prepaid Income
This is income that is not yet due but cash has been received for it. This happens
where an income is payable in advance e.g. Rent payable 3 months in advance.
A prepaid income should not be reported in the current financial period but
should be carried forward and reported in the period it relates to.
The accounting treatment will be to show it as a current liability.
Example 4.2
A firm receives rent income of 5,000 per month payable quarterly in advance.
Assuming that the firms rental income began in 1 st March and the financial year,
end is on 31st Dec. The ledger accounts will be:
15,000

15,000

15,000

1.3
1.3

15,000

1.6

15,000

1.9

1.12

Lesson Four

113

Cashbook
Year 1
1/3
Rent
1/6
Rent
1/9
Rent
1/12 Rent

Year 1

15,000
15,000
15,000
15,000
Rent Income

Year 1

Year 1

1/3
Cashbook
15,000
1/6
Cashbook
15,000
P&L (10 x 5,000) 50,000
1/9
Cashbook
15,000
31/12
Bal c/d
10,000
1/12
Cashbook
60,000
60,000

15,000

Rent for the 4 quarters of 12 months has been received as per the cashbook but
because the end of the financial year is at 31 Dec, rent for 2 months is pre-paid.
This 10,000 is not charged in the P&L but is carried forward as current liability in
the balance sheet.
Prepaid Expenses
A prepaid expense is an expense that is not payable but cash has already been
paid. A prepaid expense should not be charged in the P&L a/c but should be
carried forward to the next financial period and should be shown in the balance
sheet as a current asset.
Example
Assume as in the previous illustration, that all the facts are as stated except that
rent is an expense. The ledger accounts is as follows:
Year 1

Cashbook
Year 1
1/3
Rent
1/6
Rent
1/9
Rent
1/12
Rent

15,000
15,000
15,000
15,000

Rent Expenses
Year 1
1/3
C/B (Mar, April, May)
1/6
C/B (June, July, Aug)

Year 1
15,000
15,000

114

1/9
1/12

Adjustment to Final Accounts

C/B (Sept, Oct, Nov)


15,000
C/B (Dec, Jan, Feb)
15,000
60,000

P&L
(10 x 5,000) 50,000
31/12 Bal c/d (2 x 5,000) 10,000
60,000

Rent of 10,000 for 2 months is carried forward to the next financial period and
shown in the balance sheet as a current asset.

Lesson Four

115

The following is the summary of treatment for Accruals and Prepayments:


P&L
B/Sheet
Accrued - Report as
Current
Income
Assets
Income
Prepaid -Not
reported
Current
Liability
Accruals/
Prepayments
Accrued

- Charge as

Current
an expense
Liability
Expense
Prepaid - Not charged
Current
In P& L

Assets

Accrued Incomes and Expenses and Prepaid Incomes and Expenses are shown in
the Balance Sheet as follows:
Balance Sheet Extracts
Current Assets

Stock
Debtors
Accrued Incomes/Prepaid Expenses
Cash at bank
Cash in hand

x
x
x
x
x
x

Current Liabilities
Bank overdraft
x
Creditors
x
Prepaid Incomes/Accrued Expenses
x
X

116

Adjustment to Final Accounts

The accruals and expenses items may also be adjusted in the relevant income and
expense accounts so that the correct amount of expense or income is reported in
the profit and loss account for the year.

Example 4.4
The financial year of H Seamers ended on 31 December 2002. Show the ledger
accounts for the following items including the balance transferred to the necessary
part of the final accounts, also the balances carried down to 2003:
a) Motor expenses: Paid in 2002 7,440; Owing at 31 December 2002
2,800.
b) Insurance: Paid in 2002 42,000; Prepaid as at 31 December 2002
3,500.
c) Stationery: Paid during 2002 18,000; Owing as at 31 December 2001
25,000; Owing as at 31 December 2002 49,000.
d) Rates: Paid during 2002 95,000; Prepaid as at 31 December 2001
2,200; Prepaid as at 31December 2002 2,900.
e) Seamers sub-lets part of the premises. Receives 5,500 during the
year ended 31 December 2002. Tenant owed Seamers 1,800 on 31
December 2001 and 2,100 on 31 December 2002
a)
19X6
31/12

Cashbook
Bal c/d

b)

7220
7200
280

Insurance
19x6

Cashbook
19x7
1/1

c)

Motor Expenses
19X6
7,440
280 P/L a\c
7200
19x7
1/1 Bal b/d

19x6

4,200
31/12
31/12
4,200
Bal b/d

350
===
Stationery

P&L a/c
Bal c/d

3850
350
4200

Lesson Four

19x6
Cashbook
31/12 Bal c/d
20,400

117

19x6
18,000 1/1
Bal b/d
4,900
22,900
====

2,500
P&L a/c
22,900
====

19x7
1/1Bal b/d

4,900

118

Adjustment to Final Accounts

d)

Rates
19x6
1/1
Bal b/d
Cashbook

19x6
2200
P&L
9500 31/12 Bal c/d
11,700

8800
2900

11,700
19x7
1/1
Bal b/d
e)

2900
Rent Income

19x6
1/1
Bal b/d
P&L
19x7
1/1
Bal b/d

19x6
1800
Cashbook
5800
31/12 Bal c/d
7600

5500
2100
7600

2100

b) BAD AND DOUBTFUL DEBTS


Some debtors may not pay up their accounts for various reasons e.g. a debtor may
go out of business. When a debtor is not able to pay up his/her account this
becomes a bad debt. Therefore the business/firm should write it off from the
accounts and thus it becomes an expense that should be charged in the profit &
loss account.
In practice a firm may also be unable to collect all the amounts due from debtors.
This is because a section of the debtors will not honor their obligations. The
problem posed by this situation is that it is difficult to identify the debtors who are
unlikely to pay their accounts. Furthermore the amount that will not be collected
may also be difficult to ascertain. These debts that the firm may not collect are
called doubtful debts. A firm should therefore provide for such debts by charging
the provision in the profit and loss account. Provision for doubtful debts maybe
specific or general. Specific relate to a debtor whom we can identify and we are
doubtful that he may pay the debt (if one of our debtor goes out of business).
Accounting For Bad & Doubtful Debts.
Bad debts
When a debt becomes bad the following entries will be made:

Lesson Four

i. Debit bad debts account


Credit debtors account with the amount owing.
ii. Debit Profit and Loss Account.
Credit bad debts account to transfer the balance on the bad debts
account to the Profit and Loss Account.

119

120

Adjustment to Final Accounts

Doubtful Debts
A provision for doubtful debts can either be for a specific or a general provision. A
specific provision is where a debtor is known and chances of recovering the debt
are low.
The general provision is where a provision is made on the balance of the total
debtors i.e. Debtors less Bad debts and specific provision.
The accounting treatment of provision for doubtful debts depends on the year of
trading and the entries will be as follows. If it is the 1 st year of trading (1st year of
making provision):
i.
ii.

Debit P&L a/c.


Credit provision for doubtful debts (with total amount of the provision).

In the subsequent periods, it will depend on whether if it is an increase or decrease


required on the provision.
If it is an increase:
i.
ii.

Debit P&L a/c.


Credit provision for doubtful debts (with increase only).

If it is a decrease:
i.
Debit provision for doubtful debts.
ii.
Credit P&L a/c (with the decrease in provision only).
Example
Debtors
Bad debts

(x)
x
Specific Provision (x)
x
General Provision (x)
x
A firm started trading in the year 1999, the balance on the debtors account was
400,000. Bad debts amounting to 40,000 were written off from this balance,
there was a specific provision of 5,000 to be made to one of the debtors and a
general provision of 5% was to be made on the balance of the debtors. The
ledger accounts of 1999 were as follows:
Debtors
1999

1999

Provision for doubtful debts

1999

1999

Lesson Four

121

Bal B/d 400,000


Bad debts 40,000
P&L
22,750
Bal c/d
360,000
400,000

400,000

31/12 Bal c/d

22,750 31/12

122

1999
Debtors

Adjustment to Final Accounts

Bad debts
1999
40,000 31/12

P&L

40,000

Debtors
Bad debts
Specific Provision
General Provision (5%)

400,000
(40,000)
360,000
(5,000)
355,000
(17,750)
337,250

Profit & Loss A/C (Extract) for the year ended 31/12/99

Expenses:
Bad debts
40,000
Increase in provision for D/debts
22,750

Balance Sheet (Extract) as at 31/12/99


Current Assets
Stocks
Debtors
Provision for D/debts

360,000
(22,750)
337,250

337,250
337,250

In the year 2,000, the debtors balance goes up to 500,000 from which bad debts
of 50,000 needs to be written off there is no specific provision but the general
provision is to be maintained at 5%. The ledger accounts will be as follows:
Debtors
Bad debts
General Provision (5%)

500,000
(50,000)
450,000
22,500
427,500

Lesson Four

123

124

Adjustment to Final Accounts

Debtors
2000
500,000 Bad Debts
______ Bal c\d
500,000

2000
Bal b\d

50,000
450,000
500,000

Provision for Doubtful Debts


2000
250 1\1 Bal b\d
22,500
22,750

2000
P\L
Bal c\d

22,750
22,750

Bad Debts
2000
50,000 31\12 P& L

2000
Debtors

50,000

Profit And Loss Account (Extract) for year ended 31/12/2002.

Incomes
Decrease in provision for D/debts

250

Expenses
Bad debts

50,000

Balance Sheet (Extract) as at 31/12/2002

Current Assets
Debtors
Provision for bad debts

450,000
(22,500)
427,500

In the year 2001 the debtors balance goes up to 600,000 from which bad debts of
50,000 need to be written off, there is no specific provision but the general
provision is to be maintained at 5% the ledger accounts is as shown:

Debtors
Bad debts

600,000
(50,000)
550,000

Lesson Four

125

General provision %
(27,500)
522,500

2001
Bal b\
600,000

Debtors
2001
Bad Debts

50,000

______ Bal c\d


600,000

550,000
600,000

Provision for Doubtful Debts


2001
1\1 Bal b\d
27,500 P& L
22,500

2001
Bal c\d

Bad Debts
2001
50,000 31\12 P& L

2001
Debtors

22,500
5,000
27,500

50,000

Profit And Loss Account (Extract) for the year ended 31/12/2001

Expenses
Bad debts
Increase in provision

50,000
5,000

Balance Sheet (Extract) as at 31/12/2001

Current Assets
Debtors
Less: Provision for Doubtful Debts

550,000
(27,500)

522,500

Example 4.6
In a new business during the year ended 31 December 2002 the following debts
are found to be bad, and are written off on the dates shown:
30 April
31 August
31 October

H Gordon
D Bellamy Ltd
J Alderton

1,100
640
120

126

Adjustment to Final Accounts

On 31 December 2002 the schedule of remaining debtors, amounting in total to


68,500, is examined, and it is decided to make a provision for doubtful debts of
2,200.
You are required to show:
a. The Bad Debts Account, and the Provision for Doubtful Debts Account.
b. The charge to the Profit and Loss Account.
c. The relevant extracts from the Balance Sheet as at 31 December 2002.

Lesson Four

127

Bad Debts

Debtors

70,036

2002

2002

Bad debts
1860

(1,860)

Bad debts

68,500

1860 31/12 P\L


Provision for

D/Debt (2,200)
66,300

Provision for doubtful debts


2002
2002

31/12 Bal c/d


2,200 31/12
P&L

2,200

Profit & Loss Account (Extract)

Expenses
Bad debts
Increase in provision for Doubtful debts

1,860
2,200

Balance Sheet (Extract)

Current Assets
Debtor
Less: Provision for D/Debts

8,500
(2,200)
6,300

Example 4.2
A business started trading on 1 January 2001. During the two years ended 31
December 2001 and 2002 the following debts were written off to the Bad Debts
Account on the dates stated:
31
30
28
31
30

August 2001
September 2001
February 2002
August 2002
November 2002

W Best
S Avon
L J Friend
N Kelly
A Oliver

850
1,400
1,800
600
2,500

On 31 December 2001 there had been a total of debtors remaining of 405,000. It


was decided to make a provision for doubtful debts of 5,500.
On 31 December 2002 there had been a total of debtors remaining of 473,000. It
was decided to make a provision for doubtful debts of 6,000.
You are required to show:

128

i.
ii.

Adjustment to Final Accounts

The Bad Debts Account and the Provision for Doubtful Debts Account for
each of the two years.
The relevant extracts from the Balance Sheet as at 31 December 2001 and
2002.

Solutions
Bad debts =
Provision

2,250
405,000
(5,500)
399,500
Bad Debts
2001
850
1400 31\12 P&L
2250

2001
31\8 W.Best
30\9 S.Aron

2250
2250

Provision for D/Debts


2001
550 31\12 P&L

2001
31\12 Bal c\d
2001
1\1

550

2001
1\1 Bal b\d
600 31\12 P&L
600

Bal c\d

550
50
600

2001
28/2 J. Friend
31/8 N. Kelly
30/11 A. Oliver

Bad Debts
2001
1,800
600
2,500
31/13 P&L
4,900

4,900
4,900

Lesson Four

129

Profit & Loss Account (Extract)


19x6

Expenses
Bad debts
Provision for Doubtful Debts

2,250
5,000

19x7
Bad debts
4,900
Increase in provision for D/Debts

500

Balance Sheet as at 19x6

Current Assets
Debtors
Less provision

405,000
(5,500)

399,500

19x7
Debtors
Less: provision

473,000
(6,000)

467,000

Provision for discounts allowable.


In some cases a firm may create a provision for discounts allowable in addition to
provision for doubtful debts. This happens where a firm anticipates that some of
the debtors may take up cash discounts offered by the firm. The accounting
treatment is similar to accounting for provision for doubtful debts. The provision
should be made after creating a provision for doubtful debts (debtors figure less
either general/specific provision for doubtful debts).
Debtors
Bad debts
Specific provision

x
(x)
x
(x)
x
(x)
x

Provision for discount allowed (on balance)

(x)
x

130

Adjustment to Final Accounts

Profit & Loss Account (Extract)

Incomes
Decrease in provision for D/Debts
Decrease in provision for discounts allowed
Expenses
Bad debts
Increase in provision for D/Debts
Increase in provision for discounts allowed

x
x
x
x
x

Balance Sheet (Extract)


Current Assets
Debtors
Less: provision for Doubtful Debts
Less: provision for discounts allowed

x
(x)
(x)

Bad Debts Recovered


A firm may be able to recover a debt that was previously written off. The following
entries will be made if this happens:
i.
Debit Debtors
Credit credit bad debts recovered account to restore the bad debt
recoverable.
N/B: This should be the amount to be recovered.
ii.
Debit Cashbook
Credit Debtors with the cash received.
iii.
Debit bad debts recovered account.
Credit P & L account with the same balance as bad debts account.
Example:
A firm recovers debts amounting to 10,000 that had been written off in the
previous periods. In the same financial period the firm writes off bad debts
amounting 30,000. The ledger accounts will be as follows:
Bad debts

Debtors
30,000 Bad Debt Recovered
10,000
P\L
20,000
30,000
30,000

Bad Debt

Bad debts recovered

10,000 Debtors

10,000

Lesson Four

131

c) BANK RECONCILIATION STATMENTS


The cashbook for cash at bank records all the transactions taking place at the bank
i.e. the movements of the account held with the bank. The bank will send
information relating to this account using a bank statement for the firm to
compare.
Ideally, the records as per the bank and the cashbook should be the same and
therefore the balance carried down in the cashbook should be the same as the
balance carried down by the bank in the bank statement.
In practice however, this is not the case and the two (balance as per the bank and
firm) are different. A bank reconciliation statement explains the difference
between the balance at the bank as per the cashbook and balance at bank as per
the bank statement.
Causes of the differences:
Items Appearing In The Cashbook And Not Reflected In The Bank
Statement.
Unpresented Cheques: Cheques issued by the firm for payment to the creditors
or to other supplies but have not been presented to the firms bank for payment.
Uncredited deposits/cheques: These are cheques received from customers and
other sources for which the firm has banked but the bank has not yet availed the
funds by crediting the firms account.
Errors made in the cashbook
These include:
Payments over/understated
Deposits over/understated
Deposits and payments misposted
Overcastting and undercasting the Bal c/d in the cashbook.
ii) Items appearing in the bank statement and not reflected in the
cashbook:
Bank charges: These charges include service, commission or cheques.
Interest charges on overdrafts.
Direct Debits (standing orders) e.g. to pay Alico insurance.
Dishonored cheques
A cheque would be dishonored because:

Stale cheques

Post dated cheques

Insufficient funds

Differences in amounts in words and figures.


Direct credits

132

Interest Income/Dividend incomes

Adjustment to Final Accounts

Lesson Four

133

Errors of The Bank Statement (Made By The Bank).


Such errors include:
Overstating/understating.
Deposits
Withdrawals
The Purposes of a bank reconciliation statement.
1. To update the cashbook with some of the items appearing in the bank
statement e.g. bank charges, interest charges and dishonoured cheques and
make adjustments for any errors reflected in the cashbook.
2. To detect and prevent errors or frauds relating to the cashbook.
3. To detect and prevent errors or frauds relating to the bank.
Steps in preparing a bank reconciliation statement.
1. To update the cashbook with the items appearing in the bank statement and
not appearing in the cashbook except for errors in the bank statement.
Adjustments should also be made for errors in the cashbook.
2. Compare the debit side of the cashbook with the credit side of the bank
statement to determine the uncredited deposits by the bank.
3. Compare the credit side of the cashbook with the debit side of the bank
statement to determine the unpresented cheques.
4. Prepare the bank reconciliation statement which will show:
a) Unpresented cheques
b) Uncredited deposits
c) Errors on the bank statement
d) The updated cashbook balance.
The format is as follows:
(Format 1)
Name:
Bank Reconciliation Statement as at 31/12

Balance at bank as per cashbook (updated)


Add: Un presented cheques
Errors on Bank Statement (see note 1)

x
x
x

x
x

Less: Uncredited deposits


Errors on Bank Statement (see note 2)
Balance at bank as per Balance Sheet

x
x

(x)
x

Note 1: These types of errors will have an effect of increasing the balance at bank
e.g. an overstated deposit or an understated payment by the bank.
Note 2: These types of errors will have an effect of decreasing the balance at bank
e.g. an understated deposit or an overstated payment by the bank, or making an
unknown payment.

134

Adjustment to Final Accounts

Format 2
Name:
Bank Reconciliation Statement as at 31/12

Balance at bank as per bank statement


Add: Uncredited deposits
x
Add errors on bank statement (note 2)

x
x

x
x

Less: Unpresented cheques


Errors on bank statement (note 1)
Balance at bank as per cashbook (updated)

x
x

(x)
x
===

Example 4.8
Draw up a bank reconciliation statement, after writing the cashbook up to date,
ascertaining the balance on the bank statement, from the following as on 31 March
2003:

Cash at bank as per bank column of the cashbook (Dr)


38,960
Bankings made but not yet entered on bank statement
6,060
Bank charges on bank statement but not yet in cashbook
280
Un presented cheques C Clarke
1170
Standing order to ABC Ltd entered on bank statement, but not in cash book
550
Credit transfer from A Wood entered on bank statement, but not yet in cashbook
1,890
Solution
19X9
31/3 Bal b/d
38960
A Wood (credit transfer)

Cashbook Bank
19X9
Bank charges
280
ABC (standing order)
1890 31/3 Bal C/D
40,850

550
40,020
40,850

Lesson Four

135

Bank Reconciliation as at 31/03/2003

Balance at bank as per cashbook


Add: Unpresented cheques

40,020
1,170
41,190
(6,060)
35,130
=====

Less: Uncredited deposits


Balance at bank as per Balance Sheet
Example 4.9

The following are extracts from the cashbook and the bank statement of J
Richards. You are required to:
a) Write the cashbook up to date, and state the new balance as on 31
December 2002, and
b) Draw up a bank reconciliation statement as on 31 December 2002.
2002
Dec 1
Dec 7
Dec 22
Dec 31
Dec 31

Dr
Balance b/d
J Map
J Cream
115
K Wood
M Barrett

Cashbook
2002
Cr
1,740 Dec 8
A Dailey
88
Dec 15
R Mason
73
Dec 28
249

Dec 31

349
33
G Small

Balance c/d
178

1,831

2,328
2,328
Bank Statement
2002
Dec
Dec
Dec
Dec
Dec
Dec
Dec

Dr

1
7
11
20
22
31
31

Balance b/d
Cheque
A Dailey
349
R Mason
33
Cheque
Credit transfer: J Walters
Bank charges

Cr

88

Balance

1,740
1,828
1,479
1,446
73
54

22

1,519
1,573
1,551

Cashbook Bank
2002
31/12 Bal b/d
31/12 J. Walters (C/T)
1,863

2002
1,831
31/1 Bank charges
54
31/12 Bal C/D

22

136

Adjustment to Final Accounts

1,885
1,885

Lesson Four

137

J. Richards
Bank Reconciliation Statement as at 31/12/2002

Balance at bank as per cashbook bank


Add: Unpresented cheques (G Small)

1,863
115
1,978

Less: Uncredited deposits


K Wood
M. Barret
Balance at bank as per balance sheet
OR:
Balance at bank as per balance sheet
Add: Uncredited deposits:
K. Wood
M. Barret
Less: Unpresented cheques
Balance at bank as per cashbook bank

249
178

(427)
1,551
1,551
249
178
1,978
(115)
1,863

Exam Type Question: Nov 2001 Q4


QUESTION FOUR
(a) Explain the term bank reconciliation and state the reasons for its
preparation.
(b) Ssemakula, a sole trader received his bank statement for the month of June
2001. At that
date the bank balance was Sh. 706,500 whereas his cash book balance was
Sh.2,366,500.
His accountant investigated the matter and discovered the following
discrepancies:
1. Bank charges of Sh.3, 000 had not been entered in the cashbook.
2. Cheques drawn by Ssemakula totaling Sh.22, 500 had not yet been
presented to the bank.
3. He had not entered receipts of Sh.26, 500 in his cashbook.
4. The bank had not credited Mr Ssemakula with receipts of Sh.98, 500
paid into the bank on 30 June 2001.
5. Standing order payments amounting to Sh.62, 000 had not been entered
into the cashbook.
6. In the cashbook Ssemakula had entered a payment of Sh.74, 900 as
Sh.79, 400.
7. A cheque for Sh.15, 000 from a debtor had been returned by the bank
marked refer to drawer but had not been written back into the
cashbook.
8. Ssemakula had brought forward the opening cash balance of Sh.329, 250
as a debit balance instead of a credit balance.
9. An old cheque payment amounting to Sh.44, 000 had been written back
in the cashbook but the bank had already honored it.

138

Adjustment to Final Accounts

10.Some of Ssemakulas customers had agreed to settle their debts by


paying directly into his bank account. Unfortunately, the bank had
credited some deposits amounting to Sh.832, 500 to another customers
account. However acting on information from his customers Ssemakula
had actually entered the expected receipts from the debtors in is
cashbook.

Required:
i.
A statement showing Ssemakulas adjusted cashbook balance as at 30 June
2001. (9 marks)
ii.
A bank reconciliation statement as at 30 June 2001.
(5marks)
(Total: 20 marks)
Solution
a) Bank reconciliation is an attempt to explain the difference between the cash at
bank balance
as per the cashbook and the cash at bank balance as per the bank statement.
Reasons for preparing a bank reconciliation statement are:
1. To update the cashbook with some of the relevant entries in the bank
statement.
2. To detect and prevent errors or frauds that relate to the cashbook.
3. To detect and prevent any errors or frauds that relate to the bank.
b)

ADJUSTED CASHBOOK
2001

Sh.

2001

Bal b/d
3,000
Receipts omitted

2,366,500

Bank charges

26,500

Standing orders

Sh.

62,000
cheques)

Payment overstated
15,000

4,500

Debtors (dishonored

Error on opening balance


Balance C/F
Cheque payment
Balance C/D
2,397,500
2,397,500
SSEMAKULA
Bank Reconciliation Statement as at 30 June 2001.

329,250
329,250
44,000
1,615,000

Lesson Four

139

Sh.
Cash at bank as per the updated cashbook
Add: Unpresented cheques
Less: Uncredited cheques
Error on bank statement
Balance as per the bank statement

98,500
832,500

Sh.
1,615,000
22,500
1,637,500
(931,000)
706,500

140

Adjustment to Final Accounts

Exam type Question: Nov 96 Q4


QUESTION FOUR
(a) What is the purpose of preparing a bank reconciliation statement?
(4marks)
(b) The following is the bank statement of Kakamega Retail Traders for the month
of October
1996:
Date
1996

Particulars

Debit
Sh.

Credit
Sh.

October 1
2
334,875
2
331,327
2
318,327
2
400,327
4
4
4
340,212
4
347,492
7
7
288,992
7
325,092
8
8
344,092
October 9
9
9
387,512
15
15
405,272
16
340,272
16
358,286

Balance b/d
Cheque no. 63

31,000

Cheque no. 67

3,548

Cheque no. 65

13,000

Deposit

82,000

365,875

Cheque no. 69
Cheque no. 68
Cheque no. 64
Deposit

Cheque no. 66
Deposit

394,327
391,212

7,000
51,500

340,492

9,000

316,092

36,100
28,000

Cheque no. 72
Cheque no. 73
Deposit

51,000

Cheque no. 74
Deposit

20,560

1,330
6,250
2,800

Cheque no. 75
Deposit

6,000
3,115
51,000
7,280

Cheque no. 70
Cheque no. 71
Deposit

Balance
Sh.

65,000
18,014

342,762
336,512
384,712

Lesson Four

141

17
392,786
19
384,286
19
427,036
21
21
412,120
21
21
401,620
21
410,620
23
394,380
23
457,380
26
455,880
26
26
527,130
28
491,630
28
481,230
28
454,230
28
431,730
28
444,755
31
31
472,055

Deposit

34,500

Cheque no. 76

8,500

Deposit

42,750

Cheque no. 79
Cheque no. 77
Cheque no. 78
Cheque no. 81

6,500

Deposit

9,000

2,410
12,506

424,626

4,000

408,120

Cheque no. 82

16,240

Deposit

63,000

Cheque no. 84

1,500

Dividends
Deposit

8,750
62,500

Cheque no. 88
Standing order

464,630
35,500

10,400

(Insurance)
Cheque no. 85

27,000

Cheque no. 87

22,500

Deposit

13,025

Service charge
Deposit

750
28,050

444,005

The following is the bank column of the cashbook:


Date
Particulars
Debit
Date Particulars
1996
Sh.
1996
October 1
1
3
5
8
10
15
15

Balance b/d
Deposited at
Deposited at
Deposited at
Deposited at
Deposited at
Deposited at
Deposited at

bank
bank
bank
bank
bank
bank
bank

365,875
7,280
36,100
28,000
51,000
20,560
18,014
34,500

October 1
1
1
2
4
5
5
7

Credit
Sh.

Cheque
Cheque
Cheque
Cheque
Cheque
Cheque
Cheque
Cheque

no.
no.
no.
no.
no.
no.
no.
no.

65 13,000
66
9,000
67
3,548
68
3,115
69
6,000
70
7,000
71 51,500
72
1,330

142

Adjustment to Final Accounts

17
19
19
22
24
27
28
29
31

Deposited
Deposited
Deposited
Deposited
Deposited
Deposited
Deposited
Deposited
Deposited

at
at
at
at
at
at
at
at
at

bank
bank
bank
bank
bank
bank
bank
bank
bank

42,750
15,700
9,000
36,000
26,500
13,025
28,050
171,010
31,525
22
23
26
28
28
28
28
30
31
31
31
934,889

8
Cheque no. 73
6,250
10 Cheque no. 74
2,800
11 Cheque no. 75 65,000
15 Cheque no. 76
5,800
18 Cheque no. 77 12,506
19 Cheque no. 78
4,000
19 Cheque no. 79
2,410
19 Cheque no. 80 3,860
19 Cheque no. 81
6,500
Cheque no. 82 16,240
Cheque no. 815,000
Cheque no. 84
1,500
Cheque no. 85 27,000
Cheque no. 86 10,520
Cheque no. 87 22,500
Cheque no. 88 53,500
Cheque no. 89 2,500
Cheque no. 90 64,529
Cheque no. 91 15,500
Balance c/d 502,481
934,889

Notes:
1. The bank reconciliation on 30 September 1996 showed that one deposit was
in transit and two cheques had not yet been presented to the bank.
2. Deposits of Sh.62, 500 and Sh.36, 000 had been entered in the cashbook as
Sh.26, 500 and Sh.36, 000 and in the bank statement as Sh.62, 500 and
Sh.63, 000, respectively.
3. A cheque from Mkulima for Sh.15, 700 was deposited on 18 October 1996
but was dishonored and the advice was received on 4 November 1996.
4. Counterfoils for cheques no. 76 and no. 88 showed they had been drawn for
Sh.5, 800 and Sh.33, 500 respectively.
Required:
a) A correct cashbook balance.
(8 marks)
b) A bank reconciliation statement on 31 October 1996.
(8
marks)
(Total: 20 marks)

Lesson Four

143

No 96 Q4
CASHBOOK (ADJUSTED)
1996
31.10 Bal b/d
10,400
Dividends
750
Error on deposit
15,700
Error on cheque 88
538,381
565,231

Sh.

1996
502,481

8,750

Sh.
Standing order (insurance)

Service charge
36,000

Dishonored cheques (debtor)

18,000
565,231

Bal c/d

Bank Reconciliation Statement as at 1 October 1996. (Previous period)


Sh.
Balance as per the cashbook
Add: Unpresented cheques 63
64

Sh.
365,875

31,000
51,000

82,000
447,875

Less: Uncredited cheques


Deposits
Balance as per the bank statement

(82,000)
365,875

Bank Reconciliation Statement as at 31 October 1996


Sh.
Balance as per the correct cashbook
Add: Unpresented cheques
Cheque no. 80
Cheque no. 83
Cheque no. 86
Cheque no. 89
Cheque no. 90
Cheque no. 91
Error on bank statement

Sh.
538,381
3,860
15,000
10,520
2,500
64,529
15,500
27,000

138,909
677,290

Less: Uncredited Cheques


Deposits

Error in bank statement


Balance as per the bank statement

171,010
31,525
2,700

(205,235)
472,055

144

Adjustment to Final Accounts

d) CAPITAL AND REVENUE EXPENDITURE


Capital Expenditure: This is the amount spent on the acquisition of a non-current
asset or adding value to a non-current asset.
Examples of expenses incurred in acquisition:
i.
Purchase price/cost of the asset.
ii.
Delivery/carriage inwards costs (e.g. shipping charges or import taxes).
iii.
Costs incurred to get the asset in use (e.g. assembly, testing)
iv.
Installation
v.
Demolition costs in order to construct a new building.
vi.
Architect fees for construction and supervision
vii.
Legal fees incurred in acquisition of a new asset (e.g. lease agreement)
Examples of expenses incurred in adding value to an asset:
i.
Modify plant to increase its useful life.
ii.
Upgrading plant to improve quality of output.
iii.
Adopting or upgrading the production process to improve or reduce costs.
Revenue Expenditure: Theres an amount spent by the firm in the normal trading
process or to assist in earning revenues or income. Examples:
i.
Postage and stationery.
ii.
Carriage outwards (sales).
iii.
Repairs and maintenance.
Example 4.10
For the business of K Spinns,a wholesaler, classify the following between capital
and revenue expenditure:
a) Purchase of an extra motor van.
b) Cost of rebuilding warehouse wall, which had fallen down.
c) Building extension to the warehouse.
d) Painting extension to warehouse when it is first built.
e) Repainting extension to warehouse three years later than that done in (d).
f) Carriage costs on bricks for new warehouse extension.
g) Carriage costs on purchases.
h) Carriage costs on sales.
i) Legal costs of collecting debts.
j) Legal charges on acquiring new premises for office.
k) Fire insurance premium.
l) Costs of erecting new machine.
Solution.
a) Capital expenditure
b) Revenue expenditure
c) Capital expenditure
d) Capital expenditure
e) Revenue expenditure
f) Capital expenditure
g) Revenue expenditure

Lesson Four

h)
i)
j)
k)
l)

145

Revenue expenditure
Revenue expenditure
Capital expenditure
Revenue expenditure
Capital expenditure.

e) DEPRECIATION
It is the loss of value of a non-current asset throughout its period of use by the
firm. IAS 16 on property, plant and equipment defines depreciation as the
allocation of a depreciable amount of a non-current asset over its estimated useful
life.
Under the matching concept, all incomes or revenues and expenses for a
particular period should be reported in the financial statements and because
depreciation is an expense of the business therefore, it will be charged in the P&L
A/C.
Causes of Depreciation
1. Physical Factors
a) Wear and tear: Some non-current assets depreciate or lose value due to
use overtime
e.g. machinery and motor vehicles.
b) Rot/decay/rust:: This happens on assets that are not well maintained by
the firm e.g.
Some machines.
2. Economic Factors
a) Inadequacy:
Some assets lose value due to them becoming
inadequate e.g. when a
business grows or expands then some buildings may become
inadequate due to space. Also some machines that are unable
to manufacture a large number of goods.
b) Obsolescence: Some assets become obsolete due to change in technology
or different
methods of production e.g. computers.
3. Time Factors
Some assets have a legal fixed time e.g. properties on lease.
4. Depletion
This occurs when some assets have a wasting character due to extraction of raw
materials, minerals or oil. Such assets include mines, oil wells, and quarries.
Methods of Calculating Depreciation
These are the methods developed to assist in estimating the amount of
depreciation to be charged in the P&L a/c as an expense.
The methods chosen by a firm should be in accordance with the agreed accounting
practice, accounting standards and suit the firms non-current assets. There are 2
main methods of estimating depreciation and 5 others that will apply in a firms
situation.
The main methods are: Straight-line method and Reducing Balance method. The
other 5 methods include:

146

i.
ii.
iii.
iv.
v.

Adjustment to Final Accounts

Sum of the digits methods uses a formular.


Revaluation method applies to a non-current asset of low value.
Machine-Hour method depreciation is based on number of hours a
machine is expected to operate (manufacturing process).
Unit of output method depreciation is based on the number of units a
machine is expected to produce.
Depletion of units depreciation is based on number of units extracted from
the asset.

Lesson Four

147

Straight-Line Method
This method ensures that a uniform amount of depreciation is charged in the P&L
a/c for a particular asset and is based on the following formular:
Depreciation for year
20,000

Cost of asset Residual Value =

Estimated useful life

100,000 8

= 10,000 per
year.
Cost of Asset Residual Value
Estimated useful life of asset.
Residual Value
The amount the firm expects to sell the asset after the period of use in the firm,
also called Sales Value / Scrap Value.
Estimated Useful Life
The period the asset is expected to be used in the firm.
Example 4.1
A firm buys a machine for 100,000 which it expects to use in the firm for eight
years. After the eight years the machine will be sold for 20,000. Under the
straight-line method, the depreciation amount will be computed as follows:
This means for this asset 10,000 will be charged in the P&L account as
depreciation expense on the machine.
The straight line method assumes that benefits accruing on use of a non-current
asset are spread out evenly over the life of the asset e.g. buildings use straight-line
method.
Percentage rate based on cost as opposed to number of years can also be used to
calculate the depreciation.
Reducing Balance Method
The firm determines a fixed percentage rate that is applied on the cost of the asset
during the first period of use. The same rate is applied in the subsequent financial
periods but the rate is applied on the reduced value of the asset. (Cost of asset
total depreciation provided to date).
This method ensures that higher amount of depreciation are charged in the P&L
account in the earlier periods of use and lower amounts in the latter periods of use
as shown in the following example:
Example 4.12

148

Adjustment to Final Accounts

Assume a firm buys machinery for 100,000 and provides depreciation on


machines at 20% p.a. on reducing balance method. The depreciation charged to
the P&L will be as follows for the next 3 years.

Lesson Four

149

Year 1

Cost
Depreciation 20% of 100,000
Balance to YR 2

80,000
(16,000)

P&L YR 2

64,000

Year 3
Depreciation 20 % of 64,000
Balance to YR 4

P&L YR 1

80,000

Year 2
Depreciation 20% of 80,000
Balance to YR 3

100,000
(20,000)

64,000
(12,800)

P&L YR 3

51,200

Reducing balance method (diminishing balance method) assumes that benefits


accruing from the use of an asset are higher in the first periods of use and
lower in the latter periods e.g.
Fixtures, furniture and fitting.
Plant and machinery.
Motor vehicles.
ACCOUNTING TREATMENT ON DEPRECIATION
When non-current assets are depreciated, a new account for each type of asset is
opened; this account is called a provision for depreciation whereby the following
entries will be made:
Debit P&L a/c
Credit Provision for depreciation a/c
With the amount of depreciation charged for the period.
Example on straight-line method
The entries will be as follows:
Debit P&L a/c with 10,000
Credit Provision for depreciation. Machines a/c with 10,000 being depreciation
provided for the machine.
The ledger accounts will be as follows:
Machinery
Machinery

Cashbook 100,000
31/12 Bal c/d
P&L
10,000

Provision for Depreciation

100,000

31/12 Bal c/d 10,000

150

Adjustment to Final Accounts

The final accounts extracts will be shown as follows:


(a) Profit And Loss Account (Extract) for the year ended
Expenses

Depreciation:
Buildings
x
Plant and machinery
Furniture, Fixtures and Fittings
Motor vehicles

10,000
x
x

(b) Balance sheet (Extract) as at________


Non Current Assets
Value)

Cost

Land
x
Buildings
x
Plant and Machinery
Furniture, Fixtures & fittings
Motor vehicles
x

Total

NBV (Net Book

Depreciation ()
(x)
x
x
x

x
x

(x)
(x)
(x)
x

x
x
x
x

Example 4.13
A company starts in business on 1 January 2002. You are to write up the motor
cars account and the provision for depreciation account for the year ended 31
December 2002 from the information given below. Depreciation is at the rate of 20
per cent per annum. Using the basis of one months ownership needs one months
depreciation.
2002

Bought two motor vans for 12,000 each on 1 January


Bought one motor van for 14,000 on 1 July.
Motorcars a/c

2002
1/1
Cashbook
1/7
Cashbook
38,000

2002
24,000
14,000
31/12
38,000

Bal c/d
38,000

Lesson Four

151

Calculation for depreciation


1/1

24,000 x 20 x 12
100 12

= 4,800 + 1/7( 14,000 x 20 x 6


100 12
= 4,800 + 1,400 = 6,200

= 1,400 )

152

2002

Adjustment to Final Accounts

Provision- Depreciation for Motor cars A/c


2002

31/12 Bal c/d

6,200

31/12

P&L

6,200

Profit And Loss Account (Extract) for the period.


Expenses

Depreciation:
Motor vans 6200

Balance Sheet (Extract) as at 31/12/2002


Non-current Assets
Motor vans

Cost
38,000

Total
Depreciation
(6200)

NBV
31,800

Example 4.14
A company starts in business on 1 January 1999, the financial year end being 31
December.
You are to show:
a. The plant account.
b. The provision for depreciation account.
c. The balance sheet extracts for each of the years 1999, 2000, 2001, 2002.
The machinery bought was:
1999 1 January
2000 1 July
1 October
2002 1 April

1 plant costing 8,000


2 plant costing 5,000 each
1 plant costing 6,000
1 plant costing 2,000

Depreciation is at the rate of 10 per cent per annum, using the straight-line
method, plant being depreciated for each proportion of a year.

Lesson Four

153

Plant a/c
199
8000
31/12

1999
1/1
Cashbook
2000
1/1
Bal b/d
1/7
Cashbook
1/10 Cashbook

Bal c/d

2000
8000
10,000
6,000
24,000

2001
1/1
Bal b/d
24,000
2002
1/1
Bal b/d
1/4
Cashbook

8000

31/12

2001
24,000

Bal c/d

31/12

2002
24,000
2,000 31/12
26,000

24,000
24,000

Bal c/d

Bal c/d

26,000
26,000

Calculation for Depreciation


1999
8,000 x 10/100 x 12/12

800

2000
10,000 x 10/100 x 6/12

500

6,000 x 10/100 x 3/12 =

150

8,000 x 10/100 x 12/12

=
800
1,450

2001
24,000 x 10/100 x 12/12

2400

2002
24,000 x 10/100 x 12/12

2400

2,000 x 10/100 x 9/12 =

150
2,250

Accumulated Depreciation
800

2,250
4,650

7,200

154

1999
31/12 Bal c/d

Adjustment to Final Accounts

Provision Depreciation Machines


1999

800 31/12 P&L


800

2000

31/12 Bal c/d

2000
1/1
Bal b/d
2,250
P&L

2,250
2001

31/12 Bal c/d

31/12 Bal c/d

800
1,450
2,250

2001
1/1
Bal b/d
4,650
P&L

4650
2002

2,250
2,400
4650

2002
1/1
Bal b/d
7,200
P&L

7,200

4,650
2,550
7,200

Balance Sheet (Extract) as at 31/12/99 31/12/02


Non Current Assets

Cost

Total
Depreciation

NBV

1999
Motor vans

8,000

1999
Motor vans

24,000

(2,250)

21,750

1999
Motor vans

24,000

(4,650)

19,350

1999
Motor vans

26,000

(7,200)

18,800

(800)

7,200

DISPOSALS OF ASSETS
A firm may dispose off its non-current assets in the following 3 ways:
i. Selling the asset.
ii. Asset being written-off from damage/accident/theft.

Lesson Four

155

iii. Asset is scrapped/not used anymore.


When an asset is disposed and is no longer used by the firm, the appropriate
entries should be made in the asset account and the total depreciation provided to
date on the asset and the entries required will depend on the type of disposal.
When the asset is sold, the following entries will be made:
(a) Debit asset disposal a/c
Credit asset a/c
With the cost of the asset being disposed.
(b) Debit provision for depreciation of asset a/c.
Credit asset disposal a/c
With the total depreciation provided to date on the asset.
(c) Debit cashbook.
Credit asset disposal a/c
With the cash received on disposal.
When an asset is written off as a result of damage/accident/theft. If it was insured
and the insurance company accept liability but by the end of the period the
insurance company has not yet paid.
(a) Debit asset disposal a/c
Credit asset a/c
With the cost of the asset damaged.
(b) Debit provision for depreciation of asset a/c
Credit asset disposal a/c
(c) Debit insurance receivable a/c
Credit asset disposal a/c
With the amount expected from the insurance.
If the insurance pays before the end of the financial period, it will not be necessary
to create an insurance debtor so the following entries will be made:
Debit cashbook.
Credit asset disposal a/c
If the asset is not used anymore or scrapped by the firm, the appropriate entries
will be made in the asset account and provision for depreciation a/c only.
Debit asset disposal a/c
Credit asset a/c
With the cost of the asset no longer in use.
Debit provision for depreciation for asset
Credit asset disposal a/c
With the total depreciation provided to date.

156

Adjustment to Final Accounts

The balance in the disposal a/c after the above entries will either be a debit
balance or a credit balance. A credit balance represents a profit on disposal,
which is reported in the profit and loss a/c together with other incomes. The entry
will be:
Debit asset disposal a/c
Credit P&L a/c
With the balance in the account.
A debit balance in the asset disposal a/c is loss on disposal which is reported in
the P&L a/c as an expense and therefore the entry will be.
Example 4.15
A firm has a motor vehicle costing 1,000 total depreciation provided to date is
800. The firm decides to trade in the motor vehicle with a new one the value of
the new one being 500. The supplier of the new vehicle agree with the firm that
the old motor vehicle is worth 300, therefore the difference will be paid by cash.

Bal b/d
Disposals
Cashbook

Motor vehicle a/c

1,000 Motor vehicle disposal


300
200 Bal c/d
500
1,500
1,500
=====
====

1,000

Motor Vehicle Disposal a/c

Motor vehicle
P&L

a/c
1,000
Provision for depreciation
100
Motor vehicle
300
1,100
1,100

JOURNAL ENTRIES

Debit motor vehicles disposal


1,000
Credit motor vehicles a/c
(Motor vehicle being traded in now transferred to disposal a/c)
Debit Provision for depreciation motor vehicles
Credit Motor vehicle disposal a/c
(Total depreciation provided for motor vehicle)
Debit Motor vehicle a/c
Credit Asset disposal a/c
- Cashbook

800

1,000

800
800
500
300
200

Lesson Four

157

(New motor vehicle acquired by trade-in value


of 300 and cheque payment of 200)
Debit Asset disposal a/c
Credit P&L
(Profit made on disposal)

100
100

In case of a loss,
Debit P&L a/c
Credit asset disposal a/c
If the firm trades in an old asset for a new one, the following entries will be made
in addition to the movements in the asset and depreciation a/c.
Debit asset a/c (value of the new asset)
Credit cashbook (cash paid as difference of new value i.e. trade in value of old
asset)
Asset disposal a/c (with trade-in value of old asset)
Example 4.16
A company depreciates its plant at the rate of 20 per cent per annum, straight line
method, for each month of ownership. From the following details draw up the
plant account and the provision for depreciation account for each of the years
1999, 2000, 2001 and 2002.
1999 Bought plant costing 900 on 1 January.
Bought plant costing 600 on 1 October.
2001 Bought plant costing 550 on 1 July.
2002 Sold plant which had been bought for 900 on 1 January 1999 for the
sum of
275 on 30 September 2002.
You are also required to draw up the plant disposal account and the extracts from
the balance sheet as at the end of each year.
Example
1999
1/1
Cashbook
1/10 Cashbook

900
600
1,500

2000
1/1
Bal b/d
1,500

2001
1/1
Bal b/d
1/7
Cashbook
2,050

Plant a/c
1999

31/12 Bal c/d


2000
1,500

2001
1,500
550 31/12

1,500
1,500

31/12

Bal c/d

Bal c/d

158

Adjustment to Final Accounts

2,050
2002
1/1
Bal b/d
900

2,050
2002
2,050
31/12

30/9

Disposal

Bal c/d

1,150
2,050

2,050

Plant Provision for Depreciation a/c


1999
31/12

Bal c/d

1999
210
31/12

Bal c/d

2000
1/1
510

2000
31/12

P&L

210

Bal b/d
P&L

210
300

510
2001
31/12

510
2001
1/1
865

Bal c/d

Bal b/d
P&L

865
2002
31/12

Disposals
Bal c/d

675
1,230

510
355
865

2002
1/1
555

Bal b/d
P&L
365
1,230

865

Calculation for Depreciation


Date

Cost

Months Depreciation charge

1999
1/1
1/10

900
600

12
3

20/100 x 900 x 12/12


20/100 x 600 x 3/12

=
=
210

180
30

Lesson Four

2000
1/1
300
2001
1/1
300
1/2

159

1,500

12

20/100 x 1,500 x 12/12

1,500

12

20/100 x 1,500 x 12/12

550

20/100 x 550 x 6/12

55
355

2002
30/9
31/12
31/12

900
550
600

2002
Plant a/c
P&L

9
12
12

900
50
950

20/100 x 900 x 9/12


20/100 x 550 x 12/12
20/100 x 600 x 12/12

=
=
=
365

Plant Disposal a/c


2002

30/9
Provision for depreciation
30/9
Cashbook
275
950

135
110
120

675

Balance Sheet (Extract)


Total
Non Current Assets
1999 Plant

1,500

Cost
(210)

Depreciation
1,290

NBV

2000 Plant

1,500

(510)

990

2001 Plant

2,050

(865)

1,695

2002 Plant

1,150

(555)

595

CHANGE OF DEPRECIATION POLICY


A firm may change its depreciation policy in several ways e.g. from straight line to
reducing balance or vice versa, or it may increase/decrease the number of
estimated useful years of an asset. A firm should always follow the depreciation
policy adopted consistently and incase there is need to change the policy may be
due to a new accounting standard or change in circumstances. This change should
be disclosed in the financial statements.
When there is change in the depreciation policy this may result in an increase or a
decrease in the depreciation to be charged in the Profit and loss account .IAS 16
requires that depreciation should be based on the remaining net book value at the
start of the period.
Example 4.17
A firm buys a machine for 100,000 for which it expects to use for the next 10
years. The firm depreciates the machines on a straight-line basis on the years of

160

Adjustment to Final Accounts

the number of estimated useful years. In the 4th year, the estimated useful life of
the machine is now reduced to 8 years. year.
Required:
Show the charge in the provision for depreciation a/c and the balance carried
down for year 4. Change for 10yr 8 yr is same as change from 10% to 12.5%

Lesson Four

161

Provision for Depreciation


Year 1
31/12

Bal c/d
10,000

Year 1
10,000

Year 2
31/12

Year 2
1/1
20,000

Bal c/d
10,000

20,000
Year 3
1/1
30,000

Year 3
31/12

Bal c/d
10,000

31/12

Bal b/d

31/12

Bal c/d

Year 4
1/1
31/12
44,000

10,000
P&L
20,000

Bal b/d
31/12

30,000
Year 4

P&L

20,000
P&L
30,000

Bal b/d
P&L

44,000

30,000
14,000
44,000

Workings:
The net book value at the beginning of Year 4 is 70,000 (100,000- 30,000). And
the remaining
useful life is 5 (8 years- 3 years). The charge for year 4 for depreciation will be
70,000 = 14,000.
5
Assuming that in this example the life of the machine does not decrease
but increases from 10 years to 13 years.
Required: Show the provision of depreciation account in year 4

162

Adjustment to Final Accounts

Provision for Depreciation


Year 1
31/12

Year 1
10,000

Bal c/d
10,000

Year 2
31/12

31/12

Year 2
1/1
20,000

Bal c/d
10,000

P&L

Bal b/d

10,000
P&L

20,000
Year 3
31/12

20,000
Year 3

Bal c/d

30,000

1/1

Bal b/d

20,000
_____

P&L

10,000

30,000
30,000
Year 4

Year 4
1/1 Bal b/d

30,000
31/12
7,000

Bal c/d

37,000

31/12

P&L

37,000

37,000

REVALUATION OF NON CURRENT ASSETS


Some of the non-current assets in a firm tend to appreciate in value rather than
depreciate e.g. land and buildings. IAS 16 on property, plant and equipment
requires that such assets may be carried in the accounts at the revalued amounts
(may be based on the their market price).
Land is not depreciated, and therefore the adjustments required are minimal, but
for buildings, changes should be made at the cost and depreciation reserve
account is usually opened for the purpose of these adjustments.
Example 4.18
A firm has the following assets as part of the non-current assets:
Asset

Cost

Depreciation

Lesson Four

163

(a)

Land

(b)

Buildings

1,000,000
800,000

40,000

Illustration 1
The firm decides to revalue these two assets to reflect their current market prices
and these are revalued at:
Land
- 1,200,00
Buildings - 900,000
The following entries would be made
(a) Debit Land A/c with revaluation gain - 200,000
Credit Revaluation Reserve a/c with the same - 200,000
(Revaluation gain on the land 1,200,000 1,000,000)
(b) Debit Building a/c with revaluation gain - 100,000
Credit Revaluation Reserve a/c with the same - 100,000
(Revaluation gain on buildings
(c)

900,000 800,000)

Debit Provision for depreciation for buildings a/c with 40,000


Credit Revaluation Reserve a/c with the same 40,000

Total credit depreciation charged to date on buildings now transferred to


revaluation reserve a/c
The ledger a/c will be as follows:
Land a/c

Bal B/D
Revaluation
reserve

1,000,000
__200,000

Bal C/D

1,2000,000

1,200,000
1,200,000

Buildings a/c

Bal B/D

800,000

Revaluation
reserve

100,000

Bal C/D

900,000

164

Adjustment to Final Accounts

900,000

900,000

Revaluation Reserve a/c

Bal C/D

340,000

Land

200,000

Buildings

100,000

Provision for depr.

340,000

40,000
340,000

Provision for depreciation (Buildings)

Revaluation

40,000

Bal B/D

40,000

Bal c/d

45,000

P&L

45,000

85,000

85,000

The balances in the Land and Building a/c will be shown as cost in the
Balance Sheet and the revaluation reserve a/c appears together with the
capital as a revaluation reserve (especially used in company accounts.
Land
Buildings

1,200,000 1,000,000 = 200,000


900,000 760,000 = 140,000

340,000

Any depreciation to be charged for the buildings should be based on the


revalued amount (900,000)
If we assume depreciation of 5% for buildings, we shall have 45,000
charged in the P & L and will also be the Bal c/d in the provision for
depreciation a/c.
Assume again that the firm decides to revalue its non-current assets or land
and buildings downwards in year 3 to the following values:
Land : 900,000
Buildings: 700,000
These amounts are to be reflected in the accounts for year 3.
The Ledger accounts will be as follows:
Land

Lesson Four

165

Year 3

1/1 Bal B/D

1,200,000

Year 3

31/12 Revaluation

200,000

P&L

100,000

________

Bal C/D

__900,000

1,200,000

1,200,000

Buildings
Year 3
1/1 Bal B/D

900,000

_______

Year 3

31/12
Revaluation

100,000

P&L

100,000

Bal C/D

700,000

900,000

900,000

Revaluation Reserve
Year 3

Year 3
1/1/ Bal B/D

31/12 Land

200,000

340,000

31/12 Building

100,000

31/12 Prov. For depr.

_40,000

_______

340,000

340,000

Exam Type Question 4.19 (December 1995 ) Question 4


James Mbuvi started a taxi business in Nairobi March 1990 under the firm name
Mbuvi Taxis. The firm had two vehicles KA and KB, which had been purchased
forSh.560, 000, and Sh.720, 000 respectively earlier in the year.
In February 1992 vehicle KB was involved in an accident and was written off. The
insurance company paid the firm Sh.160, 000 for the vehicle. In the same year the
firm purchased two vehicles, KC and KD for Sh.800, 000 each.

166

Adjustment to Final Accounts

In November 1993 vehicle KC was sold for Sh.716, 000. In January 1994 vehicle
KE was purchased for Shs.840,000. In March 1994 another vehicle KF was
purchased for
Sh.960, 000.
The firms policy is to depreciate vehicles at the rate of 25 per cent on cost on
vehicles on hand at the end of the year irrespective of the date of purchase.
Depreciation is not provided for vehicle disposed of during the year. The firms
year ends on 31 December.
Required:
a) Calculate the amount of depreciation charged in the profit and loss
account for each of the five years.
(7 marks)
b)

Prepare the motor vehicle account (at cost).


(8 marks)

c) Calculate the profit and loss on disposal of each of the vehicles disposed
of by the company.
(5 marks)
(Total: 20 marks)

a
Vehicle

1990

1991

1992

1993

KA

560,000

560,000

560,000

560,000

KB

720000

720,000

KC

800,000

KD

800,000

800,000

800,000

KE

840,000

KF

960,000

Total cost

1,280,00
0

1,280,00
0

2,160,00
0

1,360,00
0

2,600,00
0

320,00
0

320,00
0

540,00
0

340,00
0

650,00
0

Depreciation at
25%

1994

Lesson Four

167

Motor Vehicle
1990
1/3

Sh
Cashbook

1,280,00
0

1991
1/1

31/12

Sh
Bal c/d

1,280,000

Bal c/d

1,280,000

1991
Bal b/d

1,280,00
0

1992
1/1

1990

31/12
1992

bal b/d

1,280,00
0

1/2

Disposal

Cashbook

1,600,00
0

31/12

Bal c/d

2,880,00
0
1993
1/1

720,000
2,160,000
2,880,000

1993
Bal b/d

2,160,00
0

1/11

Disposal

________

31/12

Bal c/d

2,160,00
0
1994

800,000
1,360,000
2,160,000

1994

1/1

Bal b/d

1,360,00
0

1/1

Cashbook

840,000

1/3

Cashbook

960,000
3,160,00

31/12

Bal c/d

3,160,000
3,160,000

168

Adjustment to Final Accounts

Provision For Depreciation M/V


1990
31/12

Sh
Balc/d

320,000

1991

1990
31/12

Sh
P&L

320,000

1/1

Bal b/d

320,000

31/12

P& L

320,000

1991

1992
31/12

Bal c/d

640,000
640,000

1992

640,000
1992

1/2

Disposal

360,000

1/1

Bal b/d

640,000

31/12

Bal c/d

820,000

3/12

P&L

540,000

1,180,00
0
1993

1,180,000
1993

1/11

Disposal

200,000

1/1

Bal b/d

820,000

31/12

Bal c/

960,000

31/12

P&L

340,000

1,160,00
0
1994

1,1
60,000
1994
31/1

31/12

Bal c/d

1,610,00
0
1,610,00
0

Bal b/d

960,000

P&L

650,000
1,610,000

Note:
KA is fully depreciated by 1994,so no depreciation is charged for that asset. Cost
still remains until the asset is disposed. So depreciation ;
= 25% x 2,600,000
= 650,000
Exam type Question

Lesson Four

169

Pentland Limited complies its financial statements for the year to 30 June each
year.
At 1 July 1999 the companys balance sheet included the following figures:
l

Accumulate
d

Net book

Depreciatio
n

Value

000

000

000

Land

4,000

Nil

4,000

Buildings

2,200

800

1,400

Plant and
machinery

1,600

600

1,000

600

200

400

Cost

Motor vehicles

Depreciation is charged at the following annual rates (all straight line):


Land

Nil

Buildings

2%

Plant and machinery


Motor vehicles

15%

20%

Appropriate depreciation charge is made in the year of purchase, sale or


revaluation of an asset
During the year ended 30 June 2000 the following transactions took place:
1. I January 2000
The company decided to adopt a policy of revaluing its
buildings; and they were revalued to 3.4m.
2. 1 January 2000 Plant which has cost 300,000 was sold for 50,000.
Accumulated depreciation on this plant at 30 June 1999 amounted to
230,000.New plant was purchased at a cost of 400,000.
3. 1 April 2000 A new motor vehicle was purchased for 30,000. part of the
purchase price was settled by part exchanging another motor vehicle, which
had cost 20,000, at an agreed value of 12,000. the balance of 18,000 was
paid in cash.
4. The motor vehicle given in part-exchange had a net book value (cost less
depreciation) at 30 June 1999 of 10,000
Required:
Prepare ledger accounts to record these transactions in the records of Pentland
Limited.

170

Adjustment to Final Accounts

(16 marks)
Land
1999
1/7

1999
Bal b/d

4,000

2000
1/1

2000
Revaluation

1,200 30/6

Bal c/d

5,200

1999
1/7

Bal b/d

2000
1/1

Revaluation

2000
30/6

Bal C/D

30/6

5,200

Buildings
1999
2,200
2000
1,200 30/6
3,400

Bal c/d

Revaluation Reserve
2000
1/1
Buildings
2,022 1/1
Provision for
depr.
2,022

3,400
3,400

1,200
822
2,022

Provision for Depreciation - Building


1999

1/7
Bal b/d
800

1999
2000
1/1

5,200

Revaluation

Bal c/d

2000
82 30/6
2
34
_

P&L
2,200 x x
15
3,400 x x
15

56_

Lesson Four

171

85
6

1999
1/7
2000
1/1

Plant
1999
1,600

Bal B/D

2000
400 1/1
_____ 30/6
2,000

Cashbook

Disposal
Bal c/d

1999
1/7
Bal b/d

1/4
1/4

2000
1/4

2000
Disposal
Cash book

Motor Vehicle
P&L

1999

Disposal
Bal c/d

300
1,700
2,000

Provision for Depreciation - Plant


1999
1/7
Bal b/d

1999
2000
1/1

856

2000
252.50 30/6
595.00
847.50

P&L

1/4
30/6

247.50
847.50

Motor Vehicles

1999
600

12
18
630

600

2000
Disposal
Bal C/D

Motor Vehicle Disposal


2000
20 1/4 Provision for
depr.
5 1/4 Motor Vehicle
25
Provision for depreciation - Vehicle
1999
1/7
Bal b/d

20
610
630

13
12
25

200

172

Adjustment to Final Accounts

2000
1/4
30/6

2000
1/1
P&L

Disposal
Bal c/d

Plant

2000
13 1/4
307.5 30/6
320.50

P&L
Bal C/D

Plant - Disposal
2000
300 1/1 Provision for
depr.
2.50 Cash book
25

120.5
______
320.50

252.50
50___
302.50

Property, Plant and Equipment Schedule (Formerly fixed asset movement


schedule)
The property, plant and equipment schedule is a summary report on the balances
and transactions of the asset and provision for depreciation account as per the
requirements of IAS 16 to be reported in the published accounts of companies.
The format is as follows:

Lesson Four

Cost/
Valuation

173

Property, Plant and Equipment Schedule:


Freehold
Leasehold Property
Plant
Fixture,
property
and
Furniture
()

Bal as at
1/1/01
Additions
Revaluations
(gains)
Reclassificati
ons
Disposals
Bal as at
31/12/01
Depreciation
/
Amortization
Bal as at
1/1/10
Change for
year
Revaluation
Eliminated
on Disposal
Bal as at
31/12/01
N.B. V as at
31/12/01
NBV as at
31/12/01

Total

Short
lease
()
x

Machine
ry ()

And fittings
()

()

Long
leases
()
x

xx
xx

xx
-

xx
-

xx
-

xx
-

xx
xx

(xx)

xx

(xx)

(xx)

(xx)

(xx)

(xx)

(xx)

xx

xx

xx

xx

xx

xx

xx

xx

xx

xx

xx

xx

xx

xx

xx

xx

(xx)

(xx)

(xx)

(xx)

(xx)

(xx)

(xx)

(xx)

(xx)

(xx)

(xx)
xx

xx

(xx)
xx

(xx)
xx

(xx)
xx

(xx)
xx

xx

xx

xx

xx

xx

xx

Additional information is in this schedule called reclassifications where some of the


non-current assets are transferred into a different class. (e.g.) some of the
properties hold under long leases (over 50 years) will be transferred to the short
leases classes when their term becomes less than 50 years. This is a
reclassification from long lease to short lease and so is shown in the schedule at
the value of transfer as a deduction in the long lease class and on addition in the
short lease class
Exam Type Questions
May 2000 Question Three
a) Briefly explain the nature and purpose of accounting for depreciation.

174

Adjustment to Final Accounts

b) The chief accountant of Jitegemea Ltd has encountered difficulties while


accounting for fixed assets and the related depreciation in the companys draft
accounts for the year ended 30 April 2000. He has decided to seek your
professional advice and presented the following balances of fixed assets as at 1
May 1999:

Lesson Four

Furniture
Trucks
Plant and machinery
Land
Buildings

175

Acquisition

Accumulated

Cost
Sh.
900,000
3,525,000
7,387,500
2,775,000
2,925,000

Depreciation
Sh.
300,000
1,470,000
4,462,500
292,500

Depreciatio
n
Rates
%
12.5
25
10
Nil
2.5

The following additional information was also available:


1. It is the companys policy to write off cost of the assets using above percentages
on cost.
2. Depreciation is fully charged in the year of acquisition and none in the year of
disposal.
3. A three year old machine acquired for sh.187,500 was sold for sh.15,750.
4. It has been decided to adjust and charge depreciation on buildings at 4%.
5. A used delivery truck purchased three years ago for sh.248,250 was traded in
during the year at a value of sh.157,500 in part exchange of the new delivery
truck costing sh.450,000.
6. Land, buildings and machinery were acquired for sh.1,350,000 from a company
that went out of business. At the time of acquisition sh.90,000 was paid to have
the assets revalued by a professionally qualified valuer. The revaluation
indicated the following market values.
Sh.
Land
900,000
Buildings
600,000
Machinery
300,000
Required:
A schedule of movement of fixed assets as requested by the Chief Accountant for
inclusion in the companys accounts for the year ended 30 April 2000.
(10 marks)
(Total: 15 marks)
SOLUTION
Depreciation is the loss of value of an asset (non-current) throughout the period of
use by the firm. IAS 16 on property plant and equipment defines depreciation as
allocation of a depreciable amount of a non-current asset throughout its useful
life.
Under the matching concept, all revenues should be matched with all the expenses
that relate to a particular financial period and therefore because the firm to earn
revenue or income uses the assets, then the loss of value should be marched with
these revenues.
A charge is made in the Profit and Loss account as a depreciation expense for the
non-current asset.

176

Adjustment to Final Accounts

Property, Plant & Equipment Schedule:


Cost/Valuation
Land,
Furniture
Buildings
And
Machinery
Sh.
Sh.
Bal as at 1/5/99
13,087,5
900,000
00
Additions
1,350,000
Revaluation
450,000
Disposals
(187500)
_____Bal as at 30/4/2000
14,700,000
900,000
Depreciation
Bal as at 1/5/99
Charge for the
year
Eliminated on
disposal
Bal as at 30/4/2000

Total

Sh.
3,225,000

Sh.
17,512,500

450,000
(248,250)
3,726,750

1,800,000
450,000
(435,750)
19,326,750

300,000
112,500

1,470,000
931,687.5

6,525,000
2,110,687.5

-______

(124,125)

(161,625)

5,784,000

412,500

8,474,062.5

8,332,500
8,916,000

600,000
487,500

2,277,562.
5
2,055,000
1,449,187.
5

4,755,000
1,066,500
(37,500)

NBV 1/5/99
NBV 30/4/2000

Motor

10,987,500
10852,687.
8

Workings:
Depreciation on Furniture = 900,000 x 12.5% = 112,500
Motor vehicle

= cost 3,525,000
Add 450,000
3,726,750 x 25% = 931,687.5

Buildings

At 2.5%
4%

= (292,500 + 600,000) x 4%
= 141,000
= 2,925,000 x 2.5% x 4 = 292,500
= 292,500 x 4% x 4
= 468,000
175,500

Machinery: cost

c/f + Additions Disposals = Bal x 10%

Lesson Four

177

73,787,500 + 300,000 (187,500) = 7,500,000 x 10%


= 750,000

178

Adjustment to Final Accounts

REINFORCING QUESTIONS
QUESTION ONE
Otter Limited operates a computerized accounting system for its sales and
purchases ledgers. The control accounts for the month of September 1999 are in
balance and incorporate the following totals:

Sales ledger:
Balances at 1 September
1999: Debit

386,430
190

Credit
Sales
Cash received
Discounts allowed
Sales returns inwards
Credit balances at 30
September 1999
Purchases ledger:
Balances at 1 September
1999: Credit

163,194
158,288
2,160
590
370
184,740
520

Debit
Purchases
Cash payments
Discounts received
Purchases returns outwards
Debit balances at 30
September 1999

98,192
103,040
990
1,370
520

Although the control accounts agree with the underlying ledgers, a number of
errors have been found, and there are also several adjustments to be made. These
errors and adjustments are detailed below:
1. Four sales invoices totaling 1,386 have been omitted from the records.
2. A cash refund of 350 paid to a customer, A Smith, was mistakenly treated
as a payment to a supplier, A Smith Limited.
3. A contra settlement offsetting a balance of 870 due to a supplier against
the sales ledger account for the same company is to be made.
4. Bad debts totaling 1,360 are to be written off.
5. During the month, settlement was reached with a supplier over a disputed
account. As a result, the supplier issued a credit note for 2,000 on 26
September. No entry has yet been made for this.
6. A purchases invoice for 1,395 was keyed in as 1,359.

Lesson Four

179

7. A payment of 2,130 to a supplier, B Jones, was mistakenly entered to the


account of R Jones.
8. A debit balance of 420 existed in the purchases ledger at the end of August
1999. The supplier concerned cannot now be traced and it has been decided
to write off this balance.
Required:
Prepare the sales ledger and purchases ledger control accounts as they should
appear after allowing, where necessary, for the errors and adjustments listed.
QUESTION TWO
April showers sells goods on credit to most of its customers. In order to control its
debtor collection system, the company maintains a sales ledger control account. In
preparing the accounts for the year to 31 October 20X3 the accountant discovers
that the total of all the personal accounts in the sales ledger amounts to 12,802,
whereas the balance on the sales ledger control account is 12,550.
Upon investigating the matter, the following errors were discovered:
1. Sales for the week ending 27 March 20X3 amounting to 850 had been
omitted from the control account.
2. A debtors account balance of 300 had not been included in the list of
balances.
3. Cash received of 750 had been entered in a personal account as 570.
4. Discounts allowed totaling 100 had not been entered in the control
account.
5. A personal account balance had been undercast by 200.
6. A contra item of 400 with the purchase ledger had not been entered in the
control account.
7. A bad debt of 500 had not been entered in the control account.
8. Cash received of 250 had been debited to a personal account.
9. Discounts received of 50 had been debited to Bells sales ledger account.
10.Returns inwards valued at 200 had not been included in the control
account.
11.Cash received of 80 had been credited to a personal account as 8.
12.A cheque for 300 received from a customer had been dishonored by the
bank, but no adjustment had been made in the control account.
Required:
Prepare a corrected sales ledger control account, bringing down the amended
balance as at 1 November 20X3.
Prepare a statement showing the adjustments that are necessary to the list of
personal account balances so that it reconciles with the amended sales ledger
control account balance.
QUESTION THREE

180

Adjustment to Final Accounts

George had completed his financial statements for the year ended 31 March 1999,
which showed a profit of 81,208, when he realized that no bank reconciliation
statement had been prepared at that date.
When checking the cashbook against the bank statement and carrying out other
checks, he found the following:
1. A cheque for 1,000 had been entered in the cashbook but had not yet been
presented.
2. Cheques from customers totaling 2,890 entered in the cashbook on 31
March 1999 were credited by the bank on 1 April 1999.
3. Bank charges of 320 appear in the bank statement on 30 March 1999 but
have not been recoded by George.
4. A cheque for 12,900 drawn by George to pay for a new item of plant had
been mistakenly entered in the cash book and the plant account as 2,900.
Depreciation of 290 had been charged in the profit and loss account for this
plant.
5. A cheque for 980 from a credit customer paid in on 26 March was
dishonoured after 31 March and George decided that the debt would have to
be written off as the customer was now untraceable.
6. A cheque for 2,400 in payment for some motor repairs had mistakenly been
entered in the cash book as a debit and posted to the credit of motor
vehicles account. Depreciation at 25% per annum (straight line) is charged
on motor vehicles, with a full years charge calculated on the balance at the
end of each year.
7. The total of the payments side of the cash book had been understated by
1,000. On further investigation it was found that the debit side of the
purchases account had also been understated by 1,000.
George had instructed his bank to credit the interest of 160 on the deposit
account maintained for surplus business funds to the current account. This the
bank had done on 28 March. George had made an entry on the payments side of
the cashbook for this 160 and had posted it to the debit of interest payable
account.
George had mistakenly paid an account for 870 for repairs to his house with a
cheque drawn on the business account. The entry in the cashbook had been
debited to repairs to premises account.
George had also mistakenly paid 540 to Paul, a trade supplier, to clear his account
in the purchases ledger, using a cheque drawn on Georges personal bank account.
No entries have yet been made for this transaction.
The cashbook showed a debit balance of 4,890 before any correcting entries had
been made. The balance in the bank statement is to be derived in your answer.
Required:
1. Prepare an adjusted cash book showing the revised balance which should
appear in Georges balance sheet at 31 March 1999.

Lesson Four

181

(6 marks)
2. Prepare a bank reconciliation statement as at 31 March 1999.
(2
marks)
3. Draw up a statement for George showing the effect on his profit of the
adjustments necessary to correct the errors found.
(8
marks)
4. Prepare journal entries to correct items (9) and (10). Narratives are
required.
(4 marks)
QUESTION FOUR
1. Name and explain four types of errors which are not disclosed by the trial
balance.
(8 marks)
The trial balance of S Juma, a sole trader, did not balance on 30 April 1995.
The difference was put in the suspense account. The final accounts which
were then prepared showed a net profit of Sh. 64,000. During audit, the
following errors were noted:

A loan from ABD Bank of Sh 10,000 was entered correctly in cash


book but was not posted to the ledger.
A cheque of Sh. 4,000 for rent was not entered in the books.
Closing stock was overvalued by Sh 1,500.
Discount allowed of Sh 500 was entered in the discount-received
account.
The opening stock was understated by Sh 3,200.
Prepaid insurance of Sh 220 had been included in the profit and loss
account.
Goods destroyed by fire amounting to Sh 12,000 were written off in
the profit and loss account. However, the insurance company has
agreed to compensate the full amount.

Required:
1. Journal entries to correct the errors.
2. Statement of corrected profit.
3. Suspense account.

(8 marks)
(2 marks)
(2 marks)
(Total: 20 marks)

QUESTION FIVE
The following Trial Balance was taken from the ledger of P Spike, a sole trader, on
31st December 2002:

182

Adjustment to Final Accounts

Capital
Purchases
Sales
Salaries
Opening stock
Insurance
Rent
Buildings
Furniture
Debtors
Other expenses
Creditors
Commission

40,000

26,154
36,246
4,814
4,307
820
965
25,000
14,500
6,140
1,060
_____
82,795

4,638
__946
82,795

Adjustments:
1.
2.
3.
4.
5.
6.
7.

Salaries due, 350


Insurance was paid for one year up to 31st March 19-2.
Rent received for January 19-2, 165.
Commission accrued but not yet received, 120.
Furniture to be depreciated by 10%.
5% of debtors are doubtful.
Stock on 31st December 19-1 was valued at 5,008.

Required:
Prepare a 10 column worksheet.

QUESTION SIX
1. Explain the purposes for which control accounts are prepared in a business
organization.
(3 marks)
XML Ltd maintains control accounts in its business records. The balances and
transactions relating to the companys control accounts for the month of December
1994 are listed below:
Balance at 1 December 1994:
Sales ledger

6,185,0 (debit)
00
52,500 (credit)

Lesson Four

Purchases ledger
Transactions during December
1994:
Sales on credit
Purchases on credit
Returns inwards
Returns outwards
Bills of exchange payable
Bills of exchange receivable
Cheques received from
customers
Cheques paid to suppliers
Cash paid to suppliers
Bill payable dishonoured
Charges on bill payable
dishounered
Cash received from credit
customers
Bad debts written off
Cash discounts allowed

183

16,500 (debit)
4,285,0 (credit)
00
8,452,0
00
5,687,5
00
203,50
0
284,00
0
930,00
0
615,00
0
7,985,0
00
4,732,0
00
88,500
400,00
0
10,000

Bill receivable dishonoured

153,00
0
64,500
302,00
0
88,500

Balances at 31 December 1994:


Sales ledger
Purchases ledger

44,000 (credit)
23,500 (debit)

Required:
Post the sales ledger and the purchases ledger control accounts for the month of
December 1994 and derive the respective debit and credit closing balances on 31
December 1994.
(17 marks)
CHECK YOUR ANSWERS WITH THOSE GIVEN IN LESSON 9 OF THE
STUDY PACK

Acknowledgement

184

LESSON FIVE
FURTHER ADJUSTMNETS TO ACCOUNTS
(a) CONTROL ACCOUNTS
Control accounts are so called because they control a section of the ledgers. By
control we mean that the total on the control accounts should be the same as the
totals on the ledger accounts. There are two main types of control accounts:
(i)

(ii)

Sales ledger control Account also called total debtors. The balance on
the sales ledger control account should be the same as the total of the
balances in the sale ledger.
Purchases Ledger Control Account also called total creditors .The
balance carried down (Bal c/d) on the purchases Ledger Control Account
should be the same as the total of the balances in the purchases ledger.

Example (Sales Ledger Control a/c)


Sales Ledger Control A/c
Sales

1400 CashBook
Bal C/D
1400

700
700
1400

Sales = 200 + 300 + 400 + 500


Cashbook = 50 + 100 + 250 +
300
Balance c/d = 150 + 150 + 200

SALES LEDGER
Debtor A a/c
Sales

200 C/B
Bal c/d
200

50
150
200

Debtor B a/c
Sales

400 C/B
Bal c/d
400

250
150
400

Debtor C a/c
Sales

300 C/B
Bal c/d
FINANCIAL ACCOUNTING 1

100
200

Lesson Five

185

300

300

186

Further Adjustments to Accounts

Debtor D a/c
Sales

500 C/B
Bal c/d
500

300
200
500

Example: Purchases Ledger Control a/c


Purchases Ledger Control a/c
C/B
Bal c/d

1900 Purcha
ses
700
2600

2600
2600

PURCHASES LEDGER
Creditor A
C/B
Bal c/d

400 Purcha
ses
200
600

600
600

Creditor B
C/B
Bal c/d

450 Purchas
es
250
700

700
700

Creditor C
C/B
Bal c/d

350 Purchas
es
150
500

500
500

Creditor D
C/B
Bal c/d

700 Purchas
es
100

800

Lesson Five

187

800

800

Purpose of Control Accounts


1. Provide for arithmetical check on the postings made in the individual
accounts (either in the sales ledger or purchases ledger.)
2. To provide for a quick total of the balances to be shown in the trial balance
as debtors and creditors.
3. To detect and prevent errors and frauds in the customers and suppliers
account.
4. To facilitate delegation of duties among the debtors and creditors clerks.
FORMAT OF A SALES LEDGER CONTROL
Sales Ledger Control a/c
1. Balance b/d of the total debit
balances from previous
period
2. Total credit sales for the
period (from the sales
journal)
3. Refunds to customers (from
cashbook)
4. Dishonored cheques (from
cashbook)
5. Bad debts recovered (from
general journal)

6. Total credit balances of the


sales Ledger carried forward

1. Total credit balances of the


sales ledger brought forward
2. Total cash received from
credit customers/debtors
(from cash book)
3. Total cheques received from
credit customers/debtors
(from cash book)
4. Total returns-inwards
(returns-inwards journal)
5. Total cash discount allowed
to customers (from cash
book)
6. Bad debtors written-off (from
general journal)
7. Cash received from bad
debtors recovered (cash
book)
8. Purchases Ledger contra
9. Allowances to customers
(price reduction in excess to
discounts allowed)
10.Total debit balance carried
down to the next period to
be derived after posting all
those transactions

Refunds to Customers
Sometimes a firm can refund some cash on the customers account. This takes
place when there is a credit balance on the debtors a/c and the customer is not a
creditor too.
The entry will be:

188

Further Adjustments to Accounts

Dr. Debtors a/c


Cr. Cashbook
Example:
Debtor A

1000 Cashbook
950
100 Discounts
50
Returns
100
1100
1100
If the firm has not paid this amount owed to the customer, then its carried forward
to the next period then is a credit balance in the customers a/c. Therefore, if a
firm has several customer, this information will be shown in the control a/cs as
total balance c/f
(debit side).
Sales
(Refunds) C/B

Contra against the purchases ledger balances:


Some debtors may also be creditors in the same firm and therefore, if the amount
due to them as creditors is less than what they owe as debtors, then the credit
balance is transferred from their creditors a/c to their debtors a/c as a contra
entry.
Example:
Debtor (A)
Sales

2000 Contrapurchases
Bal c/d
2000

1000
1000
1100

Creditor (A)
Contra Debtor

1000 Purchases

1000

FORMAT OF A PURCHASES LEDGER CONTROL ACCOUNT


Purchases Ledger Control A/C
1. Total debit balances from
1. Total credit balance brought
purchases ledger brought
forward (of purchases ledger

Lesson Five

189

forward from previous period


2. Total cash paid to creditors
(from cash book)
3. Total cheques paid to creditors
(from cash book)
4. Total cash discounts received
(from cash book)
5. Allowances by suppliers

from the previous period)


2. Total credit purchases for the
period (from purchases
journal)
3. Refunds from suppliers
(from cash book)

6. Sales ledger contra


7. Total returns outwards
(from returns-outwards
journal)
8. Total credit balance
(to be derived after posting
entries)

4. Total debit balances (of the


purchases ledger carried
forward)

NOTES:
The following notes should be taken into consideration:
1) Cash received from CASH SALES should NOT be included in sales ledger
control a/c.
2) Only cash discounts (allowable & receivables) should be included. Trade
discounts should NOT be included.
3) Provision for doubtful debts is NOT included in the sales ledger control a/c.
i.e. increase or decrease in provisions for doubtful debts will not affect this
account.
4) Cash purchases are NOT posted to the Purchases Ledger Control A/C.
However in some cases it can be included especially where there are
incomplete records (Topic to be covered later).
5) Interest due that is charged on over due customers account may also be
shown on the debit side of the sales ledger control. However when trying to
determine the turnover under incomplete records then it is wise to omit it.
Example 5.1
You are required to prepare a purchases ledger control account from the following
for the month of June. The balance of the account is to be taken as the amount of
creditors as on 30 June.
2003
June 1

Purchases ledger balances


Totals for June:
Purchases journal
Returns outwards journal
Cheques paid to suppliers
Discounts received from
suppliers

36,760
422,570
10,980
387,650
8,870

190

Further Adjustments to Accounts

Purchases ledger balances

June 30
Solution
2003
Returns out
Bank
Discounts received
Bal c/d (30/6)

Purchases Ledger Control A/C


2003

10,980 Bal b/d (1/6)


36,760
387,95
0
8,870
51,830 Purchases
422,570
459,33
459,330
0

Example 5.2
Prepare a sales ledger control account from the following:

2003
May 1

May 31

Debit balances
Totals for May:
Sales journal
Cash and cheques received from debtors
Discounts allowed
Debit balances in the sales ledger set off
against credit balances in the purchases
ledger
Debit balances
Credit balances

64,200
128,000
103,700
3,950
1,450
?
500

Solution
2003
1/5

Bal b/d
Sales

31/5

Bal c/d

Sales
2003
64,20
0
128,0
00
500 31/5
192,7
00

Ledger Control A/C


Cash book
Discounts
allowed
Purchases
contra
Bal c/d

103,70
0
3,950
1,450
83,600
192,70
0

Lesson Five

191

Example 5.3 (Exam type question November 1997 Question 2)


(a) Explain the purposes for which control accounts are prepared. (3 marks)
(b) The balances and transactions affecting the control accounts of Kopesha Ltd.
for the month of November 1997 are listed below:Sh.
Balances on 1 November 1997:
Sales ledger
Purchases ledger
Transactions during November 1997:
Purchases on credit
Allowances from suppliers
Receipts from customers by
cheques
Sale on credit
Discount received
Payments to creditors by cheques
Contra settlements
Bills of exchange receivable
Allowances to customers
Customers cheques dishonored
Cash received from credit
customers
Refunds to customers for
overpayments
Discounts allowed
Balances on 30 November 1997
Sales ledger
Purchases ledger

9,123,00
0
211,000
4,490,00
0
88,000

(debit)
(credit)
(credit)
(debit)

18,135,0
00
629,000
27,370,0
00
36,755,0
00
1,105,00
0
15,413,0
00
3,046,00
0
6,506,00
0
1,720,00
0
489,000
4,201,00
0
53,000
732,000
136,000 (credit)
67,000 (debit)

Required:
The sales ledger and purchases ledger control accounts for the month of
November 1997 and show the respective debit and credit closing balances on 30
November 1997.
(17 marks)
(Total: 20 marks)

192

Further Adjustments to Accounts

(a)
i)

Provide for arithmetical check on the postings made in the


individual accounts (either in the sales ledger or purchases ledger.)

ii)

To provide for a quick total of the balances to be shown in the trial


balance as debtors and creditors.

iii)

To detect and prevent errors and frauds in the customers and


suppliers account.

iv)

To facilitate delegation of duties among the debtors and creditors


clerks.

Lesson Five

193

Kopesha Ltd
199
7
1/11

Bal b/d
Sales
Dishonored
cheques
Refunds to
customers

30/1
1

199
7
1/11

Bal c/d

Bal b/d
Allowances from
suppliers
Discounts
received
Bank
Contra settlement

30/1
1

Bal c/d

Sales Ledger Control A/C


Sh 199
7
9,123,00 1/11 Bal b/d
0
36,755,0
Bank
00
489,000
53,000

136,000 30/1
1
46,556,0
00

Contra
Bills of exchange
receivable
Allowances
Cash
Discounts allowed
Bal c/d

Purchases Ledger Control A/C


Sh 199
7
88,000 1/11 Bal b/d
Purchases
629,000
1,105,00
0
15,413,0
00
3,046,00
0
2,411,00 30/1 Bal c/d
0 1
22,692,0
00

Sh
211,000
27,370,00
0
3,046,000
6,506,000
720,000
4,201,000
732,000
2,770,000
46,556,00
0

Sh
4,490,000
18,135,000

67,000
22,692,000

Example 5.4 (Exam Question May 2000


Question 4)
Poesha Limited keeps sales and purchases control accounts in the General Ledger.
The transactions for the month ended 30 April 2000 were as follows:
Sh
Credit balances on 1 April 2000
-Sales ledger
154,000
-Purchases
569,000
ledger
Debit balances on 1 April 2000
-Sales ledger
956,000
-Purchases
196,000
ledger

194

Credit balances on 30 April 2000


Debit balances on 30 April 2000

Further Adjustments to Accounts

-Sales ledger

178,000

Purchases
ledger

189,000

Credit purchases

2,450,000

Credit sales

4,563,000

Cheques received from debtors

3,140,000

Cash received from debtors

1,367,000

Cheque payments to creditors

1,994,000

Cash payments to creditors


Bad debts written off

352,000
68,000

Discounts received

104,000

Discounts allowed

169,000

Contra entry to sales ledger from


purchases ledger

234,000

Refunds to debtors

62,000

Returns outwards

138,000

Returns inwards

231,000

Required:
Sales ledger and purchases ledger control accounts for the month ended 30 April
2000.
(20 marks)
ERRORS ON ACCOUNTS
There are two types of errors in accounts:
Errors that dont affect the trial balance
Errors that affect the trial balance
Errors that dont affect the trial balance
The trial balance produced from the accounts appears to be okay/correct, i.e the
debits are the same as the credits. However, on taking a close check on the
balances and transactions posted, errors may have been made and therefore the
balances shown on the trial balance may be incorrect i.e. under/over stated.
There are 6 main types of errors that dont affect the trial balance and these are
explained as follows:

Lesson Five

195

a) Error of omission
Here, a transaction is completely omitted from the accounts and therefore the
double entry is not made e.g. a sales invoice of 400 is not posted in the sales
journal therefore no entry is made in the debtors account and the sales account
i.e. both debit of 400 in debtors account and credit of 400 in the sales account.
The effect of the error is understates both the debtors and the sales.
To correct this error, the transaction is posted in the books by:
Debiting debtors
Crediting sales

400
400

b) Error of Commission
This error occurs when a transaction is posted to a wrong account but the account
is of the same class. Example: a credit sale to T Thompson is posted to L
Thompsons account for an amount of 200. Instead of a debit to T Thompsons
account it is made to L Thompsons account and the corresponding credit in the
sales account is correct.
Although the debit entry is made into the wrong account, the two accounts are of
the same class i.e. debtors.
To correct this error a transfer is made from L Thompsons account to T Thompson
by:

(i)
(ii)

Debit T Thompson a/c


Credit L Thompson a/c

200
200

c) Error of principle
In this type of error a transaction is posted not only to the wrong account but also
of a different class e.g. Motor vehicle purchased for 400 is posted to the motor
vehicle expenses a/c. (Instead of debiting motor vehicles, we debited motor
vehicle expenses a/c and the credit entry in the cashbook is correct)
The motor vehicles account is a non-current asset, and motor vehicles expenses a/c
is an expense account. Therefore a capital expenditure has been posted as
revenue expenditure.
To correct this error a transfer is made from the motor expenses account to the
motor vehicles a/c by:

(i)
Debit Motor vehicles a/c 400
(ii)
Credit Motor expenses a/c
400
d) Complete reversal of entries
A transaction is posted to the correct accounts but to the wrong sides of the
accounts i.e. a debit is posted as a credit and a credit is posted as a debit.

196

Further Adjustments to Accounts

Example: cash drawn from the bank of 150 for business use is posted as a debit in
the bank account and credit in cash in hand.
To correct this error, two entries are made in the relevant accounts:
(i)
Correct the error
(ii)
Post the transaction correctly
The entries will therefore be as follows:
(i)

Debit Cash in hand by


150
Credit bank by
150

To correct the error of 150 posted in the wrong sides of these account
(ii)

Debit cash by
150
Credit bank by
150
To post the entries correctly

e) Error of Original entry


Here a transaction is posted to the correct accounts but the amount posted is not
correct i.e. it is either under/over stated. In some cases, this is known as a
transposition error e.g. cash received from a debtor of 980 is credited/posted to
the customers account as 890.
To correct this error, the amount understated or overstated is posted to these
accounts to reflect the correct balance. In this case, we will:

Debit cash book


Credit debtors

90
90

f) Compensating Errors
These are errors that tend to cancel out each other i.e. if the effect of one error is
to understate the debits or credits then another error may take place to overstate
the debits or credits by the same amount, hence canceling out each other. E.g. if
the balance c/d of the purchases a/c is 3,980 but shown in the trial balance as
3,890 and another error carried to the trial balance of fixture amounting to
4,540 instead of 4,450:

Lesson Five

Purchases

197

3,980
3,890
(90)

Fixtures

4,450
(4,540)
90

This type of error is corrected by use of a suspense account.


Example 5.5
Give the journal entries needed to record the corrections of the following.
Narratives are required.
a)
b)
c)
d)
e)

Extra capital of 10,000 paid into the bank had been credited to Sales account.
Goods taken for own use 700 had been debited to General Expenses.
Private insurance 89 had been debited to Insurance account.
A purchase of goods from C Kelly 857 had been entered in the books as 587.
Cash banked 390 had been credited to the bank column and debited to the
cash column in the cashbook.
f) Cash drawings of 400 had been credited to the bank column of the cashbook.
g) Returns inwards 168 from M McCarthy had been entered in error in J
Charltons account.
h) A sale of a motor van 1,000 had been credited to Motor Expenses.

198

Further Adjustments to Accounts

Solution
THE JOURNAL
Debit
10,000

Sales
Capital
Additional capital passed into
sales a/c now transferred to
capital a/c
Drawings
General expenses
Drawings debited in general
expense now transferred to
drawing a/c
Drawings
Insurance
Private insurance transferred
from insurance a/c to drawings
a/c
Purchases
C Kelly
Purchases and creditors amount
to 857 initially entered as 587
Bank
Cash
Correct error in posting
Bank
Cash
To post the cash banked correctly
Bank
Cash
Cash drawings correctly started
from bank to cash
J Charlton
M McCarthy
Returns in from McCarthy
entered in error in J Carlton now
transferred to his a/c
Motor expenses
Motor disposal a/c
To correct error in recording
sales proceeds In expense
account

Credit
10,000

700
700

89
89

270
270
390
390
390
390
400
400
168
168
1000
1000

Example 5.6 (Exam type question May 200 Question 2)

Lesson Five

199

The balance sheet of N Patel, a sole trader, as at 31 March 2000 was as follows:

Sh000
Capital 1 April 1999
Profit for the year
ended 31 March
2000
Deduct: drawings
Creditors
Bank overdraft

450
150

Sh000
1,890 Land and
buildings (at
valuation)
Machinery (at
cost)
300 Deduct:
depreciation
630 Stock at cost
270 Debtors
3,090

Sh000

Sh00
0
1,650

1,200
750
570
420

450
990
3,090

Further investigation reveals the following information:


1. The closing stock includes damaged goods which, although they had cost Sh.
10,000 have an estimated sale value of Sh.7, 500.
2. Debtors include Sh. 20,000 in respect of a customer who has gone bankrupt.
A provision for doubtful debts of 2 % is also required on the balance of the
debtors.
3. The machinery was acquired five years ago and is being depreciated to its
scrap value on a straight-line basis over eight years. A more realistic
estimate indicates that the life span will be 10 years.
4. Wages owing at 31 March 2000 amounted to Sh. 9,500 but this has not been
reflected in the accounts.
5. Charges for the bank overdraft, amounting Sh 8,000 have not been reflected
in the accounts.
6. In arriving at the profit for the period, a drawing of Sh 100,000 paid to Mr.
Patel had been deducted as an expense.
7. Sh 20,000 rent owing to Mr. Patel for the letting of part of his business
premises to external party had not been received and no entry had been
made in the books in respect of this item.
Required:
a) Journal entries to correct errors and omissions.
(10
marks)
b) A statement of revised profit for the year ended 31 March 2000.
(8
marks)

200

Further Adjustments to Accounts

c) A revised balance sheet as at 31 March 2000.


marks)

(7
(Total: 25 marks)

Solution

a)

THE JOURNAL
Debit
2,500

Trading account
Stock
Being a reduction in stock for damaged
goods
Profit and loss(Bad debts)
Debtors
Debtors gone bankrupt written off
Profit and loss)
Provision for doubtful debts
Being a provision for doubtful debts created
at 20%.
Provision for depreciation
Profit and loss
A change in estimated lifespan for machinery
Profit and loss( wages )
Accrued expenses
Wages owing omitted in the accounts
Profit and loss (Bank overdraft
charges)
Bank overdraft
Changes for overdraft not reflected in the
accounts.
Drawings

Credit
2,500

20,000
20,000
10,000
10,000
150,000
150,000
9,500
9,500
8,000
8,000
100,000

Lesson Five

201

Profit and loss


Drawing to Mr. Patel deducted as an
expense.
Accrued income
Profit and loss (rent income)
Rent receivable owing not reflected in the
accounts.

100,000
20,000
20,000

b) STATEMENT OF ADJUSTED NET PROFIT


Sh
Net profit as per the account
Add: Provision for depreciation
50,000
Drawings
100,000
Accrued income (rent)
20,000
Less: Stock reduction
Bad debts
Provision for doubtful
debts
Accrued expenses
Bank charges
Net profit (revised)

Sh
450,000
170,000
620,000

2,500
20,000
10,000
9,500
8,000

(50,000)
570,000

REVISED BALANCE SHEET AS AT 31 MARCH 2000


Sh
Sh
Land and buildings
1,650,000
Machinery
1,200,000
(700,000)
2,850,000
700,000
Add: Current Assets
Stock
567,500
Debtors
400,00
Less: Provision for doubtful
(10,000)
390,000
debts
Accrued rent income
20,000
977,500
Less Current liabilities
Creditors
630,000
Accrued wage expense
9,500
Bank overdraft
278,000
(917,500)

Sh
1,650,000
500,000
2,150,000

60,000

202

Further Adjustments to Accounts

2,210,000

Capital
Add Net Profit

1,890,000
570,000
2,460,000

Less drawings

(250,000)
2,210,000

Errors That Affect The Trial Balance And The Suspense Account
These types of errors are reflected on the trial balance because the debits will not
be same as the credits. The debits may be more than the credits and vice versa.
Examples include:
1. Transaction is posted on one side of the accounts i.e. only a debit entry or a
credit entry. Example cash received from a debtor is debited to the cashbook
and no other entry is made in the account, i.e. no credit entry on the debtors
a/c.
2. A transaction is posted on one side of both the accounts i.e. two debits or two
credits. Example a payment to a creditor of 300 is credited in the cashbook
and also credited in the creditors accounts.
3. A transaction is posted correctly but different amounts i.e. debit is not the same
as the credit. Example cash received from a debtor of 450 is debited in the
cashbook as 450 and credited as 540 in the debtors a/c.
4. Error on balances of accounts i.e. understatement or overstatement of an
account balance due to mathematical errors.
5. Balance on an account is shown on the wrong side of the account when opening
the ledger accounts or when taken up to the trial balance. Example Bal c/d in
the cash book for cash at bank of 2000 is shown as a credit i.e. an overdraft,
instead of a debit in the trial balance. The balance may also be brought down
as an overdraft instead of a debit balance in the trial balance.
6. A balance is omitted from the trial balance on the accounts in total.
To correct the above errors, the appropriate or the adjusting entries are made
through an account called a suspense account.
The difference in the accounts is posted to this account and the entries to correct
the accounts are posted here. The balance to be shown on the suspense accounts
depends on which side the error is shown on the trial balance.
If the debits credits, then an amount is included on the credit side of the trial
balance so that the debits = credits. This is a credit balance and will be taken to
the suspense account on the credit side.

Lesson Five

203

Example:
DR
240
240

Total
Suspense

CR
200
40
240

Suspense a/c

Difference as per T/B

40

If the credits are more than the debits this is a debit balance and therefore we
require an amount to be added to the total of the debits for the two side to be
same. This debit balance is posted to the debit side of the suspense a/c.
Total
Suspense

Difference as per
T/B

DR
260
40
300

CR
300
300

Suspense a/c

40

Posting the correct entries should eliminate the balance on the suspense account.
In some cases, after checking for all errors that can affect the trial balance, the
suspense a/c has a balance. This balance depends on whether it is a credit or debit
and whether it is material or not for purposes of proper accounting treatment. The
following is the recommended approach:
Balance
Debit
Credit

Material
Show as an asset (eg)
other debtors
Show as a liability (eg)
other creditors

Not Material
Charge in P& L as an
expense
Report as income in
P&L

Example 5.7
A bookkeeper extracted a trial balance on 31 December 2002 that failed to agree
by 3,300, a shortage on the credit side of the trial balance. A suspense account
was opened for the difference.
In January 2003 the following errors made in 2003 were found:
(i)
(ii)
(iii)
(iv)

Sales daybook had been undercast by 1,000.


Sales of 2,500 to J Church had been debited in error to J Chane account.
Rent account had been undercast by 700.
Discounts received account had been under cast by 3,000.

204

(v)

Further Adjustments to Accounts

The sale of a motor vehicle at book value had been credited in error to Sales
account 3,600.

You are required to:


a) Show the journal entries necessary to correct the errors.
b) Draw up the suspense account after the errors described have been
corrected.
c) If the net profit had previously been calculated at79,000 for the year ended
31 December 2002, show the calculations of the corrected net profit
Solution
THE JOURNAL

1,000

Suspense
Sales
Sales under cast of 100 now corrected
J Church
J Chane
Sale to J Church posted to J Chane corrected
Rent
Suspense
Under cast in rent balance now corrected
Suspense
Discount received
Under cast in discount received balance now
corrected
Sales a/c
Disposal
Sale of motor vehicle entered in sales a/c now
corrected

Sales
Discount received

Suspense a/c

1,000 Bal b/d


3,000 Rent
4,000

STATEMENT OF CORRECTED NET

Net profit as per


account
Add:
Sales
1,000
Discount received
3,000
Less:
Rent
700
Sales
3,600
Corrected net profit

,1000

2,500
2,500
700
700
3,000
3,000
3,600
,3600

3,300
700
4,000

PROFIT

79,000

4,000
(4,300)
78,700

Lesson Five

205

Example 5.8
Chi Knitwear Ltd is an old fashioned firm with a handwritten set of books. A trial
balance is extracted at the end of each month, and a profit and loss account and
balance sheet are computed. This month, however, the trial balance did not
balance, the credits exceeding debits by 1,536.

206

Further Adjustments to Accounts

Your are asked to help and after inspection of the ledgers discover the following
errors:
(i)
(ii)
(iii)
(iv)
(v)
(vi)
(vii)

A balance of 87 on a debtors account has been omitted from the schedule


of debtors, the total of which was entered as debtors in the trial balance.
A small piece of machinery purchased for 1,200 had been written off to
repairs.
The recipiets side of the cashbook had been under cast by 720.
The total of one page of the sales daybook had been carried forward as
8,154, whereas the correct amount was 8,514.
A credit note for 179 received from a supplier had been posted to the
wrong side of his account.
An electricity bill in the sum of 152, not yet accrued for, is discovered in a
filing tray.
Mr. Smith, whose past debts to the company had been the subject of a
provision, at last paid 731 to clear his account. His personal account has
been credited but the cheque has not yet passed through the cashbook.

Solution
Opening balance
Sales - under
record

Suspense a/c

1,536.0 Debtors
0
360.00 Cashbook under cast
Creditors error
Creditors (correct)
Cashbook: smiths debt
paid
1,896.0
0

87.00
720.00
179.00
179.00
731.00
1,896.00

i.
Increase total for debtors by 87.
ii.
Add 1,200 to fixed assets and reduce repair costs by 1,200 therefore an
increase in
profits.
iii. Increase sales by 360.
iv. Reduce the creditors by 358.
v.
accruals by 152 and reduce profits by the same.
vi. Increase the cash balance by 731.

Lesson Five

207

Example 5.9 (Exam type question May 2002 question 1).


On 31 December 2001, an inexperienced bookkeeper working for Wanji, a sole
trader extracted a trial balance. Due to errors committed by the bookkeeper, the
trial balance failed to balance by Sh 369,400. He placed the difference in a
suspense account as shown below:
Wanji trial balance as at 31 December 2001
Sh
Sh
Fixed assets cost
832,000
Stocks:
1 January 2001
148,000
31 December
98,800
2001
Trade debtors
76,000
Prepayments
10,000
Trade creditors
34,600
Bank overdraft
15,200
Accruals
16,000
Drawings
359,600
Capital
1,054,000
Sales
1,043,200
Provision for
166,400
depreciation
Purchases
733,000
Operating expenses
126,000
Provision for doubtful
3,800
debts
Discounts received
5,000
Discounts allowed
5,800
Suspense account
________
369,400
2,548,400
2,548,400
Investigations carried out after preparing the above trial balance detected the
following errors:
1.
2.

3.
4.
5.
6.

The total of the sales daybook for December 2001 was overcast by Sh 25,700.
On July 2001, the business purchased office equipment for Sh 40,000. These
were debited to purchases account. Depreciation on the equipment is at the
rate of 10% per annum on cost and based on the period (months) of usage in
the year.
A payment to a creditor by cheque of Sh. 8,500 was erroneously credited to the
creditors account.
A payment of Sh. 4,500 for telephone expenses was debited to telephone
account as Sh 5,400.
An amount of Sh 15,000 received from a debtor was not posted to the debtors
account from the cashbook.
Purchases daybook for October 2001 was under cast by Sh 28,000.

208

Further Adjustments to Accounts

Assume the business had reported a net profit of Sh 85,800 before adjusting for
the above errors.
Required:
(a) The adjusted trial balance and the correct balance of the suspense account.
(6 marks)
(b) Journal entries to correct the errors (Narrations not required)
(6 marks)
(c) Suspense account starting with the balance determined in the adjusted trial
balance in (a) above.
(4 marks)
(d) The adjusted net profit for the year.
(4
marks)

Solution:
Adjusted Trial Balance
Fixed assets cost
Stock - 1 January 2001
Trade debtors
Prepayments
Trade creditors
Bank overdraft
Accruals
Drawings
Capital
Sales
Provision for
depreciation
Purchases
Operating expenses
Provision for doubtful
debts
Discounts received
Discounts allowed
Suspense account

Sales
Suspense
Office equipment
Purchases

Sh
832,000
148,000
76,000
10,000

Sh

34,600
15,200
16,000
359,600
1,054,000
1,043,200
166,400
733,000
126,000
3,800
5,000
5,800
47,800
2,338,200

_______
2,338,200

THE JOURNAL
Dr
25,700

Cr
25,700

40,000
40,000

Lesson Five

209

Provision for depreciation


Profit and loss

2,000
2,000

Creditors
Suspense
Creditors
Suspense

8,500
8,500
8,500
8,500

Suspense
Telephone

900
900

Suspense
Debtor

15,000
15,000

Suspense
Discounts allowed

2,500

Suspense
Discounts received

2,500

2,500
2,500

Purchases
Suspense
2001
1 Jan

Bal b/d
Telephone
Debtors
Discount
allowed
Discount
received
Bal c/d

28,000
28,000
SUSPENSE ACCOUNT
Sh 2001
47,800 1 Jan
Sales
900
Creditors
15,000
Creditors
2,500
Purchases
2,500
2,000
70,700

______
70,700

STATEMENT OF ADJUSTED NET PROFIT


Sh
Net profit as per the accounts
Add
Purchases
Telephone expenses
Discount allowed + received
Less
Sales
Depreciation
Purchases

Sh
25,700
8,500
8,500
28,000

Sh
85,800

40,000
900
5,000

45,900
131,700

25,700
2,000
28,000

(55,700)

210

Further Adjustments to Accounts

Corrected Net Profit

76,000

c) STOCK VALUATION (IAS 2 INVENTORIES)


inventories in a firm includes:
(a) Finished goods (assets held for sale)
(b) Work in progress (assets still in production for purposes of sale)
(c) Raw materials (to be used in production process).
The cost of inventories should include all costs of purchase. (Purchase price and
other taxes like import duties), costs of conversion (e.g. direct labour) and other
costs incurred in bringing the inventories into their present location and condition
(carriage inwards).
Inventories or stock is a sensitive area, as it does not form part of the double entry.
In most cases either carrying out stocktaking or checking the stock records that
the firm is kept determines the value of stock at the end of the financial period.
Stocktaking involves counting the number of units of finished goods, work in
progress or raw materials available or in the stores/warehouse/saleroom.
The value of stock to the final accounts is then derived by multiplying the cost per
unit to the total number of units available.
Example.
A firm has three products A, B and C whose costs are shs.200, shs.300 and shs.400
each respectively. At the end of year 2002, stocktaking was carried out and the
following units were available:
Product A 200,000 units
Product B 20,000 units
Product C 30,000 units

Lesson Five

211

Required:
Compute the cost of stock to be included in the final accounts.
Solution:
(200,000 x 200) + (20,000 x300) + (30,000 x 400) = shs.58, 000,000
Cost Formular:
The cost of the different units of stock that a firm has should be assigned to each
unit as far as the business can be able to identify each item.
For those units that the business cannot identify the specific cost due to the
number of transactions and changes in the cost price, IAS 2 on inventories
recommends the use of the following estimates:
(i)
First In First Out (FIFO)
The business assumes that items of stocks that were purchased first are sold first
and therefore, items left as part of closing stock were purchased recently.
(ii)
Weighted Average Cost (AVCO)
Under this method, the cost of each item is determined from the weighted average
of the cost of similar items at the beginning of the period and the cost of similar
items purchased during the period.
(iii)
Last In First Out (LIFO)
This method assumes that items of stock which were purchased last are sold first
and therefore, the closing stock shows items that were bought first.
Net Realizable Value (SP- Expenses)
This is the estimated selling price in the ordinary course of business less the
estimated costs of completion and the estimated costs necessary to make the sale.
In some cases, the value of stock may decline where below the cost price (either
actual or estimated under the different methods) and if the firm was to sell the
stock, then it will fetch an amount below this cost.
IAS 2 requires that closing stock should be stated at the lower of cost or net
realizable value.
Example:
A firm has a closing stock of Sh 300,000 (cost) out of which stock valued Sh
20,000 is damaged. This stock can fetch the firm Sh 22,000 after repairs and
packaging that will cost Sh 4,000.
Required:
What value will be attached on this damaged units and the total closing stock for
the final accounts purposes.
Sh
Cost
20,000
Selling price
22,000
Repairs
4,000
NRV (22-4)
18,000

212

Further Adjustments to Accounts

The NRV (22,000 4,000) is lower than the cost of Sh. 20,000 and therefore, this
damaged unit will be shown as Sh 18,000. The balance of the stock of Sh 280,000
+ 18,000 of the damaged stock will be included in the final accounts and shown
together as Sh 298,000.

Lesson Five

213

d) WORKSHEETS
A work sheet is a simple report that shows the final accounts inclusive of the trial
balance in column form. A work sheet has 8-10 columns and the simple headings
are as follows:
TRIAL
BALANC
E
Dr
Cr

ADJUSTME
NT
Dr
Cr

TRADING
ACCOUN
T
Dr
Cr

PROFIT &
LOSS
ACCOUNT
Dr
Cr

BALANCE SHEET
Assets Liabilities +
Capital

Example 5.10
Mr Chai has been trading for some years as a wine merchant. The following list of
balances has been extracted from his ledger as at 30 April 19X7, the end of his
most recent financial year.

Capital
83,887
Sales
259,870
Trade creditors
19,840
Returns out
13,407
Provision for bad debts
512
Discounts allowed
2,306
Discounts received
1,750
Purchases
135,680
Returns inwards
5,624
Carriage outwards
4,562
Drawings
18,440
Carriage inwards
11,830
Rent, rates and insurance
25,973
Heating and lighting
11,010
Postage, stationery and telephone
2,410
Advertising
5,980
Salaries and wages
38,521
Bad debts
2,008
Cash in hand
534
Cash at bank
4,440
Stock as at 1 May 19x6
15,654
Trade debtors
24,500
Fixtures and fittings at cost
120,740
Provision for depreciation on fixtures
and fittings as at 30 April 19X7
63,020
Depreciation
12,074
The following additional information as at 30 April 19X7 is available:

214

Further Adjustments to Accounts

(a)
(b)
(c)
(d)
(e)

Stock at the close of business was valued at 17,750.


Insurances have been prepaid by 1,120.
Heating and lighting is accrued by 1,360.
Rates have been prepaid by 5,435.
The provision for bad debts is to be adjusted so that it is 3% of trade
debtors.

Required:
MR CHAI
Trial Balance
WORKSHEE
T

Dr

Cr
83,887

Dr

Capital
Sales
Trade
creditors
Returns
outwards
Provision for
B debts
Discounts
allowed
Discounts
received
Purchases
Returns
Inwards
Carriage
outwards
Drawings
Carriage
inwards
Rent, rates &
insurance
Heating &
lighting
Postage,
stationery
and
telephone
Advertising
Salaries and
wages
Bad debts
Cash in hand

Adjustments

Trading
account

Cr

Dr

Profit & loss


Balance
a/c
sheet

Cr

259,87
0
19,840

259,8
70

13,407

13,40
7

Dr

223
2,306

1,750

1,750

135,68
0
5,624

135,6
80
5,624

4,562

4,562

18,440

18,44
0

11,830

11,83
0

25,973

5,980
38,521
2,008
534

Cr
83,8
87

735

2,
306

2,410

Dr

19,8
40

512

11,010

Cr

6,55
5
1,360

19,41
8
12,37
0
2,410

5,980
38,52
1
2,008
534

Lesson Five

Cash at bank
Stock at 1
May 19X6
Trade
debtors
Fixtures &
fittings at
cost
Provision for
depreciation
Depreciation

4,440
15,654

4,440
15,65
4

24,500

24,50
0
120,7
40

120,74
0
63,020

63,0
20

12,074
442,28
6

Stocks
30.04.19X7
asset
Stocks
30.04.19X7
Cost of Sales
Insurance
prepaid
Heating and
lighting
accrued
Rates
prepaid
Provision for
bad debts

215

12,07
4
442,28
6
17,75
0
17,7
50

17,75
0

1,120

1,120
1,36
0

1,36
0

5,435

5,435

223
25,88
8

Gross profit
(Balancing
figure)

17,75
0

223
25,8
88
122,2
39
291,0
27

122,2
39
291,0
27

Net profit
(Balancing
figure)

Prepare a worksheet for the year to 30 April 19X7

24,11
7
123,9
89

123,9
89

24,1
17
192,9 192,
59 959

216

Further Adjustments to Accounts

Solution
This marks the end of the session on preparing final accounts with adjustments. In
the next session we shall prepare the final accounts incorporating these
adjustments. Some adjustments will affect the format of final accounts and
therefore they will look as follows:

FORMAT OF FINAL ACCOUNTS WITH ADJUSTMENTS


NAME
TRADING, PROFIT AND LOSS ACCOUNT FOR THE YEAR ENDED 31 DEC

Sales
XX
Less
(XX)
Returns
inwards
XX
Less cost
of sales
Opening
XX
stock
Purchases XX
Add
XX
carriage
in
XX
Less
(XX
XX
Returns
)
out
XX
Less
(XX)
(XX)
closing
stock
Gross
XX
profit
Discount
XX
received
Other
XX
incomes
(rent,
interests,
dividends)
Profit on
XX
disposal
of noncurrent

Lesson Five

217

assets
Reduction
in
provision
for
doubtful
debts
Reduction
in
provision
for
discount
allowable
Interest
on
overdue
debtors
balances

XX

XX

XX

XX
Less
Expenses
Bad debts
Depreciat
ion: (eg)
Plant

XX
XX
XX

Motor
vehicle
Increase
in
provision
for
doubtful
debts
Increase
in
provision
for
discount
allowable
Loss on
disposal
of non
current
assets
Loss of
other
assets
(eg) stock
Interest

XX

XX

XX

XX

XX

218

Further Adjustments to Accounts

charged
by
creditors
Other
expenses:
Rent
Insurance
Postage
Interest
on loan
etc
NET
PROFIT

XX
XX
XX
XX

(XX)
XX

BALANCE SHEET AS AT 31 DEC.

Non current
assets

Land
Buildings
Plant and
machinery
Fixtures,
furniture and
fittings

XX
XX
XX

(XX)
(XX)

XX
XX
XX

XX

(XX)

XX

Motor vehicle

XX

(XX)

XX

XX

Current assets
Stock
Debtors
Less provision for
doubtful debts

XX
XX
(XX)

XX

Accrued income

XX

Prepaid expenses

XX

Cash at bank

XX

Cash in hand

XX

XX
XX

Lesson Five

219

Current
liabilities
Bank overdraft

XX

Trade creditors

XX

Prepaid income

XX

(XX)

Net current
assets
Net assets

X
X
XX

Capital
Add net profit

XX

X
X

XX
(XX)

Less drawings

XX
Non current
liabilities
Loan

X
X
XX

Non current
liabilities
Loan
XX

X
X

Non current
liabilities
Loan

X
X
XX

Non current
liabilities
Loan
XX
Non current liabilities
Loan

X
X
XX

220

Further Adjustments to Accounts

Example 5.11
Given the question 5.10, the final accounts for the year ended 30 April 19X2 will be
as follows:
Mr Chai
Trading and Profit and Loss Account for year ended 30 April 19X7

Sales
259,8
70
Less
(5,62
Returns
4)
inwards
254,2
46
Less
cost of
sales
Openin
15,654
g stock
Purchas
135,680
es
Add
carriag
e in
147,510
Less
(13,407)
134,103
Returns
out
Cost of
149,757
goods
availabl
e for
sale
Less
(17,750)
(132,
closing
007)
stock
Gross
122,2
profit
39
Add:
1,750
Discoun
t
receive
d
123,9
89
Less
Expens

Lesson Five

es
Discoun
t
allowed
Carriag
e
outward
s
Rent,
rates
and
insuran
ce
Heating
and
lighting
Postage
,
statione
ry and
telepho
ne
Advertis
ing
Salaries
and
Wages
Bad
debts
Provisio
n for
bad
debts
Provisio
n for
depreci
ation
fixtures
and
fitting
Net
profit
Mr Chai
Balance Sheet as at 30 April 19X7

221

2,306
4,562

19,418

12,370

2,410

5,980
38,521
2,008
223

12,074

99,87
2
24,11
7

222

Further Adjustments to Accounts

Non
current
asset
Fixtures
and
fittings
Current
assets
Stock
Debtors
Less
provision
for
doubtful
debts
Prepaym
ents
Cash at
bank
Cash in
hand
Current
liabilitie
s
Creditors
Accruals

120,7
40

(63,020)

57,720

17,750
24,50
0
(735)

23,765

6,555
4,440
5
34
53,044

19,84
0
1,360

(21,200)

31,844
89,564
83,887
24,117

Capital
Add net
profit

108,004
(18,440)

Less
drawings

89,564
Example 5.12
The following trial balance has been extracted from the ledger of Mr. Yousef, a
sole trader.
Mr. Yousef
Trading and Profit and Loss Account for the year ended 31 May 19X6.

Sales
Purchases

82,350

138,078

Lesson Five

Carriage
Drawings
Rent, rates and insurance
Postage and stationery
Advertising
Salaries and wages
Bad debts
Provision for bad debts
Debtors
Creditors
Cash in hand
Cash at bank
Stock at at 1 June 19X5
Equipment
At cost
Accumulated depreciation
Capital

223

5,144
7,800
6,622
3,001
1,330
26,420
877
130
12,120
6,471
177
1,002
11,927
58,000
______
216,770

19,000
53,091
216,770

The following additional information as at 31 May 19X6 is available:


(a) Rent is accrued by 210.
(b) Rates have been prepaid by 880.
(c) 2,211 of carriage represents carriage inwards on purchases.
(d) Equipment is to be depreciated at 15% per annum using the straight line
method.
(e) The provision for bad debts to be increased by40.
(f) Stock at the close of business has been valued at 13,551.
Required:
Prepare a trading and profit and loss account for the year ended 31 May 19X6 and
a balance sheet as at that date.
Solution:

224

Further Adjustments to Accounts

Mr. Yousef
Trading and Profit and Loss Account for the year ended 31 May 19X6.

Sales
138,078
Less cost of sales
Opening stock
11,927
Purchases
82,350
Carriage inwards
2,211 84,561
96,488
Less closing stock
(13,551
(82,937
)
Gross profit
55,141
Less expenses
Carriage outwards
2,933
Rent, rates and insurance
5,952
Postage and stationery
3,001
Advertising
1,330
Salaries and wages
26,420
Bad debts
877
Increase in provision for bad
40
debts
Depreciation equipment
8,700
(49,253
Net profit
5,888

Mr. Yousef
Balance Sheet as at 31 May 19X6.
Non Current assets
Equipment
Current Assets
Stocks
Debtors
Less provision for doubtful
debts
Prepayments
Cash in hand
Cash at bank
Current Liabilities
Creditors
Accruals
Capital
Add: Net Profit
Less Drawings

58,000

(27,700)

30,300

13,551
12,120
(170)

11,950
880
177
1,002
27,560

6,471
210

6,681

20,879
51,179
53,091
5,888
58,979
(7,800)

Lesson Five

225

51,179

226

Further Adjustments to Accounts

Example 5.13
The following trial balance has been extracted from the ledger of Herbert Howell,
a sole trader, as at 31 May 20X9, the end of his most recent financial year.
Herbert Howell
Trial Balance As At 31 May 20x9
Property at cost
Equipment at cost
Provision for depreciation (as at 1
June 20X8)
Property
Equipment
Stock as at 1 June 20X8
Purchases
Sales
Discounts allowed
Discounts received
Wages and salaries
Bad debts
Loan interest
Carriage out
Other operating expenses
Trade debtors
Trade creditors
Provision for bad debts
Cash on hand
Bank overdraft
Drawings
13% loan
Capital, as at 1 June 20X8

Dr

90,000
57,500

Cr

12,500
32,500
27,400
259,600
405,000
3,370
4,420
52,360
1,720
1,560
5,310
38,800
46,200
33,600
280
151
14,500
28,930
______
612,901

12,000
98,101
612,901

The following additional information as at 31 May 20X9 is available:


(a) Stock as at the close of business was valued at 25,900.
(b) Depreciation for the year ended 31 May 20X9 has yet to be provided as follows:
Property
1% using the straight-line method
Equipment -15% using the straight-line method
(c) Wages and salaries are accrued by 140.
(d) Other operating expenses include certain expenses prepaid by 500. Other
expenses included under this heading are accrued by 200.
(e) The provision for bad debts is to be adjusted so that it is 0.5% of trade debtors
as at 31 May 20X9.

Lesson Five

227

(f) Purchases include goods valued at 1,040, which were withdrawn by Mr


Howell for his own personal use.
Required:
Prepare Mr. Howells trading and profit and loss account for the year ended 31
May 20X9 and his balance sheet as at 31 May 20X9.
(20 marks)
Solution:

Sales
Less cost of sales
Opening stock
Purchases
Less closing stock
Gross profit
Discounts received
Decrease in provision for bad
debts

27,400
258,560
285,960
(25,900)

405,000

(260,060)
144,940
4,420
____49
149,409

Less expenses
Depreciation: Property
Equipment
Discounts
allowed
Wages and
salaries
Bad debts
Loan interest
Carriage out
Other operating
expenses
NET PROFIT

900
8,625
3,370
52,500
1,720
1,560
5,310
38,500

(112,485)
86,924

Herbert Howell
Balance Sheet as at 31 May 2000

Non current Assets


Property
Equipment

90,000
57,500
147,50
0

(13,400)
(41,125)
54,525

Current Assets
Stock
Debtor
Less provision
Prepayments

25,900
46,200
(231)

45,969
500

76,600
16,375
92,975

228

Further Adjustments to Accounts

151
Cash in hand
72,520
Current liabilities
Bank overdraft

14,500

Creditors

33,600

Accruals

__340

(48,440)
24,080
117,055

Capital

98,101

Add net profit

36,924
135,025

Less drawings

(29,975)
105,055

Non current liabilities


Loan (13%)

12,000
117,055

Workings:
1) Depreciation for:
Property
Equipment

1% X 90,000
15% X 57,500

2) Provision for bad debts


0.5% X (46,200)
=231
Decrease in provision for bad debts
280 231= 49
3) Wages and salaries
Paid
52,360
Accruals
140
52,500
4) Other operating expenses
Paid
8,800
Pre-paid
(500)

=
=

900
8,625

Lesson Five

Accruals
5) Purchases:
Drawings:

229

8,300
200
8,500
259,600 1,040
28,930 + 1,040

=
=

258,560
29,990

230

Further Adjustments to Accounts

REINFORCEMENT QUESTIONS
QUESTION ONE
David Dolgellau, a sole trader has prepared the following balance as at 31 March
2001

Sales
Discount Received
Rent Received
Returns outwards
Creditors
Bank Overdraft
Capital
Purchases
Salaries and Wages
Office expenses
Insurance premiums
Electricity
Stationery
Advertising
Telephone
Business Rates
Discounts allowed
Returns Inwards
Stocks as at 1 April 2000
Warehouse, shop and office
Fixtures and fittings
Debtors
Cash in till
Drawings

378,500.00
2,400.00
7,500.00
7,700.00
18,700.00
30,000.00
287,500.00
261,700.00
45,700.00
8,400.00
3,100.00
1,600.00
6,200.00
8,400.00
2,100.00
7,500.00
600.00
4,100.00
120,600.00
210,000.00
12,800.00
13,000.00
500.00
26,000.00

The following further information was obtained:

Closing stock was 102,500.00


Electricity charges accrued 700.00
Advertising expenses accrued 500.00
Insurance premiums paid in advance 900.00
Business rates prepaid 1,500.00

Required:
Prepare a trial balance, trading, profit and loss account for the year ended 31
March 2001 and balance sheet as at that date.

Lesson Five

231

232

Further Adjustments to Accounts

QUESTION TWO
Donald Brown, a sole trader, extracted the following trial balance on 31 December
20X0.
TRIAL BALANCE AS AT 31 DECEMBER 20X0
Debit

Capital at 1 January
20X0
Debtors
Cash In Hand
Creditors
Fixtures and fittings at
cost
Discounts allowed
Discounts received
Stock at 1 January 20X0
Sales
Purchases
Motor Vehicles at cost
Lightning and heating
Motor expenses
Rent
General expenses
Balance at bank
Provision for
depreciation
Fixtures and
fitting
Motor vehicles
Drawings
The following information as

Credit

26,094

42,737
1,411
35,404
42,200
1,304
1,175
18,460
491,620
387,936
45,730
6,184
2,862
8,841
7,413
19,861

_26,568
591,646

2,200
15,292
_______
591,646

at 31 December is also available:

a) 218 is owing for motor expenses.


b) 680 has been prepaid for rent.
c) Depreciation is to be provided of the year as follows:
Motor vehicles: 20% on cost
Fixtures and fittings: 10% reducing balance method
d) Stock at the close of business was valued at 19,926.
Required
Prepare Donald Browns trading and profit and loss account for the year ended 31
December 20X0 and his balance sheet at that date.

Lesson Five

233

QUESTION THREE
The following trial balance has been extracted from the accounts of Brenda Bailey,
a sole trader.
Brenda Bailey
Trial Balance As At 30 June 20x9
Dr

Sales
Purchases
Carriage inwards
Carriage outwards
Wages and salaries
Rent and rates
Heat and light
Stock at 1 July 20X8
Drawings
Equipment at cost
Motor vehicles at cost
Provision for
depreciation:
Equipment
Motor vehicles
Debtors
Creditors
Bank
Sundry expenses
Cash
Capital

Cr

427,726

302,419
476
829
64,210
12,466
4,757
15,310
21,600
102,000
43,270
22,250
8,920
50,633
41,792
3,295
8,426
477
______
626,873

122,890
626,873

The following information as at 30 June 20X9 is also available.


a)
b)
c)
d)

350 is owing for heat and light.


620 has been prepaid for rent and rates.
Depreciation is to be provided for the year as follows:
Equipment - 10% on cost
Motor vehicles - 20% on cost
Stock at the close of business was valued at 16,480

Required

234

Further Adjustments to Accounts

Prepare Brenda Baileys trading and profit and loss account for the year ended
30June 20X9 and her balance sheet at that date.

QUESTION FOUR
On 10 January 19X9, Frank Mercer received his monthly bank statement for
December 19X9. The statement showed the following.
MIDWEST BANK
F Mercer: Statement of Account
Date
Particulars
19X8
Dec 1
Balance
Dec 5
417864
Dec 5
Dividend
Dec 5
Bank Giro
Dec 8
Credit
Dec 10
417866
Dec 11
417867
Dec 14
Sundry Credit
Dec 20
Standing Order
Dec 20
417865
Dec 21
Bank Giro
Dec 21
Credit
Dec 24
417868
Dec 27
416870
Dec 28
Bank charges
Dec 29
Bank Giro
Dec 29
Credit
Dec 31
Direct Debit
417873
Bank Giro
Credit
417871

Debits
$

Credits
$

243
26
212
174
17
32
307

185

95
161
18

118

88
12

47

25

279

Balance
$
1,862
1,619
1,645
1,857
1,683
1,666
1,851
1,819
1,512
1,630
1,535
1,374
1,356
1,403
1,315
1,303
1,582
1,557

Lesson Five

235

His cashbook for the corresponding period was as follows.


CASH BOOK
19x8
Dec 1

Balance
b/d
J
Shannon
M Lipton

Dec 4
Dec 9
Dec
Dec
Dec
Dec
Dec

19
26
27
29
30

G Hurst
M Evans
J Smith
V Owen
K
Walters

$ 19x8
1,862 Dec 1
212 Dec 2
185 Dec 5
118
47
279
98
134

Dec
Dec
Dec
Dec
Dec

6
10
14
16
20

Dec 21
Dec 22
Dec 31
_____

Cheque
No
Electricit
y
P
Simpson
D
Underhill
A Young
T Unwin
B Oliver
Rent
M Peters
L Philips
W
Hamilton
Balance
c/d

$
243

864
865

307

866

174

867
868
869
870
871

17
95
71
161
25

872
873

37
12
1,793
2,935

2,935
Required
a)
b)

Bring the cash book balance of $1,793 up to date as at 31 December 19X8.


(10 marks)
Draw up a bank reconciliation statement as at 31 December 19X8
(5 marks)

236

Further Adjustments to Accounts

CHECK YOUR ANSWERS WITH THOSE GIVEN IN LESSON 9 OF THE


STUDY PACK

Acknowledgement

237

LESSON SIX
OTHER ASPECTS OF FINAL ACCOUNTS
(a)INCOMPLETE RECORDS
An incomplete record situation is whereby, the accounting system falls short of the
double entry. This may be due to:
Lack of records at all; or
Insufficient records that will facilitate the preparation of final accounts.
Reasons for incomplete records:
a) Managers or owners may not have the skills or expertise in preparing and
maintaining an accounting system (records and procedures).
b) It may not be economical for the business to maintain accounting records
due to the volume or/and nature of transactions (small scale businesses)
c) Records are destroyed (e.g. through fire), stolen or misplaced.
There are 4 main approaches in preparing final accounts where there are
insufficient records.
a) Estimating income from the net assets.
b) Estimating income from the use of ratios.
c) Use of a simple cashbook and bank statement.
d) Use of control accounts.
N/B: approach number c and d are normally used together.
(a) Estimating Income from the Net Assets
Where the available records are so deficient (i.e. it is impossible to compile a
reasonable complete cash summary, the only method of estimating the profits or
loss for the period, is to prepare statement of affairs showing the net worth of the
business at the beginning and at the end of the period.
The profit/loss is estimated by use of the following formulas:
Profit or loss = Closing
Capital

Opening
Capital

Drawings

Additional
Capital

Or where there are no non current liabilities then this optional formula can be used
Profit or loss = Closing
Additional
Net Asset

Opening

Net Asset

FINANCIAL ACCOUNTING 1

Drawings

Capital

238

Other Aspects of Final Accounts

Example: 6.1
A sole traders capital position is as follows:
31 December

Motor vehicle:
Cost
Depreciation
Stock
Debtors
Bank
Cash
Creditors
Net assets

19X2

19X3

7,500
3,000
4,500
2,960
1,150
925
__263
9,798
2,860
6,938

7,500
4,500
3,000
3,450
2,060
2,125
___54
10,689
3,340
7,349

He has estimated his drawings for 19X3 at 12,500. Estimate his net profit for the
year.
Solution:
Net profit =
Closing
- Opening
Net Asset
Net Asset

+ Drawings
Net assets

Additional

= 7,349 6,938 + 12,500


= 12,911
(b) Use of Ratios
There are 3 important ratios to be looked at:
1) Gross profit margin
2) Mark up
3) Stock turnover
If a firm has a uniform Gross Profit for all the items sold then any information
available on sales or purchases can be used to derive the total Gross Profit for the
period and incase there is sufficient information on expenses, then the Net Profit
can also be derived.
The above ratios are computed as follows:
1) Gross Profit Margin = Gross Profit x 100
Sales (selling price)
E.g. If the selling price of a unit is 100 and Gross Profit made per unit is 25, the
Gross Profit Margin will be:

Lesson Six

239

= 25 x 100
100
= 25%

240

Other Aspects of Final Accounts

If a firm sells 1,000 units in a financial period, then the Gross Profit will be:
= 25% (100,000)
= 25,000
2) Mark up
= Gross Profit x 100
Cost of Sales (cost price per unit)
In the above example, the mark up will be:
= 25 x 100
75
= 33.33%
N/B: 75 = 100 25
Cost = selling price gross profit
3) Stock Turnover
Measures the rate at which a firm uses its stocks to make sales or turnover.
The formula is:

Average stock

Cost of Sales
Average Stocks
expressed as number of times
=

Opening Stock + Closing Stock


2

Example: A firm has the following data for the period:


Opening stock
Purchases
Closing stock

20,000
300,000
30,000

Required: The Stock Turnover Ratio.


Average Stock = 30,000 + 20,000
2
= 25,000
Cost of sales = (20,000 + 300,000) 30,000
= 290,000

Lesson Six

241

Stock Turnover
= 300,000
25,000
= 11.6 times
Example 6.2
M Jones gives you the following information as at 30 June 2002

Stock 1 July 2001


6,000
Purchases
54,000
Joness mark-up is 50% on cost of goods sold. His average stock during the year
was 12,000. Draw up a trading and profit and loss account for the year ended 30
June 2002.
a) Calculate the closing stock as at 30 June 19X7.
b) State the total amount of profit and loss expenditure Jones must not exceed
if he is to maintain a net profit on sales of 10%.
Solution
a) Average Stock = Opening Stock + closing stock
2
12,000
= 6,000 + C
2
C
= 24,000 6,000
= 18,000
Gross profit = 50%
Cost of Sales = 42,000
Gross Profit = 50%
42,000
Gross Profit = 21,000.
MEMORANDUM TRADING ACCOUNT

Sales
63,000
Less cost of sales
(42,000)
Gross profit
21,000
Expenses
(14,700)
Net profit
6,300
Example 6.3
W Whites business has a rate of turnover of 7 times. Average stock is 12,600.
Trade discount (i.e. margin allowed) is 33% off all selling prices. Expenses are
66 % of gross profit.

242

You are to calculate:


(a) Cost of goods sold.
(b) Gross profit margin.
(c) Turnover.
(d) Total expenses.
(e) Net profit.

Other Aspects of Final Accounts

Lesson Six

243

Solution:
Profit schedule
Turnover
Cost of goods sold
Gross profit
Expenses
Net profit

132,300
88,200
44,100
(29,400)
14,700

Turnover = Cost of Sales


Average stock
Margin = Gross Profit
Sales
7

= Cost of Sales
12,600

Cost of Sales = 88,200


(c) Use of Cashbook and Bank Statement (in addition) Control Accounts.
If there is sufficient information relating to cash payments and receipts, then a
simple cashbook for both cash in hand and cash at bank can be prepared in
confirmation of deposits and payments made from the bank statement.
The information can then be posted to the relevant accounts e.g. any income
received to the relevant income accounts, expenses to relevant expense accounts
and assets and liabilities to relevant accounts.
Information relating to amounts owed to suppliers/creditors and amounts due from
debtors can be posted in summary to the control accounts.
The preparation of the cashbook and control accounts will enable one to estimate
any cash sales or credit sales and cash purchases or credit purchases.
Steps in Preparing the Final Accounts
1)

Prepare a statement of affairs at the beginning of the period (a list of all


assets and liabilities) to determine the beginning capital.
2) Open and post the balances and transactions to these 3 relevant accounts (i.e.
the cashbook (for both cash in hand and bank), sales ledger control account
and purchases ledger control account.
Any other account can be opened where necessary.
3) Make adjustments for any accruals or prepayments.
4) Extract a list of the balances. (Trial balance).
5) Prepare the final accounts.
Example 6.4
Hobbs does not keep proper books of account. You ascertain that his bank
payments and receipts during the year to 31 December 19X8 were as follows:

244

Other Aspects of Final Accounts

Reciepts
Balance 1 Jan
19X8
Cheques for sales
Cash banked
Balance 31 Dec
19X8

Payments

572 Purchases
10,007
13,17 Expenses
9
14,00 Drawings
5
3,751 Delivery van
31,50
7

2,950
11,250
7,300
31,507

From a cash notebook you ascertain:

62

Cash in hand 1 January


19X8
Cash takings
Purchases paid in cash
Expenses paid in cash
Cash in hand 31
December 19X8
Drawings by proprietor in
cash

16,300
1,850
375
65
Unknown

You discover that assets and liabilities were as follows:


1 Jan 19X8
Debtors
Trade creditors
Stock on hand

1,850
1,250
2,650

31 Dec
19X8

2,070
1,420
2,990

Depreciation on the van is to be provided at the rate of 20% per annum.


Statement of Affairs as at 1 January 19x8

CURRENT ASSETS
Cash at bank
Cash in hand
Debtors
Stock
CURRENT LIABILITIES

572
62
1,850
2,650
5,134

Lesson Six

245

Creditors
Net Assets

(1,250)
3,884

Capital

3,884

Balance b/d
Sales

Sales Ledger Control Account

1,850
Cash
16,300
Takings
29,69
Bank
13,179
9
______
Bal c/d
2,070
31,54
31,549
9

246

Other Aspects of Final Accounts

Purchases Ledger Control Account

Cash purchases
1,850 Bal b/d
1,250
Bank
10,00 Purchases
12,027
7
13,27
13,277
7

Cash in Hand Account

Balance b/d
62 Creditors
Debtors/sales
16,30 Expenses
0
Bank
Bal c/d
_____ Drawings
16,36
2

1,850
375
14,005
65
___67
16,362

The capital invested at any point of time in a business by the owner is


represented by the difference between the assets and liabilities at that time.
The difference between the capital at the end and the capital at the
beginning of the trading period represents the trading profit made during
that period, unless there were withdrawals or investments of additional
capital.

Hobbs
Trading and Profit and Loss Account for the year ending 31 December 19X8

Sales
29,699
Less cost of goods sold:
Opening stock
2,650
Add purchases
12,027
14,677
Less closing stock
(2,990)
11,687
GROSS PROFIT
18,012
Less Expenses:
Expenses (375 + 2,950)
3,325
Depreciation
1,460
(4,785)
NET PROFIT
13,227

Lesson Six

Fixed Assets
Delivery van

247

Hobbs
Balance Sheet as at 31 December 19X8

Cost Depreciation
NBV
7,300
1,460
5,840

Current Assets
Stock
Debtors
Cash
Less current liabilities
Creditors
Bank overdraft
Financed by:
Capital
Add net profit
Less drawings (11,250 + 67)

2,990
2,070
___65
5,125
1,420
3,751

5,171
5,794
3,884
13,2
27
17,111
11,317
5,794

Example 6.5 (Exam Type Questions May 2001 Question 3)


Kimeu commenced his business of making furniture on 1 April 2000. Due to his
limited accounting knowledge he has not maintained proper books of account. You
have been engaged to examine his records and prepare appropriate accounts there
from. You perform an examination of the records and from interviews with Kimeu
you ascertain the following information.
1.

2.
3.
4.
5.

6.

At the commencement of business on 1 April 2000, he deposited Sh 1,200,000


into business bank account. On the same day he brought into the firm his
pickup and estimated that it was worth Sh 660,000 and then that from 1 April
2000 it will have useful life of three years.
To increase his working capital he borrowed Sh 400,000 at 15% interest per
annum on 1 July 2000 from his sister but no interest has yet been paid.
On 1 April 2000, Sally was employed as a clerk at a salary of Sh. 720,000 per
annum.
He had drawn Sh 18,000 per week from the business account for private use
during the year.
He purchased timber worth Sh 1,960,000 out of which Sh 158,000 worth of
stock was retained in the workshop on 31 March 2001. He also spent Sh
960,000 on the purchase of some equipment at the commencement of the
business which he estimates will last him five years.
Electricity bills received up to 31 January 2001 were Sh 240,000. Bills for the
remaining two months were estimated to be Sh 48,000. Motor vehicle
expenses were Sh 182,000 while general expenses amounted to Sh 270,000 for

248

Other Aspects of Final Accounts

the year. Insurance premium for the year to 30 June 2001 was Sh 160,000. All
these expenses have been paid by cheque.
7.
8.

9.

Rates for the year to June 2001 were Sh 36,000 but these had not been paid.
Sally sent out invoices to customers for Sh 6,178,000 but only Sh 5,080,000
had been received by 31 March 2001. Debt totaling to Sh 17,000 were
abandoned during the year as bad. Other customers for jobs too small to
invoice have paid Sh 726,000 in cash for work done of which Sh 560,000 was
banked. Kimeu used Sh 75,000 of the difference to pay for his familys
foodstuff, bought Kenya Charity Sweepstake tickets worth 24,000 and Sally
used the rest on general expenses except for Sh 30,100 which was left in the
office on 31 March 2001.
You agree with Kimeu that he will pay you Sh 55,000 for accountancy fee.

Required:
(a) Profit and loss account for the year ended 31 March 2001.
marks)
(b) Balance sheet as at 31 March 2001.
marks)

(10
(10
(Total: 20 marks)

Solution:

Capital
Loan
Debtors
Cash

Bal c/d

Cash
Sh
1,200,00
0
400,000
5,080,00
0
560,000

book Bank
Salary
Drawings
Timber

Equipment
Electricity
Motor vehicle
expenses
General expenses
Insurance
________ Bal c/d
7,240,00
0
Capital
Sh
Bank
1,860,00 Pick up
0
1,860,00
0

Sh
120,000
936,000
1,960,000
960,000
240,000
182,000
270,000
160,000
1,812,000
7,240,000

Sh
1,200,000
660,000
1,860,000

Lesson Six

249

Sales

Sales

Debtors
Sh
6,178,00 Bank
0
Bad debts
________ Bal c/d
6,178,00
0
Cash book - cash in hand
Sh
726,000 Bank
Drawings
Drawings
General Expenses
______ Bal c/d
726,000

Sh
5,080,000
17,000
1,081,000
6,178,000

Sh
5,080,000
17,000
1,081,000
36,900
30,100
726,000

250

Other Aspects of Final Accounts

Loan interest

400,000 x 15% x 9/12

Rates

36,000 x 9/12

Accruals

Electricity bills
Rates
=
Agency fees =
Loan interest

=
48,000
27,000
55,000
=
45,000
175,000

27,000

Kimeu
Profit and Loss Account For the year ended 31 March 2001
Sh
Sh
Sales (cash + credit)
6,904,000
Less expenses
Timber used (1,960,000
1,802,000
158,000)
Depreciation motor vehicle
220,000
- Equipment
192,000
Loan interest
45,000
Salary
720,000
Electricity bills
288,000
Motor vehicle expenses
182,000
General expenses
306,900
Insurance premium
120,000
Rates
27,000
Bad debts
17,000
Accountancy fees
55,000
(3,974,900)
Net profit
2,929,100

Lesson Six

Non current Asset


Equipment
Motor vehicle

251

Kimeu
Balance Sheet as at 31 March 2001
Sh
Sh
960,000
192,000
660,000
220,000
1,620,000
412,000

Current Assets
Stock
Debtors
Insurance prepayments
Cash at bank
Cash in hand

Sh
768,000
440,000
1,208,000

158,000
108,000
40,000
181,200
30,000
3,121,100

Less current liabilities


Accruals

175,000

Capital
Add net profit

2,946,100
4,154,100
1,860,000
2,929,100
4,789,100
1,035,000
3,754,100

Less drawings
Non current liability
Loan 15%

400,000
4,154,100

Example 6.6 (Exam Type) June 1995 Question 2


Abi, a proprietor of a grocery and general store has not previously engaged an
accountant. He informs you that this year his bankers have insisted on a proper
set of accounts. Abi supplies you with his trading results for the year ended 30
June 1994 which are as follows:
Payments for goods
Payments for expenses
Profits

Sh
4,747,50 Takings
0
565,000
152,500
5,465,00
0

Sh
5,465,000
________
5,465,000

Abi instructs you to examine his records and prepare accounts. From your
examination of the records and interview with your client, you ascertain the
following information:
1.

The takings are kept in a drawer under the counter; at the end of each day the
cash is counted and recorded on a scrap of paper; at irregular intervals Mrs.
Abi transcribes the figures into a notebook; a batch of slips of paper was
inadvertently destroyed before the figures had been written into the notebook,

252

Other Aspects of Final Accounts

but Mr. And Mrs. Abi carefully estimated their takings for that period, and the
estimated figure is included in the total of Sh. 5,465,000.
2.

Mr. Abi involved himself in betting for 30 weeks of the year, spending Sh. 500
per week with cash taken from the drawer. His winnings totaled Sh. 29,500.

3.

The following balances are ascertained as correct:


30 June
1994
1993
Sh
Sh
Cash in hand
43,500
22,500
Balance at bank
109,500
78,000
Sales debtors
245,500
229,000
Creditors for purchases of
121,500
139,500
stock
Stock at cost
950,000
975,000

4.

Debts totaling Sh. 178,000 were abandoned during the year as bad; the takings
included Sh 12,500 recovered in respect of an old debt abandoned in the
previous year.
Mr. Abi rents the shop for living accommodation at Sh. 1,500 per week for 52
weeks in a year; the rent is included in expenses of Sh 565,000. The living
accommodation comprises one-third of the building.
The total expenses also include:

5.
6.

7.
8.
9.

10.
11.
12.
13.

Sh. 17,500 running expenses of Abis private car;


Sh. 30,000 for exterior decoration of the whole premises;
Sh. 80,000 for alterations to the premises to enlarge the storage
accommodation.

Mr. Abi takes Sh. 5,000 per week from the business for his wifes personal
expenses. This excludes the amount indicated in note 8.
Mr. Abi draws Sh. 750 per week for cigarettes and beer.
During the year, Mr. Abi bought a secondhand car (not for use in the business)
from a friend; the price agreed was Sh. 175,000, but as the friend owed Mr. Abi
Sh. 33,500 for goods supplied from the business, the difference was settled by
cheque.
An insurance policy for Mr. Abis life matured and realized Sh. 320,500.
Mr. Abi cashed a cheque for Sh. 50,000 for a friend; the cheque was
dishonored and the friend is repaying the Sh. 50,000 by installments. He had
paid Sh. 20,000 by 30 June 1994.
Other private payments by cheque totaled Sh. 48,000 plus a further sum of Sh.
55,000 for income tax.
You are to provide Sh. 21,000 for accountancy fees.

N.B. All receipts and payments of Mr. Abi are made through his business
account.

Lesson Six

253

Required:
(a) Mr. Abis balance sheet for the business at 30 June 1993.
(4 marks)
(b) Mr. Abis profit and loss account for the year ended 30 June 1994.
(12 marks)
(c) Mr. Abis balance sheet for the business at 30 June 1994.
(6 marks)
(Total: 20 marks)

254

Other Aspects of Final Accounts

Solution:
Abi
Balance Sheet as at 30 June 1993
Current Assets
Stock
Debtors
Cash at bank
Cash in hand
Current liabilities
Creditors

Sh
97,500
229,00
0
78,000
22,500
1,304,5
00

Sh

(139,50
0)

1,165,0
00
1,165,0
00

Capital

Balance b/d
Sales ledger control
a/c
Insurance (drawings)
Drawings
Drawings
Debtors

1,165,0
00

Cash at Bank
Sh
78,000 Drawings personal expense
for wife
12,500 Drawings cigarettes and
beer
320,500 Expenses
50,000 Drawings second hand car
20,000 Cash in hand
5,591,00 Drawings friend
0
Creditors
Dishonored cheque
drawings
Drawings
Income tax
________ Balance c/d
6,072,00
0
Cash in Hand

Balance b/d

Sh
22,500

Drawings

Sh
15,000

Sh
260,000
39,000
565,000
141,500
6,500
50,000
4,747,500
50,000
48,000
55,000
109,500
6,072,000

Lesson Six

Drawings betting
Bank

Balance b/d
Bad debts recovered
Credit sales

Bank
Balance c/d

Rent
Motor running
expenses
Decoration
Alterations
Other expenses

255

12,500
6,500
58,500

Balance c/d

Sales Ledger Control


Sh
229,000
Bad debts
12,500
Bank
5,819,00 Drawings
0
Bank
________
Balance c/d
6,060,50
0

43,500
58,500
A/c
Sh
178,000
12,500
33,500
5,591,000
245,500
6,060,500

Purchases Ledger Control A/c


Sh
Sh
4,747,50 Balance c/d
139,500
0
121,500 Credit
4,729,500
purchases
4,869,00
4,869,000
0
Total
78,000
17,500

Expenses
Business
52,000
-

30,000
80,000
359,500
565,000

20,000
80,000
359,500
532,500

Private
26,000
17,500
10,000
_____
53,500

Abi
Trading Profit and Loss Account for the year ended 30 June 1994

Sales
5,819,000
Less cost of sales
Opening stock
975,000
Purchases
4,729,500
57,040,500
Less closing stock
950,000
4,754,500
Gross profit
1,064,500
Less expenses
Rent
52,000
Decoration
20,000
Alterations
80,000

256

Other Aspects of Final Accounts

359,500
Other expenses
Bad debts

165,500

Accountancy fees

21,000

Net profit

Current Assets:
Stock
Debtors
Cash at bank
Cash in hand
Current Liabilities
Creditors
Accruals
Capital
Add net profit
Less drawings

(698,000)
366,500

Abi
Balance Sheet as at 30 June 1994
Cost Depreciatio Book Value
n

950,000
245,500
109,500
43,500
1,348,500
121,500
21,000

(142,500)

1,206,000
1,165,000
366,500
1,531,500
(325,500)
1,206,000

NON PROFIT MAKING ORGANIZATIONS (Club, Associations and Others)


These are some form of organizations that are set up to promote or to cater for the
welfare of the members involved and not to make a profit. These include clubs,
(e.g. football clubs, sports clubs), welfare associations and any other societies
(charitable institutions).
Because these organizations are not trading, the types of accounts to prepare are
different from the ones of trading organizations.
Example:
1.
2.
3.

Instead of a cashbook, the clubs will maintain a receipts and payments which
has similar entries to those of a cashbook.
Instead of profit and loss account, we have an income and expenditure account.
Because the club is not formed by any one owner (has no owner), it is funded
by members contributions, donations, income from investments to get an
accumulated fund instead of capital.

From the income and expenditure account, if the incomes are more than the
expenditures for the period, then the club has a surplus and not a net profit.

Lesson Six

257

If the expenditure is more than incomes, then the club has a deficit and not a loss.
The club may carry out some trading activities on a small scale to finance some of
the clubs activities and incase a firm has a trading activity, then in addition to the
income and expenditure account and the balance sheet, prepare a Bar Trading
Account.

258

Other Aspects of Final Accounts

Format of the Final Accounts


Name
Income and Expenditure Account for the year ended 31 December
Incomes

Profit from trading activities

XX

Subscriptions

XX

Income from investments

XX

Donations

XX

Income from other activities


[dinner dance, raffles,
festivals]

XX
XX

Expenditure
Depreciation

XX

Salaries and wages

XX

Expenses on other activities


[prizes]

XX

Loss from trading activities

XX

All other expenses

XX

SURPLUS/( DEFICIT )

(XX)
XX/(XX)

Lesson Six

BALANCE SHEET
Non current Assets
Buildings
Fixtures, fittings and
equipment
Motor vehicle

259

NAME
AS AT 31 DECEMBER

XX
(XX)
XX
XX
(XX)
XX
XX
XX

Investments
Current Assets
Stocks
Debtors
Prepayments and accrued
income
Cash at bank/hand (receipts +
payments)

(XX)
(XX)

XX
XX
XX

XX
XX
XX
XX
XX

Current liabilities
Creditors
Accrued expenses and prepaid
income
Bank overdraft

XX
XX
XX

(XX)

Accumulated fund balance b/f


Add/less surplus / deficit
Other funds
Life membership fund
Building fund
Education fund

XX
XX
XX

XX
XX
XX
XX/(XX)

XX
XX

Notes To The Above Format:


1. Subscriptions:
These are the amounts received by the club from the members to renew their
membership. It is often paid on an annual basis.

It is income for the club and therefore reported in the income and expenditure
account.
Depending on the policy of a club, any subscriptions due but not received are
shown as accrued income (debtors for subscriptions) in the balance sheet.
Any amounts prepaid are shown as prepaid (creditors for subscriptions).
Some clubs will not report subscriptions as income until it is received in form of
cash.

2. Income from Investments:


Some clubs invest excess cash in the bank (fixed deposit account), shares of limited
companies, treasury bills and any other investment that may be available.

260

Other Aspects of Final Accounts

If the club is investing with no specific intention (i.e a general investment) then
income from this investment should be reported in the income and expenditure
account.
If the investment is for a specific purpose and relates to a specific fund (e.g
building fund) it will not be reported in the income and expenditure account but
credited directly to the fund.

3. Other funds
These are funds set up for a specific purpose and not general. They will be
shown together with the accumulated fund.
Any incomes relating to these funds, will be credited directly to the funds and
any expenses will be taken off from these funds e.g. building fund, education
fund.
Life Membership Fund
Some members may pay some amount to become life members of the club and if
this happens, there may be a need to spread out this income over the expected life
of the members in the club.
Depending on the policy of a club, the following accounting treatment may be
allowed:
i. The full amount is reported in the Income and Expenditure account in the
year it is received and therefore no balance is retained in the life
membership account.
ii. The amount is shown separately in the life membership fund with no
transfer in the Income and Expenditure account and hence no balance in
the life membership account.
iii. To transfer some amounts from the life membership funds to the income
and expenditure account over the expected life of membership to the club.
Example 6.7
The following is the receipts and payments account of the Friendship Club for the
year ended 31 December 19X1:

Balance at bank
31 December
19X0
Entrance fees
Subscriptions:
19X0
19X1
19X2
Bar Sales

102 Bar purchases


42 Wages
25 Rent
305 Heating and
lighting
35 Postage and
stationery
5,227 Insurance

4,434
416
186
128
33
18

Lesson Six

261

Sale of
investments

750 General expenses

46

Payments on
account of new
furniture
Balance at bank,
_____ 31 December
19X1
6,486

450
775
6,486

The following information is also supplied:


(1)

31 December
19X0
Bar stock, at cost
Creditors for bar purchases
Rent due
Heating and lighting
expenses due
Subscriptions due
Insurance paid in advance

31 December
19X1
272
306
18
16
25
5

315
358
36
19
40
7

2) On 31 December 19X0, the club held investments which cost 500. During the
year ended 31 December 19X1, these were sold for 750.
3) Furniture was valued at 300 on 31 December 19X0. On June 19X1, the club
purchased additional furniture at a cost of 520. Depreciation of all
furniture is to be provided for at the rate of 10% per annum.
Required:
(a) Prepare an income and expenditure account for the year ended 31 December
19X1.
(b) Prepare a balance sheet at that date.
Solution:
Friendship Club
Accumulated Fund As at 1.1.19X1
Assets
Stock
Subscriptions due
Insurance prepaid
Investments
Furniture
Balance at bank

272
25
5
500
300
102

262

Other Aspects of Final Accounts

1,204
Liabilities
Creditors
Rent due
Heating and lighting expenses
Accumulated fund

Receipts and
payments
Balance c/d

306
18
16

Creditors

4,434 Balance b/f


358 Purchases
4,792

(340)
864

306
4,486
4,792

Subscriptions
Balance b/d
Income &
expenditure
Balance c/d

25 Receipts &
payments
345

365

35 Balance c/d
405

40
405

Lesson Six

263

Friendship Club
Bar, Trading Account for the year ended 31 December 19X1

Sales

5,227

Less: Cost of Sales


Opening stock
Purchases

272
4,486
4,758

Less closing stock


Gross profit to income &
expenditure a/c

(315)

(4,443)
784

Friendship Club
Income and Expenditure Account for the year ended 31 December 19X1

Profit from bar trading


784
Entrance fees
42
Subscriptions
345
Profit from sale of investments
250
1,421
Expenditure
Wages
416
Rent
204
Heating and lighting
131
Postage and stationery
33
Insurance
16
General expenses
46
Depreciation furniture
56
(902)
Surplus
519

264

Other Aspects of Final Accounts

Friendship Club
Balance Sheet as at 31 December 19X1

Non current Assets


Furniture
820
(56)
Current Assets
Stock
Subscriptions due
Prepaid expense
Cash at bank
Current liabilities
Creditors
Prepaid subscriptions
Accrued expenses
Creditors fixtures
Accumulated fund b/f
Add surplus

764

315
40
7
775
1,137
398
35
55
70

(518)

619
1,383
864
519
1,383

Example 6.8 (Exam Type) November 2001


(a) State and briefly explain any three distinguishing features between (i) a
receipts and payments account and (ii) an income and expenditure account.
(6 marks)
(b) The accountant of Mamba Sports Club has extracted the following information
from the books of account for the year ended 31 March 2001.

Lesson Six

265

Receipts

Payments
Sh

Balance brought
forward

288,000 Salaries and wages

Subscriptions
Year: 1999/2000
2000/2001
2001/2002

Sh

New equipment

254,000
565,000

249,000 Repairs and


maintenance

124,000

2,050,000 Office expenses

415,000

194,000 Printing and


stationery

168,000

Dinner dance

723,000 Purchase of
beverages

497,000

Beverage sales

657,000 Dinner dance


expenses

315,000

Investments income

400,000 Refund of
subscriptions

45,000

Sports prizes

25,000

Transport
Investments
_______ Balance carried
forward

248,000
1,500,000
_405,000

4,561,000

4,561,000

31 March
2000

31 March
2001

Furniture and fittings


(net)

240,000

Equipment (net)

690,000

Balances as at

266

Other Aspects of Final Accounts

3,500,000

Subscriptions in
arrears

300,000

375,000

Salaries accrued

68,000

72,000

162,000

184,000

85,000

Investment at cost

Stock of beverages
Subscriptions in
advance
Additional information:
1.
2.
3.

Subscriptions in arrears are written-off after twelve months.


Depreciation is provided for on reducing balance method at 10% and 20% per
annum on furniture and fittings and equipment respectively.
Investments, which had cost Sh. 500,000, were sold on 30 March 2001 for Sh.
625,000. No entries have been made in the books in this respect.

Required:
(a) Income and expenditure account for the year ended 31 March 2001.
(8 marks)
(b) Balance sheet as at 31 March 2001.
(6
marks)
(Total: 20 marks)
Solution:
Mamba Sports Club
Statement of Affairs
Assets

Sh

Sh

Furniture and fittings

240,000

Equipment

690,000

Receipts and payments

288,000

Investment at cost

3,500,000

Subscriptions in arrears

300,000

Stock of beverages

162,000

Lesson Six

267

5,180,000
Liabilities
Subscriptions accrued

85,000

Accrued salaries

68,000

(153,000)
5,027,000

268

Other Aspects of Final Accounts

Mamba Sports Club


Trading Account for the year ended 31.3.2001
Sh
Sales
Less cost of sales
Opening stock
162,000
Purchases
497,000
659,000
Less closing stock
(184,000)
Profit to income and
expenditure
2001
Balance b/d
Receipts and
payments
Income &
expenditure
Balance c/f

Subscriptions
Sh 2001
300,000 Balance b/f
45,000 Receipt and
payment
2,465,000 Income &
expenditure
194,000 Balance c/f
3,004,000

Sh
657,000

(475,000)
182,000

85,000
2,493,00
0
51,000
375,000
3,004,00
0

Mamba Sports Club


Income and Expenditure Account for the year ended 31 March 2001
Incomes
Profit from trading account
Subscriptions
Dinner dance
Investment income
Profit on sale of investments

Sh
182,000
2,465,000
723,000
400,000
125,000
3,895,000

Lesson Six

269

Example 6.9 (Exam Type) DECEMBER 2000 QUESTION 3


The following trial balance was extracted from the books of Literary and
Philosophical Society as at 30 September 2000:
Sh
Sh
Balance at bank: current account
724,800
Accumulated fund 1 October 1999
5,771,200
Land and buildings, at cost
3,700,000
Debtors for subscription
62,000
Furniture and fittings
1,874,000
Provision for depreciation of
284,000
furniture & fittings
Subscriptions
1,450,800
Lecturers fees
920,000
Lecturers travel and accommodation
358,000
expenses
Donations
108,000
Camera and projector repairs
17,000
Projectors, cameras and audio
190,400
equipment
Depreciation of equipment
54,400
Rates and water
277,000
Lighting and heating
367,200
Rental of rooms
495,000
Wages Caretaker
880,000
- Restaurant
1,600,000
- Bar staff
800,000
Purchase of food
1,565,800
Stock bar 1 October 1999
473,600
Bar receipts
4,032,000
Bar purchases
2,842,000
Restaurant receipts
3,642,000
Loan
1,600,000
Deposit account bank
1,000,000
Interest payable and receivable
36,000
Creditors for bar and food
________
178,400
17,651,800
17,651,800
Additional information:
1. The bar stock was valued at Sh. 642,800 as at 30 September 2000.
2. It is expected that, of the debtors for subscriptions, Sh. 43,600 will not be
collectable.
3. The interest account is net. The loan is at a concessional rate of 4% while 10%
has been earned on the deposit account. No changes have taken place all year
in the principal sums involved.
4. An invoice for Sh. 43,000 of wine had been omitted from the records at the
close of the year although the wine had been included in the bar stock
valuation.
5. Depreciation for the year is to be provided as follows:
Furniture and fittings Sh. 194,000
Projectors, cameras, etc. Sh. 19,000.

270

Other Aspects of Final Accounts

Required:
(a) Bar and restaurant trading account for the year ended 30 September 2000
(6 marks)
(b) An income and expenditure account for the year ended 30 September 2000
(8 marks)
(c) A balance sheet as at 30 September 2000
(6
marks)
(Total: 20 marks)

Lesson Six

271

Solution:
Literary and Philosophical Society
Bar and Restaurant Trading Account for the year ended 30 September
2000
Sh
Sales
Less cost of sales
Opening stock
Add purchases
Less closing stock
Profit to the income and
expenditure

Sh
7,674,000

473,600
4,450,800
4,924,400
(642,800)

(4,281,600)
3,392,400

Literary and Philosophical Society


Income and Expenditure Account for the year ended 30 September 2000
Income
Profit on trading account
Interest on bank deposit
account
Subscriptions
Donations
Rental of rooms
Expenditure
Lecturers fees
Depreciation on furniture and
fitting
Equipment
Lecturers travel and
accommodation exp.
Camera repairs
Rates and water
Lighting and heating
Caretakers wages
Interest on loan
Provision for subscription
Surplus

Sh

Sh
992,400
100,000
1,450,000
108,000
495,000
3,146,200

920,000
194,000
19,000
358,000
17,000
277,000
867,200
880,000
64,000
43,600

(3,139,800)
6,400

272

Other Aspects of Final Accounts

Literary and Philosophical Society


Balance Sheet as at 30 September 2000
Non current Assets
Land and buildings
Fixtures and fittings
Equipment
Current assets
Stock
Debtors of subscription
Balance at bank deposit a/c
- Current a/c
Current liabilities
Creditors
Accumulated fund b/f
Add surplus
Non current liabilities
4% loan

Sh
3,700,000
1,874,000
190,400
5,764,400

Sh
(478,000)
(73,400)
551,400

Sh
3,700,000
1,396,000
117,000
5,213,000

642,800
18,400
1,000,000
724,800
2,386,000
(221,400)

2,164,600
7,377,600
5,771,200
___6,400
5,777,600
1,600,000
7,377,600

(c ) Manufacturing Accounts
Some firms may manufacture or produce goods rather than buy due to savings in
operational costs. (i.e. it is cheaper to produce the goods rather than buy).
Due to additional costs involved in the production process, additional information
is reported in the final accounts.
Therefore, in addition to a trading, profit and loss account, a new account called
manufacturing account is shown before these others.
The purpose of the manufacturing account is to report all the costs incurred in
producing the goods. These costs are divided into 2 classes:
1) Direct costs (prime costs)
2) Indirect costs (overheads)
Direct Costs/Prime Costs
This is a cost that can be traced directly to a unit that has been produced. This
include
1) Direct material
2) Direct labour (wages)
3) Direct expense
Indirect costs/Production overheads
These are all other costs incurred in the production of manufacturing of goods but
cannot be traced directly to any particular unit. Example:
1) Rent for the factory

Lesson Six

273

2) Salaries to supervisors and factory managers


3) Depreciation of plant and machinery used in production
The manufacturing account will show the factory cost of goods produced that will
be shown in the trading account in place of purchases.

274

Other Aspects of Final Accounts

FORMAT
Name
Manufacturing Trading Profit and Loss Account for the year ended 31
December

Raw Materials
Opening stock of raw materials
XX
Purchases of raw materials
XX
Add carriage inwards
XX
XX
Less returns outwards
(XX)
XX
Cost of raw materials available for use
XX
Less closing stock of raw materials
(XX)
Raw materials consumed
XX
Direct labour (factory wages)
XX
Direct expenses
XX
Prime cost
XX
Factory overheads
Salary to factory manager
XX
Depreciation on Plant and machinery
XX
- Factory buildings
XX
Other expenses Factory power
XX
Lighting and heating
XX
Water
XX
Cleaners wages
XX
XX
Total cost of production
XX
Add: opening Work In Progress
XX
Less: closing Work In Progress
(XX)
XX
Factory cost of production (cost of
XX
Note 1
finished goods)
FACTORY PROFIT
XX
Finished goods at a transfer price
XX
Note 2
Sales
Less returns inwards
Less cost of sales
Opening stock finished goods
Factory cost of production/transfer price
Less closing stock of finished goods
Gross profit
Add factory profit
Other incomes discount received
- Profit on disposal

XX
(XX)
XX
XX
XX
XX
(XX)

(XX)
XX
XX
XX
XX
XX

Lesson Six

275

Less expenses
Salaries and wages administration &
non production
Rent for administration building
Depreciation - Delivery vans
- Fixtures and distribution
Other selling and distribution costs
Net profit/(net loss)

XX
XX
XX
XX
XX

(XX)
XX/(XX)

For the balance sheet, the format is the same for all the assets and liabilities
except for the current assets section whereby the stock at the end of the period
should be shown for each type of stock as per this format:
Current Assets
Stock: raw materials
Work in progress
Finished goods

XX
XX
XX

XX

Note 1: This represents the total costs of all the units produced during the period
and therefore will be taken to the trading account as the goods are transferred to
the selling department.
Note 2: If the firm transfers the goods to the selling department at a price higher
than the cost of production, then this generates a factory profit. The goods will be
shown in the trading account at the transfer price and the factory profit is added to
the Gross Profit of the period.
Expenses can also be classified into:
1) Administration Expenses
These are expenses incurred in running or managing the affairs of the firm and
includes managers salaries (not factory managers), legal and accounting fees,
depreciation of furniture and fixtures and equipment not used in production,
finance cost e.g. loan interest.
2) Selling and Distribution
These are expenses incurred to generate sales income e.g.

Salaries and commission to the sales manager and staff


Carriage outwards (i.e. to deliver goods to the customers
Depreciation on motor vehicles (used for the delivery purpose)
Advertising
Bad debts

276

Other Aspects of Final Accounts

Example 6.10
B spikes
Trial Balance as on 31 December 2002
Stock of raw materials 1.1.2002
Stock of finished goods 1.1.2002
Work in progress 1.1.2002
Wages(direct 180,000: factory
indirect145,000)
Royalties
Carriage inwards (on raw materials)
Purchases of raw materials
Productive machinery (cost
280,000)
Accounting machinery (cost 20,000)
General factory expenses
Lighting
Factory power
Administrative salaries
Sales representatives salaries
Commission on sales
Rent
Insurance
General administration expenses
Bank charges
Discounts allowed
Carriage outwards
Sales
Debtors and creditors
Bank
Cash
Drawings
Capital as at 1.1.2002

Dr
21,000
38,900
13,500
325,000

Cr

7,000
3,500
370,000
230,000
12,000
31,000
7,500
13,700
44,000
30,000
11,500
12,000
4,200
13,400
2,300
4,800
5,900
142,300
56,800
1,500
20,000
______
1,421,800

1000,000
125,000

29,680
1,421,800

Notes at 31.12.2002
1.
2.
3.

Stock of raw materials 24,000, stock of finished goods 40,000, work in


progress 15,000.
Lighting, and rent and insurance are to be apportioned: factory 5/6ths,
administration 1/6th.
Depreciation on productive and accounting machinery at 10 per cent per
annum on cost.

Required:

Lesson Six

277

Prepare a manufacturing, Trading Profit and Loss Account for the year ended 31
December 2002.

278

Other Aspects of Final Accounts

Solution:
B Spikes
Manufacturing, Trading Profit and Loss Account for the year ended 31
December 2002
Raw Materials

Opening Stock of raw materials


21,000
Purchases
370,000
Carriage inwards on raw materials
3,500
373,500
394,500
Less: closing stock of raw materials
(24,000)
Raw materials consumed
370,500
Direct wages
180,000
PRIME COST
550,500
Factory Overheads
Wages
145,000
Royalties
7,000
Depreciation: productive machinery
28,000
General factory expenses
31,000
Lighting( 5/6 x 7,500)
6,250
Factory power
13,700
Rent(5/6 x 12,000)
10,000
Insurance( 5/6 x 4,200 )
3,500
24,4,450
Total cost of production
794,950
Add: opening work in progress
13,500
808,450
Less: closing work in progress
(15,000)
Factory cost production per finished
793,450
goods
Sales
1,000,000
Less cost of sales
Opening stock of finished goods
38,900
Factory cost of production
793,450
832,350
Less closing stock of finished goods
(40,000)
792,350
Gross profit
207,650
Expenses
Accounting machinery depreciation
2,000
Lighting (1/6 x 7,500)
1,250
Administrative salaries
44,000
Sales representatives salaries
30,000
Commission on sales
11,500
Rent ( 1/6 x 12,000)
2,000
Insurance (1/6 x 4200)
700
General administrative expenses
13,400
Bank charges
2,300

Lesson Six

279

Discounts allowed
Carriage outwards
Net profit

4,800
5,900

(117,850)
89,800

B Spikes
Balance Sheet as at 31 December 2002
COST
Non current Assets
Productive machinery
Accounting machinery
Current Assets
Stock: raw materials
Finished goods
Work in progress
Debtors
Cash at bank
Cash in hand
Current liabilities
Creditors
Capital
Add net profit
Less drawings

280,000
20,000
300,000
24,000
40,000
15,000

DEPRECIATIO
N

(78,000)
(10,000)
88,000

NET BOOK
VALUE

202,000
10,000
212,000

79,000
142,300
56,800
1,500
279,600
(125,000)

154,600
366,600
296,800
89,800
386,600
(20,000)
366,600

280

Other Aspects of Final Accounts

Example 6.11 (Exam Type June 1986 Question Two)


Bibi Maridadi owns and manages a small manufacturing business. The following
balances have been extracted from her books of account at 31 January 1986:
Dr
Sh
Capital at 1 February 1985
Accounts payable
Bank and cash balance
Accounts receivable
Drawings
Administration expenses
Advertising expenses
Factory direct wages
Factory indirect wages
Factory power
Furniture and fittings (all offices)
Heat and light
Plant and equipment
Motor vehicle (used by salesmen)
Plant hire
Provision for bad debts
Provision for depreciation 1 February
1985:
Furniture and fittings
Plant and equipment
Motor vehicle
Raw material purchases
Rent rates
Sales
Selling and distribution expenses
Inventories at cost, 1 February 1985:
Raw materials
Work in progress
Finished goods

Cr
Sh
171,120
86,000

5,400
92,000
60,000
150,360
12,000
60,000
24,000
36,000
18,400
16,000
276,800
144,000
4,000
3,200
9,200
138,400
24,000
228,000
20,000
829,440
66,400
8,000
16,000
24,000
1,261,360

_______
1,261,360

The following additional information is provided:


(i) Accruals at 31 January 1986 were:
Factory power
Sh.1,600
Rent and rates
Sh. 4,000
There was also prepayment of Sh. 800 for salesmens motor vehicle insurance.
(ii) Inventories at 31 January 1986, were valued at cost as follows:

Lesson Six

281

Raw materials
Work in progress
Finished goods

Sh. 15,200
Sh. 30,400
Sh. 45,600

(iii) Depreciation is to be charged on plant and equipment, motor vehicle,


furniture and fittings at the rates of 20%, 25% and 10% per annum
respectively on cost.
(iv) Expenditure on heat and light, and rent and rates is to be apportioned
between the factory and office in the ratio of 9 to 1 and 3 to 2 respectively.
(v) The provision for bad debts is to be made equal to 5% of accounts receivable
at 31 January 1986.
Required:
Using the vertical method, prepare Bibi Maridadis manufacturing, trading and
profit and loss account for the year ended 31 January 1986 and a balance sheet as
at that date. (22 marks)
Solution:
Bibi Maridadi
Manufacturing, Trading and Profit and Loss Account for the year ended 31
January 1986
Sh
Sh
Direct materials
Opening stock of raw materials
8,000
Add: purchases of raw materials
228,000
236,000
Less: closing stock of raw materials
(15,200)
Raw materials consumed
220,800
Factory direct wages
60,000
PRIME COST
280,800
Factory overheads
Factory indirect wages
24,000
Factory power
37,600
Plant hire
4,000
Heat and light
14,400
Rent and rates
14,400
Depreciation on plant
55,360
149,760
430,560
Add opening work in progress
16,000
446,560
Less closing work in progress
(30,400)
Factory cost of production
416,160
Sales
829,440
Less cost of sales
Opening stock of finished goods
24,000
Add factory cost of production
416,160
440,160
Less closing stock of finished goods
(45,600)
394,560
Gross profit
434,880
Less expenses
Increase in provision for doubtful
1,400

282

Other Aspects of Final Accounts

debts
Rent and rates

9,600

Heat and light

1,600

Depreciation: motor vehicle

36,000

Furniture and fittings

1,840

Selling and distribution expenses

65,600

Administration expenses

150,360

Advertising expenses

12,000

Net profit

278,400
156,480

Bibi Maridadi
Balance Sheet as at 31 January 1986
COST
Non current Assets
Plant and equipment
Furniture and fittings
Motor vehicle
Current Assets
Stock: Raw materials
Work in progress
Finished goods
Debtors
Less: provision for doubtful
debts
Prepayments
Cash in hand and bank
Current liabilities
Creditors
Accruals
Capital
Add net profit
Less drawings

276,800
18,400
144,000
439,200
15,200
30,400
45,600
92,000
(4,600)

DEPRECIATIO
N

(193,760)
(11,040)
(60,000)
(264,800)

NET BOOK
VALUE

83,040
7,360
84,000
174,400

91,200
87,400
800
5,400
184,800

86,000
5,600

(91,600)

93,200
267,600
171,120
156,480
327,600
(60,000)
267,600

Lesson Six

283

UNREALISED PROFITS ON CLOSING STOCK


In most cases where business transfers finished goods at a profit to the selling
department and the goods are reflected in the balance sheet at the transfer price,
then the closing stock includes a profit that has not been earned or realised. If the
mark up profit (the profit based on cost of production is always uniform, then any
changes in the value of closing stock will result in a reduction or an increase in the
unrealised profits.
If there is an increase on unrealised profit on the closing stock, then this increase
will be reduced from the gross profit from our profit and loss account and if there
is a reduction in unrealised profits, then this reduction will be added to the gross
profit in our profit and loss account.
Any unrealised profit of closing stock should be deducted from the closing stock in
the balance sheet.
The slight change in the format of the Profit and Loss Account and Balance Sheet
will be as follows

284

Other Aspects of Final Accounts

Increase in unrealised profit in closing stock (UPCS)


Profit and loss (extract) Account for year ended..

Gross profit
Add: factory profit
Add: other expenses
Less expenses
Other expenses
Increase in unrealised profit on
closing stock
Net profit

X
X

X
X
X
X
(X)
X

Decrease in UPCS
Profit and Loss Account (extract) for year ended

Gross profit
Add: factory profit
Add: other incomes
Add: decrease in UPCS
Less expenses
Other expenses
Net profit

X
X
X
X
X
(X)
X

Example:
A firm always values its stock (finished goods) at a mark-up of 20% on cost of
production. The opening stock of finished goods for the period was valued at Sh.
100,000. (The marked up cost) The closing stock at the end of the financial period
was Sh.160,000.
Opening Stock:
100,000 (marked up)
=
120%
(16,667)
=
(20%)
83,333
=
100%
Closing Stock
160,000 (marked up)
=
120%
(26,667)
=
(20%)
133,333
=
100%

Balance
c/d

UPCS
Balance b/f
26,667 Profit and loss
a/c
26,667

16,667
10,000
26,667

Lesson Six

285

286

Other Aspects of Final Accounts

Profit and Loss (Extract)


Less: Expenses:
Sh
Increase in unrealized profits on closing stock

Sh
10,000

Balance Sheet (Extract)


Current Assets
Stock:
Raw materials
Work in progress
Finished goods
Less: UPCS

Sh

Sh

X
X
160,000
(26,667)

133,333

(d) DEPARTMENTAL ACCOUNTS


Some organizations have various departments carrying out trade and therefore the
profitability of each department needs to be established. For each department,
trading, profit and loss account should be prepared. The final accounts will be
very important for the management to assess the performance of each department.
The expenses in relation to a specific department should be charged in the Profit
and Loss account for that department. The accounts will be represented in
columnar form and the format will be as follows: (Assume a firm has departments
A and B).
Name
Trading Profit and Loss account for the year ended 31 December

Sales
Less cost of sales
Opening stock
Purchases
Less closing stock
Gross profit
Other incomes
Less expenses
Salaries and wages
Depreciation
Other expenses
Managers
commission
NET PROFIT

Department A

XX
XX
XX
XX
(XX)

XX
XX
XX
XX

(XX)
XX
XX
XX

(XX)
XX

Department B

XX
XX
XX
XX
(XX)

XX
XX
XX
XX

(XX)
XX
XX
XX

(XX)
XX

Department C

XX
XX
XX
XX
(XX)
XX

XX
XX
XX
XX

(XX)
XX
XX
XX

(XX)
XX

Lesson Six

287

The balance sheet will reflect the position of the whole organization and therefore
a departmental balance sheet is not required.

When departments in a firm are sharing resources, then the various expenses need
to be apportioned between or among the different departments e.g. if the
departments are sharing a building, then rent expense should be apportioned
among the departments.
The following guidelines can be followed in apportioning the expenses among the
departments.
Type of Expense
1) Rent, rates, heat, light, repairs
to buildings,
depreciation of buildings and
insurance.

Basis of apportionment
Floor area occupied by each
department.

2) Depreciation, insurance and


maintenance of equipment

Cost or net book value of the


equipment in each department.

3) Salaries, canteen expenses,


welfare and other expenses
relating to employees.

Number of employees in each


department

4) Carriage inwards.

Purchases in each department.

5) Advertising, depreciation and


maintenance of delivery van.

Value of sales in each department.

6) Increase in provision for


doubtful debts, bad debts and
discounts allowed.

Sales or debtors in each


department.

Example 6.12
J Spratt is the proprietor of a shop selling books, periodicals, newspapers and
childrens games and toys. For the purposes of his accounts, he wishes the
business to be divided into two departments:
Department A
Department B

Books, periodicals and newspapers


Games, toys and fancy goods.

The following balances have been extracted from his nominal ledger at 31 March
19X9:
Dr
Sales Department A
Sales Department B

Cr
15,000
10,000

288

Other Aspects of Final Accounts

250
Stocks Department A, 1 April
19X8
Stocks Department B, 1 April
19x8

200

Purchases Department A
Purchases Department B
Wages of sales assistants
Department A
Wages of sales assistants
Department B
Newspaper delivery wages
General office salaries
Rates
Fire insurance buildings
Lighting and air conditioning
Repairs to premises
Internal telephone

11,800
8,200
1,000

Cleaning
Accountancy and audit charges
General office expenses

30
120
60
25,000

750
150
750
130
50
120
25
25

25,000

Stocks at 31 March 19X9 were


valued at:
Department A 300
Department B 150
The proportion of the total floor area occupied by each
department was:
Department A one fifth
Department B four-fifths
Prepare J Spratts trading and profit and loss account for the year ended 31 March
19X9, apportioning the overhead expenses, where necessary, to show the
Department profit or loss.
The apportionment should be made by using the methods as shown:
Area Rates, Fire insurance, Lighting and air conditioning, Repairs,
Telephone, Cleaning:

Lesson Six

Turnover -General office salaries, Accountancy, General office expenses.

289

290

Other Aspects of Final Accounts

Solution:
J Sprat
Trading, Profit and Loss Account for the year ended 31 March 19X9

Sales
Less cost of sales
Opening stock
Purchases
Less closing stock

Department A

15,000
250
11,800
12,500
(300)

Gross profit
Less expenses
Wages
Newspaper delivery
wages
General office salaries
Rates
Fire insurance
buildings
Lighting and airconditioning
Repairs to premises
Internal telephone
Cleaning
Accountancy or audit
charges
General office
expenses
NET PROFIT

(11,750
)
3,250

Department B

10,000
200
8,200
8,400
(150)

(8,250)

Department C

25,000
450
20,000
20,450
(450)

1,750

1,000
150

750
-

1,750
150

450
26
10

300
104
40

750
130
50

24

96

120

5
5
6
72

20
20
24
48

25
25
30
120

36

(1,784)

24

(1,426)

1,466

Workings:
1) General Office Salaries:

A = 15,000 X 750 = 450


25,000

B = 10,000 X 750 = 300


25,000
2) Rates:

A = 1/5 X 130 = 26
B = 4/5 X 130 = 104

3) Fire Insurance:

A = 1/5 X 50 = 10
B = 4/5 X 50 = 40

324

60

(20,000
)
5,000

(3,210)
1,790

Lesson Six

291

4) Lighting:

A = 1/5 X 120 = 24
B = 4/5 x 120 = 96

5) Repairs:

A = 1/5 X 25 = 5
B = 4/5 X 25 = 20 etc.

292

Other Aspects of Final Accounts

Interdepartmental Trading
A department may buy goods from another department in the same firm and
therefore the departments trade with one another. Example, in 4.16 above,
department A sells goods to Department B. (Department B is buying from
department A). Interdepartmental sales and purchases should be excluded from
the total sales and total purchases of the whole firm. If we assume that A sold
goods to B amounting to 1,000 and this figure is included in sales of A and
purchases of B, the trading account for the whole firm will be as follows (other
items will remain the same):

Sales
24,000
Less cost of sales
Opening stock
450
Purchases
19,000
19,450
Less closing stock
(450)
(19,000)
Gross profit
5,000
Managers Commission
A commission based on the net profit made in each department may reward
managers of each department.
The commission is normally a percentage of the net profit but it may be a
percentage on the net profit before or after charging the commission.
1)

Percentage Before Charging Commission

If we assume in example 4.16 that the managers in each department is paid a


commission of 5%, before charging such commission, the commission will be as
follows:
Net profit before
commission
Managers commission
@ 5%
Net profit after
commission

Department A
1,466

Department B
324

Total
1,790

(73.3)

(16.2)

(89.5)

1,392.7

307.8

1,700.5

2) Percentage After Charging Commission


Assume the commission is 5% of the net profit after charging such commission:
Net profit before
commission
Managers commission

Department A
1,466

Department B
324

Total
1,790

(69.8)

(15.4)

(85.2)

Lesson Six

@ 5%
Net profit after
commission

293

1,392.2

308.6

1,704.5

Note:
If we use percentage for each commission assuming a 5% rate, the commission will
be computed as follows:

Net profit before


commission
Commission of 5%
Net profit after
commission

Before
charging
commission
100

After charging
commission

__5
95

__5
100

105

REINFORCEMENT QUESTIONS
QUESTION ONE
Dare is a grocer who had not kept a full set of books.
The following was a summary of his bank statement for the year ended 31
December 19X6:
Amounts credited by
bank

32,050 Balance at 1 January


19X6
Payments to trade
creditors
Rent and rates
Fixtures
Lighting and heating
General expenses
Loan interest
Drawings
Customers cheques
dishonoured
_____ Balance at 31 December
19X6
32,050

892
27,380
475
100
210
800
120
900
180
993
32,050

You are given the following information:


1.

Trading receipts consisted partly of cash and partly of cheques. During the
year, Dare had paid, out of his takings, wages for part-time staff amounting to
2,914 and sundry expenditure of 140. He retained between 2 and 5 per
week pocket money and maintained a balance of 20 in the till for change. The

294

2.
3.
4.

5.
6.

Other Aspects of Final Accounts

balance of his takings, together with cheques amounting to 250, which he had
cashed out of his takings for the convenience of certain friends, was paid into
the bank.
Cheques drawn payable to trade creditors, but not presented at 1 January
19X6, amounted to 280 and at 31December 19X6 to 320.
All dishonoured cheques were re-presented and honoured during the year.
The loan interest was paid to a close friend, Bryant, who had lent Dare 4,000
some years ago at a nominal rate of interest of 3% per annum. The interest
was duly paid half-yearly on 31st March and 30 September, and the loan was
still outstanding at the end of the year.
Discounts allowed by trade creditors amounted to 480 and those allowed to
debtors were 520.
Other balances at 1 January and 31 December 19X6 are given below:
1 January

Stocks
Trade debtors
Accrued general
expenses
Rates paid in advance
Fixtures valued at
Trade creditors
Creditors for heating
and lighting
7.

4,500
2,800
240
40
2,800
1,800
80

31
December

5,800
3,200 (including a bad debt of
200 to be written off)
190
50
2,550 (including those
purchased during the
year)
2,200
70

There is a standard gross profit margin of 25% on sales.

You are required to prepare:


(a) a statement of Dares capital on 1 January 19X6;
(b) a profit and loss account for the year ended 31 December 19X6;
(c) a balance sheet as on that date.
QUESTION TWO
You have agreed to take over the role of bookkeeper for the AB Sports and Social
Club.
The summarized balance sheet on 31.12.94 as prepared by the previous
bookkeeper contained the following items. All figures are in s.
Assets
Heating oil for clubhouse
Bar and caf stocks
New sports ware, for sale, at cost
Used sports ware, for hire, at

1,000
7,000
3,000
750

Lesson Six

valuation
Equipment for grounds person
cost
depreciation
Subscriptions due
Bank current account
- deposit account
Claims
Accumulated fund
Creditors bar and caf stocks
- Sports ware

295

5,000
3,500

1,500
200
1,000
10,000
23,150
1,000
300

296

Other Aspects of Final Accounts

The bank account summary for the year to 31.12.95 contained the
following items:
Receipts:
Subscriptions
Bankings bar and sale
Sale of sports ware
Hire of sports ware
Interest on deposit account
Payments
Rent and repairs of clubhouse
Heating oil
Sports ware
Grounds person
Bar and caf purchases
Transfer to deposit account

11,000
20,000
5,000
3,000
800
6,000
4,000
4,500
10,000
9,000
6,000

You discover that the subscriptions due figure as at 31.12.94 was


arrived at as follows:
Subscriptions unpaid for 1993
Subscriptions unpaid for 1994
Subscriptions paid for 1995

10
230
40

Corresponding figures at 31.12.95


are:
Subscriptions
Subscriptions
Subscriptions
Subscriptions

unpaid for 1993


unpaid for 1994
unpaid for 1995
paid for 1996

10
20
90
200

Subscriptions due for more than 12 months should be written off with
effect from 1.1.95
Asset balances at 31.12.95 include:
Heating oil for club house
Bar and caf stocks
New sports ware, for sale, at cost
Used sports ware, for hire, at
valuation

700
5,000
4,000
1,000

Closing creditors at 31.12.95 are:


For bar and caf stocks

800

Lesson Six

For sports ware


For heating oil for clubhouse

297

450
200

/3 rds of the sportswear purchases made in 1995 had been added to stock of new
sportswear in the figures given in the list of assets above, and 1/3 had been added
directly to the stock of used sportswear for hire.
Half of the resulting new sportswear for sale at cost at 31.12.95, to transfer these
older items into the stock of used sportswear, at a valuation of 25% of their original
cost.
No cash balances are held at 31.12.95. The equipment for the grounds person is to
be depreciated at 10% per annum, on cost.
Required:
Prepare income and expenditure account and balance sheet for the AB Sports club
for 1995, in a form suitable for circulation to members. The information given
should be as complete and informative as possible within the limits of the
information given to you. All workings must be submitted.
(23 marks)
QUESTION THREE
Mr Cherono trades as a retailer of electric lamps and related products under the
name of Chero Hardware. Most goods in which he trades are purchased from
various suppliers in a finished form. In addition, a separate department of the firm
manufactures various types of lampshades from purchased raw materials. When
finished, the lampshades are transferred to the shop at an agreed transfer price for
sale. No lampshades are sold other than through the shop.
The firms Accounts Assistant presents you with the following trial balance at 30
June 1988:
Sh
Sh
Capital account Cherono
740,000
Drawings Cherono
95,000
Long term loan (interest at 15%
240,000
p.a)
Fixtures and fittings at cost
900,000
Accumulated depreciation at 1 July
350,000
1987
Motor vehicle at cost
208,000
Accumulated depreciation at 1 July
60,000
1987
Stock at 1 July 1987 (at cost):
Raw materials for lampshades
40,000
Completed lampshades
20,000
Other goods
328,000
Trade debtors and creditors
122,000
Bank balance
98,000
Sales
4,100,000

298

Other Aspects of Final Accounts

855,000
Purchases raw materials for
lampshades
- other goods
Wages
Rent and rates
Water and electricity
Motor expenses
Repairs
Interest on loan
Bank charges
Insurance
Sundry expenses

2,400,000
254,000
96,000
47,000
60,800
12,000
18,000
4,000
18,000
21,200
5,597,000

_______
5,597,000

Lesson Six

299

Additional Information:
(i) Rent and rates include a prepayment of rates of Sh. 6,000.
(ii) The insurance includes a premium for the period ending 31 October 1988.
(iii)
A trade debt of Sh. 14,000 is not expected to be realized.
(iv)
During the year a pick-up van, which was bought for Sh. 86,000, was
sold for Sh. 30,000, and replaced with another pick-up van costing Sh.
152,000. Both transactions have been posted to the motor vehicle account.
No disposal account has been opened. The straight-line rates of
depreciation based on cost are 25% p.a. for motor vehicle and 10% p.a. for
fixtures and fittings. A full years depreciation is charged in the year of
acquisition and none in the year of disposal.
(v) Accruals at 30 June 1988 were:
Water and electricity
Sh. 5,000
Sundry expenses
Sh. 4,000
(vi)

Stocks at 30 June 1988 were:


Sh.

Lampshades raw materials


Lampshades (at transfer price)
Other goods at cost
(vii)

80,000
30,000
252,000

(a) The agreed transfer price for lampshades produced was Sh. 1,000,000. The
workshop produced 50,000 lampshades during the year.
(b) Wages include those of the lampshades making employee who has been
paid Sh. 50,000 for the year. In addition, she is entitled to a commission on
the annual profit of her department of 10% p.a. after charging such
commission. Shop assistants wages were Sh. 108,000.
(c) The apportionment of rent and rates; and water and electricity to the
lampshades is 25% of the total.
Required:
(a) Prepare a manufacturing, trading and profit and loss accounts for the year
ended 30 June 1988, disclosing clearly (i) the profit earned by the lampshadesmaking department and (ii) the gross profit earned by the shop.
(b) Prepare a balance sheet as at 30 June 1988.
QUESTION FOUR
On 2 November 1983, the Treasurer of the Olympiad Athletics Club died. The
financial year of the club, which had been formed to provide training facilities for
both field and track event athletes, had ended two days previously on 31 October
1983. An extraordinary general meeting was convened for the purpose of
appointing a new treasurer whose task it would be to prepare the annual accounts
for that financial year.
An enthusiastic club member, Guy Rowppe, was duly appointed but, having only an
elementary knowledge of bookkeeping, soon found himself in difficulty.
He sought your assistance, which you agreed to give. During your conversation he
said, The previous treasurer maintained a Cash and Bank account. I have

300

Other Aspects of Final Accounts

summarized the detailed entries into what I think you call a Receipts and Payments
Account, and have rounded the figures to the nearest 1.
At this point he supplied you with a copy of the following document:

Lesson Six

301

Olympiad Athletics Club


Receipts and Payments Account for 12 months ended 31 October 1983
Not
Receipts
Not
Payments
e
e
No.
No.
Cash
Bank
Cash
Bank

Balance c/d
73
Balance b/d
105
Membership fees:
(4) Insurance
premiums paid to
580
brokers
(1) Entrance
80
170 (7) Payments to
suppliers of
5,270
sporting requisites
(1) Annual
215 4,465 (5) Wages of grounds
3,600
subscriptions
man
(2) Life membership
530 (8) Postages and
692
telephones
(3) Training ground
454 7,206 (9) Stationery
629
fees
Insurance:
World-wide
Athletics Club
50
affiliation fee
(4) Premiums
638 (10) Rates of training
846
ground
(4) Commissions
53
Upkeep of training
ground
1,200
(11 Interest received
Transfers to bank
700
)
from investments
626
(12 Sale of office
370 (11) Purchase of
5,600
)
furniture
investments
(6) Sale of sporting
8,774 (11) Short term
3,000
requisites
deposits
Advertising revenue
603
Transfers from cash
____ __700
Balances c/d
122 2,563
822 24,1
822 24,1
35
35
Balances b/d
122 2,563
After you had perused the above account, Guy Rowppe explained the numbered
items, as follows:
(1) On admittance to membership of the club, new members pay an initial
entrance fee together with their annual subscription. At 31 October 1982,
annual subscriptions of 70 had been paid in advance and 180 was owing but
unpaid; of this latter amount, 40 related to members who left during the
current year and is now no longer recoverable. The figures at 31 October 1983
are 100 subscriptions in advance and 230 subscriptions in arrear. The policy
of the club is to take credit for subscriptions when due and to write off
irrecoverable amounts as they arise.

302

Other Aspects of Final Accounts

(2) As an alternative to paying annual subscriptions, members can at any time opt
to pay a lump sum, which gives them membership for life without further
payment.
Amounts so received are held in suspense in a Life Membership Fund account
and then credited to Income and Expenditure Account in equal instalments
over 10 years; the first such transfer takes place in the year in which the lump
sum is received. On 31 October 1982 the credit balance on the Life
Membership Fund Account was 4,720, of which 850 credited was as income
for year ended 31 October 1983.
(3) The club has a permanent training ground. Non-members can use the facilities
on payment of a fee. In order to guarantee a particular facility, advance
booking is allowed. Advance booking fees received before 31 October 1983 in
respect of 1984 total 470. The corresponding amount paid up to 31 October
1982 in advance of 1983 was 325. Members can use the facilities free of
charge.
(4) Club members can take out insurances through the club at advantageous rates.
Initially, premiums are paid by members to the club. Subsequently, the club
pays the premiums to an insurance broker and receives commission. At 31
October 1982, premiums received but not yet paid over to the broker
amounted to 102 and commissions due but not yet received were 11. The
corresponding amounts at 31 October 1983 are 160 and 13 respectively.
(5) The grounds man is employed for the six months April to September only. He
is then paid a retaining fee to secure his services for the following year. At
31 October 1982 the grounds man had been paid a retainer (250) for 1983.
Included in the Wages figure (3,600) is the retainer (300) for 1984.
(6) Sporting requisites are sold only on cash terms. There are therefore no
debtors for these items.
(7) On 31 October 1982 sums owed to suppliers of sporting requisites totaled
163; the corresponding figure on 31 October 1983 was 202.
(8) Stock of unsold sporting requisites on 31 October 1982 was 811 and on 31
October 1983, was 927. In arriving at this latter figure, the sum of 137,
representing damaged and unsaleable stock at cost price, had been
excluded.
(9) Postage stamps unused at 31 October 1983 totalled 4.
(10) Stock of stationery on 31 October 1982 and 1983 was 55 and 36
respectively.
(11) Rates are payable to the District Council in two installments (in advance)
each year. 360 had been paid on 1 October 1982, 390 on 1 April 1983 and
456 on 1 October 1983.
(12) The club receives interest on investments bought a number of years ago at a
cost of 7,400 (current valuation 7,550). At the end of October 1983, the
club had acquired further investments which cost 5,600 (current valuation
5,600) and at the same time placed 3,000 in a short-term deposit account.
(13) The written down value of the furniture which had been sold during the year
was 350; it had originally cost 800.

Lesson Six

303

Other Matters:
Initially, the training ground had been acquired freehold* from a farmer at an
inclusive cost of 4,000. Subsequently, the club had some timber buildings erected
to provide various facilities for members. The total cost of these buildings was
35,000; depreciation is calculated at the rate of 10% per annum on a straight-line
basis. At 31 October 1982, the provision for depreciation account had a balance of
9,400.
At 31 October 1982, the furniture and equipment etc. was recorded in the clubs
books as 7,900 (cost) against which there was a provision for depreciation of
4,150 (calculated on the same basis as for buildings). Apart from the disposal
referred to in note (12) above there had been no other disposals or acquisitions
during the year.
Required:
Prepare the clubs Income and Expenditure Account for year ended 31 October
1983 and the Balance sheet at that date.
All workings must be shown.
*Freehold land is land held in perpetuity.
CHECK YOUR ANSWERS WITH THOSE GIVEN IN LESSON 9 OF THE
STUDY PACK

304

Other Aspects of Final Accounts

COMPREHENSIVE ASSIGNMENT No.2


TO BE SUBMITTED AFTER LESSON 6
To be carried out under examination conditions and sent to the Distance Learning
Administrator for marking by the University.
EXAMINATION PAPER.
ANSWER ALL QUESTIONS

TIME ALLOWED: THREE HOURS.

QUESTION ONE
The bank account of Fuller Ltd, prepared by the companys book-keeper, was as
shown below for the month of October 19-6:
Bank Account
1919-6
6
Oct
Oct
1
Balance c/d
91.40
2
Petty Cash
0623 36.15
13
3
McIntosh and Co
260.1
3
Fredas Fashions
0623 141.1
1
14
7
3
Malcolm Brothers
112.8
6
Basford Ltd
0623 38.04
3
15
3
Cash sales
407.5
8
Hansler Agencies
0623 59.32
4
16
14 Rodney
361.0
9
Duncans storage
0623 106.7
Photographic
2
17
5
17 Puccinis Cold Store 72.54
9
Aubrey plc
0623 18.10
Ltd
18
20 Eastern Divisional
10 Secretarial services
0623 28.42
Gas Board rebate
Ltd
19
(August direct
63.40
credit)
22 Graingers Garage
93.62 14 Trevors Auto
0623 11.75
repairs
20
29 Cash sales
235.3 15 Wages cash
0623 115.5
9
21
2
31 Balance c/d
221.5 16 Towers Hotel
0623 44.09
2
22
17 Bank charges
- 12.36
(September)
20 Broxcliffe borough
Council
SO 504.2
2

Lesson Six

305

21
24

______
1,919.
37

Eastern Area
Electricity Board

28

Eastern Divisional
Gas Board
Petty Cash

30

Wages cash

31

Salaries transfer

Nov
1

Balance c/d

DD

108.6
4

DD
0623
23
0623
24
-

41.20
119.0
7
337.7
4
______
1,919.
37
221.5
2

In early November, the companys bank sent a statement of account which is


reproduced below:

306

Other Aspects of Final Accounts

Statement of Account with Lowland Bank plc


Account: Fuller Ltd Current Account No 10501191
Date of issue: 1 November 19-6
19-6
Oct

Description

BCE

CR

Debit

Credit

Balance

90.45

175.02

265.47

062310

111.34

154.13

062312

9.18

144.95

062309

15.41

129.54

CR

062313

780.48
36.15

910.02
873.87

10

ADJ

12.90

15

062315

38.04

848.73

16

062314

141.17

707.56

17

CR

20

SO

504.22

646.90

21

062317

106.75

540.15

21

DD

196.83

343.32

21

062320

11.75

331.57

22

141981

212.81

118.76

22

ADJ

10.00

108.76

22

062319

28.42

80.34

443.56

886.77

1,151.12

Lesson Six

22

062320

22

CR

24

ADJ

27

INT (loan a/c)

27

307

11.75

68.59
93.62

162.21

212.81

375.02

26.35

348.67

062321

115.52

233.15

28

062322

44.09

189.06

28

DD

108.64

80.42

30

CGS

9.14

71.28

31

ADJ

Abbreviations:
BCE = Balance
Adjustment
INT = Interest

11.75

SO = Standing Order
DD = Direct Debit

83.03

CR = Credit

ADJ =

CGS = Charges

Required:
Prepare the companys bank reconciliation statement as at 31 October 19-6.
(Chartered Association of Certified Accountants)
QUESTION TWO
PAUL RUDYERD
The following balances have been extracted from the accounting records of Paul
Rudyerd, a wholesale fruiter, at the end of his financial year ended on 31 May
19X1.

Purchases and sales


104,310
146,200
Stocks
3,010
Motor vehicles at cost
26,360
Provision for depreciation on motor vehicles as
12,960
at 1 June 19X0
Warehouse equipment at cost
20,000
Debtors and creditors
25,250
21,200
Bank
3,200
Motor vehicle expenses
11,960
Rent and rates
11,220
Advertising
2,500
Sundry expenses (including insurance and
3,470
electricity)
Drawings
6,600

308

Other Aspects of Final Accounts

______

31,120

214,680

214,680

Capital as at 1 June 19X0

Lesson Six

309

Additional information and opinions are given as follows:


(a) Stocks at 31 May 19X1 were valued at 2,600. This amount includes a
consignment of rare fruit from abroad which cost 300, which would normally
sell for approximately 660, but which is badly bruised and could be sold as
juice pulp for 100.
(b) Depreciation on motor vehicles is normally charged at an annual rate of 20%,
using the reducing balance method. The motor vehicles at cost figure includes
a new car purchased during the year for 9,600 for Rudyerds personal use
which it is estimated will last four years with an estimated residual value of
4,000.
(c) Expenses prepaid and accrued at 31 May 19X1 were estimated as follows:
Prepayments
Accruals

Rates
230
Rent
160
Insurance
180
Electricity
200
(d) A bad debt of 250 is to be written off. A provision for doubtful debts of 1% of
outstanding debtors should be created.
(e) A recording error has resulted in a second-hand delivery van, purchased on 2
June 19X0 for 9,000, being treated as a motor vehicle expense.
(f) No record has been made of fruit, estimated to have cost 520, withdrawn
from the business by Rudyerd for his personal use.
(g) No adjustment should be made, in preparing the answer to part (a) for the new
warehouse equipment purchased during the year.
Required:
(a) Prepare a draft trading, profit and loss account for Paul Rudyerds wholesale
fruit business for the year ended 31 May 19X1 and a draft balance sheet as at
31 May 19X1.
(15 marks)
(b) Briefly explain what accounting concepts and conventions would be important
in considering the treatment of the new warehouse equipment.
(4 marks)
(c) Itemize the additional information that you would wish to know before you
could make the appropriate adjustments to the above financial statements in
respect of the new warehouse equipment.
(3 marks)
QUESTION THREE
ABC LTD
You have just been appointed as an accounting assistant to ABC Ltd. A week after
your arrival the finance director is rushed into hospital; the auditors are about to
arrive to prepare the accounts for the recently-ended financial year; you cannot
find any working papers for the previous years accounts; and the other accounts
staff are too busy to assist you in preparing for the auditors visit.
The eight situations described below are detailed on a notepad left by the finance
director and their treatment in the accounts needs to be considered by you.

310

Other Aspects of Final Accounts

(a) A supply of office stationery was purchased five months before the year-end at
a cost of 1,000. At the year-end it is estimated there is about 250 worth left
in stock.
(b) An electronic typewriter was purchased during the year at a cost of 270. It is
estimated to have a useful life of five years.
(c) A batch of goods was produced to a customers special order. The goods cost
5,800 but have not been delivered as it transpires the customer is now
bankrupt. A buyer for the goods has been found, who will pay 4,500 but
modifications costing 1,200 will have to be made to the goods.
(d) Three technical staff have spent the last six months exclusively on a new
product design project; their salaries for this period amount to 22,000. At the
year-end it is known that the design stage will take another month, to be
followed by market research; after this the directors will decide whether the
project should proceed to production and marketing. The companys chief
engineer is confident that sales of the new product will start in the next
financial year and will last for at least four years.
(e) A freehold property was purchased on the first day of the financial year at a
cost of 650,000. The building is estimated to have a useful life of ten years
when it is expected it will have to be demolished for redevelopment. It is
estimated that the freehold land, at the time of purchase, was worth 500,000.
(f) A specialist machine was purchased seven years ago for 200,000. It has been
depreciated, using the straight-line method, at 10% per annum since then. To
the beginning of the year under review 120,000 depreciation has been
provided. The chief engineer has advised that the machine is worn out and
would need to be rebuilt to last more than another two years. The directors
have already decided the machine should not be rebuilt but scrapped one year
after the end of the year under review.
(g) The debtors ledger shows balances totalling 52,000 at the year-end. Two
debts, totalling 2,000, are known to be bad. Another customer has gone into
liquidation owing 3,000; it is expected he will be able to pay 60p of every
owed to his creditors. The sales director recommends a general bad debt
provision of 2% in respect of the remaining debtor balances.
(h) The company has undertaken a heavy advertising campaign throughout the
year under review to promote its corporate image and product range. The
sales and managing directors feel that this campaign will benefit the company
for at least a further six months after the year end. You determine that the
campaign cost 150,000 and has been fully paid for before the year-end.
Required:
For each of the eight situations described above:
(a) Describe what action should be taken in respect of:
(i) The amount to be charged or credited to the years profit and loss account
(if any);
(ii) The value to be placed on the asset in the balance sheet at the year end (if
any);

Lesson Six

311

(8 marks)
(b) State what accounting assumptions, conventions or concepts could be involved
and give reasons, where there is a conflict between two or more of them, why
you have chosen the action you propose.
(9 marks)

312

Other Aspects of Final Accounts

QUESTION FOUR
The following final balance was extracted from the books of J Yeats, a trader, at 31
December 19X9:
Ksh
Carriage inwards
Capital account at 1 January
19X9
Motor vans
Stock at 1 January 19X9
Balance at bank
Purchases
Sales
Trade debtors
Trade creditors
Rent and rates
Salaries
General expenses
Motor expenses
Discounts allowed
Discounts received
Insurance
Bad debts
Provision for doubtful debts 1
Jan 19X9
Provision for depreciation on
vans
Drawings
Disposal
Returns inwards

Ksh
6,310
500,000

200,000
164,000
116,860
1,593,690
2,224,000
290,000
157,600
56,080
350,400
44,720
25,600
40,400
37,600
17,600
30,400
8,000
60,000
50,000
7,140
2,993,200

6,000
_______
2,993,200

The following matters should be taken into account:


(a) After examination of the debtors account, it was decided to:
Write off a bad debt of Ksh 12,000
Make a specific provision in the accounts for the following doubtful debts,
Ksh 5,000 from Wordsworth
Ksh 3,000 from Coleridge
Make a general provision of 5% on the debtors.
(b) Goods unsold at 31 December 19X9 had cost Ksh 201,600 but Yeats expected
to sell them at Ksh 232,470.
(c) Salaries accrued at 31 December 19X9 amounted to Ksh 32,000.

Lesson Six

313

(d) The rent of the premises is Ksh 40,000 a year, payable quarterly in arrears, but
the instalment due on 31 December 19X9 was not paid until 15 January in the
next year.
(e) Insurance paid in advance at 31 December 19X9 amounted to Ksh 2,000.
(f) Depreciation is to be provided for on the motor truck at the rate of 20% per
annum straight line on cost.
(g) General expenses include Ksh 3,060 relating to the telephone account which is
made up of:
- Rent three months in advance from 30 November 19X9 at Ksh. 420.
- Calls three months ended 30 November 19X9 at Ksh 2,640.
(h) It has been agreed with Inland Revenue (Taxation Office) that 12.5% of the rent
sand rates relate to private use.
Prepare a trading and profit account for the year to 31 st December 19X9, and a
balance sheet as at 31 December 19X9.
QUESTION FIVE
The balance sheet of Johnsons shop at 1 October 19X7 was as follows:
Non current
assets
Shop premises

45,000

Shop fittings
Delivery van

12,000
4,000

Ksh

Current assets
Stock in trade
Cash in hand

14,000
2,000

Ksh

Ksh
Capital as at 1
Oct

Ksh
51,000

61,000
Current
liabilities
Trade creditors
16,000 Bank overdraft
77,000

12,000
14,000

26,000
77,000

The following is a summary of the transactions which took place during the year to
30 September 19X8:
1.

Sales were made, all for cash, of Ksh 145,000. The stock in trade sold cost Ksh
83,000.
2. Stock in trade bought, all on credit for Ksh 78,000.
3. Cash of Ksh 113,000 was taken from the till (cash register) and paid into the
bank.
4. The trade creditors were paid Ksh 73,000 by cheque.
5. Johnson borrowed Ksh 30,000 from Black, which was paid into the bank. The
loan is for 5 years.
6. Wages of Kshs 17,000 were paid in cash.
7. Rates of Ksh 2,900 were paid by cheque.
8. Sundry expenses of Ksh 6,000 were paid in cash.
9. Electricity bills of Ksh. 1,600 were paid by cheque.
10. The owners of the business withdrew Ksh 9,000 in cash.
At 30 September 19X8 you discover the following:

314

1.
2.
3.
4.

Other Aspects of Final Accounts

Interest Ksh 2,500 due to Black for the year was unpaid.
Shop fittings are to be depreciated at 10% per annum on the total at the yearend; the delivery van is to be depreciated at 20% per annum of the total at the
year-end.
The rates payment during the year included Ksh 1,000 in respect of the period
1/10/19X8 to 31/3/19X9.
The electricity bill for the quarter to 30/09/19X8 for Ksh 500 was unpaid.

Prepare a balance sheet as at 30 September 19X8 and a profit and loss account for
the year to that date.

END OF COMPREHENSIVE ASSIGNMENT No.2


NOW SEND TO THE DISTANCE LEARNING CENTRE FOR MARKING

Acknowledgement

315

LESSON SEVEN
PARTNERSHIPS
A partnership is a relationship that subsists between two or more persons carrying
on a business common with a view to making profit.
Reasons for partnership
1) Additional capital incase a sole trader or one person is not able to raise
sufficient capital.
2) Incase there is need for skills or expertise in certain areas of the business.
3) To involve more persons in the business especially for a family.
Membership
A partnership has minimum membership of two (2) maximum of fifty (50) except
for professional firms (e.g.) lawyers, doctors, accountants etc. whose maximum
membership is twenty (20) persons.
Partnership deed
Where two or more persons wish to form a partnership, then it is recommended
that they agree on the terms upon which the partnership will be run and the
relationship between each other. This is done in writing and signed off as agreed
by all the partners and therefore it becomes a partnership deed or agreement.
Contents of partnership agreement
1) Name(s) and address(s) of both the firm and the partners
2) Capital to be contributed by each partner
3) The profit sharing ratios that may be expressed as a fraction or as a
percentage.
4) Salaries to be paid to any partners who will be involved in the active
management of the business
5) Any interest to be charged on drawings made by the partners.
6) Interests to be given to the partners on their capital balances.
7) Procedures to be taken on the retirement or admission of a partner.
Accounting for partnerships.
The interest of the partners in the business is either long term or short-term.
The long-term interest is the capital contributed by each partner and the balance is
expected to remain fixed. It will only change when the partners agree or incase of
any changes in the partnership like admission of or retirement of a partner.
The short-term interest is reflected in form of a current account which is affected
by the trading activities of the partnership (i.e.) the profits or losses and any
drawings made by the partners.
In most partnerships, both a capital and a current account are maintained and
therefore the capital account becomes a fixed capital account. When there is no
FINANCIAL ACCOUNTING 1

Lesson Seven

316

distinction between a capital account and a current account then any short- term
changes are passed through the capital account therefore the capital account
becomes a fluctuating capital account.
Some of the transactions to be passed through the capital account and the current
account are shown in the following formats.
(Assume a firm of 3 partners A, B and C)

Loss or
revaluation
Goodwill written
off
Bal c/d

CAPITAL ACCOUNT
A
B
C

xx xx xx Bal b/d
xx

xx

xx

xx
xx

xx
xx

xx
xx

xx

xx

xx

Additional capital
xx
(c/book or asset)
Gains on revaluation xx
Goodwill
xx
xx
Bal b/d
xx

xx

xx

xx
xx
xx
xx

xx
xx
xx
xx

CURRENT ACCOUNT
B
C
A

xx
Bal b/d
xx
xx xx xx Interest on capital
xx
A

Bal b/d
Interest on
drawings
Drawings
Bal c/d

xx

xx

xx

xx

xx

xx

xx

xx
xx

Salaries
xx
Share of profits xx
Loan interest
Bal c/d
xx
Bal b/d
xx

xx
xx
xx
xx
xx
xx

xx
xx
xx
xx
xx
xx
xx

Format For Final Accounts:


Profit and Loss Account
The profit and loss account is exactly as the one for the sole trader and in addition
to the profit and loss account, a new section called the Appropriation account is
included and this account shows how the partners share the Net Profit for the
period. (In addition to other expenses in the profit and loss, an expense for
interest on loan given by one of the partners is included and the credit entry is
made on the partners current account.)
The format for the Appropriation account is as follows:

317

Partnerships

Net Profit for the year


Add: Interest on drawings.
A
B
C

xx
xx
xx

Less: Interest on capital.


A
B
C

xx
xx
xx

xx

xx
xx

(xx)

xx

Less: Salaries
A
B
C

xx
xx
xx

Balance of profit to be shared in percentage ratio


A (ratio)
xx
B (ratio)
xx
C (ratio)
xx

(xx)
xx

(xx)

Balance sheet
The balance sheet also the same as that for a sole trader but the interest of each
partner in the business should be shown separately and any loan given by a
partner to the firm is also shown separately in the non-correct liability section
therefore, the format will be as follows.

xx
xx
xx
xx
xx

Net assets.
Capital: A
B
C
Current account A
B
C (debit balance).
Non-current liabilities
10% loan B
10% loan bank

xx
xx
(xx)

xx
xx
xx

xx

xx
xx

Lesson Seven

318

Example 7.1
Read the following and answer the questions below.
A and B own a grocery shop. Their first financial year ended on 31 December
2002.
The following balances were taken from the books on that date:
Capital:
Partnership salaries:
Drawings:

A- 60,000;
A - 9,000;
A - 12,000;

B - 48,000.
B - 6,000.
B - 13,400.

The firms net profit for the year was 32,840.


Interest on capital is to be allowed at 10% per year.
Profits and losses are to be shared equally.
(a) From the information above prepared the firms appropriation account and the
partners current accounts.
SOLUTION
A and B
Profit and Loss Appropriation account for the year ended 31 Dec 2002

Net Profit for the year


32,840
Less: Interest on capital
A
B

6000
4800

(10,800)
22,040

Less: Salaries
A
9000
B
6000
(15,000)
Balance of profit to be shared in Profit Share Ratio
A

3520
B

3520
(7,040)

Drawings

12,86
0

Bal c/d
5,660
18,52
0

CURRENT ACCOUNT
B

13,400 Interest on
capital
Salaries
920 Profit shared.
14,320

7,040

6,000

4,800

9,000
3,520

6,000
3,520

18,520

14,32
0

319

Partnerships

Bal b/d

5,660
920

Lesson Seven

320

EXAMPLE 7.2
Draw up a profit and loss appropriation account for the year ended 31 December
19X7 and balance sheet extracts at the date, from the following:
i.
ii.
iii.
iv.
v.
vi.
vii.
viii.

Net profits 30,350


Interest to be charged on capitals: W 2,000; P 1,500; H 900
Interest to be charged on drawings; W 240; P 180; H 130
Salaries to be credited: P 2,000; H 3,500.
Profits to be shared: W 50%; P 30%; H20%.
Current accounts: balances b/f W 1,860; P 946; H 717
Capital accounts: balances b/f W 40,000; P 30,000; H 18,000
Drawings: W 9,200; P 7,100; H 6,900.

SOLUTIONS
W,P and H
Profit and Loss Appropriation Account for the year ended 31 December
2002

Net profit for the year


Add: Interest on drawings
W
P
H

240
180
130

Less: Interest on capital


W
P
H

2,000
1,500
900

Less: Salaries
P
H
Balance of profit to be shared
W
50%
Pl
30%
H
20%

W
Interest on
draw
Drawings

240

2,000
3,500
10,500
6,300
4,200

30,350

550
30,900

(4,400)
26,500
(5,500)
21,000
(21,000)

Current Account
H

130 Bal b/d

1,860

180
9,200

717

946
6,900

Interest on

2,000

900

321

Partnerships

7,100

capital
Salaries

Bal c/d

1,500
3,500
2,000

Share of
profits
Bal c/d

4,920
14,360

4,200
6,300

2,287
3,466
10,74
6

9,317

Balance sheet (extract) as at 31 Dec 2002

Net Assets
Capital
W
P
H
Current Accounts
W
Pl
H

10,000

14,360

10,74
6

9,317

xx
40,000
30,000
18,000
88,000

4,920
3,466
2,287

10,673
98,673

Example 7.3
The following list of balances as at 30 September 19X9 has been extracted from
the books of Brick and Stone, trading partnership, sharing the balance of profits
and losses in the proportions 3:2 respectively.

Printing, stationery and postage


3,500
Sales
322,100
Stock in hand at 1 October 19X8
23,000
Purchases
208,200
Rent and rates
10,300
Staff salaries
36,100
Telephone charges
2,900
Motor vehicle running costs
5,620
Discounts allowable
950
Discount receivable
370
Sales returns
2,100
Purchases returns
6,100
Carriage inwards
1,700
Carriage outwards
2,400
Fixtures and fittings: at cost
26,000
Provision for depreciation
11,200
Motor vehicles: at cost
46,000
Provision for depreciation
25,000
Provision for doubtful debts
300

Lesson Seven

Drawings:

Brick
Stone
Current account balances
At 1 October 19X8:
Brick
Stone
Capital account balances
At 1 October 19X8:
Brick
Stone
Debtors
Creditors
Balance at bank

322

24,000
11,000
3,600
2,400

credit
credit
33,000
17,000
9,300
8,400
7,700

Additional information
1. 10,000 is to be transferred from Bricks capital account to a newly opened
Brick Loan Account on 1 July 19X9.
2. Interest at 10 per cent per annum on the loan is to be credited to Brick.
3. Stone is to be credited with a salary at the rate of 12,000 per annum from 1
April 19X9.
4. Stock in hand at 30 September 19X9 has been valued at cost at 32,000.
5. Telephone charges accrued due at 30 September 19X9 amounted to 400 and
rent of 600 prepaid at that date.
6. During the year ended 30 September 19X9 Stone has taken goods costing
1,000 for his own use.
7. Depreciation is to be provided at the following annual rates on the straight line
basis:
Fixtures and fittings
10%
Motor vehicles
20%
Required:
(a)
Prepare a trading and profit loss account for the year ended 30 September
19X9.
(b)
Prepare a balance sheet as at 30 September 19X9 which should include
summaries of the partners capital and current accounts for the year ended
on that date.
Note: In both (a) and (b) vertical forms of presentation should be used.
SOLUTION
Brick And Stone.
Trial Balance As At 30 September 19x9
Debit

Printing and stationery and postage


3,500
Sales
Stock (1 October 19X8)
23,000

Credit

322,100

323

Purchases
Rent and rates
Heat and light
Staff salaries
Telephone charges
Motor vehicle running expenses
Discounts allowable
Discounts receivable
Sales returns
Purchases returns
Carriage inwards
Carriage outwards
Fixtures and fittings at cost
Provision for depreciation
Motor vehicles at cost
Provision for depreciation
Provision for doubtful debts
Drawings: Brick
Stone

Partnerships

208,200
10,300
8,700
36,100
2,900
5,620
950
370
2,100
6,100
1,700
2,400
26,000
11,200
46,000
25,000
300
24,000
11,000

Lesson Seven

324

Current accounts:
Brick
Stone
Capital accounts:
Brick
Stone
Debtors
Creditors
Balance at bank

3,600
2,400
33,000
17,000
9,300
8,400
7,700
429,470
429,470

TRADING AND PROFIT LOSS ACCOUNT FOR THE YEAR ENDED 30


SEPTEMBER 19X9

Sales
322,100
Less: Sales returns
2,100
320,000
less cost of sales
Opening Stock
23,000
Purchases (adjustment)
207,200
Add: Carriage inwards
1,700
208,900
Less: Purchases returns
(6,100)
202,800
225,800
Less: Closing Stock
(32,000)
(193,800)
Gross profit
126,200
Discount receivable
370
Less Expenses
Telephone charges (adjustment))
3,300
Printing and stationery and postage
3,500
Rent and rages (adjustment)
9,700
Heat and light
8,700
Staff salaries
36,100
Motor vehicle running expense
5,620
Discount allowable
950
Carriage outwards
2,400
Depreciation on fixtures and fittings
2,600
Depreciation on motor vehicles
9,200
Interest on loan (adjustment)
250
(82,320)
Net profit
44,250
Less: Salaries
Stone (adjustment)
(6,000)
Balance of profit to be shared
38,250
Brick 3 5
22,950
Stone

15,300

(38,250

325

Partnerships

Balance sheet as at 30 September 19X9


Non current Asset

Fixtures and fittings


26,000
(13,800)
12,200
Motor vehicles
46,000
(34,200)
11,800
72,000
48,000
24,000
Current Asset
Stock
32,000
Debtors
9,300
Less: Provision
(300)
9,000
Payments
600
Cash at bank
7,700
49,300
Current Liabilities
Creditors
8,400
Accruals
400
(8,800)
40,500
64,500
Capital
Brick (adjustment)
23,000
Stone
17,000
Current:
Brick adjustment
2,800
Stone
11,700
14,500
54,500
Non-Current Liabilities
10% loan Brick
10,000
64,500
Current Account
Drawings

Brick

24,00
0

Bal c/d

Stone

12,000
(adj)
11,700

Bal b/d
Interest on loan
Salaries.

Brick

3,600
250

2,800
Share profits
26,80
0

26,800

Stone

2,400

22,950
26,800

6,000
15,30
0
23,70
0

Lesson Seven

326

EXAMPLE 7.4
Mack and Spencer are in partnership sharing profits and losses equally. The
following is the trial balance as at 30 June 2003.
Dr.
Cr.

Buildings (cost 750,000)


500,000
Fixtures at cost
110,000
Provision for depreciation: Fixtures
33,000
Debtors
162,430
Creditors
111,500
Cash at bank
6,770
Stock at 30 June 19X8
419,790
Sales
1,236,500
Purchases
854,160
Carriage outwards
12,880
Discount allowed
1,150
Loan interest: King
40,000
Office expenses
24,160
Salaries and wages
189,170
Bad debts
5,030
Provision for bad debts
4,000
Loan from J King
400,000
Capitals: Mack
350,000
Spencer
290,000
Current accounts: Mack
13,060
Spencer
2,890
Drawings: Mack
64,000
Spencer
56,500
2,446,040 2,446,040
Required:
Prepare a trading and profit and loss appropriation account for the year ended 30
June 19X9 and a balance sheet as at that date.
a)
b)
c)
d)
e)
f)
g)

Stock, 30 June 2003, 563,400


Expenses to be accrued: Office Expenses 960; Wages 2,000
Depreciate fixtures 10 per cent on reducing balance basis, buildings 10,000
Reduce provision for bad debts to 3,200.
Partnership salary: 8,000 to Mack. Not yet entered
Interest on drawings: Mack1,800; Spencer 1,200.
Interest on capital account balances at 10 per cent.

327

Partnerships

Mack and Spencer


Q 5.2 Trading and Profit Loss Account for the year ended 30 June 2003

Sales
1,236,500
Less cost of sales
Opening Stock
419,790
Add: Purchases
854,160
1273,950
Less: Closing Stock
(563,400)
710,550
Gross Profit
525,950
Reduction in provision for bad debts (400-300)
800
526,750
Less Expenses
10
Depreciation: Fixtures & Fittings (110,000-33,000 x 100
)7,700
Buildings
10,000
Carriage outwards
12,880
Discount allowed
1,150
Office expenses (24160 + 960)
25,120
Loan interest
40,000
Salaries and wages (18,9170 + 2000)
191,170
Bad debts
5,030 (293050)
Net Profit for the period
233,700
Add: Interest on drawings:
Mack
1,800
Spencer
1,200
3,000
236,700
Less: Salaries Mack
(8,000)
228,700
Less: interest on capital
Mack
35,000
Spencer
29,500
(64,500)
164,200
Balance of profits
Mack
82,100
Spencer
82,100
164,200
Mack Current Account

Drawings
64,000
balance b/d
13,060
Interest on drawings
1,800
salary
8,000
Interest on capital
35,000
Profit
82,100
bal c/d
72,360
138,160
138,160

Lesson Seven

328

Spencer Current Account

Drawings
56,500
Interest on drawings
1200
Bal c/d

bal b/d
2980
Interest on capital 29,500
Profit
82,100
56,880
114,580
114,580

Balance Sheet as at 30 June 19X9


Non Current Assets
Buildings
Fixtures

Cost
750,000
110,000
860,000

Current Assets
Stock
Debtors (16,243 320)
Cash at bank
Current Liabilities
Creditors
Accruals (200 + 96)

56,3400
15,9230
6770
72,9400
111,500
2,960

Capital Accounts: Mack


Spencer
Current Accounts: Mack
Spencer
Loan from J. King

Depreciation
NBV
(260,000)
490,000
(40,700)
69,300
(300,700)
559,300

(114,460)

61,4940
1,174,240

35,000
29,500

64,500

56,880

129,240

72,360
400,000
1,174,240

329

Partnerships

Example 7.5
JUNE 1998 QUESTION 4
The balance sheet of the partnership of Kombo and Nzuki as at 31 March
1997 was as follows:
Capital accounts:
sh.
sh
Kombo
1,400,000
Nzuki
1,400,000
2,138,000
Current Accounts:
Kombo
136,000
Nzuki
(81,200)
Current Liabilities:
Creditors
501,600
Accruals
25,600
Suspense account
1,570,300

Shs.

Sh.

Fixed asset

(at cost less depreciation


2,800,000 Premises
1,200,000
Equipment
520,000
Vehicles
418,000

54,800
Current Assets:
Stock
894,200
527,200 debtors
475,900
Provision
(46,400)
429,500
Prepayments
28,600
326,300 Bank and cash
281,000
3,708,300
3,708,300

After a lengthy check of all the entries, the following errors were identified
1. Discounts received, sh.26,400 had been debited to discounts allowed.
2. The sales account had been under cast by sh.200,000.
3. A credit sale of Sh.29,400 had been debited to a customers account as
Sh.42,900.
4. A vehicle bought originally for sh.140,000 four years ago and depreciated
at 20% by straight line method on an assumed residual value of
Sh.20,000 had been sold at Sh.60,000 but no entries, other than in the
bank account had been passed through the books.
5. An accrual of Sh.11,200 for electricity charges had completely been
omitted.
6. A bad debt of Sh.31,200 had not been written off an provision for bad
debts should have been maintained at 10% of debtors.
7. Kombos current account had been credited with a partnership salary of
Sh.60,000 which should have been credited to Nzukis current account.
8. Kombo had withdrawn, for personal use, goods to the value of Sh.39,200.
No entries had been made in the books.
9. The partners share of profits and losses as follows:
Kombo 60% and Nzuki 40%
Required:
a)
A statement of adjustments to show the correct net profit for the y
(12 marks)

Lesson Seven

b)

330

A suspense account showing how the balance is eliminated from the


books.
(2 marks)
c)
A corrected balance sheet as at 31 March 1997.
(8 marks)

331

Partnerships

SOLUTION
The following journal can be included although not required in the question.
DR
i)

DR: Suspense Account


CR: Discount Allowed Account
DR: Suspense Account
CR: Discount receive
Making the correct in the accounts

ii)

DR: Suspense Account


CR: Sales Account
Sales undercast now corrected

iii)

DR: Suspense Account


CR: Debtors Account
Being an overstatement of debtors
account now corrected

iv)

DR:
CR:
DR:
CR:
DR:
CR:
DR:
CR:

v)

DR: Profit and Loss Account


CR: Accrue expenses Account

vi)

CR
26,400
26,400
26,400
26,400

200,000
200,000
13,500
13,500

Asset Disposals Account


140,000
Asset Account
140,000
Provision for depreciation Account
Asset Disposal Account
Suspense Account
60,000
Asset Disposal Account
60,000
Asset Disposal Account
16,000
Profit and Loss Account
16,000
11,200

DR:
CR:
DR:
CR:

Profit and Loss Account


Debtors Account
Provision for doubtful debts Account
Profit and Loss Account

DR:
CR:
DR:
CR:
39,200

Kombos Current Account


Nzukis Current Account
Kombos Current Account
Profit and Loss Account (Purchases)

11,200
31,200
31,200
3,280
3,280
60,000
60,000
39,200

Lesson Seven

(a)

332

Kombo and Nzuki Partnership


Statement of Corrected Net Profit for the year
Sh.
Sh.
Adjustments
Discount allowed
26,400
Discount received
26,400
Sales undercasted
200,000
Profit on disposal of asset
16,000
Accrued electricity charges
(11,200)
Bad debts
(31,200)
Decrease in provision for bad debts
3,280
Drawings (goods)
39,200
Net adjustments to Net Profit
268,880

Bal b/d

Kombo
Shs
-

Nzuki

Partners Current Account


Nzuki
Shs.
Bal b/d
81,200
Kombo

Kombo
Shs.
136,00
0
-

60,000
Drawings
Bal c/d

39,200
198,12
8

86,352

297,32
8

167,55
2

(b)
Discount allowed
Discount received
Sales
Debtors
Motor vehicle disposal

Net profit
adjustments

Suspense Account
Shs.
26,400 bal b/d
26,400
200,000
13,500
60,000
326,300

Nzuki
Shs.
-

161,32
8

60,000
107,55
2

297,32
8

167,55
2

Shs.
326,300

326,300

333

(c)

Partnerships

Kombo and Nzuki


Balance Sheet as at 31 March 1997
Fixed Assets
Shs.
Shs.
Premises
Equipment
Vehicles
Current Assets
Stocks
Debtors (431,200 43,120)
Prepayments
Bank and Cash
Current Liabilities
Creditors
501,600
Accruals (25,600 + 11,200)
Capital Accounts
Kombo
Nzuki
Current Accounts
Kombo
Nzuki

Shs.
1,200,000
520,000
374,000
2,094,000
894,200
388,080

28,600
218,000
1,528,880
36,800

(538,400)
3,084,480

990,480

1,400,000

2,800,000

1,400,000

198,128
86,352

284,480
3,084,480

NB
This is a very good question on partnership as it combines both errors on the
accounts and Partnerships. Please study it carefully and follow up the entries and
adjustments.
The next example is still on past paper and combines both incomplete records and
partnerships.
EXAMPLE 7.6 JUNE 1997
Question 1
Kefa and Mark are partners sharing profits and losses equally. They do not
maintain proper books of accounts. The following information has been obtained
from the available records on 31 March:
1996
1997
Sh.
Sh.
Balance at bank
94,800
169,680
Stock in trade
541,200
488,640
Trade debtors
612,000
?
Trade creditors
?
305,760
Furniture
360,000
Motor vehicles (book value)
1,920,000

Lesson Seven

334

Total sales during the year ended 31 March 1997 amounted to Sh.3,849,120 while
purchases, all on credit for the same period were Sh.2,952,480. On 31 March
1996 Kefas capital was Sh.200,000 less than that of Mark. The analysis of the
cash book for the year ended 31 March 1997 shows the following:
Receipts:
Cash from credit sales
Additional capital by Kefa
Cash sales
Payments:
For purchases
Salaries paid
Rent paid (for 6 months to 30.9.96)
Rates paid (for 6 months to 30.6.97)
Electricity charges
Advertising
Motor vehicle expenses
Sundry expenses
Drawings Kefa
Mark

3,491,520
240,000
586,800
3,070,080
420,000
144,000
120,000
60,000
41,760
119,520
33,600
132,480
102,000

On 31 March 1997 liabilities were as follows:


Sh.
Electricity charges
12,480
Advertisement
Sundry expenses
3,600

6,240

On 20 March 1997 the firm decided to dispose of two of its motor vehicles. One
vehicle was sold on credit for Sh.640, 000 while the other was taken over by Kefa
at a valuation of sh.250, 000. the combined book value of the two vehicles was
Sh.660,000. the transaction has not been recorded in the books.
Depreciation at the rate of 10 percent is to be provided on furniture and motor
vehicles on hand at 31 March 1997. No depreciation is to be provided for the
vehicles, which were disposed of.
Required:
a) Trading, profit and loss account for the year ended 31 March 1997.
(10 marks)
b) Balance sheet as at 31 March 1997.
(8 marks)
c) Partners capital accounts
(4 marks)
(Total: 20 marks)
SOLUTION
June 1997 Question 1
KEFA and MARK
STATEMENT OF AFFAIRS AS AT 31 March 1997
Assets
Sh.
Sh.

335

Partnerships

Bank
Stock
Debtors
Furniture
Motor vehicle
Liabilities
Creditors
Net Assets
Capital

94,800
541,200
612,000
360,000
1,920,000
3,528,000
(423,360)
3,104,640

Kefa
Mark

1,452,320
1,652,320

3,104,640

Trading, Profit and loss account for the year ended 31 March 1997
Sh

Sh.
3,849,120

Sales
Less cost of sales
Opening stock
Purchases
Less: Closing stock
Gross profit
Profit on disposal adjustment

541,200
2,952,480
488,640)

Less Expenses
Salaries
Rent adjustment
Rates
Electricity
Advertising
Motor vehicle
Sundry expense
Depreciation Furniture
- Motor vehicle
Net Loss should in PSR
Kefa
Mark

(3,005,040)
844,080
230,000
1,074,080

420,000
288,000
60,000
72,480
48,000
119,520
37,200
36,000
126,000
(66,560)
(66,560)

(1,207,200)
(133,120)
133,120

Balance sheet as at 31 March 1997


Non-current Assets
Furniture
Motor vehicle

Sh.
360,000
1,620,000

Current Assets
Stock
Debtors Trade Adjustment
- others vehicle

Sh.
(36,000)
1,260,000
(162,000)
488,640
382,800
640,000

Sh.
324,000
(126,000)
1,134,000
1,458,000

Lesson Seven

336

Prepayments
Bank
Current Liabilities
Creditors
Accruals
Capital -

Kefa
Mark

60,000
169,680
1,741,120
305,760
166,320

(472,080)

1,269,040
2,727,040
1,243,280
1,483,760
2,727,040

337

Partnerships

Kefa
Shs.
Drawings
132,480
Disposal

Capital Account
Mark
Shs.
Bal b/d
102,000
Cash

250,000

Kefa
Shs.
1,452,3
20

mark
Shs.
1,652,3
20

240,000

Loss shared
Bal c/d

66,560
1,243,28
0
1,692,32
0

66,560
1,483,76
0
1,652,32
0

1,692,3
20

1,652,3
20

GOODWILL AND REVALUATION OF ASSETS


This is defined as the advantage, whatever it may be, a person gets by
continuing to be entitled to represent to the outside would that he is
carrying on a business which has been carried on for sometime previously.
Judge Warey in Hull V Frases
Goodwill is the element that arises from a business due to its reputation and
therefore, enjoys benefits that a new business may not get.
(e.g.) A new business may not make profits easily during the first year of
trading.
(c)

Factors that contribute to goodwill


1. Quality of products/Services
2. Good personnel
3. Marketing
4. Location
5.
In accounting, goodwill is very important for ascertaining the element or the
share of a partners effort to improve the business. The problem is normally
to ascertain the value or cost of goodwill.
There are two types of goodwill:
1. Non-Purchase goodwill
Non- purchased goodwill is determined by using subjective estimates.
There are various approaches to these. Goodwill maybe arrived at by
taking the average profits for lets say three previous years of trading.
Due to this subjective estimate, this type of goodwill is not maintained or
shown in the accounts.
2. Purchased goodwill
This is less subjective because it is the excess amount paid for a business
above its net assets.
This is less subjective because it is the excess amounts paid for a
business above its net assets.

Lesson Seven

338

(e.g) If a business pays Sh.3.5 m to acquire the net assets (i.e. in these
case the net assets will be total assets less total liabilities) of another
business that is still trading on and the value of the net asset is 3 M,
therefore the purchased goodwill may be shown in the accounts as an
intangible asset. Purchased goodwill can be treated in the following three
main ways:
1) Goodwill is written off from the accounts
2) Is carried at its value an amortized over a period of time
3) Carried at its value without being amortized.
The practice is normally to carry it in the accounts together with the
other assets (as an intangible asset) and amortize it over estimated
period of time.

339

Partnerships

In a partnership, there are normally three situations where goodwill is accounted


for in the accounts:
a) If there is a change in the profit sharing ratio.
b) On admission of a new partner.
c) On retirement of an old partner.
d)
Example (when there is a change in profit sharing ratio)
When there is a change in the profit sharing ratio, then goodwill is introduced in
the accounts by
Dr. Goodwill account
Cr. Partners capital account ( the credit is based on the
old
profit sharing ratio.)
The goodwill may remain in the accounts and therefore no partner entries will be
made.
If the goodwill is to be written off from the accounts, this will be done by
Debiting partners capital account (in the New profit sharing ratio)
Crediting goodwill account
Example:
A and B have been trading as partners sharing profits and losses equally. They
decided to change profit sharing ration to 3:2. The capital balances are:
A: - Sh.1,000,000
B: - Sh.1,500,000
Goodwill has been agreed at Sh.500,00.
Required: The partners capital balances assuming that:
1) Goodwill is to be retained in the accounts
2) Goodwill is to be written off form the accounts.
Solution:
1)

CAPITAL ACCOUNT
B
Bal b/d

A
1,000,0
00

B
1,500,0
00

250,000

250,000

Goodwill(OPSR)
Bal c/d

2)
Goodwill

12,500,0
00
12,500,0
00

1,750,0
00
1,750,0
00

CAPITAL ACCOUNT
A
B
300,000
Bal b/d

A
1,000,000

B
1,500,00

Lesson Seven

340

(NPRS)
Bal c/d (NPSR)

200,000
1,550,00
0

950,000
12,500,00
0

0
Goodwil
l
(OPSR)

1,750,00
0

250,000
250,000
12,500,00
0

1,750,00
0

REVALUATION OF ASSETS.
The business may revalue some of the assets to reflect their fair values (e.g.) based
on market price.
The revaluation is normally done when a new partner is to be admitted or an old
partner is retiring.
Any revaluation gains or losses are passed through a new account (i.e) a
Revaluation account and the balance on this account profit or low on revaluation is
transferred to the partners capital accounts in the existing profit sharing ratio.
Example:
(A, B, and C are trading as partners sharing profits and losses in the ratio of 2:2:1.
They have the following assets and liabilities at the book values and they wish to
restate these values at market values and agreed values.
Assets/Liabilities
Buildings
Fixtures, Fittings & furniture
Motor vehicle
Stock
Debtors
(50,000)
Creditors

Book value Market price/Agreed value Gain)

Loss
2,000,000
2,500,000
100,000
900,000
800,000
(100,000)
1,200,000
1,150,000
(50,000)
700,000
650,000
(50,000)
450,000
400,000
800,000

700,000

100,000

Required:
Prepare Revaluation account and the partners capital account given the
partners balances as
A 3,000,000
B 2,500,000
C 1,500,000
REVALUATION ACCOUNT
Fixtures
Motor vehicles
Stock
Debtors
Capital A/C A 2 5
B

100,000
50,000
50,000
50,000
140,000
140,000

buildings
Creditors

500,000
100,000

341

Partnerships

70,000
600,000

600,000

Goodwill

A
000

Bal c/d

3,140
3,140

CAPITAL ACCOUNT
C
000
Bal b/d
2,640 1,510 Revaluatio
n
2,640 1,570
B
000

A
000
3,000
140

B
000
2,500
140

C
000
1,500
70

3,140

2,640

1,570

If there is a profit on revaluation, then the profit will be transferred to the


partners capital account by:
Dr. Revaluation
Cr. Partners capital account in the profit share ratio
If there is loss then
Dr. Partners capital account
Cr. Revaluation in the profit share ratio
EXAMPLE 7.7
Alan, Bob and Charles are in partnership sharing profits and losses in the ratio
3:2:1 respectively.
The balance sheet for the partnership as at 30 June 19X6 is as follows;
Fixed Assets
Premises
Plant
Vehicles
Fixtures
Current Assets
Stock
Debtors
Cash
Capital
Alan
Bob
Charles
Current account

62,379
34,980
___760

90,000
37,000
15,000
2,000
144,000

98,119
242,11
9
85,000
65,000
35,000
185,000

Lesson Seven

342

3,714
Alan
Bob
Charles

Loan Charles

Current liabilities
Creditors
Bank overdraft

(2,509)
4,678

5,883

28,000

19,036
4,200
242,11
9

Charles decides to retire from the business on 30 June 19X6, and Don is admitted
as a partner on that date. The following matters are agreed:

343

Partnerships

a) Certain assets were revalued;


Premises 120,000
Plant 35,000
Stock 54,179
b) Provision is to be made for doubtful debts in the sum of 3,000.
c) Goodwill is to be recorded in the books on the day Charles retires in the sum of
42,000. The partners in the new firm do not wish to maintain a goodwill
account so that amount is to be written back against the new partners capital
accounts.
d) Alan and Bob are to share profits in the same ratio as before, and Don is to
have the same share of profits as Bob.
e) Charles is to take his car at its book value of 3,900 in part payment, and the
balance of all he is owed by the firm in cash except 20,000 which he is willing
to leave as a loan account.
f) The partners in the new firm are to start on an equal footing so far as capital
and current accounts are concerned. Don is to contribute cash to bring his
capital and current accounts to the same amount as the original partner from
the old firm who has the lower investment in the business.
The original partner in the old firm who has the higher investment will draw out
cash so that his capital and current account balances equal those of his new
partners.
Required;
a) Account for the above transactions, including goodwill and retiring partners
accounts.
b) Draft a balance sheet for the partnership of Alan, Bob and Don as at 30 June
19X6.
Solution:

Goodwill
written
off
Motor
vehicle
Cashbook
Bal c/d

Don

Alan

Bob

12,0
00
-

18,00
0
-

12,0
00
-

21,00
0
67,00
0
106,0
00

67,0
00
79,0
00

Don

Alan

67,0
00
79,0
00

Bob

Charl
es

Capital Accounts
Don
Alan

- Bal b/d
3,900 Goodwil
l
38,10 Cash
0 book
42,00
0

Charl

Bob

Charl
es

85,00
0
21,00
0
-

65,0
00
14,0
00
-

35,00
0
7,000

106,0
00

79,0
00

42,00
0

79,0
00
79,0
00

Current Accounts
Don
Alan

Bob

Charl

Lesson Seven

Bal b/d
Cash
book

344

2,50
9

9,02
3

es

es

- Bal b/d

3,714

4,678

7,478 Revaluation
a/c

8,400

5,60
0

2,800

3,09
1

3,09
1

12,11
4

5,60
0

7,478

Cash book
Bal c/d

3,09
1
3,09
1

3,091
12,11
4

3,09
1
5,60
0

7,478

345

Plant
Stock
Debtors
Profits shared:
Alan
Bob
Charles

Partnerships

Revaluation Account

2,000 Premises
30,000
8,200
3,000
8,400
5,600
2,800
30,000

_____
30,000

Cash book
Bal b/d
Don - capital
account
Current account

760 Charles capital


account
79,000
Loan
3,091

Current
account
Alan capital
account
Current account
Bal c/d

38,100
8,000
7,478
21,000
9,023
******

Cash book

Don - capital
account
Current account

Bal b/d
79,000 Charles capital
account
3,091
Loan account
Current account
Alan capital
account
Current account

4,200
38,100
8,000
7,478
21,000
9,023

Lesson Seven

346

Alan, Bob and Don Partnership


Balance Sheet as at 30 June 19X6
Fixed Assets

Cost

Depreciatio
n

Premises
Plant
Vehicles
Fixtures

120,000
35,000
1,100
2,000
168,100

Current Assets
Stock
Debtors
Cash

54,179
31,980
__760
86,919

Less Current
Liabilities
Creditors
Bank overdraft

19,036
5,710

Capital accounts
Alan
Bob
Don

67,000
67,000
67,000

Current Accounts
Alan
Bob
Don
Non current
liabilities
Loan Charles

NBV

3,091
3,091
3,091

(24,746)

62,173
230,273

201,000

9,273
210,273
20,000
230,273

NOTE:
i.
ii.
iii.
iv.

Goodwill introduced shared among the partners in the old partnership in


current profit sharing ratios.
Same case applies for any gain or loss in the revaluation of assets.
Goodwill written off in the new profit sharing ratios against the capital
accounts only for the new partners.
When there is no enough cash to be paid to the retiring partners, his balance
remains in the business as a loan.

347

Partnerships

(d)
Admission of a new partner.
When a new partner is admitted into the firm, this marks the end of the old
partnership and the beginning of a new one.
The new partner will have to bring in the capital that is due from him as per the
agreement and also pay for a share of the goodwill.
Goodwill is credited to the partners account(only the old) and is again written off
by debiting the partners account(inclusive of the new one in the new Profit
Sharing Ratio).
If the admission is taking place part way through the financial period, then the new
partner will be entitled to the profits or losses for the remaining part of the
financial period. (i.e from the point of joining the partnership).
Care should be taken when apportioning interest on capital, salaries and profits
because of the changes
Example:
The following was the partnership trial balance as at 30 April 2001:
Sh.
Sh.
Fixed capital accounts
Rotich
750,000
Sinei
500,000
Current accounts
Rotich
400,000
Sinei
300,000
Leasehold premises (purchased 1 May 2000)
2,250,000
Purchases
4,100,000
Motor vehicle (cost)
1,600,000
Balance at bank
820,000
Salaries (including partners drawings)
1,300,000
Stocks: 30 April 2000
1,200,000
Furniture and fittings (cost)
300,000
Debtors
225,000
Accountancy and audit fees
105,000
Wages
550,000
Rent, rates and electricity
310,000
General expenses (Sh.352,400 for the six months
to 31 October 2000)
660,000
Cash introduced Tonui
1,250,000
Sh.
Sh.
Sales (Sh.3,500,000 to 31 October 2000)
8,750,000
Accumulated depreciation: 1 May 2000
Motor vehicle
300,000
Furniture and fittings
100,000
Creditors
1,070,000
13,420,000
13,420,000
Additional information:
1.
On I November 2000 Tonui was admitted as a partner and from that date
profits and losses were to be shared on the ratio 2:2:1. For the purposes of
this admission, the value of goodwill was agreed at Sh.3, 000,000. No
account for goodwill was to be maintained in the books, adjusting entries for

Lesson Seven

2.
3.
4.
5.
6.
7.

8.
9.

348

transactions between the partners being made in their current accounts. On


that date, Tonui introduced Sh.1,250,000 more into the firm of which
Sh.375,000 comprised his fixed capital and the balance was credited to his
current account.
Interest on fixed capitals was still to be allowed at the rate of 10% per annum
after Tonuis admission. In addition, after Tonuis admission, no interest was
to be charged or allowed on current accounts.
Any apportionment of gross profit was to be made on the basis of sales.
Expenses, unless otherwise indicated were to be apportioned on a time basis.
A charge was to be made fro depreciation on motor vehicle and furniture and
fittings at 20% and 10% per annum respectively, calculated on cost.
On 30 April, the stock was valued at Sh.1,275,000.
Salaries included the following partners drawings:
Rotich Sh.150,000, Sinei Sh.120,000 and Tonui Sh. 62,500
A difference in the books of Sh.48,000 had been written off at 30 April 2001 to
general expenses, which was later found to be due to the following clerical
errors:
Sales returns of Sh. 32,000 had been debited to sales returns but had not
been posted to the account of the customer concerned;
The purchases journal had been undercast by Sh.80,000
Doubtful debts (for which full provision was required) amounted to Sh.30,000
and Sh.40,000 as at 31 October 2000 and 30 April 2001 respectively.
On 30 April 2001, rates and rent paid in advance amounted to Sh.50,000 and
a provision of Sh.15,000 for electricity consumed was required.

Required:
a) Trading and profit and loss account for the year ended 30 April 2001.
(9
marks)
b) Partners current accounts for the year ended 30 April 2001
(4 marks)
c)
Balance sheet as at 30 April 2001
(7 marks)
(Total: 20 marks)
Solution
a) ROTICH, SINEI AND TONUI
TRADING, P ROFIT AND LOSS ACCOUNT FOR THE YEAR ENDED 30
APRIL 2001
Sh.
Sh.
Sales
8,750,000
Less: cost of sales
Opening stock
1,200,000
Purchases
4,180,000
5,380,000
Less: Closing stock
(1,275,000) 4,105,000
Gross
Profit
4,645,000

349

Partnerships

Gross profit
S
TS

GP

Expenses
Dep. Motor
Vehicle
Furniture
Salaries
Accountancy
fees

1.3.20003.10.2000
Sh
Sh
1,858,00
0

1.11.200030.4.2001
Sh
Sh
2,787,00
0

Sh

160,0
00

160,00
0

320,00
0

15,00
0
483,7
50

15,000

30,000

483,75
0
52500

967,50
0
105,00
0

275,00
0
137,50
0
359,60
0

550,00
0
275,00
0
612,00
0

52,50
0
275,0
00
137,5
00
362,4
00

Wages
Rent, rates,
electricity
General
expenses
Prov. For
depreciation

30,00
0

(1,506,1
50)

10,000

(1,393,3
50)

40,000

Net Profit
351,850

Sh
4,645,00
0

2,899,50
0
1,745,50
0

1,393,65
0

Less: Interest on
capital
Rotich
37,50
0

37,500

75,000

25,00
0
-

25,000

50,000

Sinei
Tonui

(62,500)
Balance of profit
shared
Rotich

Sinei

18,750

289,350
192,9
00
96,45
0

(81,250)

18,750

1,312,40
0
524,96
0

717,86
0

524,96
0

621,41
0

(143,750
)
1,601,75
0

Lesson Seven

Tonui -

350

(289,350
)

262,48
0

(1,312,4
00)

262,48
0

(1,601,7
50

b)

Goodwill
w/o
Capital
A/C

R
Sh.
1,200,0
00

S
Sh.
1,200,0
00

Current Account
R
Bal b/d
Sh.

T
Sh.
600,00
0
375,00
0

Drawings
150,00
0

120,00
0

62,500

400,00
0
Cash
book
Goodwill
(2:1)
Interest
on
capital
Profit
share

Bal c/d

1,842,8
60
3,192,8
60

c)

651,41
0
1,971,4
10

S
Sh.
300,00
0

493,73
0
1,531,2
30

C
Sh.
-

1,250,00
0
2,000,0
00

1,000,0
00

75,000

50,000

18,750

717,86
0

621,41
0

262,480

3,192,8
60

1,971,4
10

1,531,23
0

Rotich, Sinei and Tonui


Balance Sheet as at 30 April 2001
Non-Current Assets
Leasehold premises
Furniture and Fittings
170,000
Motor vehicle

Sh
2,250,000
1,600,000
4,150,000

Sh.
300,000

Sh.
2,250,000
(130,000)

(620,000
980,000
(750,000) 3,400,000

351

Partnerships

Current Assets
Stock
Debtors
Less Provision
Prepayments
Balance at bank
Current Liability
Creditors
Accruals

1,275,000
193,000
(40,000)

1,070,000
15,000

153,000
50,000
820,000
2,298,000
(1,085,000)
1,213,000
4,613,000

Capital: Rotich
Sinei
Tonui
Current Account: Rotich
Sinei
Tonui
(d)

750,000
500,000
375,000
1,625,000
1,842,860
651,410
493,730

2,988,000
4,613,000

The adjusting entries on admission of a new partner should be made to the


capital account (i.e) for any introduction of goodwill and revaluation of assets
Some of the adjustments may also be made in the current accounts if
adjustments are made in the capital account and the admission is partway
through the financial period, then any interest to be charged on capital will
be based on the adjusted capital balance.
If the adjustments are made in the current account then there will be no
change on the capital balance and therefore no change on the interest
charged on the capital balances.

(e)

Retirement of a partner
When a partner retires (i.e.) leaves the firm and the others partners are left to
continue with the business then the retirement marks the end of one
partnership and the start of a new one.
The partner who is leaving should be paid all the amounts due to him. This
include:
1)
Capital balance
This will be all the amounts the partner has invested in the firm. Some firms
may not be able to refund the amount in full and therefore it may be
transferred t o a loan account whereby interest will be paid on the balance.
2) Goodwill
Because this partner contributed to the improvement (existence) of the
partnership therefore it will be fair to pay him his share of the goodwill.
Goodwill is introduced to the accounts in the old profit sharing ratio ((i.e.)
credited to all the partners capital accounts in the old profit sharing ratio),
then written off from the accounts by debiting the capital accounts of the
remaining partners in the new profit share ratio.

Lesson Seven

352

2) Credit balance on the current account


This amount due to the partner is paid directly from the cashbook or
transferred to the
capital account whereby the total cash payable is to be determined.
The transfer is made by:
Dr. Current account
Cr. Capital account
4) Share of profits
If the retirement takes place during the financial period, then the retiring
partner is entitled to take profits made up to the point of retirement. Any
interest of capital, salaries and balance of profit shared in profit share ratio
will be credited to the partners current account.
Therefore the profit and loss account will be split between the two periods
and appointment of profits done and this will be based on the terms of the
partnership in each period.
EXAMPLE 7.9
May 2002 Question 3
Kyamba, Onyango and Wakil were partners in a manufacturing and retail
business and shared profits and losses in the ratio 2:2:1 respectively
Given below is the balance sheet of the partnership as at 31 March 2001.
Balance sheet as at 31 March 2001
Assets
Sh.
Sh.
Non-current assets:
Fixed assets
465,000
Current assets:
Stock
294,000
Debtors
209,000
503,000
968,000
Capital and liabilities:
Capital accounts:
Kyamba
160,000
Onyango
140,000
Wakil
200,000
500,000
Current accounts:
Kyamba
65,300
Onyango
49,000
Wakil
53,000
167,300
667,300
Current Liabilities:
Bank overdraft
48,000
Trade creditors
252,000
300,700
968,000

353

Partnerships

Additional information:
1. On 1 April 2001, Wakil retired from the partnership and was to start a business
as a sole trader while Kyamba and Onyango continued in partnership.
2. On retirement of Wakil, the manufacturing business was transferred to him
while Kyamba and Onyango continued with the retail business
The assets and liabilities transferred to Wakil were as follows:
Net book value
Transfer value
Sh
Sh.
Fixed assets
260,000
306,000
Stocks
166,000
157,000
Debtors
172,000
165,000
Creditors
156,000
156,000
Wakil obtained a loan from a commercial bank and paid into the partnership
the net amount
due for him.
3. On retirement of Wakil form the partnership, goodwill was valued at Sh.200,
000 but was not to be maintained in the books of the partnership of Kyamba
and Onyango.
4.
After retirement of Wakil on 1 April 2001, Kyamba and Onyango agreed on the
following terms and details of the new partnership.

Kyamba and Onyango to introduce additional capital of Sh.48, 000 and


Sh.68, 000 respectively.
Each partner was entitled to interest on capital at 10% per annum with
effect from 1 April 2001 and the balance of the profits be shared equally
after allowing for annual salaries of Sh.72, 000 to Kyamba and Sh.60, 000
to Onyango.

5. The profit of the new partnership before interest on capitals and partners
salaries was Sh.240,000 for the year ended 31 March 2002.
6. The profits made by the new partnership increased stocks by Sh.100,000,
debtors by Sh.90,000 and bank balance by Sh.50,000.
7. Drawings by the partners in the year were Kyamba Sh.85,000 and Onyango
Sh.70,000.
Required:
a)
Profit and loss and appropriation account for the year ended 31 March 2002.
(4 marks)
b)
Capital accounts for the year ended 31 March 2002
(4 marks)
c)
Current accounts for the year ended 31 March 2002.
(4 marks)
d)
Balance sheet of the new partnership as at 31 March 2002.
(8 marks)
(Total: 20 marks)

Lesson Seven

354

SOLUTION
a) Kyamba and Onyango
Profit and loss appropriation account for the year ended 31.3.2002
Sh
Net profit for the year
Less: Interest on capital
Kyamba
Onyango

Sh.
240,000

20,000
20,000

(40,000)
200,000

Less: Salaries
Kyamba
72,000
Onyango
60,000
Balance of profits shared in PSR
Kyamba
34,000
Onyango
34,000
b)
(2) Goodwill
in New PSR
(4) Fixed
Assets

K
100,00
0

O
100,00
0

Stocks
Debtors

Bal c/d

200,00
0

(132,000)
68,000
(68,000)

CAPITAL ACCOUNT
K
Bal b/d
160,0
00
306,0 (1)Goodwill in
00
old PSR
80,00
0
157,0 Cashbook
00
48,00
0
165,0 Profit on
00
transfer in
12,00
old PSR
0
Creditors
W

200,00
0

O
140,0
00

W
200,0
00

80,00
0

40,00
0
-

68,00
0
12,00
0

156,0
00

Current
account (3)

53,00
0

Cash book
(**)
300,00
0

c)

3000,0
00

628,0
00

CURRENT ACCOUNT

6,000

300,0
00

300,0
00

173,0
00
628,0
00

355

Partnerships

Capital

K
Sh
-

O
Sh
-

W
Sh
53,000

Bal b/d

K
Sh
65,300

O
Sh
49,00
0

Drawing
s

85,00
0

70,000

Interest on
capital
Salaries

20,000

W
sh
53,00
0
-

20,00
0
72,000

60,00
0

Bal c/d

106,3
00

93,000

191,3
00

163,00
0

Share of
profits

53,000

34,000

191,300

KYAMBA AND ONYANGO


Balance Sheet as at 31 March 2002.
Non-Current Assets
Current Assets
Stock
Debtors
127,000
Bank
490,300
Liabilities
Creditors
(96,000)
Capital:
Kyamba
Onyango

Sh.
Sh.
205,000
228,000
135,300
394,300
599,300
200,000
200,000
400,000

Current:
Kyamba
Onyango

106,300
93,000

b)

199,300
599,300

Bank
Working capital
Kyamba- capital
Onyango capital
Increase

34,00
0

173,000
Bal b/d 48,700
48,000
Drawings
68,000 Kyamba
85,000
50,000 Onyango
10,000
_______ Bal c/d
135,300
339,000
339,000

163,0
00

53,00
0

Lesson Seven

356

Workings:
Non Current Assets:
Bal b/f
Transfer
Balance

465,000
260,000
205,000

Stock:
Bal b/f
Transfer
Increase

294,000
(166,000)
100,000
228,000

Bal b/f
Transfer
Increase

209,000
(172,000)
90,000
127,000

Bal b/f
Transfer

252,000
156,000
96,000

Debtors:

Creditors:

EXAMPLE 7.10
Upp and Downe are in partnership. The following trial balance has been
extracted from their books of account as at 31 March 19 2 after their trading
and profit and loss account has been prepared, but before any consequent
adjustments have been made to the partners respective capital accounts.
Dr.
Cr.
Capital accounts (as at 1 April 19 1):

Upp
60,000
Downe
40,000
Cash
6,600
Creditors
29,250
Debtors
201,000
Downe: goods withdrawn
400
Drawings:
Upp (all at 31 December 19 1)
20,000
Downe (all at 30 September 19 1) 15,000
Fixed assets: at cost
200,000
Accumulated depreciation
90,000
Accrued interest on Upps Loan account
10,000
Loan account: Upp
50,000
Net profit for the year to 31 March 19 2)
179,750
Salary: Downe
12,000

357

Partnerships

Stocks
3,500
Upp: private expenses paid (on 31 March 19 2)
459,000

500
459,000

Additional information
1.
The partnership agreement contains the following provisions:
a) Profits and losses are to be shared equally;
b) Current accounts are not to be kept;
c) The partners will be entitled to interest on their capital account balances as
at 1 April in each year at a rate of 15% per annum;
d) The partners will be charged interest on any cash drawings made during the
year at a rate of interest of 10% per annum;
e) Downe is to be allowed a salary of 16,000 per annum;
f) A specific loan made by any partner is to bear interest at a rate of 20% per
annum;
g) Upon the retirement of a partner the partnership assets and liabilities ar to
be revalued at their market value as at the date of retirement of the partner.
2.

Upp decided to retire at 31 March 19 2. In accordance with the partnership


agreement, the assets and liabilities were revalued as follows:

Car (to be retained by UPP)


10,000
Remaining fixed assets taken over by the new partnership
50,000
Stocks
5,000
Debtors
180,000
Creditors
35,000
Goodwill
40,000
Legal and other expenses connected with the partnership change
4,750
3.
Following Upps decision to retire, Downe invited Side to join him in
partnership as fro 1 April 19 2. Side agreed to pay 75,000 into the new
partnership as at that date as his capital contribution. Profits and losses are
to be shared in the proportion Downe 75% and side 25%. Goodwill is not to
be retained in the books of the partnership.
4.
Upp agreed to leave half of the total amount owing to him on his retirement as
a long run term loan in the new partnership, the other half being paid to him
in cash.
5.
It may be assumed that all of the transactions relating to the changes in the
respective partnerships take place on 1 April 19 2. The legal and other
expenses connected with the partnership changes were due for payment on
30 April 19 2.
Required:
Prepare:
a. Upp and Downes profit and loss appropriation account for the year to 31
March 19 2.
b. Upp, Downe and Sides respective capital accounts sufficient to reflect all of
the above transactions. and
c. Downe and Sides balance sheet as at 1 April 19 2 immediately after all of
the above transactions have

Lesson Seven

358

been settled.
(Detailed working should be submitted with your answer).
SOLUTION
(a)
Upp and Downe
Profit and loss appropriation account for the year ended 31 March 19-2

Net profit b/d


Add interest on drawings
Upp [3/12 x (10% x
20,000)]
Downe [16/12 x (10% x
15,000)]

179,750

500
750

1,250
181,000

Less:
Interest on capital
Upp [15% x 60,000]
Downe [15% x 40,000
Less: Salary Downe
Balance of profits shared
in PSR
Capital Upp (1/2)
- Downe (1/2)

9,000
6,000

(15,000)
166,000
(16,000)
150,000
75,000
75,000

150,000
_____-

359

Partnerships

(b)
Upp

Capital Accounts
Downe

Balances b/d

500

750

Loan interest

12,000
15,000

Appropriation
-salary

400

-interest on
capital
-residual profit

Appropriation
- interest on
drawings
Salary
Drawings
Private
expenses/goods
Car
Revaluation (deficit)
(W1)
[see workings after
(c)]
Loan (balancing
figure)
Balance c/d

Goodwill written
back (W2)
Balances c/d

20,00
0
500
10,00
0
20,00
0
103,0
00
_____154,0
00
Side

10,00
0
65,00
0
75,00
0

Upp

60,00
0
10,00
0

Downe

40,000

16,000
9,000

6,000

75,00
0

75,000

88,850
137,00
0

______
154,0
00

______
137,00
0

Downe

30,000

Balance b/d

Side

Downe

88,850

58,850

Cash

75,00
0
75,00
0

____-

20,000

88,850

(c)
Balance Sheet as at 1 April 19-2

Non current assets


Current assets
Stocks
5,000
Debtors
180,000
Cash (W3)
__5,100
190,100

50,000

88,850

Lesson Seven

360

Current liabilities
Creditors [35,000 + 4,750]

39,750

Working capital

150,350

Net assets employed


Financed by
Capital
Downe
Side

200,350
58,850
65,000
123,850

Loan
Upp (W4)

_76,500
200,350

Workings
W1
Revaluation
Debtors
Fixed assets (cost)
Stocks
Legal etc expenses
Creditors

201,00
0
200,00
0
3,500
4,750
35,000

______
444,25
0
Balance b/d

40,000
_____
40,000

Creditors
Provision for
depreciation
Capital Upp (car)
Fixed assets
Stocks
Debtors
Goodwill
Balance c/d (deficit)

Capital
-Downe (1/2)
-Upp (1/2)

29,250
90,000
10,000
50,000
5,000
180,00
0
40,000
40,000
444,25
0
20,000
20,000
40,000

W2
Goodwill
Revaluation

40,000

Capital
-Downe (75%)

30,000

361

Partnerships

_____
40,000

-Side (25%)

10,000
40,000

W3
Cash
Balance b/d
Capital
-Side
Bal b/d

6,600
75,000
81,600
5,100

Loan
-Upp [1/2 x
153,000]
Balance c/d

76,500
5,100
81,600

W4
Loan - Upp
Cash
Balance c/d

76,500
76,500

Balance b/d
Capital
-Upp

153,00
0
Balance b/d

50,000
103,00
0
153,00
0
76,500

Lesson Seven

362

REINFORCEMENT QUESTIONS
QUESTION ONE
1.

K. Kimeu and M. Maingi are in partnership as manufactures of Tick Toys,


Kimeu being responsible for the factory and Maingi for the sales. All
completed toys are transferred from the factory to sales department at
agreed price. Profits are shared on the following basis:
Factory
Sales Department
Kimeu
80%
40%
Maingi
20%
60%
The following trial balance has been extracted from the books at 31 March

1992:
Sh.
Freehold factory at cost
Factory plant, at cost
Provision for depreciation 1 April 1991
Delivery van, at cost
Provision for depreciation 1 April 1991
Stocks at 1 April 1991
Raw materials
Work-in-progress
Toys completed (30,000 at Sh.40)
Sales (45,500 toys)
Purchases of raw materials
Factory wages
Sales department wages
Expenses:
Factory
Sales Department
Provision for doubtful debts 1 April 1991
Trade debtors and creditors
Bank overdraft
Capital accounts:
Kimeu
Maingi
Drawings:
Kimeu
Maingi

Sh.
1,053,750
843,750
151,250
401,250
86,250

100,700
85,000
1,200,000
2,775,500
716,250
375,500
150,750
301,750
250,500
450,000

40,000
150,000
176,200
1,400,000
1,425,000

150,000
125,000
6,204,200

6,204,200

363

Partnerships

Additional information:
i
38,000 toys at Sh.45 each were manufactured and transferred to Sales
Department during the year. Tys in stock at the end of the year were to be
valued at Sh. 45 each. Stock of raw materials was Sh.79.50 and work-inprogress was valued at prime cost of Sh.126, 250 at 31 March 1992.
ii
Accrued expenses outstanding at 31 March 1992:
Factory
Sales Department
Sh.
Sh.
Expenses
52,250
27,000
Factory wages 7,000
iii
Provision for depreciation is to be made as follows:
- Factory plant
10% p.a. on cost
- Delivery van
20% p.a. on cost
iv
The general provision for bad debts is to be maintained at 10% of the trade
debtors.
Required:
Manufacturing, trading and profit and loss accounts for the year ended 31 March
1992 and a balance sheet as at that date.
(20 marks)
QUESTION TWO
Amis, Lodge and Pym were in partnership sharing profits and losses in the
ratio 5:3:2. The following trial balance has been extracted from their books
of accounts as at 31 March 19-8:

Bank interest received


Capital accounts (as at 1 April 19-7):
Amis
80,000
Lodge
15,000
Pym
5,000
Carriage inwards
4,000
Carriage outwards
12,000
Cash at bank
4,900
Current accounts:
Amis
1,000
Lodge
500
Pym
400
Discount allowed
10,000
Discount received
4,530
Drawings:
Amis
25,000
Lodge
22,000
Pym
15,000
Motor vehicles:
80,000
Accumulated depreciation (at 1 April 19-7)
20,000
Office expenses
30,400

Lesson Seven

364

Plant and machinery:


At cost
100,000
Accumulated depreciation (at 1 April 19-7)
36,000
Provision for bad and doubtful debts
(at 1 April 19-7)
420
Purchases
225,000
Rent, rates, heat and light
8,800
Sales
404,500
Stock (at 1 April 19-7)
30,000
Trade creditors
16,500
Trade debtors
14,300
583,300
583,300
Additional information:
1. Stock at 31 arch 19-8 was valued at 35,000.
2. Depreciation on the fixed assets is to be charged as follows:
a. Motor vehicles 25% on the reduced balance
b. Plant and machinery 25% on the original cost.
There were no purchases or sales of fixed assets during the year to 31
March 19-8.
3. The provision for bad and doubtful debts is to be maintained at a level
equivalent to 5% of the total trade debtors as at 31 March 19-8.
4. An office expense of 405 was owing at 31 March 19-8, and some rent
amounting to 1,5000 had been paid in advance as at that date. These items
had not been included in the list of balances shown in the trial balance.
5. Interest on drawings and on the debit balance on each partners current
account is to be charged as follows:

Amis
1,000
Lodge
900
Pym
720
6. According to the partnership agreement, Pym is allowed a salary of 13,000
per annum. This amount was owing to Pym for the year to 31 March 19-8,
and needs to be accounted for.
7. The partnership agreement also allows each partner interest on his capital
account at a rate of 10% per annum. There were no movements on the
respective partners capital accounts during the year to 31 March 19-8, and
the interest had not been credited to them as at that date.
Required:
a) Prepare the Partners trading, profit and loss account for the year ended
31 March 19-8
b) The partners current accounts and a balance sheet as at 31 March 19-8

365

Partnerships

QUESTION THREE
Amber and Beryl are in partnership sharing profits in the ratio 60:40 after
charging annual salaries of 20,000 each. The regularly make up their
accounts to 31 December each year.
On July 1996 they admitted Coral as a partner and agreed profits shares
from that date of 40% Amber, 40% Beryl and 20% Coral. The salaries
credited to Amber and Beryl ceased from 1 July 1996.

Lesson Seven

366

The partnership trial balance at 31 December 1996 was as follows:

Capital accounts as at 1.1.96:


Amber
Beryl
Capital account Coral (see note (d) below)
Current accounts as at 1.1.96
Amber
Beryl
Drawing accounts
Amber
Beryl
Coral
Loan account Amber
Sales
Purchases
Stock 1.1.96
Wages and salaries of staff
Sundry expenses
Provision for doubtful debts at 1.1.96
Freehold land at cost (see not (e) below)
Buildings: cost
Aggregate depreciation 1.1.96
Plant, equipment and vehicles: cost
Aggregate depreciation 1.1.96
Trade debtors and creditors
Cash at bank

280,000
210,000
140,000
7,000
6,000

28,000
24,000
15,000
50,000
2,000,000
1,400,000
180,000
228,000
120,000
20,000
200,000
250,000
30,000
240,000
420,000
38,000
3,143,000

50,000
350,000
3,143,000

In preparing the partnership accounts the following further information is to


be taken into account:
a) Closing stock at 31 December 1996 was 200,000
b) Debts totaling 16,000 are to be written off and the provision for
doubtful debts increased by 10,000.
c) Provision is to be made for staff bonuses totaling 12,000.
d) The balance of 140,000 on corals capital account consists of 100,000
introduced as capital and a further sum of 40,000 paid for a 20% share
of the goodwill of the partnership. The appropriate adjustments to deal
with the goodwill payment are to be made in the capital accounts of the
partners concerned, and no goodwill account is to remain in the records.
e) It was agreed that the freehold land should be revalued upwards on 30
June prior to the admission of Coral from 200,000 to 280,000. The
revised value is to appear in the balance sheet at 31 December 1996.
f) Ambers loan carries interest at 10% per annum and was advanced dot
the partnership some years ago.
g) Provide depreciation on the straight-line basis on cost as follows:
Buildings
2%

367

Partnerships

Plant, equipment and vehicles


h) Profits accrued evenly during the year.

10%

Require:
a) Prepare a trading account, profit and loss account and appropriation
account for the year ended 31 December 1996 and a balance sheet as at
that date.
(17 marks)
b) Prepare the partners capital accounts and current accounts for the year
in columnar form.
(7 marks)
(Total: 24
marks)
QUESTION FOUR
Duke and Earl are in partnership operating a garage business named Aristocratic
Autos.
In addition to selling petrol and oil, the garage has a workshop where car repairs
and maintenance are carried out and also a small showroom form which new and
second hand cars are sold.
For accounting purposes, each of these three activities is treated as a separate
department.
At 30th September 1986 balances extracted from the ledgers of Aristocratic Autos
comprised:

Cash sales:
Workshop (repair charges)
Petrol and oil
Showroom (car sales)
Credit sales:
Workshop (repair charges)
Petrol and oil
Showroom (car sales)
Stocks (at 1 October 1985):
Workshop (repair materials)
Petrol and oil
Showroom (cars)
Credit purchases:
Workshop (repair materials)
Petrol and oil
Showroom (cars)
Fixed assets (at 1 October 1985):
*Freehold buildings:
Workshop
Petrol and oil
Showroom
Plant, equipment and vehicles:
Workshop
Petrol and oil

32,125
32,964
8,500
65,892
41,252
81,914
1,932
3,018
20,720
23,860
41,805
52,100
12,600
14,200
38,000
65,180
22,900

Lesson Seven

Showroom
Provisions for depreciation (at 1 October
1985):
Freehold buildings:
Workshop
Petrol and oil
Showroom
*Note Freehold held in perpetuity
Plant, equipment and vehicles:
Workshop
Petrol and oil
Showroom
Fixed asset acquisitions during year (at
cost):
Plant and equipment:
Workshop
Petrol and oil
Showroom
Fixed asset disposal proceeds during the
year (see note (3)):
Plant and equipment:
Workshop
Salaries:
Showroom
Rates
Electricity
General expenses
Wages:
Direct:
Workshop
Petrol and oil
Indirect:
Workshop
Showroom
Creditors:
Workshop
Petrol and oil
Showroom
Bank/Cash:
Workshop
Petrol and oil
Showroom
Debtors:
Workshop
Petrol and oil
Drawings:
Duke
Earl
Current accounts (at 1 October 1985)
(credit balances):

368

17,450

5,060
7,100
19,390
48,254
17,077
9,451

26,210
4,250
1,060

5,200
10,200
26,738
9,453
10,692
34,050
5,602
6,810
4,160
4,225
5,602
15,250
316
1,605
30,470
1,365
537
12,190
9,740

369

Partnerships

Duke
Earl
Capital accounts:
Duke
Earl

9,750
10,477
50,000
40,000

Lesson Seven

370

Notes at 30 September 1986


1) Stocks at 30 September 1986:
Workshop
2,752
Petrol and oil
2,976
Showroom
25,310
2) Depreciation is calculated using the straight-line method (assuming no
residual value) and is applied to the original cost of the asset at eh end of
the financial year, using the following rates:
%
Freehold buildings
20
Plant, equipment and vehicles
20
The depreciation charges for the current year have not yet been posted to
the accounts.
The freehold buildings are temporary structures with a five year life.
3) No entries have yet been made to transfer the cost (19,500) and
accumulated depreciation (15,633) of the workshop plant sold during the
year.
4) Accruals at 30 September 19861

Wages:
Direct:
Workshop
113
Petrol and oil
83
Indirect:
Workshop
214
Showroom
231
Electricity
517
General expenses
1,304
5) Prepayments at 30 September 1986

Rates
13,300
6) Rates and electricity are apportioned over departments on the basis of the
original cost of freehold buildings at the end of the current financial year.
7) General expenses are apportioned over departments on the basis of turn
over for the current year.
8) Duke and Earl are credited with interest on their respective capital account
balances at the rate of 5% per annum.
Required:
Prepare, using separate columns for each department and the business as a whole;
a) A departmental trading and profit and loss account for Aristocratic Autos for
the year ended 30 September 1986.
(20
marks)
b) A departmental balance sheet for Aristocratic Autos as at 30 September
1986.
(14 marks)
(Total: 34 marks)

371

Partnerships

Lesson Seven

372

QUESTION FIVE
Reg, Sam and Ted are in partnership, sharing profits and losses equally. Interest
on capital and partnership salaries is not provided. The position of the business at
th end of its financial year is:

Capital
accounts:
Reg
Sam
Ted
Current
Accounts:
Reg
Sam
Ted
(debit)
Creditors

Balance Sheet 30 June 19-6

Buildings
9,000
8,000
8,000

Equipment
Stock
Debtors
25,00 Bank
0

17,000
3,300
900
2,020
2,840

140
200
340
100
240
___82
0
26,06
0

_____
26,060

Reg died suddenly on 31 October 19-6.


The partnership agreement provides that in the event of the death of a partner the
sum to be paid to his estate will be the amount of his capital and current account
balances at the last financial year-end adjusted by his share of profit or loss since
that date together with his share of goodwill. A formula for calculation of goodwill
is given, and its application produced a figure of 7,500. no goodwill account is to
remain in the books after any change of the partnership constitution.
The stock value at 31 October has been calculated and all other accounts balanced
off, including provisions for depreciation, accrued expenses and prepaid expenses.
This results in the following position at 31 October.

buildings
17,000
Equipment (including additions of 400)
Stock
1,100
Debtors
2,230
Bank balance
3,370
Creditors
980

3,480

373

Partnerships

There were no additions to, or reductions of, the capital accounts during the four
months, but the following drawings have been made:
Reg
Sam
Ted

2,000
1,600
1,800

Lesson Seven

374

It has also been agreed that the share of a deceased partner should be
repaid in three equal installments, the first payment being made as on the
day after the day of death.
The surviving partners agree that Abe (son of Reg) should be admitted as a
new partner with effect from 1 November, and it is agreed that he will bring
into the business 4,000 as his capital together with a premium for his share
of the goodwill (using the existing valuation). The new profit-sharing
agreement is: Sam, two-fifths; Ted, tow-fifths; and Abe one-fifth.
Show the partnership Balance Sheet as at 1 November 19-6, on the
assumption that the above transactions have been completed by that date.

CHECK YOUR ANSWERS WITH THOSE GIVEN IN LESSON 9 OF THE


STUDY PACK

375

Partnerships

COMPREHENSIVE ASSIGNMENT No.3


TO BE SUBMITTED AFTER LESSON 7
To be carried out under examination conditions and sent to the Distance Learning
Administrator for marking by the College.
EXAMINATION PAPER.
THREE HOURS.

TIME ALLOWED:

ANSWER ALL QUESTIONS


QUESTION ONE
The Dohray Amateur Musical Society has a treasurer who is responsible for
receipts and payments, which he records in cash and bankbooks. Periodically,
these books are handed over to the firm of certified accountants that employs you.
One of your tasks is to prepare the final accounts of the Society. As a preliminary
step, you have prepared the receipts and payments account (rounded to the
nearest 1) for the year ended 31 May 1985. This is shown below, together with
the explanatory notes which the treasurer has supplied to enable you to
understand the nature o f some of the items.
Dohray amateur Musical Society
Receipts and Payments Account
For the year ended 31 May 1985
Receipts
Cash

Payments
Ban
k

Cash

Bank

Opening balances
b/d
Debtors:
members
Joining fees
(note 1)
Annual
subscriptions
(Note 2)

31

190

285

309 Creditors: trade


Fixed assets (note
4)
160
Musical
instruments
Trophies
70 Creditors: trade

522
83

Lesson Seven

Annual concert
(note 3)
Takings
Sales of goods
(note 4)
Musical
instruments
Prize moneys
(note 7)
Sponsorship grant
(Note 5)
Refreshment sales
Raffle profits
PAC grants (note
6)
Revenue
Capital

376

1,791
287
190

113
64

Purchase for
resale
(Note 4)
Sheet music

118

Annual concert
(note 3)
Hall booking fees

490

Printing of
publicity
300
Posters
Hire professional
Soloists
Musicians
100
Adjudication fees
400 Musical Festivals
(note 7)
2,91
Entrance fees
0
Hire of buses
Honoraria (note 8)
Secretary
Treasurer
R.M.F.C affiliation
fee
(Note 9)
Rent of societys
premises
(Note 10)
Refreshment
purchases
Bank charges
Sundry expenses
Transfers to bank
a/c

Transfers from
cash a/c

Closing balances c/d


3,091

4,24
9

112
236
174

250
281
150
100
72
510
72
42
60
2,910
49

723

3,091

4,249

Explanatory notes supplied by the treasurer


1) On joining the Society, members pay a non-returnable fee of 10 (before 1
June 1982, the fee had been ). It has been found from experience that, on
average, members remain in the Society for five years. On this basis, one

377

Partnerships

fifth of each joining fee is credited to Income and Expenditure account each
year.
New members statistics are
During the
year
Ended 31
May
1981
1982
1983
1984
1985

Number
of
new
members
No.
20
24
32
27
35

Joining fees in
Suspense at 31
May 1984

20
48
192
216
Nil
476

2) Annual subscriptions are due on 1 June each year. It is Societys policy to


credit these to income and expenditure account on an actual receipts basis,
not an accruals basis. However, if subscriptions are received in advance, the
amounts are credited to income and expenditure account for the year, which
they are paid.
3) The Societys major money raising event is its annual public concert. This is
given in a large hall, which the Society hires. The society also hires
professional musicians and soloists and has to pay the fees of the
adjudicators (judges).
4) The society buys trophies (silver bowls and shield) to present to the winners
of individual musical items at the annual concert. It also buys musical
instruments some of which are for use by the members and others for resale
to the members. Musical scores and sheets are also bought for resale to the
members.
5) A local building company has given a grant to the Society for a period of
three years in return for publicity. This sponsorship grant was received in
full on 1 June 1984 and is being credited to income and expenditure account
in equal installments in each o the three years to 31 May 1987.
6) The performing Arts Council (PAC) has awarded the Society an annual grant
towards the running costs. In addition the PAC makes capital grants. The
societys policy is to hold capital grants in suspense and to release each
years grant to income and expenditure account over a period of five years,
from the year of grants onwards. At 31 May 1984 capital grants held in
suspense were analyzed as follows:
In respect of
year
Ended 31
May
1981
1982

Capital
grants
Suspense

30
70

Lesson Seven

1983
1984

378

120
120
340

7) Throughout the year, the Society competes at various musical festivals.


Cash prizes won by individual members are retained by the Society and
credited to income and expenditure account in order to reduce the cost of
attending the festivals.
8) The offices of secretary and treasurer are unpaid but the society gives each
of them an ex-gratia (honorary) cash award, termed an honorarium.
9) In order to participate in the musical festivals, the Society has to be
affiliated to the Regional Musical Festival Community (RMFC). The annual
fee, which has remained the same for a number of years, is paid on 1 March
in each year.
10)
The Society pays rent for its premises. The rental, which is inclusive of
rates, heating, lighting, cleaning etc. is reviewed annually on 31 March. The
payment shown in the receipts and payments account represents quarterly
payments in advance, as follows:
1984
30 June
30
Septembe
r
31
December

Payment

120
120
120
120

1985
31 March

150
510

The treasurer supplied further information as follows:


1) Creditors at 31 May

1984

Fixed assets
Musical instruments
Trophies
Purchases for resale
Sheet music
Musical instruments
2) Subscriptions
Payments in advance included in the
actual receipts for the year
3) Stocks at 31 May
Goods for resale

1985

79

119
23

14
45

13

20
39
30

40

379

Partnerships

Sheet music
31
52
Musical instruments
70
94
Refreshments not brought into account on the grounds that
It is not material in amount
4) Fixed assets (at cost) at 31 May
musical instrument
Trophies

1,378
247

There were no fixed asset disposals during the year


5) Provision for depreciation at 31 May
Musical instruments
704
Trophies
96
Depreciation is calculated on the cost of these assets at the end of the
financial year. The straight-line method is employed using the following
assumed asset lives.
Musical instruments
Trophies

5 years
10 years

Required:
Prepare for the Dohray Amateur Musical Society
a) The Income and Expenditure account for year ended 31 May 1985,
showing the surplus or deficit on each of the activities:
and
b) The Balance Sheet at that date.
Note:
WORKINGS are an integral part of the answer and must be shown.
(34 marks)
QUESTION TWO
A client of the firm of accountants by which you are employed is interested in
buying a road transport business from the widow of its deceased owner.
The senior partner of the practice is investigating various aspects of the business
and has delegated to you the task of discovering the amount of investment in
vehicles at the end of each of the financial years ended 30 September 1980 to 1983
inclusive. The business had commenced operations on 1 October 1979.
The only information available to you is the fact that the owner calculated
depreciation at a rate of 20% per annum, using the Reduction Balance method,
based on the balance at 30 September each year, and copies of certain ledger
accounts which are reproduced below:

1981

Provision for depreciation of vehicles

1980

1
Balance b/
32,000
Oct.
1981

Lesson Seven

30
Sept.

380

Balance c/d

57,600 30
Sept.
57,600

Profit and
loss

57,60
0

1982
30
Sept.

Disposals
Balance c/d

1983
30
Sept.

Disposals
Balance

1
Oct.
10,800 1982

Balance b/d

73,440 30
Sept.

Profit and
loss
includes
10,000
(depreciation
on 1982
acquisitions)

26,640

___
__
84,24
0

29,280 1
Oct.
79,328 1983
30
Sept.

1
Oct.

Vehicles
(vehicles
Originally
acquired
On 1
October

57,600

_____
84,24
0

_______
108,6
08

1982
30
Sept.

25,600

Disposals

1982
30
Sept.
30,000

Balance b/d

73,440

Profit and
loss
(includes
20,000
Depreciation
on
1983
acquisitions)

35,168

Balance b/d

______
10860
8
79,328

Provision for
Depreciation

10,800

Bank

16,000

381

Partnerships

1979)
Profit and
loss
______
30,00
0

______
30,00
0

1983
30
Sept.

Vehicles
(vehicles
Originally
acquired
On 1
October
1979)
Profit and
loss

3,200

1983
300
Sept.

60,000

Provision for
Depreciation

29,280

Bank

42,000

11,280
______
71,28
0

______
71,28
0

Required:
a) Calculate the cost of asset, vehicles, held by the business at 30
September in each of the years 1980 to 1983 inclusive
(4 marks)
b) Show the detailed composition of the charge for depreciation of the
vehicles to profit and loss account at 30 September 1981, 1982 and 1983.
(9 marks)
All workings must be shown.
(13
marks)

QUESTION THREE
The trial balance of Happy Bookkeeper Ltd, as produced by its bookkeeper
includes the following items:
Sales ledger control account 110,172
Purchase ledger control account
Suspense account (debit balance)

78,266
2,315

You have been given the following information:


i.

The sales ledger debit balances total 111,111 and the credit balances total
1,234.

Lesson Seven

382

ii.
The purchase ledger credit balances total 77,777 and the debit balances total
1,111.
iii.
The sales ledger includes a debit balance of 700 for business X, and the
purchase
ledger includes a credit balance of 800 relating to the same
business X. Only the net amount will eventually be paid.
iv.
Included in the credit balance on the sales ledger is a balance of 600 in the
name of H. Smith. This arose because a sales invoice for 600 had earlier
been posted in error from the sales daybook to the debit of the account of M.
Smith in the purchase ledger.
v.
An allowance of 300 against some damaged goods had been omitted from
the appropriate account in the sales ledger. This allowance had been included
in the control account.
vi.
An invoice for 456 had been entered in the purchase daybook as 654.
vii.
A cash receipt from a credit customer for 345 had been entered in the
cashbook as 245.
viii.
The purchase daybook had been overcast by 1,000.
ix.
The bank balance of 1,200 had been included in the trial balance, in error, as
an overdraft.
x.
The bookkeeper had been instructed to write off 500 from customer Ys
account as a bad debt, and to reduce the provision for doubtful debts by
700. By mistake, however, he had written off 700 from customer Ys
account and increased the provision for doubtful debts by 500.
xi.
The debit balance on the insurance account in the nominal ledger of 3,456
had been included in the trial balance as 3,546.
Required:
Record corrections in the control and suspense accounts. Attempt to reconcile the
sales ledger control account with the sales ledger balances, and the purchase
ledger control account with the purchase ledger balances. What further action do
you recommend?
(25 marks)

383

Partnerships

QUESTISON FOUR
Ray Dyo, Harry UII and Val Vez are in partnership, trading under the name of
Radtel Services, as radio and television suppliers and repairers, sharing profits and
losses in the ratio one half, one third and one sixth, respectively. Val Vez works
full-time in the business with responsibility for general administration for which
she receives a partnership salary of 4,000 per annum.
All partners receive interest on capital at 5% per annum and interest on any loans
made to the firm, also at 5% per annum.
It also had been agreed that Val Vez should receive not less than 4,000 per annum
in addition to her salary. Any deficiency between this guaranteed figurer and her
actual aggregate of interest on capital, plus residual profit (or less residual loss)
less interest on drawings, is to be borne by Dyo and UII in the ratio in which they
share profits and losses; such deficiency can be recouped by Dyo and UII at the
earliest opportunity during the next two consecutive years provided that Val Vez
does not receive less than the guaranteed minimum described above. During the
year ended 30 September 1983, Dyo and UII had jointly contributed a deficiency of
1,500.
Radtel Services rents two sets of premises - one, a workshop where repairs are
carried out, the other, a shop from which radio and television sets are sold. The
offices are situated above the shop and are accounted for as part of the shop.
The workshop and shop are regarded as separate departments and managed,
respectively, by Phughes and Sokkitt who are each remunerated by a basic salary
plus a commission of one ninth of their departments profits after charging their
commission.
On 30 September 1984, the trial balance of the firm was:

Stocks at 1 October 1993:


Shop (radio and television sets)
Workshop (spares, components etc.)
Purchases:
Radio and television sets
Spares, components etc.
Turnover:
Sales of radio and television sets

19,750
8,470
155,43
0
72,100
232,60
0
127,00
0

Repair charges
Wages and salaries (employees):
Shop and offices
Workshop
Prepaid expenses (at 30 September
1984)
Accrued expenses (at 30 September

54,640
18,210
640
3,160

Lesson Seven

1984)
Provision for doubtful debts at 1 October
1983
Rent and rates:
Shop and offices
Workshop
Stationery, telephones, insurance:
Shop and offices
Workshop
Heating and lighting:
Shop and offices
Workshop
Debtors
Creditors
Bank
Cash
Other general expenses:
Shop and offices
Workshop
Depreciation:
Shop and offices (including vehicles)
Workshop
Shop fittings (cost)
Workshop tools and equipment (cost)
Vehicles (cost)
Discount received:
Shop
Workshop
Bank loan (repayable in 1988)
Loan from Harry UII
Capital Accounts:
R. Dyo
H. UII
V. Vez
Current Accounts (after drawings have
been debited):
R. Dyo
H. UII
V. Vez
Loan interest:
Bank loan
Loan from H. UII
Provision for depreciation:
Shop fittings
Workshop tools and equipment
Vehicles

384

920
7,710
8,450
2,980
1,020
4,640
3,950
4,460
15,260
48,540
960
3,030
2,830
2,400
2,580
17,060
55,340
27,210
420
390
15,000
10,000
40,000
40,000
20,000
290
1,040
920
2,400
500
3,190
10,020
5,670
525,5
90

525,5
90

385

Partnerships

The following matters are to be taken into account:


1) Managers commissions.
2) Partnership salary (Vez).
3) Interest on partners capital accounts (these have not altered during the
year).
4) Interest on partners drawings; Dyo 70; UII 30; Vez 20.
5) Closing stocks: shop 31,080; workshop 10,220.
6) Provision for doubtful debts at 30 September 1984, 540.
7) Residual profits/Losses.
N.B. Loan interest and the movement in the provision for bad debts are
regarded as shop items.

Lesson Seven

386

Required:
a) Prepared columnar departmental trading and profit and loss accounts
and a partnership appropriation account for he year ended 30 September
1984 and the partnership balance sheet at that date.
(21 marks)
b) Complete the posting of the partners current accounts for the year.
(4 marks)
(25 marks)
QUESTSION FIVE
Ernie is a building contractor, doing repair work for local householders. His wife
keeps some accounting records but not on a double-entry basis.
The assets and liabilities of the business at 30 June 1997 were as follows:

Assets
Plant and equipment: cost
Depreciation to date
Motor Van: cost
Depreciation to date
Stock of materials
Debtors
Rent of premises paid in advance to 30
September 1997
Insurance paid in advance to 31 December
1997
Bank balance
Cash in hand
Liabilities
Creditors for supplies
Telephone bill owing
Electricity owing

12,600
5,800
9,000
6,500
14,160
9,490
750
700
1,860
230
3,460
210
180

His cash and bank transactions for the year from 1 July 1997 to 30 June 1998
are as follows:
Receipts
Opening balances
Receipts from
customers
Loan received
Proceeds of sale of
vehicles

Cash and Bank summary


Cash
Bank
Payments

230 1,860 Suppliers


52,64
0

150,8 Rent of premises


80
10,00 Insurance (to 31.12.98)
0
Purchase of plant and
equipment

Cash

Bank

83,99
0
3,600
1,600
8,400

387

Partnerships

Held at the beginning


of year
Cash paid into the
bank
Cash withdrawn from
bank
Closing balance

3,000 Purchase of new vehicle

48,26
0

24,04 Telephone
0
Electricity
2,100 Wages of repair staff
Miscellaneous expenses
Drawings by Ernie
Refund to customer
Cash paid into bank
Cash withdrawn from
bank
Closing balance

101,1
30

12,80
0
860

191,8
80

890
68,20
0
1,280
8,000
400
24,04
0
890
101,1
30

48,26
0
29,80
0
191,8
80

The following further information is available


1) Plant and equipment is to be depreciated at 25% per annum on the reducing
balance with a full years charge in the year of purchase.
2) The new motor vehicle was purchased on 1 January 1998. Ernies
depreciation policy is to charge depreciation at 25% per annum on the
straight-line basis with a proportionate charge in the year of purchase but
not in the year of sale.
3) The rent of the premises was increased by 20 % from 1 October 1997.
4) The loan of 10,000 was obtained from Ernies brother on 1 April 1998. It
carries interest at 10% per annum, payable on 30 September and 31 March.
5) At 30 June 1998, Ernie owed the following amounts:

Suppliers
4,090
Telephone
240
Electricity
220
Miscellaneous expenses
490
6) At 30 June 1998, amounts due from customers totaled 10,860. Of this
amount, Ernie considered that debts totaling 1,280 were bad and should be
written off.
7) Stock of materials at 30 June was 12,170
8) Ernie agreed to pay his wife 5000 for her assistance with his office work
during the year. This amount was actually paid in August 1998.
Required:
Prepare Ernies trading profit and loss account for the year ended 30 June
1998 and a balance sheet as at that date.

Lesson Seven

388

END OF COMPREHENSIVE ASSIGNMENT No.3


NOW SEND YOUR ANSWERS TO THE DISTANCE LEARNING CENTRE FOR
MARKING

Acknowledgement

389

LESSON EIGHT
COMPANY ACCOUNTS
Introduction:
COMPANY ACCOUNTS:
Limited companies come into existence because of the growth in size of business
and the need to have many investors in the business.
Partnerships were not suitable for such businesses because the membership is
limited to 20 persons.
Types of companies
There are 2 principle types of companies:
Private companies
These have the words limited at the end of the name. Being private, they cannot
invite the members of the public to invest in their ownership.
Public companies
There much larger in size as compared to private companies. They have the words
public limited company at the end of their name.
They can invite the members of the public to invest in their ownership and the
companies may be quoted on the stock exchange.
Share capital of a company.
The owners interest in a limited company consists of share capital. The share
capital is divided into shares. The investor will then pay for and be issued with the
shares and therefore, they become owners.
Each share has a flat value called Par value/face value/nominal value. (e.g.) If a
company decides to set up a share capital of Sh. 200,000, it may decide to issue:
200,000 shares of Sh. 1 each per value.
100,000 shares of Sh. 2 each per value.
400,000 shares of Sh. 50 each per value.
There are 2 main types of share capital
Preference share capital
This is made up of preference shares and a preference share carries the right to a
final dividend, which is expressed as a percentage of their par value. E.g. 10%
preference shares.
Preference shares do not carry a right to vote and therefore no control in the
company.
Ordinary Share capital
These are the most common shares. They carry no right to a fixed dividend but are
entitled to residual value of the business during winding up, and all profits after
the claim on all of the preference dividend have been paid. The more the no. of
ordinary share held, the higher the control.

FINANCIAL ACCOUNTING 1

Lesson Eight

390

Share capital may also have the following meaning:


Authorized share capital
Also called, registered or nominal capital. Is the total of the share capital which the
company is allowed to issue to shareholders. A company cannot issue more shares
than the amount that is authorized.
Issued share capital
This is the total of the share capital actually issued to the shareholders.
Called up share capital
This is the amount the shareholders have been asked to pay where the amount of
capital required is less than the issued share capital.
(e.g.) If a firm issues ordinary shares of 1 each and request the shareholders to
pay 60p. Assuming that the issued 100,000 shares, then the called up share capital
will be:
60p 100,000 = 60,000
Uncalled share capital
This is part of the issued share capital for which the company has not requested
for payment and therefore these amounts will be received in the future.
In the above (e.g.) because the firm had not requested for 40p, therefore the
uncalled capital is 40p 100,000 = 40,000.
Paid-up share capital
This is the total of the share capital, which has been paid for by the shareholders.
Illustration
A limited has an authorized share capital of 200,000 shares of 1 each out of which
only 150,000 share have been issued, Although the firm requested the
shareholders to pay 80p per share, the shareholders were able to pay 50p per
share.
Required:
Determine the:
Authorized share capital
Issued share capital
Called up share capital
Uncalled up share capital
Paid up share capital
Authorized share capital
200,000 = 200,000
Issued share capital
150,000 1 = 150,000
Called up share capital
150,000 80p = 120,000
Uncalled up share capital

391

Company Accounts

150,000 20 p = 30,000
Paid up share capital
150,000 50p = 75,000
The principal distinctions between unlimited partnerships and limited companies
are:
Unlimited Partnerships

Limited Companies

No separate Legal Entity


apart from its members

Separate legal entity,


which is not affect by
changes in its
membership. A company
may contract, sue or be
sued in its own name.
If the company is limited
by share, each
shareholder is limited to
the amount he has agreed
to pay the company for
share allotted.
A limited company must
have at least 2 members.
The maximum number of
shares is restricted to the
companys authorized
share capital.
Rights to management are
delegated to directors who
alone can act on behalf of
and bind the company.

Liability of each member


for debts of the firm is
unlimited.

Number of partners
limited to 20 except for
professional firms.

Every partner can


normally take part in the
management of the
business. He can legally
bind the firm by his
action.
Copy of accounts need
not be filed with the
Registrar of Companies
Although a written
Partnership deed is
desirable it is not
mandatory.
A partnership is subject
to the partnership Act
which can be varied by
mutual agreement.

Copies of accounts must


be registered with the
Registrar of Companies
A company is required to
have a memorandum and
articles of association
which defines powers and
duties of directors.
A company is subject to
the Companies Act the
provisions of which cannot
be varied.

Lesson Eight
The partners contribute
the capital by
agreement. The amount
need not be fixed.

A share in a partnership
cannot be transferable
except by the consent of
all partners.

A partnership is not
obliged to keep statutory
books of account and an
audit is not compulsory.

392

The authorized share


capital is fixed by the
memorandum of
association. It can be
altered by passing
ordinary resolution or by
the court.
In public companies
shares are freely
transferable. In private
companies share transfer
are subject to any
restrictions imposed by
the articles of association.
A company is required to
keep specialized
accounting records and is
subject to compulsory
audit.

Format Of Final Account


The P & L of a company, is the same as that of a sole trader, but there are
additional expenses that are unique to the company and therefore, they should be
included in the P & L A/C.
(e.g.)
Directors fees salaries and other expenses
Audit fees
Amortization e.g. goodwill
Debenture interest
In addition to the P & L A/C, just like a partnership has an appropriation A/C which
shows the allocation of the net profit for the period. Therefore, the format will be
as shown:
Format for Company Accounts
B Limited
Trading, profit and loss and Appropriation Account for the year ended
31.12

Sales
x
Less Returns inwards
(x)
x
Less Cost of Sales
x
Opening Stock
x
Purchases
x
Add Carriage in
x
(x)
x
Less purchase returns
x
(x)
(x)
Less Closing stock
x
Gross Profit

393

Company Accounts

Add incomes
Discount received
Profit on disposal (sale of Assets)
Income from investment (can also be
shown below)
Other incomes e.g. interest received from
bank
Less Expenses
Other expenses
Directors salaries/fees/---Audit fees
Debenture Interest
Amortization of good will
Operating profit for the period
Add investment income
Profit before tax
Taxation: Corporation tax
Transfer to deferred tax
Under or over provision
Profit after tax
Less: transfer to the general reserve
Less: Dividends
Preference dividend:
Final proposed

x
x
x

x
x
x
x
x

Ordinary dividend:
Interim paid
Final proposed
Retained profit for the year
Retained profit b/f
Retained profit c/d

B Limited
Balance sheet as at 31.12

Intangible Assets
Goodwill
Copyrights, patents
(Longterm) Investments (mkt
value sh x)

x
x
x
x
x
(x)
x
x
x

Interim paid

Non current Assets


Land & Building
Plant and Machinery
Fixtures, Furniture & Fittings
Motor vehicle

x
x
x
x
x
x

x
x
x
x
x

(x)
(x)
(x)
(x)
x

x
x
x
x
x

x
x
x

(x)
(x)
x

x
x
x
x

(x)
x
(x)
x

(x)
x
x
x

Lesson Eight
Current Assets
Stock
Debtors
Less provision for bad debts
Prepayments
(Short term) Investments
Cash at bank
Cash in hand
Current liabilities
Bank overdraft
Creditors
Accruals
Interest payable(debenture
interest)
Tax payable
Dividends payable

394

x
x
(x)

x
x
x
x
x
x

x
x
x
x
x
x

(x)

x
x
x
(x)
x

Financed by
Authorized share capital
100,000 ordinary shares of 1
each
100,000 preference shares of 1
each
Issued and Fully paid
80,000 ordinary shares of 1 each
50,000 10% preference shares of
1 each
Capital Reserves
Share premium
Revaluation Reserve
Capital Redemption Reserve
Revenue Reserves
General Reserve
Profit and loss A/C

x
x

x
x
x
x
x
x
x

x
x

x
x
x
x
x
x

Deffered tax A/C


Non Current Liabilities
10% debenture
Other Long term Loans
Directors salaries:
Salaries, fees and other expenses in relation to the directors are expenses as far as
company accounts are concerned.
This is different from that of Partnerships & Sole traders which are shown as
appropriations expenses.

395

Company Accounts

Audit fees
All companies are required to prepare the accounts which should be audited and
therefore any fees paid in relation to audit and accountancy is an expense.
Debenture interest
Loans taken up by companies are called debentures. The interest paid on these
loans are charged as an expenses and unpaid amount are shown as current
liabilities in the business.
The debenture is classified under non-current liability.
Corporation tax
Companies pay corporation tax on the profirs they earn. This is shown in the
accounts because a company is a separate legal entity unlike for sole traders and
partnerships whose tax is shown as drawings.
The tax is listed under those 3 items as shown in the appropriation (under/over
provision for previous period, transfer to deferred tax corporation tax for the year).
The under provision and corporation tax relate to direct liability to the government
and therefore is a deduction from the net profit for the period .
Transfer to deferred tax is to cater for future possible tax liability.
Assume that a firm had estimated that the corporation tax for the year ended
31.12.99 is 150,000. In 2000, the liability is now agreed at 160,000, which the
company pays and at the end of the year 2000, the company estimates that the tax
liability is 140,000.
Prepare a tax A/C and show the amount to be deducted as tax for the year (ignore
deferred tax).
(e.g.)

Taxation Account
Cashbook
Bal c/d

Under provision
Corporation tax

160,000
140,000
300,000

Bal b/d
150,000
Appropriation
150,000
300,000

10,000 (160 -150)


140,000

DIVIDENDS
Shareholders are also entitled to a share of profits made by the company and this
is because the shareholders do not make drawings from the company.
A company may pay dividends in 2 stages during the cause of the financial period:
Interim dividends
Is paid part way --- the financial period. (e.g.) after the 6 ----Final proposed
Is paid after the year-end or after the completion to final accounts.
If a company pays in these 2 stages then the dividend section of the P & L
appropriation should disclose interim paid and final proposed.

Lesson Eight

396

CAPITAL RESERVES
Amounts reflected in Capital reserves cannot be paid out or distributed to
shareholders. The three types of capital reserves are:
Share Premium: A share premium arises when accompany issues shares at a
price that is more than the par value. The share Premium may be applied in:

Paying un issued shares.


Writing off preliminary expenses.
Write off discounts on shares.

Example:
A Ltd wishes to raise capital by issuing 100,000 ordinary shares at 1 each (per
value) and the issue price (selling price) is 1.5 each.
The following are the entries to be made in the A/C.
Dr Cashbook
(100,000 1.5) 150,000
Cr Ordinary shares capital
(100,000 1)
100,000
A
Cr Sahre Premium /C
(100,000 0.5)
50,000
Issue of shares at a premium of 0.5
Revaluation Reserve: Any gain made on revaluation of non current Assets
especially for Land and buildings. When company sills its property to realize the
gain, the amount is transferred to the Profit and Loss Account.
Capital Redemption Reserve: A reserve created after redemption or purchase of
Preference shares without issuing new shares. The transfer is made from either
the share premium or the profit and loss account.
REVENUE RESERVES
This can be distributed and includes the retained profits (P & L Accounts) and the
General Reserves. Transfers are made from the Profits to the General reserves to
provide for expansion or purchase of non current assets. The General Reserves can
also be used to issue bonus Shares.
DEBENTURE LOANS
The term debenture is used when a limited company receives money on loan, and
certificates called debenture certificates are issued to the lender.
They are also called loan stock or loan capital. Debenture interest has to be paid
whether profits are made or not. A debenture may either be redeemable of
irredeemable. Redeemable is repayable at or by a particular date and irredeemable
is payable when the company is officially terminated.
BONUS SHARES
Shares issued to existing shareholders free of charge. They are paid out from
either the share premium, balance of retained profits of the General Reserves.

397

Company Accounts

A scrip issue is similar to bonus issue only that a scrip issue gives the shareholder
the choice of receiving cash or stock dividends. In a bonus issue the shareholder
has no choice but to take up the shares.
Example
A Ltd has 100,000 shares at 1 each to form an ordinary share capital of 100,000
and a balance on the share premium A/C of 50,000. It issues some bonus shares to
existing shareholders at a rate of 1 share for every 5shares held. This amount is to
be financed by the share premium. The entries will be as follows:

Shares to be issued:
100,000 1 =20,000
5
Dr share premium A/C [20,000 1 ] 20,000
Cr ordinary share capital
20,000
A bonus issue of 20,000 shares
Balance sheet (extract)
Ordinary shares of 1
Capital Reserves
Share premium

120,000
30,000

Rights Issue
A right issue is an option on the part of the shareholder given by the company to
existing shareholders at a price lower than the market price.
It involves selling ordinary shares to existing shareholders of the company on a
prorata basis. When the rights are issued the shareholders have 2 options
available.
Buy the new shares and exercise their rights
Sell the rights in the market,
Ignore the rights.
A rights issue therefore gives the shareholder the right (but not an obligation) to
buy the new shares issued by the company.
Example:
A Ltd has a share capital of 200,000 trade up of 100,000shares of 2 each. The
balance on the share premium is 60,000. Additional capital is raised by way of a
right issue. The term are:
For every 5 shares held in the company, a shareholder can buy 2 shares at a price
of 2.5 per share.
Required:
The journal entries to reflect the above transaction assuming that all the
shareholders exercise their rights and the relevant balance sheet extract.
Shares to be issued
100,000 2 =40,000 shares

Lesson Eight

398

5
Dr cash book
[40,000 2.5 ]
100,000
Cr Ordinary share capital [40,000 2 ]
80,000
Cr Share Premium [40,000 0.5 ]
20,000
Balance sheet (extract)
140,000
Ordinary shares @ 2
Capital Reserves
Share premium
80,000

280,000

The following examples will illustrate the preparation of final Account for
companies.
Example 8.1
Just before you launch yourself into the question that follows remember that
everything you have learnt about double entry bookkeeping and the presentation
of year end accounts is valid in the context of companies, subject only to the points
we have added in this session.
The following is the trial balance of Transit Ltd at 31 March 19X8.

Issued share capital (ordinary shares of 1


42,00
each)
75,00
0
Leasehold properties, at cost
0
Motor vans, at cost (used for distribution)
2,500
Provision for depreciation on motor vans to 31
1,000
March 19X7
7,650
Administration expenses
10,00
Distribution expenses
0
Stock, 31 March 19X7
12,00
Purchases
0 206,5
Sales
138,7
00
Directors remuneration (administrative)
50
Rents receivable
3,600
Investments at cost
25,00
Investment income
0
340
7% Debentures
15,00
Debenture interest
6,750
0
Bank interest
Bank overdraft
Debtors and creditors
1,050
730
Interim dividend paid
162 24,10
Profit and loss account, 31 March 19X7
0
31,00
0 17,85
1,260
2
311,1

399

Company Accounts

311,1
22

22

You ascertain the following:


All the motor vans were purchased on 1 April 19X5. Depreciation has been, and is
to be, provided at the rate of 20% per annum on cost from the date of purchase to
the date of sale. On 31 March 19X8 one van, which had cost 900, was sold for
550, as part settlement of the price of 800 of a new van, but no entries with
regard to these transactions were made in the books.
The estimated corporation tax liability for the year to 31 March 19X8 is 12,700.
It is proposed to pay a final dividend of 10% for the year to 31 March 19X8.
Stock at the lower of cost or net realizable value on 31 March 19X8 is 16,700.
Required:
Prepare, without taking into account the relevant statutory provisions:

A profit and loss account for the year ended 31 March 19X8:
A balance sheet at that date.
(22 marks)

Lesson Eight

400

Solution:
Transit Ltd
Profit and Loss A/C for the year ended 31.3.19X8

Gross profit
Profit on disposal of van
Rent Receivable
Less: Expenses
Depreciation on motor vans
Administration expenses
Distribution expenses
Debenture interest
Bank interest
Trading profit for the year
Add investment income
Profit before tax
Taxation
Profit after tax
Less: Dividends
Interim paid
Final proposed
Retained profit for the year
Retained profit b/f
Retained profit c/d

500
32,65
0
10,00
0
1,050
162

1,260
4,200

72,45
0
190
3,600
76,24
0

(44,3
62)
31,87
8
340
32,21
8
(12,7
00)
19,51
8
(5,46
0)
14,05
8
17,85
2
31,91
0

401

Company Accounts

Transit Ltd
Balance sheet as at 31.3.19X8
Non-Current Assets
Leasehold properties
Motor vans

75,000
2,400
77,400

(960)
960

75,000
1,440
76,440
6,750
83,190

Investments
Current Assets
Stock
Debtors

16,700
31,000
47,700

Current liabilities
Bank overdraft
Creditors
Tax payable
Proposed dividends

980
24,100
12,700
4,200

Financed by:
Authorized issued and fully paid
42000 ordinary share of 1
Revenue Reserves
Profit and Loss A/C c/f

31,910
73,910
15,000
88,910

Workings
Sales
206,500
Less: Cost of sales
Opening stock
12,000
Purchases
138,750
150,750
Less Closing stock (16,700)
(134,050)
Gross profit
72,450

Motor vehicle
Bal b/f
Disposal
550

5,720
88,910
42,000

Non-Current liabilities
7% Debentures

Motor Vehicle Depreciation


Disposal
540 Bal b/d
Bal c/d
960 P & L
1,500

(41,980
)

1,000
500
1,500

2,500 Disposal

900

Lesson Eight

Cashbook

Disposal
P&L

250 Bal c/d


3,300

402

2,400
3,300

Motor vehicle Disposal


900 Motor Vehicle
550
190 Depreciation
540
1090
1090

Example 8.2
The Following Trial Balance Was Extracted From The Books Of Collins Ltd
At 31 December 19X5

Share capital authorized and issued:


80,000 ordinary shares of 1 each
Freehold premises at cost
Motor vans
Balance 1 January 19X5 at cost
Additions less sale proceeds
Provisions for depreciation of motor vans to 31
December 19X4
Stock in trade 31 December 19X4
Balance at bank
Provision for doubtful debts 31 December 19X4
Trade debtors and creditors
Directors remuneration
Wages and salaries
Motor and delivery expenses
Rates
Purchases
Sales
Legal expenses
General expenses
Profit and loss account: balance at 31
December 19X4

59,00
0

80,00
0

15,00
0
650
6,750
13,93
0
6,615
12,39
5
4,000
13,12
7
3,258
700
108,4
40
644
5,846
243,6
05

You are given the following information.:


i. Stock in trade, 31 December 19X5, 14,600.
ii. Rates paid in advance, 31 December 19X5, 140.

275
11,38
0

142,7
70
2,430
243,6
05

403

Company Accounts

iii. Debts of 1,075 to be written off and the provision to be increased to


350.
iv. On 1 January 19X5, a motor van which had cost 680, was sold for 125.
v. Depreciation provided for this van up to 31 December 19X4 was 475.
vi. Provide for depreciation of motor vans (including additions) at 20% of
cost.
vii. The balance on legal expenses account included 380 in connection with
the purchase of one of the freehold properties.
viii. The directors have decided to recommend a dividend of 5%.
Required:
With particular emphasis on presentation, prepare a trading and profit and loss
account for the year 19X5, and a balance sheet at 31 December 19X5, ignoring
taxation.
(24 marks)

Lesson Eight

404

Solution:
Trading and profit and loss account
for the year ended 31 December 19X5

Sales
Opening stock
Purchases
Less: Closing stock
Cost of goods sold
Directors remuneration
Wages and salaries
Motor and delivery expenses
Rates (700 - 140)
Legal expenses (644 - 380)
General expenses
Bad debts
Loss on disposal
Depreciation
Net profit
Proposed dividend
Retained profit brought forward
Retained profit carried forward

Balance sheet at 31 December 19X5

13,93
0
108,4
40
122,3
70
14,60
0
4,000
13,12
7
3,258
560
264
5,846
1,150
80
3,019

142,7
70

107,7
70
35,00
0

31,30
4
3,696
4,000
(304)
2,430
2,126

405

Company Accounts

Non-Current Assets
Freehold properties
Motor vans

59,380
15,095
74,475

---(9,294)
(9,294)

59,380
5,801
65,181

Current Assets
Stock
Debtors and prepayments, less
provision for doubtful debts
Cash at bank

14,600
11,110
6,615
32,325

Current liabilities
Creditors
Proposed dividends

11,380
4,000
15,380
16,945
82,126

Share capital
Ordinary shares of 1 each
Profit and loss account

80,000
2,126
82,126

Workings
Bad debts

Debtors
1,075
Balance c/f
350

Balance b/f
275
Profit and loss account
1,150

1,425

1,425
Motor vans

Balance b/f
15,000
Additions
775

Disposals
680
Balance c/f
15,095

15,775

15,775

Disposals
475
Balance c/f
9,294
9,769

Provision for depreciation

Balance b/f
6,750
Profit and loss account
3,019
9,769

Lesson Eight

406

Disposals

Motor vans
680

680

Provision for depreciation


475
Proceeds
125
Loss on capital
80
680

Example 8.3
Owik-Freez p.l.c. is a company which provides refrigerated storage facilities to
local farmers.
Services offered include the collection of produce, the use of rapid freezing
equipment, storage of the frozen produce and transport from frozen storage in
refrigerated vehicles to any point within the country. Orders for these services are
secured by the companys sales staff.
The companys revenue consists of charges for transport and freezing, and of
storage rentals. Customers may hire storage space either on a long-term contract
basis at advantageous charges (payable in advance) or on a casual basis (invoiced
monthly).
A considerable amount of electricity from the public supply is used by the company
in the freezing and storage operations. In the event of a sudden failure in this
supply, the company is able to generate its own emergency supplies from standby
generators kept for this purpose. An insurance policy has been taken out to protect
the company against the claims which would arise should any of the frozen
produce deteriorate as the result of power or equipment failure.
At the end of the companys financial year ended 30 September 1982, the assistant
accountant extracted the following balances from the ledgers.

407

Assets Account
Land and buildings (at cost)
Plant (at cost)
Vehicle (at cost)
Provision for depreciation (at 1 October
1981):
Land and buildings
Plant
Vehicles
Stock of consumable stores (at 30
September 1982)
Debtor for rentals
for charges
Bank
Cash
Liability Accounts
Trade Creditors
7% Debentures 2004/2012
Ordinary Share Capital (see note 7)
General reserve
Unappropriated profit (at 1 October
1981)
Share Premium
Revenue Accounts
Storage rentals long term contracts
Casual
Freezing charges
Transport charges
Expense Accounts
Wages, salaries and related expenses
Rates
Electricity
Transport costs
Repairs
Consumable stores
Postages, stationery, telephones
Insurance premium
Debenture interest
Sundries

Company Accounts

390,000
271,900
82,600
39,600
144,800
27,050
23,449
18,204
2,332
30,710
1,103
7,390
80,000
200,000
25,000
108,284
15,000
302,090
85,063
112,810
90,107
128,004
79,112
76,860
43,271
30,319
29,800
15,604
7,800
5,600
9,176
8,650

Other Accounts
Suspense (credit balance)
Notes at 30 September 1982:
At the beginning of the 1981-82 financial year, the company had sold refrigeration
plant (which had originally cost 26,000 and on which 20,800 had been provided
as depreciation to date of disposal) for 4,000. The only accounting entries relative

Lesson Eight

408

to this disposal which have been made so far, are a debit to Bank and a credit to
Suspense of the amount of the sale proceeds.
In April 1982, the compressor unit in No.7 storage unit failed and as a
consequence the contents deteriorated to such an extent that they had to be
disposed of by incineration. Compensation of 1,350 was paid to the farmer by
Owik Freez by cheque and debited to Suspense.
The insurance company has admitted liability under the policy but no further
ledger entries have as yet been made.
During the 1981-82 financial year, the company replaced one of its refrigerated
vehicles, which has originally cost 16,400 and on which 13,120 had been
provided as depreciation to date of disposal. A trade-in (part exchange) allowance
of 6,000 was granted in respect to this vehicle. A replacement vehicle was
acquired at a list price of 27,000. The entries relating to the disposal of the old
vehicle have not yet been made, except that the trade-in allowance has been
debited to Vehicles and credited to Suspense. The balance of the price of the new
vehicle has been paid by cheque and debited to Vehicles account.
It is the companys policy to provide for depreciation on a straight line basis
calculated on the cost of fixed assets held at the end of each financial year and
assuming no residual value. Annual depreciation rates are:
%
Building
2
Plant
10
Vehicles
25
The Buildings content of the item Land and Buildings included in asset account
balances is 120,000.
Adjustments, not yet posted to the accounts, should be made for the following
items:

Storage rentals received in advance 25,631


Insurance premium prepaid
600
Wages and Salaried accrued
1,920
Rates prepaid
28,820
Electricity accrued
5,757
Consumable stores include 4131 and Repairs include 9972 relating to vehicles.
The authorized and issued capital of the company consists of 400000 Ordinary
Shares of 0.50 per share. The directors have recommended a dividend for the
year of 0.12 per share.
Required:
Prepare, for internal circulation purposes, a Profit and Loss account for QwikFreez p.l.c.for the year ended 30 September 1982 and a Balance Sheet at that
date. All workings must be shown.
(31 marks)

409

Company Accounts

Open the Suspense account and post the entries needed to eliminate the opening
credit balance.
(2 marks)
(33 marks)
Solution:
Qwik-Freez (East Anglia) p.l.c.
Profit and Loss Account for the year ended 30 September 1982
Workings:
Revenue
Storage rentals long term
(302,090 25631)
casual
Freezing charges
Transport charges
Less:
Expenses
Wages, Salaries etc. (128,004 + 1,920)
Rates (79,112 28,820)
Electricity (76,860 5,757)
Transport costs (43,271 + 4,131 +
9,972)
Repairs (30,319 9,972)
Consumable stores (29,800 4,131)
Postages, stationery, telephones
Insurance premiums (7,800 - 600)
1,2
*Depreciation
Debenture Interest
Sundries
5
Profit (less loss) on disposal of
fixed assets
Net Profit For The Year
Retained profit brought forward
Distributed profit
Less:
Ordinary dividends proposed
Retained profit carried forward
Workings:

276,459
85,063
112,810
90,107
564,439

129,92
4
50,292
82,617
57,374
20,347
25,669
15,604
7,200
43,540
5,600
9,176

447,343
117,096
1,520
118,616
108,284
226,900
48,000
178,900

Lesson Eight

410

Land

270,0
00

Fixed Assets:
Balance 1 October 1981
(veh 82,600 (6,000 +
21,000))
Acquisitions (21,000
6,000)
Disposals
Balance 30 September
1982
Depreciation
-rate

270,0
00

2%

2,400

Buildin
gs

120,00
0

120,00
0

Plant

271,90
0
(26,00
0)
245,90
0

Vehicl
e

55,600

Total

717,50
0

27000
(16,40
0)
66,200

27,000
(42,40
0)
702,10
0

10
25

-current year charge


24,59 16,5 43,54
0
50
0
Alternatively the depreciation charge for vehicles (16,550) can be classified as a
transport cost, thereby increasing that figure to 73,924.

411

Provision for Depreciation:


Balance 1 October 1981
Disposals
Current year charge

Company Accounts

39,600

2,400

42,000

Balance 30 September
1982

Written down values at 30


September 1982

144,8
00
(20,8
00)
24,59
0
148,5
90

27,05
0
(13,1
20)
16,55
0
30,48
0

211,4
50
(33,9
20)
43,54
0
221,0
70

270,0
00

78,000

97,31
0

35,72
0

481,0
30

4,000

6,000

10,00
0

5,200

3,280

Proceeds from disposals


Less:
Written down values of
disposals
(26,000 20,800)
(16,400 13,120)
Profit/(Loss) on disposals

Qwik-Freez (East Anglisa) p.l.c


Balance Sheet as at 30 September 1982
Workings:

8,480
(1,2
00)

2,720

1,520

Lesson Eight
Fixed Assets
Land and Buildings
Plant
Vehicles
1,3,4
Current Assets
Stocks
Debtors
- for rentals
- for charges
- for insured losses
Prepaid expenses (600 + 28,820)
Bank
Cash
Less:
Current Liabilities
Creditors
Accrued expenses (1920 + 5757)
Advance receipts
Proposed dividends
Working Capital
Net Assets employed
Financed by:
Share Capital, authorized, issued and fully paid,
400000 Ordinary shares of 0.50 per share
16
Reserves
Share Premium
General Reserve
Profit and Loss account
Shareholders funds
Long-term loan
7% Debentures 2004/2012
Cost

390,000
245,900
66,200
Depreciation

42,000
148,590
30,480
Net

348,000
97,310
35,720
702,100
221,070

412

413

Company Accounts

Suspense

Fixed Asset Disposals:


`
Plant 4000 Balance b/d
8650
Vehicle
6000 Debtors (insured loss)
1350
10000
10000
Example 8.4
Mwanga and Sons Ltd is a small manufacturing firm owned by members of the
family. The following trial balance was extracted from the books of the company as
at 31 March 1993:
Freehold property, at cost (land Sh. 75,000)
Plant, at cost
Depreciation
Motor vehicle, at cost
Depreciation motor vehicle
Fittings and fixtures, at cost
Depreciation fittings and fixtures
20,000 Ordinary shares of Sh. 10 each
authorized, issued and fully paid
Share premium
General reserve
Interim dividend paid
Cash at bank and in hand
Accounts receivable and payable
15% Debentures
Discount received
Profit and loss account 1 April 1992
Purchases of raw materials
Sales of finished goods
Inventories 1 April 1992:
Raw materials
Work in progress
Finished goods
Provision for doubtful debts
Bad debts
Rates and insurance
Wages
Factory power
Light and water
Plant maintenance
Salaries
Returns of raw material
Sales returns
Advertising
Transport expenses (Sales department)
Bank charges
General expenses

Sh.
125,00
0
130,00
0

Sh.
62,000
30,500

53,000
11,790
38,600
200,00
0
50,000
120,00
0
16,000
33,570
130,54
0

942,38
0

57,430
100,00
0
3,640
103,87
0
1,254,
760

33,060
57,660
107,86
0
6,400
4,890
9,430
108,37
0
22,560
16,280
10,970
90,000

3,240

Lesson Eight

414

1,360
8,580
24,320
3,040
36,1
60
2,003,
630

2,003,
630

415

Company Accounts

Additional information:

Depreciation is to be provided for the year using the reducing balance


method and applying rates of 15% on plant, 25% on motor vehicle and 10%
on fittings and fixtures.
Building is to be depreciated at the rate of 4% using the straight-line
method. (Assume the whole building is used for manufacturing purposes).
Provision for doubtful debts is to be adjusted to a figure equal to 10% of
accounts receivable.
Light and water, insurance and general expenses are to be apportioned in
the ratio 4:1 between factory and administrative overheads.
Electricity and wateer accrued
was
Insurance prepaid was
Rates prepaid were

Sh.
860
270
780

Inventories were valued at:


Raw materials
Work in progress
Finished goods

139,630
82,450
124,320

Debenture interest has not yet been paid.


The directors require provision for a final dividend which will bring the
dividend for the year up to Sh. 2 per share.

Required:
Prepare in vertical form a Manufacturing, Trading and Profit and Loss Account for
the year ended 31 March 1983 and a Balance Sheet as at that date. (25 marks)

Lesson Eight

MWANGA AND SONS LTD


Manufacturing Account for the year ended 31 March 1993
Raw materials:
Opening stocks
33,060
Purchases
942,380
Less Returns In.
(3,240)
939,140
972,140
Less Closing stocks
(139,630)
Prime Costs
832,570
Factory Overheads:
Plant depreciation
10,200
Rates and insurance
2,000
Factory power
6,704
Light and water
22,560
Plant maintenance
13,712
General expenses
10,970
28,928
95,074
Opening W.I.P.
927,644
Less: Closing W.I.P.
57,660
Goods manufactured
(82,450)
(24,790)
902,854
Trading, Profit And Loss Account For The Year Ended 31 March 1993
Shs
Shs
Sales
1,254,760
Less: Closing stocks
(1,360)
1,253,400
Opening stock
107,860
Goods manufactured
902,854
1,010,714
Less: Closing stocks
(124,320)
886,394
367,006
Discount received
3,640
370,646
Debenture interest
15,000
Provision for bad debts
6,654
Depreciation
- Motor vehicle
5,625
- Fittings and fixtures
2,681
Dividend
- Interim
16,000
- Fianl
24,000
40,000
Retained Profit for the year
49,150
Retained Profit brought forward
103,870
Retained Profit carried forward
153,870

416

417

Company Accounts

Lesson Eight

418

Balance Sheet As At 31 March 1993


Fixed Assets
Freehold property
Plant
Motor vehicle
Fittings and fixtures
Current Assets
Stocks
- Raw materials
- work in progress
- finished goods
Debtors, less provisions
Cash at bank and in hand
Prepaid expenses
Current Liabilities
Creditors
Accruals
Dividend proposed
Net current assets
Financed by:
Authorized, and issued share
capital:
20,000 Ordinary shares each
Sh. 10
Reserves:
Share Premium
General Reserve
Profit and Loss account

Cost

125,000
130,000
53,000
38,600
346,600

Depreciat
ion

2,000
72,200
36,125
14,471
124,796

Net

123,000
57,800
16,875
24,129
221,804

139,630
82,450
124,320
346,400
117,486
33,570
1,050
498,506
57,430
15,860
24,000
97,290

401,216
623,020

200,000
50,000
120,000
153,000

323,020
523,020
100,000
623,020

15% debentures
Workings:
B/d

Rate And Insurance


9,430 Prepaid
Prepaid
Profit and Loss Account
Factory

270
780
1,676
6,704

419

Company Accounts

9,430

9,430

Lesson Eight

420

Issuance Of Shares
Issue and Forfeiture of shares:
The sale of shares by 2 PLC to members of the public can be categorized as
follows:
Sale of

Sale at a

Sale at per

Lump sum
Sale
2.

Lump sum
Sale
3.

Lump sum
Sale
4.

Lump sum
Sale
1.

When shares are sold in exchange for lump sum cash payment and this is at per
value, the entries to be made are:
DEBIT: Cashbook
CREDIT: Share Capital
When shares are sold in exchange for lump sum cash payment and this is at a
premium, the entries to be made are:
DEBIT: Cashbook
CREDIT: Share Capital
CREDIT: Share Premium
Sale of shares which are to be paid for in installments are normally dealt with as
follows:
The number of installments may vary from 2 4. Each installment is collected
through a comprehensive set of processed(called a stage). The 4 possible stages
are:
Application stage
For each stage, an account is opened.
Allotment stage
This account must close at the end of
1st Call stage
the stage. (The application stage &
2nd Call stage
allotment stage may be dealt with in a
single account called the application/
Application Stage
In this stage, the company invites members of the public to send in applications for
share they (the public) are interested in purchasing.

421

Company Accounts

The application firms must be accompanied by the 1st installment money when the
public respond to the companys offer.
When the company requests members of the public to send in application forms &
application money it will make the following entries in its books:
DEBIT: Application A/C
CREDIT: Share Capital
When the public responds by sending funds, the company will then
DEBIT: Cashbook
CREDIT: Application A/C
There may be an over or under subscription. If there is an under subscription,
DEBIT: Cashbook
CREDIT: Application
With
If there is an over-subscription, then the excess applications may either be rejected
outright and the applicants money refunded, or applications awarded on a prorata basis. (i.e. a lower number of shares allotted compared to the number applied
for)
If outright rejection, the company will:
DEBIT: Application
With refunded
CREDIT: Cashbook
money
If pro-rata issue, the company will:
DEBIT: Application
With amount required to
CREDIT: Allotment
close the application A/C.
This marks the end of the application stage.
Allotment Stage
In this stage, the company selects the applicants and informs them of their
allotment. It also requests them to bring in a second installment. As it requests for
the second installment the entries to be made are:
DEBIT: Allotment A/C.
CREDIT: Share Capital.
When the public respond by sending in the second installment money, the
company, will in its books: DEBIT: Cashbook.
CREDIT: Allotment.
Generally only the correct amount of money is collected at this stage. Since the
account has closed by this stage, the stage is deemed to be over.
1st Call Stage
Here the company requests for the third installment from the public. As the
company does this, it will:
DEBIT: 1st Call A/C .
CREDIT: Share Capital.
When the public respond by bringing in the installment money, the company, will in
its books:
DEBIT: Cashbook
CREDIT: 1st Call A/C.
It is possible that some of the allotees do not pay their 1 st installment money on
time. When this is so,
DEBIT: Cashbook with money received

Lesson Eight

422

DEBIT: Calls in Arrears with money not received


CREDIT: 1st Call A/C with total.
This marks the end of the Call stage.
2nd Call Stage
this is very similar to the 1st Call whereby the company requests for the second
(and last) call money; as it does so, it will:
DEBIT: 2nd Call A/C
CREDIT: Share Capital
When the public respond by sending in the second call money, then the company
will:
DEBIT: Cashbook
CREDIT: 2nd Call A/C
It is possible that some of the allotees do not pay up their 2 nd Call money. When this
is so:
DEBIT: Cashbook with money collected.
DEBIT: Calls in arrears with money not received
CREDIT: 2nd Call A/C.
THIS MARKS THE END OF THE NORMAL ISSUE OF SHARES
PROCEDURES.
When a debtor for share money (calls in arrears) does not pay up his dues, his
shares will be cancelled and any money he previously gave the company forfeited
(i.e. not refunded to him). This is known as Share Forfeiture. The entries to be
made when shares are forfeited are:
DEBIT: Share Capital
CREDIT: Calls in Arrears
CREDIT: Share forfeiture.
Forfeited shares may be resold as follows:
At per value
At a premium
At a discount
If resold at per:
DEBIT: Cashbook.
CREDIT: Share Capital.
If resold at a premium:
DEBIT: Cashbook
CREDIT: Share Capital.
CREDIT: Share premium
When shares are sold at a discount, a condition will have to apply. The share sale
will be expressly illegal unless:
Amounts collected from previous allotee plus an amount collected from current
allotee equals or is greater than the par value.
If the condition is fulfilled and shares are sold at a discount, then

423

Company Accounts

DEBIT: Cashbook with money received.


DEBIT: Share forfeited with deficit.
CREDIT: Share Capital with par value.
Last entry in this exercise is to transfer any balance on the share forfeiture A/C to
Share Premium A/C as follows:
DEBIT: Share forfeiture .
CREDIT: Share Premium.
Example 8.5 MAY 1999
QUESTION FOUR
Give a brief definition of memorandum of association and certificate of
incorporation.
(5 marks)
Radhi Tea Company Limited has an authorized share capital of Sh. 10,000,000
ordinary shares of Sh.10 each. The shares were issued at par as follows:
Payable
Payable
Payable
Payable

on
on
on
on

application
allotment
first call
second call

Sh.1.00
Sh.3.00
Sh.4.00
Sh.2.00

Applications were received for 1,630,000 shares.


It was decide to refund applicants monies on 130,000 shares and to allot all the
shares on the basis of two for every three applied for.
The excess application monies received from the successful applicants is not to be
refunded but is to be applied to reduce the amount payable on allotment.
The calls were made and paid in full with the exception of one member of one
member holding 5,000 shares who paid neither the first nor the second call and
another member who did not pay the second call on 1,000 shares. After requisite
action by the directors the shares were forfeited. They were later reissued at a
price of Sh.8 per share.
Required:
The necessary ledger accounts to record these transactions
(15 marks)
(Total: 20 marks)
Solution:
Memorandum of association explains the relationship between the company and
the outside world. It shows the object of the company, the name, address, the
authorized share capital, its location (its registered office) and the date of
incorporation.
Articles of association state the rules under which the company will operate e.g.
the number of directors, the structure, and meetings. It explains the relationship
between the different directors and shareholders.
Certificate of incorporation is a document issued to the company when it is
registered by the registrar of companies.

Lesson Eight

424

Radhi Tea Co.


Application A/C
Cashbook
OSC
Allotment

Sh.
130,000
1,000,000
500,000
1,630,000

Sh.
Cashbook 1,630,000
1,630,000

Allotment A/C
Sh.
OSC
Allotment

Sh.
3,000,000 Application
Cashbook
3,000,000

500,000
2,500,000
3,000,000

1st Call
OSC

Sh.
Sh.
4,000,000 Calls in arrears
20,000
Cashbook
3,980,000
4,000,000
4,000,000

425

Company Accounts

2nd Call
OSC

Bal c/d

Sh.
Sh.
1,990,00 Calls in arrears
2,000
Cashbook
1,988,000
1,990,000
1,990,000
Share Premium
Sh.
Sh.
1,630,000 Share forfeiture

Ordinary Share Capital


Sh.
Share forfeiture A/C
50,000
Allotment
3,000,000
1st Call
4,000,000
nd
2 Call
1,990,000
Bal c/d
10,000,000
10,050,000

1,630,000

Sh.
Application 1,000,000

Share forfeiture
10,050,000

60,000

Share Forfeiture A/C


Sh.
Sh.
Calls in arrears
22,000
OSC
50,000
OSC
60,000
Cashbook 48,000
Share Premium
16,000
98,000
98,000

1st Call
2nd Call

Calls in Arrears
Sh.
`
Sh.
20,000
2,000 Share forfeiture 22,000
22,000
22,000

FINANCIAL STATEMENT ANALYSIS


(RATIO ANALYSIS)
Financial statements include a profit and loss A/C (income statement) that tells us
the performance of a company throughout the financial period. It also includes a
balance sheet that shows the financial position or status of a company and lastly a
cash flow statement which shows changes in cash position of the entity,
We analyse financial statements by the use of accounting ratios. There are 5
classes of ratios:
Liquidity
Leverage/Gearing ratios
Activity Ratios
Profitability

Lesson Eight

Equity / Investor ratios.

426

427

Company Accounts

LIQUIDITY RATIOS.
These measure the firms ability to meet its short term maturing obligations.
Leverage/Gearing Ratios These measure the extent to which a firm has been
financed by non-owner supplied funds.
Activity Ratios These measure the efficiency with which the firm is using
various assets to generate sales revenue or how active has the firm been.
Profitability Ratios These measure the efficiency with which the firm uses
various funds to generate profits or returns. They also measure the managements
ability to control the various expenses in the firm.
Equity Ratios/Investor Ratios They measure the relative value of the firm and
returns expected by the owners of the firm. They also try to look at the overall
performance of the firm and going concern of the firm.
The following question will be used to illustrate the above classes of ratios
ABC ltd
Profit and Loss A/C for the year ended 31.12.1992
Sh
Sh
Sales
850,000
Less: Cost of Sales
99,500
Opening stock
559,500
Purchases
659,000
(149,000)
(510,000)
Less: Closing stocks
340,000
Gross profit
Less expenses
30,000
Selling and distribution
10,000
Depreciation
135,000
(175,000)
Administration expenses
165,000
Earnings before interest & taxes
(15,000)
Interest
150,000
Earnings before tax
75,000
Tax @ 50%
75,000
Less ordinary dividend
(0.75 per share)
(15,000)
Retained profit for the year
60,000
ABC
Balance Sheet as at 31 December 1992
Non Current
Sh.
Assets
250,00
Land and
0
Buildings
80,000
Plant &
330,00
Machinery
0
75,000
Current Assets
(4,000) 149,00
Inventory
0
Debtors

Issued share
capital
(20000 share of
Sh, 10)
Reserve
Retained profit
Long term
Current liabilities.

Sh.
200000
90000
60000
100000
130000

580,000

Lesson Eight
Less provision
Cash

428

71,000
30,000
580,00
0

Additional Note
Cash purchases amount to 14,250.
Required:
Compute the relevant ratios.
LIQUDITY RATIOS
Current Ratio = Current Assets
Current Liabilities
Current Ratio = 250,000 = 1.92 : 1
130,000
The higher the ratio then the more liquid the firm is.
Quick Ratio/Acid Test Ratio
= Current Assets - Inventories
Current Liabilities
= 250,000 149,000 = 101,000
130,000
130,000
= 0.78 : 1
this is a more refined ration that tries to recognize the fact that stakes may not be
easily converted into cash. The higher the ratio, the better for the firm as it means
an improved liquidity position.
Cash Ratio
= Cash + Marketable Securities
Current Liabilities
= 30,000 = 0.23 : 1
130,000
= 0.23 : 1
This ratio assumes that stakes may not be converted into cash easily and the
debtors may not pay up their accounts on time. The higher the ratio, the better for
the firm as the Liquidity position is improved.
Net Working Capital Ratio.
= Net Working Capital
Net Assets

429

Company Accounts

Net Working Capital =CA CL = 250,000-130,000=120,000


Net Working Capital = 120,000 = 0.27 : 1
450,000
= 0.27 : 1
The higher the ratio the better for the firm and therefore the improved Liquidity
position.
GEARING RATIOS
These measure the financial risk of a firm (the probability that a firm will not be
able to pay up its debts). The more debts a business has (non owner supplied
funds) the higher the financial risk.
Debt Ratio
= Total Liabilities
Total Assets
This ratio measures the proportion of total assets financed by non owner supplied
funds. The higher the ratio, the higher the financial risk .
= 230,000 = 0.4
580,000
40% is supplied by non owners
Debt Equity Ratio
= Total Liabilities
Networth (share holders funds)
= 230,000 = 0.66
350,000
40% is supplied by non-owners
This ratio measures how much has been financed by the non-owner supplied funds
in relation to the amount financed by the owners i.e. for every shilling invested in
the business by the owners how much has been financed by the non-owner
supplied funds.
For ABC Ltd, for every 1 shilling contributed in the business by the owner, the
creditor have put in 67 cents.
The higher the financial risk.
Long Term Debt Ratio
= Non Current Liabilities
Net Assets
= 100,000 = 0.2
450,000

Lesson Eight

430

This measures the proportion of the total net assets financed by the non-owner
supplied funds.
The higher the ratio, then the higher the financial risk.
ACTIVITY RATIO
Stock Turnover
= Cost of Sales
Average Stocks
where
Average Stocks = Opening Stock + Closing Stock
2
= 510,000 = 4.1
124,250
= 4.1 times
This is the number of times stock has been converted to sales in a financial year.
The higher the ratio the more active the firm is.
An alternative formula is
=

Sales
Closing Stock

Debtors Turnover
= Credit Sales
Average Debtors
Where
Average Debtors = Opening debtors + Closing debtors
2
Assume the opening debtors was 89,000 and all sales are on credit
Debtor Turnover = 850,000 = 10.625
80,000
The higher the ratio, the more active the firm has been (we had debtors over 10
times to generate the sales)
Note
Average Collection Period
=
360
Debtors Turnover
=

360 = 34 days
10.625

This measure the number of days it takes for debtors to pay up. The lesser the
period, the better for the firm as it improves the liquidity position.

431

Company Accounts

Creditors Turnover
= Credit Purchases
Average Creditors
= 545,250
130,000
= 42 times
The ratio tries to measure how many times we have creditors during a financial
period. The lesser the ratio the better.
Non Current Assets Turnover (Fixed Assets Turnover)
= Sales
Average Fixed Assets
A.F.A = 340,000 + 330,000 = 670,000 = 335,000
2
2
= 850,000 = 2.54 times
335,000
The ratio measures the efficiency with which the firm is using its fixed/ Non
Current Assets to generate sales.
The higher the ratio the more active the firm.
Total Assets Turnover
= Sales
Total Assets
= 850,000
580,000
= 1,046 times
Measures the efficiency with which the firm is using its total assets to generate
sales.
PROFITABILITY RATIOS
Profitability in Relation to Sales
Gross Profit Margin
= Gross Profit = 165,000 = 19%
Sales
850,000
The higher the margin, the more profitable the firm is.
Net Profit Margin
= Net Profit after tax = 75,000 = 9%
Sales
850,000
The higher the margin, the more profitable the firm is.

Lesson Eight

432

Margin affected by:


Operating expenses for the period.
Profitability in Relation to investment
Return On Investment
= Net Profit after tax
Total Assets
= 75,000 = 13%
580,000
Shows how efficient the firm has been in using the total assets to generate returns
in the business.
Return On Capital Employed
= Net Profit after tax
Net Assets
= 750,000 = 17%
450,000
How efficient the firm has been in using the net assets to generate returns in the
business.
Return On Equity
= Earnings after tax
Networth
= 75,000
850,000
= 21%
Efficiency of the firm in using the owners capital to generate returns.
NOTE
The higher the ratio the more efficient is the firm.
EQUITY RATIOS
Earnings Per Share (Eps)
EPS = Earnings attributable to ordinary shareholders
No. of ordinary shares outstanding.
= 75,000
20,000
= 3.75

433

Company Accounts

This is the return expected by an investor for every share held in the firm.
Earnings Yield
= Earnings Per Share
Market price per share
Assume that the market price for the ABCS shares is Sh20/Share.
= 3.75 100%
20
= 19%
This is the return amount expected by a shareholder for every shilling invested in
the business.
Dividend Per Share
= Total Dividend (ordinary shareholders)
Ordinary shares outstanding.
= 15,000
20,000
= 0.75 cts per share
This is the amount expected by an investor for every share held in the firm.
NOTE
The higher the amounts, the better for the firm.
LIMITATIONS ON USE OF RATIOS
It is difficult to categorise firms in the various industries due to
diversification. This makes inter-company comparison difficult.
It is difficult to compare one company with others in case of monopolist
firms.
Different, firms use different accounting policies and methods e.g. on
depreciation, provisions and other estimates so this makes comparison of
companies difficult.
Ratios are compiled at a point in time and may be affected by short term
changes. Therefore ratios are used for short term planning.
Ratios are computed from historical data and therefore are not good
indicators of the future.
DEFINITIONS
TREND ANALYSIS Comparing or assessing a companys performance over time.
CROSS SECTIONAL ANALYSIS Comparing two or more companies in the same
industry.
Example 8.6 (ACCA DEC 98)

Lesson Eight

434

Beta Ltd is reviewing the financial statements of two companies, Zeta Ltd and
Omega Ltd. The companies trade as wholesalers, selling electrical goods to
retailers on credit. Their most recent financial statements appear below.
PROFIT AND LOSS ACCOUNTS FOR THE YEAR ENDED 31 MARCH 20X8
Zeta Limited
000
Sales
Cost of sales
Opening stock
Purchases
Less: closing stock

200
3,200
3,400
400

Profit before tax


Taxation
Net profit for the period

800
4,800
5,600
800
3,000
1,000

Gross profit
Expenses
Distribution costs
Administrative expenses
Interest paid

Omega Limited
000
000
000
4,000
6,000

200
290
10

4,800
1,200
150
250
400

500
500
120
380

800
400
90
310

435

Company Accounts

Balance Sheets As At 31 March 20x8


Zeta Limited
000
Fixed assets
Tangible assets
Warehouse and office
1,200
buildings
600
Equipment and vehicles

000

Omega Limited
000
000
5,000
1,000

1,800

6,000

Current assets
Stock
Debtor trade
- sundry
Cash at bank

400
800
150
1,350

800
900
80
100
1,180

Current liabilities
Creditors trade
- sundry
Overdraft
Taxation

(800)
(80)
(200)
(120)

(800)
(100)
(90)

Long-term loan (interest


10% pa)
Share capital
Revaluation reserve
Profit and loss account

150
1,950
1,950
1,000
950
1,950

890
6,890
(4,000)
2,890
1,600
500
790
2,890

Required:
a) Calculate for each company a total of eight ratios which will assist in
measuring the three aspects of profitability, liquidity and management of the
elements of working capital. Show all workings.
(8 marks)
b) Based on the ratios you have calculated in (a), compare the two companies
as regards their profitability, liquidity and working capital management.
(8 marks)
c) Omega Ltd is much more highly geared than Zera Ltd. What are the
implications of this for the two companies?
(4 marks)
(20 marks)
Solution:

Lesson Eight

436

PROFITABILITY
Gross profit margin
Gross profit 100%
Sales
Net profit margin
Net profit 100%
Sales
Return on capital employed
Profit before interest and tax
Capital employed
Return on shareholders capital
Profit before tax
Share capital and reserves
Asset turnover
Sales
Capital employed
LIQUIDITY
Current ratio
Current assets
Current liabilities

1000 100% =
25%
4000

1200 100% =
20%
6000

500 100% =
12.5%
4000

400 100% = 6.7%


6000

510 = 26.2%
1950
500 = 25.6%
1950
4000 = 2.1
times
1950

800 = 11.6%
6890
400 = 13.8%
2890
6000 = 0.9 times
6890

1880 = 1.9:1
990

Quick ratio
Current assets stock
Current liabilities

1350 = 1.1:1
1200

Gearing
Long term loans
Capital

950 = 0.8:1
1200

Interest cover
Profit before interest and tax
Interest charges

nil = nil
1950

1080 = 1.1:1
990
4000 = 58%
6890
800 = 2 times
400
510 = 51 times
10

WORKING CAPITAL
MANAGEMENT
Debtors days
Trade debtors 365 days
Sales
Creditor days
Trade creditors 365 days
Purchases

800 365 = 73
days
4000

900 365 = 55
days
6000

800 365 = 61

437

Stock days
Average stock 365 days
Cost of sales

Company Accounts

800 365 = 91
days
3200
300 365 = 37
days
3000

days
4800
800 365 = 61
days
4800

Note. We have used average stock here. When you have the information use it.
Profitability
Zeta has a higher gross margin than Omega. This may indicate a differing pricing
policy. Omegas net margin is lower than Zetas. Omegas expenses are therefore
proportionally higher. It should be noted that Omegas bottom line profit is reduced
significantly by the interest charge.
Return on Omegas capital is around half of Zetas. Omega has a higher fixed asset
base due in part to a revaluation. It may be that a revaluation of Zetas assets will
partially close the gap.
Liquidity
Omega has nearly twice as many current assets as current liabilities. Although
both companies quick ratios are much closer, Zetas liquidity does appear to be an
issue especially as there is no cash at hand. It would be wise to examine projected
cashflows to see how readily Zetas profits will improve this situation. As Zeta has
no long-term loans they may be able to borrow in order to improve liquidity.
Working capital management
Zeta is turning stock over more quickly than Omega. This is beneficial in a market
which can be subject to obsolescence.
Zetas creditor and debtor days are a cause for concern. Debtors should be
collected within 60 days if not sooner. 60 day collection would improve cash flow
by over 140,000 reducing the debtors balance to 658,000(60/73 800,000).
Creditors should be paid at least as quickly as Omega pays theirs. Zeta risks
damaging the goodwill it has with its suppliers. Paying creditors within 60 days
would have an adverse effect on cash flow of over 270,000. The creditors balance
would be 527,000 (60/91 800,000).
Omega is highly geared whereas Zeta has no long-term loans. Omegas gearing
means that should profits fall they may not be in a position to pay the loan interest.
Zetas capital is entirely share capital and so a fixed return is not required.
Omegas loan appears to be fixed rate. This means that in times of falling interest
rates Omega will have higher interest costs than say, Zeta, if Zeta borrowed the
same amount. The converse is true in times of rising interest rates.

Lesson Eight

438

439

Company Accounts

REINFORCEMENT QUESTIONS
QUESTION ONE
The chief accountant of AZ Limited has extracted the following trial balance as at
31 October 1999.
Sh.
Sh.
Authorized and issued capital
400000
Share premium
00
8% debenture stock
5000
Profit and loss stock
00
Motor vehicles at cost
165000 100000
Provision for depreciation on motor vehicle
00
00
Plant and machinery at cost
55000
Provision for depreciation on plant and
258000
00
machinery
00
Land buildings at cost
34000
Stock in hand 1 November 1998 Finished
300000
00
goods
00
Raw materials
42000
63000
Work-in-progress
0
00
38000
0
Trade debtors
56000
Office furniture and equipment at cost
0
Provision for depreciation on office furniture
and equipment
Sh.
Sh.
Trade creditors
736000
Purchase of raw materials
0
Sales of finished goods
89000
18500
Direct wages
0
0
Direct expenses
10000
Factory expenses
00
Indirect materials
950000
Factory insurance
0 285500
Sales room expenses
00
Administration expenses
135000
Office salaries and wages
0
Vehicles running expenses
395000
Bad debts written-off
290000
Balance at bank overdrawn
350000
150000
485000
620000
840000
656000
640000
11750
00
966100 966100
00
00

Lesson Eight

440

Notes:
Closing stock includes
Finished goods
Raw materials
Work-in-progress
Accrued salaries
The directors recommended a dividend of 10% on the issued share capital and a
transfer of Sh. 2000000 to a general reserve.
Debenture interest has not been paid
Depreciation is provided on straight-line method at 10% and 25% per annum on
furniture and equipment, plant and machinery and motor vehicles respectively.
The overdraft interest of Sh. 725000 was communicated to the company by the
bank on 5 November 1999 and therefore it has not been posted in the cash book.
Required:
Manufacturing, trading, profit and loss account for the year ended 31 October
1999.
(12 marks)
Balance sheet as at 31 October 1999.
(8 marks)
(Total: 20 marks)
QUESTION TWO
The Chief Accountant of KK Ltd has extracted the following trial balance as at 31
October 1998.
Sh,00 Sh,00
Authorized and issued capital (shares of Sh. 20
0
0
each fully paid)
30,000
Share premium
350
10% premium
3,500
General reserve
2,000
Profit and loss account 1 November 1997
2,850
Motor vehicles at cost
3,500
Provision for depreciation
265
Freehold property
44,50
Trade debtors
0
Trade creditor
1,375
460
Purchases and sales
127,45
Stock in hand 1 November 1997
95,650
0
Furniture and fittings at cost
3,478
Provision for depreciation
1,540
Goodwill
138
Rent receivable
500
Salaries and wages
385
General expenses
2,285

441

Vehicles running expenses


Bad debts
Telephone and postage
Water and electricity
Rates and insurance
Cash at bank

Company Accounts

358
2,470
124
568
269
289
10,492
167,39
8

167,39
8

Notes:
1. Credit sales amounting to Sh.165,000 were made on 31 October 1998 but no
entries were made in the books.
2. Returns outwards amounting to Sh.128,000 were dispatched on 31 October
1998 but no entries were made in the books.
3. Closing stock was valued at Sh.4,398,000.
4. Accrued salaries and telephone bills amounted to Sh.134,000 and Sh.55,000
respectively.
5. Rent for the month of October 1998 amounting to Sh.35,000 had not been
received from the tenant.
6. Provision for depreciation on furniture and fittings and the motor vehicles
are 10% and 20% on cost respectively.
7. Provision for bad and doubtful debts of 5% on trade debtors should be made.
8. Corporation tax should be provided at 35% of the net profit before tax.
9. The directors propose a dividend of 15% on issued share capital and a
transfer of Sh.2,500,000 to the general reserve.
10.The debenture interest has not yet been paid.
Required:
1. Trading, profit and loss account for the year ended 31 October 1998. (13
marks)
2. Balance sheet as at 31 October 1998.
(7 marks)
(Total: 20 marks)
QUESTION THREE ACCA PILOT PAPER
The balance sheet of Grand Limited, a wholesaler, at 31 December 1995 and 1996
were as follows:
31
December
1995 1996

Lesson Eight
Tangible fixed assets
Cost of valuation
Aggregate
depreciation
Current assets
Stock
Debtors
Cash
Current liabilities
Trade creditors
Corporation tax
Proposed dividend
Net current assets
Loans (due for
repayment 1999)
Called up share capital
Share premium
Revaluation reserve
Profit and loss account

442

000
126,300
(50,000)

000
76,300

12,000
10,500
1,400
23,900

000

000

162,4
00
(64,0
00)

98,40
0

15,00
0
14,00
0
2,000
31,00
0

6,800
3,400
4,000
14,200
9,700
86,000
(60,00
0)
26,000
6,000
1,000
19,000
26,000

The stock at 31 December 1994 was 10,000,000.

9,400
5,000
6,000
20,40
0

10,60
0
109,0
00
(60,0
00)
49,00
0
10,00
0
3,000
8,000
28,00
0
49,00
0

443

Company Accounts

The summarized profit and loss accounts for the company for the years ended 31
December 1995 and 1996 were:

Sales
Cost of sales
Gross profit
Expenses
Net profit before tax

Year ended 31 December


1995
1996
000
000
64,000
108,000
40,000
75,600
24,000
32,400
10,000
12,400
14,000
20,000

Required:
a) Calculate the following accounting ratios for both years:
The gross profit percentage
The current ratio and the quick ratio (or acid test)
Debtors collection period in days
Trade creditors payment period in days (based on purchases figures
which are to be calculated)
Gearing ratio.
b) Show you full workings.
(10 marks)
c) Explain what you can deduce from the ratios as at 31 December 1996 and
from comparing them with those for 1995.
(5 marks)
d) State two points which could cause the movement in the gross profit
percentages between the two years and explain how they could bring the
change about.
(2 marks)
e) State the extent to which you agree or disagree with the following and give
brief reasons for your answers.
f) The current ratio and the quick ratio help to assess whether a company is
able to meet its debts as they fall due. Therefore the higher these ratios are
the better placed the company is.
g) A high gearing ratio is advantageous to shareholders, because they benefit
from the income produced by investing the money borrowed.
(3 marks)
(20 marks)
QUESTION FOUR
On 1 February 19X1 the directors of Alpha Ltd issued 50,000 ordinary shares of 1
each at 120p per share, payable as to 50p on application (including the premium),
40p on allotment and the balance on 1 May 19X1.
The lists were closed on 10 February 19X1, by which date applications for 70,000
shares had been received. Of the cash received, 4,000 was returned and 6,000
was applied to the amount due on allotment, the balance of which was paid on 16
February 19X1. All shareholders paid the call due on 1 May 19X1, with the
exception of one allotee of 500 shares. These shares were forfeited on 29
September 19X1 and reissued as fully paid at 80p per share on November 19X1.

Lesson Eight

444

You are required to write up the necessary accounts, excluding those relating to
cash, to record these transactions.
CHECK YOUR ANSWERS WITH THOSE GIVEN IN LESSON 9 OF THE
STUDY PACK

445

Company Accounts

COMPREHENSIVE ASSIGNMENT No.4


TO BE SUBMITTED AFTER LESSON 8
To be carried out under examination conditions and sent to the Distance Learning
Administrator for marking by the University.
EXAMINATION PAPER.
ANSWER ALL QUESTIONS

TIME ALLOWED: THREE HOURS.

QUESTION ONE
Pesa Nyingi had a retail business and employed an assistant at a weekly wage of
Shs.5,000.00. On 2 January 2002, this assistant did not report for work and it was
found that he had left, taking with him the balance in the till. It had been Pesa
Nyingis practise to bank each Monday morning the balance in the till resulting
from the previous weeks transactions. No float was maintained. The only records
kept, apart from the bank statement, were details of sales on credit and unpaid
invoices for goods.
You ascertain the following balances on 1 January 2001.
Stock
688,000.00
Creditors
988,000.00
Bank
276,000.00
Debtors
344,000.00
Cash
228,000.00
Accrued expenses 100,000.00
Fixtures and Fittings
1,000,000.00
You also ascertain the following:
An analysis of the bank statement for the year ended 31 December 2001 showed
the following
Receipts
180,000.00
Banking from debtors cheques 4,116,000.00
Cash
4,296,000.00
Payments
Creditors for goods
Rent and expenses

3,748,000.00
232,000.00
3,980,000.00

Before banking the amounts, Pesa Nyingi paid the assistant and took Shs. 4,000.00
for himself every week.
Expenses Paid out of the till could be assumed to average Shs. 8,000.00 per week
excluding wages.
Stock at the end of the period was valued at Shs.360,000.00.
The debtors summary showed that credit sales for the period amounted to Shs.
13,960,000. An amount of Shs. 336,000.00 was still outstanding.

Lesson Eight

446

Creditors for goods have always been paid by cheque. Unpaid invoices on 31
December 2001 amounted to Shs. 1,120,000.00. Creditors for expenses were Shs.
800,000.00.
Although creditors were agreed at Shs. 1,120,000.00, goods had been returned
against a cash receipt of Shs. 48,000.00. The receipt has not been recorded
There was a fixed margin of gross profit of 20% on selling price.
The insurance company has agreed to admit a claim for the amount of the theft.
A depreciation charge of 20% is to be charged on the value outstanding on the
fixtures and fittings at the end of the year.
A cheque from one of the debtors of Shs. 10,000.00 was dishonored but this fact
has not yet been reflected in the bank statement.
Assume a 50 week year
Required:
a) Prepare workings showing your calculation of the amount of the theft.
(8 marks)
b) Prepare a trading, profit and loss account for the year ended 31 December
2001 and a balance sheet as at that date.
17 marks)
QUESTION TWO
Jambo Dealers Ltd maintains a Sales Ledger and a Purchases Ledger.
The monthly accounts of the company for May 2002 are being prepared and the
following information is available.
Sales Ledger balances as at 1 May 2002
Purchases Ledger balances as at 1 May 2002
Sales Ledger balances as at 31 May 2002
Purchases Ledger balances as at 31 May 2002
Credit Sales
Credit Purchases
Cash and cheques received
Sales Ledger
Purchases Ledger
Cash and cheques received
Sales Ledger
Purchases Ledger
Credit notes issued (for returns inwards)
Debit notes received (returns outwards)
Dishonoured cheques
Discounts allowed
Discounts received
Bad debts written off in December 2001 but now
recovered

Debit
1,672,000.
00
28,000.00
?
36,500.00

Credit
114,600.00
747,000.00
67,000.00
?
18,938,000.
00
670,000.00
1,549,700.0
0
13,000.00
47,000.00
632,000.00
119,800.00
24,000.00
32,000.00
43,000.00
33,800.00
14,200.00

447

Company Accounts

It has been decided to set off a debt due from a customer, A Mutiso, of Shs.
30,000.00 against a debt due to him of Shs. 120,000 in the creditors ledger.
The company has decided to create a provision for doubtful debts of 2.5% of the
total debtors on 31 May.
Required:
a) Prepare the sales ledger control account and the purchases ledger control
account for May 2002 in the books of Jambo Dealers Ltd.
(14
marks)
b) Produce an extract of the balance sheet as at 31 May 2002 of Jambo Dealers
Ltd relating to the companys trade debtors and trade creditors.
(3 marks)
c) Briefly explain the purpose of control accounts.
marks)

(3

QUESTION THREE
The following are the summarized trading, profit and loss accounts for the year
ended 30 April 2000, 2001, 2002 and balance sheet as at 30 April 1999, 2000,
2001 2002 for James Mwendapole, a sole trader.
Trading, Profit and Loss Accounts for the year ended 31 May

Sales
Cost of sales
Gross Profit
Expenses (including loan
interest)
Net Profit

2000
Sh000
1,000.00
(600.00)

2001
Sh000
1,200.00
(720.00)

400.00
(200.00)

480.00
(300.00)

200.00

180.00

2002
Sh000
1,400.0
0
(980.00
)
420.00
(280.00
)
140.00

Balance Sheets as at 31 May


Non Current Assets

1999
Sh000
380.00

2000
Sh000
480.00

2001
Sh000
680.00

2002
Sh000
900.00

Current Assets
Stocks
Debtors
Balance at bank
Total Current Assets

140.00
100.00
900.00
330.00

160.00
180.00
130.00
470.00

200.00
400.00
390.00
990.00

290.00
520.00
160.00
970.00

(40.00)
-

(80.00)
-

(120.00)
(500.00)

(180.00)
(500.00)

Current Liabilities
Creditors

Lesson Eight
Loan (received on 31
May 2001)
Total Current Liabilities
Net Current Assets
Net Assets
Capital
Opening Capital
Add Net Profit

448

(40.00)

(80.00)

(620.00)

(680.00)

290.00
670.00

390.00
870.00

370.00
1,050.00

290.00
1,190.00

510.00
160.00
670.00

670.00
200.00
870.00

870.00
180.00
1,050.00

1,050.00
140.00
1,190.00

Required:
1. Calculate for each of the years ended 31 May 2000, 2001, 2001, the
following financial ratios.
Return on capital employed
Quick ratio
Stock turnover
Net Profit Margin
(8 marks)
2. Use two financial ratios (not referred in (a) above) to draw attention to two
aspects to the business which would appear to give cause for concern.
(6 marks)
3. Advise James MwendaPole whether, on financial grounds, he should continue
trading and whether it was a sound decision to borrow the loan.
(6
marks)
QUESTION FOUR
Bingwa and Shabiki are in partnership as manufacturers of high quality
wheelbarrows, Bingwa being responsible for the factory and Shabiki being
responsible for sales. Completed wheelbarrows are transferred from the factory to
the warehouse at agreed prises. Bingwa and Shabiki are credited with one third of
the manufacturing profit and 10% of the trading gross profit respectively and the
balance of the firms profit being shared equally. All wheelbarrows are sold at Sh.
680.00 each. No interest is credited or charged on capital accounts or drawings.
The following trial balance was extracted on 31 March 2002.

449

Company Accounts

Sh.
Capital Accounts
Bingwa
Shabiki
Drawings
Bingwa
Shabiki
Freehold factory (including land Sh.
300,000.00)
Factory Plant at cost
Delivery Van at cost
Provision for depreciation
Freehold Factory
Factory Plant
Delivery Van
Stocks on 1 February 2001
Raw materials
Work in progress
Wheelbarrows (1220 at Shs. 440)
Sales
Return inwards
Purchases of raw materials
PAYE
Factory wages
Office wages
Expenses Factory
Office
Provision for unrealized stock (in
warehouse)
Provision for doubtful debts
Debtors and creditors
Bank

96,000.0
0
87,400.0
0
708,800.
00
326,400.
00
82,000.0
0

42,000.0
0
40,200.0
0
536,800.
00

Sh.
482,000.
00
507,000.
00

307,040.
00
110,160.
00
38,400.0
0

1,237,60
0.00

13,600.0
0
291,600.
00

8,800.00

165,400.
00
48,000.0
0
126,800.
00
143,400.
00

48,800.0
0
19,200.0
0
109,200.
00
57,200.0
0

217,600.
00
2,926,00
0.00
Additional information:

2,926,00
0.00

Lesson Eight

450

a) 1540 wheelbarrows at Sh. 480 each were transferred to the warehouse


during the year.
b) Wheelbarrows in stock being balance of the current years production, were
valued at agreed price of Sh. 480 each.
c) The stock of raw materials was Sh. 34,000.00 and work in progress is valued
at Sh. 53,600.00
d) Accrued expenses on 31 March 2002 amounted to 62,400 (including
office(Sh. 32,800.00) and prepaid rates Sh,3,200.00 (including office Shs.
1,200.00 )).
e) Provision for depreciation is to be made as follows:
Factory buildings 2% p.a.
Factory Plant
10% p.a.
Motor vehicles
25% p.a.
The general provision for doubtful debts is to maintain at 10% of the trade debtors.
Required:
Manufacturing, trading and profit and Loss Accounts for the year ended 31 March
2002 and a balance sheet as at that date.
(20 marks)
QUESTION FIVE
State and explain the qualities of useful financial information.
(10
marks)
To what extent do International Accounting Standards help achieve these qualities.
(5 marks)
(25 marks)
END OF COMPREHENSIVE ASSIGNMENT No.4
NOW SEND YOUR ANSWERS TO THE DISTANCE LEARNING CENTRE FOR
MARKING

Acknowledgement

LESSON NINE
REVISION AID
INDEX

KASNEB SYLLABUS
MODEL ANSWERS TO REINFORCING QUESTIONS

LESSON 1
LESSON 2
LESSON 3
LESSON 4
LESSON 5
LESSON 6
LESSON 7
LESSON 8
MOCK EXAMINATION

FINANCIAL ACCOUNTING 1

451

452

Revision Aid

SOLUTIONS TO REINFORCEMENT QUESTIONS


Question 1
Check the balances on your ledger accounts with the trial balance as shown below:
DR
CR

Cash at bank
1,703
Cash in hand
12
Drawings
560
Postage and stationery
129
Traveling expenses
104
Cleaning expenses
260
Sundry expenses
19
Telephone
214
Electricity
190
Motor vas
2,000
Rates
320
Fixtures ad fittings
806
Capital
2,308
Purchases
3,163
Discounts received
419
Credit sales
830
Cash sales
4,764
Discount allowed
81
Provision for
depreciation:
700
Motor van
250
Fixtures ad fittings
Stock at 1 January 20X1
366
Loan - Frey
250
Debtors Brown
12
Blue
150
Stripe
48
Creditors Live
602
Negative
_____
64
10,207
10,207

Lesson Nine

453

Workings
Cash at bank
Opening balances
Bankings of cash (908 + 940 + 766 + 1,031)
Capital introduced
Received from customers
(160 + 66 + 22 + 10 + 40 + 120 + 140 + 150 + 20 + 44
+ 38 + 20) x 90%

672
3,643
500
729
5,546

Less cheque payments (telephone, electricity, rates and


van)
Payments to suppliers
(143 + 468 + 570 + 390 + 80 + 87 + 103 + 73 + 692 +
187)

(2,374)
1,703

Cash at Bank

Bal b/d
Sales (bal)

5 Bank
4,764 Drawings
Stationery
Travel
Petrol ad van
Sundry
Postage
Cleaner
Bal c/d
4,769

3,645
560
73
40
104
19
56
260
_12
4,769

Question 2
Mary Carter
Balance Sheet as at 31.12.2001
Non current assets
Freehold premises
Plant
Current assets
Stock
Debtors
Cash at Bank
Cash in hand
Current liabilities
Creditors

25,000
12,000
37,000

8,000
7,000
1,000
6,000
22,000
(10,000)

12,000
49,000

454

Revision Aid

Capital [34,000 + 5,000


10,000]

29,000

Non current liabilities


Loan from bank

20,000
49,000

Workings
Stock:
11,000 + 34,000 37,000
= 8,000
Debtors:
10,000 + 51,000 54,000
= 7,000
Cash at bank:
5,000 16,000 2,000 1,000 36,000 + 54,000 3,000 =
1,000
Cash hand: 3,000 10,000 + 9,000 + 16,000 10,000 2,000 = 6,000
Capital
Bal b/f
Add profit
Less drawings
Profit:
Sales
Cost of sales
Electricity
Rates
Wages
Sundry expenses
Bank interest
Net profit

34,000
_5,000
39,000
(10,000)
29,000
60,000
(37,000)
(2,000)
(1,000)
(10,000)
(2,000)
(3,000)
5,000

Creditors
= 12,000 + 34,000 36,000 = 10,000
Question 3
Apparent from the text
Profit is determined by redrafting the second section of the balance sheet.
Remember that net assets will be the same as capital.
Capital b/f +
additional
Add net profit
(missing figure)
Less drawings
Capital c/f

25,000
6,000
31,000
(4,500)
26,500

Profit may be also computed as follows:


Net profit
= closing capital (net assets) opening capital + drawings
additional capital

Lesson Nine

= 26,500 20,000 + 4,500 5,000


= 6,000

455

456

Revision Aid

Question 4
Brian Barmouth
Trial balance as at 30 June 2000

Sales
Purchases
Office expenses
Insurance
Wages
Rates
Heating and lighting
Telephone
Discounts allowed
Opening stock
Return inwards
Returns outwards
Premiums
Plant and machinery
Motor vehicle
Debtors
Bank balance
Creditors
Loan long term loan
Capital
Drawings for the year

47,600

22,850
1,900
700
7,900
2,800
1,200
650
1,150
500
200
150
40,000
50,000
12,000
12,500
7,800

__4,000
121,150

3,400
10,000
60,000
______
121,150

NB: The closing stock does not appear in the trial balance.

Lesson Nine

457

LESSON 2
Question 1
(a)
1-May
13-May
16-May
24-May

(b)
4 May
11 May
18 - May
(c)
2 May
9 May
17 - May

Capital
Sales
Bruce
hill

5,000
200
700
200

1-May
19-May
20-May
21-May
30-May
30-May
31-May
_____ 31-May
6,100

Store fitments
Abel
Rent
Delivery exp
Drawings
Wages
Green
Balance c/d

2,000
650
200
50
200
320
300
2,380
6,100

SALES DAYBOOK

700
580
360
1,640

Bruce
Hill
Nailor

PURCHASES DAYBOOK

650
300
800
1,750

Abel
Green
Kaye

Check the account balances with the balances shown on the trial balance.
(d)
Cash
Sales
Purchases
Debtors
Creditors
Capital
Fixtures and fittings
Rent
Delivery expenses
Drawings
Wages

Dr

2,380

Cr

1,840

1,750
740
800
5,000
2,000
200
50
200
_320
7,640

____
7,640

458

Revision Aid

Question 2
End Papers
Trading, Profit & Loss Account for the year ended 31.12.02

Sales
15,500
Less returns inwards
(1,500)
150,000
Cost of sales
Opening stock
46,000
Purchases
103,500
Less returns outwards
(3,500)
100,000
146,000
Less closing stock
(41,000)
(105,000)
Gross profit
45,000
Discount received
200
Rent received
2,000
47,200
Expenses
Salaries and wages
18,700
Office expenses
2,500
Insurance
1,100
Electricity
600
Stationery
2,400
Advertising
3,500
Telephone
800
Rates
3,000
Discount allowed
__100
(32,700)
Net profit
14,500
End Papers
Balance Sheet as at 31 December 2002
Non current assets

Premises
Fixtures and fittings
Current assets
Stocks
Debtors
Cash in hand
Current liabilities
Bank overdraft
Creditors

80,000
5,000
85,000

41,000
4,800
200
46,000
12,000
7,500

(19,500)
26,500
111,500

Lesson Nine

459

Capital
Add net profit

111,000
14,500
125,500
(14,000)
111,500

Less drawings
Question 3

K Smooth
Trading, Profit and Loss Account for the year ended 31.3.2002

Sales
Less: Cost of sales
Opening stock
Purchases
Add carriage inwards
Less returns outwards
Less closing stock

1,816,000

6,918,500
42,000
6,960,500
(64,000)

6,896,500
8,712,500
(2,239,000)

Less expenses
Wages and salaries
Carriage outwards
Rent and rates
Communication
expenses
Commission payable
Insurance
Sundry expenses
Net profit

Current liabilities
Creditors
Capital
Add net profit

(6,473,500)
2,760,500

1,024,000
157,000
301,500
62,400
21,600
40,500
31,800

K Smooth
Balance Sheet as at 31 December 2002
Non current assets

Buildings
Fixtures
Current assets
Stocks
Debtors
Bank
Cash

9,234,000

(1,638,800)
1,121,700

2,000,000
285,000
2,285,000

2,239,000
1,432,000
297,000
11,500
3,979,500
(816,000)

3,163,500
5,448,500
5,088,800
1,121,700

460

Revision Aid

6,210,500

Less drawings

762,000
5,448,500

Lesson Nine

461

Question 4
Skates
Trading, Profit and Loss Account for the year ended 31 September 2002

Sales
13,090,000
Less: returns outwards
__(55,000)
13,035,000
Cost of sales:
Opening stock
2,391,000
Purchases
9,210,000
Add carriage inwards
___21,500
9,231,500
Less returns outwards
__(30,700)
9,200,800
11,591,800
Less closing stock
(2,747,500)
(8,844,300)
4,190,700
Less expenses
Wages and salaries
1,282,000
Carriage outwards
30,900
Motor expenses
163,000
Rent and rates
297,000
Telephone
40,500
Insurance
49,200
Office expenses
137,700
Sundries
28,400
(2,027,700)
Net profit
_2,163,000
Skates
Balance Sheet as at 30September 2002
Non current assets

Office equipment
Motor van
Current assets
Stocks
Debtors
Bank
Cash
Current liabilities
Creditors
Capital
Add net profit
Less drawings

625,000
410,000
1,035,000

2,747,500
1,239,000
311,500
__29,500
4,318,500
(937,000)

3,381,500
4,416,500
3,095,500
2,163,000
5,258,500
(842,000)
4,416,500

462

Revision Aid

LESSON 3
Question 1Adequately covered in the text.
Question 2
Also covered adequately in the text.
Question 3
Materiality
Information is material if its omission or misstatement could influence users
decisions taken on the basis of the financial statements. The materiality of the
omission or misstatement depends on the size and nature of the item in question
judged in the particular circumstances of the case. Only items material in amount
or in nature will affect the true and fair view given by a set of accounts.
Example:
If a business has a bank loan of 50,000 and a 55,000 balance on bank deposit
account, it might well be regarded as a material misstatement if these two
amounts were displayed on the balance sheet as cash at bank 5,000. In other
words, incorrect presentation may amount to material misstatement even if there
is no monetary error.
Comparability
Users must be able to compare the financial statements of an enterprise over time
to identify trends and with other enterprises statements to evaluate their relative
financial position, performance and changes in financial position. It is therefore
necessary for similar events and states of affairs to be represented in a similar
manner.
Compliance with accounting standards helps to achieve comparability by ensuring
that different entities account for similar transactions and events in a similar way.
Example:
Depreciation policy must be consistent from one period to the next, unless it
becomes inappropriate.
Prudence
The prudence concept states that where alternative procedures, or alternative
valuations, are possible, the one selected should be the one which gives the most
cautious presentation of the businesss financial position or results.
IAS 1 describes the prudence concept as being that revenue and profits are not
anticipated, but are recognized by inclusion in the profit and loss account only
when realized in the form either of cash or of other assets, the ultimate cash
realization of which can be assessed with reasonable certainty; provision is made
for all known.expenses and losses whether the amount of these is known with
certainty or is a best estimate in the light of the information available.
Example:

Lesson Nine

463

If there is any doubt as to the recoverability of debts outstanding at the year-end, a


provision should be made so that the amount in question is not included in the
profit for the year.
Objectivity:
This means that accountants must be free from bias. They must adopt a neutral
stance when analyzing accounting data. This means that they should try to strip
their answers of any personal opinion or prejudice and should be as precise and as
detailed as the situation warrants. The result of this should be that any number of
accountants will give the same answer independently of each other.

464

Revision Aid

Example:
Internally generated good will should not be capitalized in the balance sheet, as its
value cannot be determined objectively.
Relevance
The Statement of Principles for Financial Reporting states that to be useful,
information must be relevant to the decision-making needs of users. Information is
relevant when it has the ability to influence the decisions of users by helping them
to evaluate past, present or future events or to confirm or correct their past
evaluations.
Example:
Suppliers and other creditors would like to have information that enables them to
determine if to lend to the firm or supply on credit.
Question 4
Information is material if its omission or misstatement could influence the
economic decisions of users taken on the basis of the financial statements.
Factors affecting materiality are:
The size of the item;
The nature of the item.
To be useful, information must be relevant to the decision-making needs of users.
Information is relevant when it influences the economic decisions of users by
helping them evaluate past, present or future events or confirming, or correcting
their past evaluations.
Neutrality means that the information in financial statements should be free from
deliberate
or systematic bias.
Prudence means that a degree of caution is needed in making estimates about
certain items.
The potential conflict between the two is that neutrality requires freedom from
bias while the exercise of prudence is a potentially biased concept since judgment
is required.
In resolving the conflict, a balance should be found that neither overstates nor
understates assets, gains, liabilities and losses.
Safeguards to ensure that a companys financial statements are free from material
error:
The fact that the financial statements have been audited by an independent
professional;
The existence of sound internal controls within the company;
The existence of an internal audit function within the company.

Lesson Nine

Question 1

465

LESSON FOUR

David Douglleu
Trading and Profit and Loss Account for the year ended 31 March 2001

Sales
378,500
Less returns
(4,100)
inwards
374,400
Less cost of sales
Opening stock
120,600
Purchases
261,700
Less returns out
(7,700)
254,000
374,600
Less closing
102,500
272,100
stock
Gross profit
102,300
Add
Discount
2,400
received
Rent received
7,500
112,200
Less expenses
Salaries and
45,700
wages
Office expenses
8,400
Insurance
2,200
premiums
Electricity
2,300
Stationery
6,200
Advertising
8,900
Telephone
2,100
Business rates
6,000
Discounts
600
(82,400)
allowed
Net profit
29,800

466

Revision Aid

Balance Sheet as at 31 March 2001


Non current assets

Warehouse shop and


office
Fixtures and fittings
Current assets
Stocks
Debtors
Prepayments
Cash in hand
Current liabilities
Creditors
Accrued expenses
Bank overdraft
Capital
Add Net Profit
Less drawings

210,000
12,800
222,800

102,500
13,000
2,400
500
118,400
18,700
1,200
30,000

(49,900)

68,500
291,300
287,500
29,800
317,300
(26,000)
291,300

Question 2
Donald Brown
Trading and Profit and Loss Account fro the year ended 31 December 20X0

Sales
491,620
Less cost of sales
Opening stock
18,460
Purchases
387,936
406,396
Closing stock
19,926
386,470
Gross profit
105,150
Discounts received
1,175
106,325
Less expenses:
Discounts allowed
1,304
Lighting and heating
6,184
Motor expenses
3,080
Rent
8,161
General expenses
7,413

Lesson Nine

Depreciation (w)
Net profit

467

13,146
39,288
67,037

Working:
Depreciation charge:
Motor vehicles: 45,730 x 20% = 9,146
Fixtures and fittings: 10% x (42,200 2,200) = 4,000
Total: 4,000 + 9,146 = 13,146
Donald Brown
Balance Sheet as at 31 December 20X0
Cost
Depreciation

Non current
asset
s
Fixtures and
42,200
6,200
fittings
Motor vehicles
45,730
24,438
87,930
30,638
Current assets
Stock
19,926
Debtors
42,737
Prepayments
680
Cash in hand
1,411
64,754
Current
liabil
ities
Creditors
35,404
Accruals
218
Bank overdraft
19,861
55,483
Net current
assets
Net assets
Financed by
Capital
Net Profit for
year
Less drawings

Net

36,000
21,292
57,292

9,271
66,563
26,094
67,037
93,131
26,568
66,563

468

Revision Aid

Question 3
Brenda Bailey
Trading and Profit and Loss Account for the year ended 30 June 20X9

Sales
427,726
Opening stock
15,310
Purchases
302,419
Carriage inwards
476
318,205
Less closing stock
16,480
Cost of sales
301,725
Gross profit
126,001
Carriage outwards
829
Wages and salaries
64,210
Rent and rates (12,466
11846
620)
Heat and light (4,757 + 5,107
350)
Depreciation
10,200
equipment
Motor
8,654
vehicles
Sundry expenses
8,426
109,272
Net profit for the year
16,729
Brenda Bailey
Balance Sheet as at 30 June 20X9
Cost
Depreciation
Non current
assets
Equipment
Motor vehicles
Current assets
Stock
Debtors
Prepayments
Cash
Current liabilities
Bank overdraft
Creditors
Accruals

102,000
43,270
145,270

Net book
value

32,450
17,574
50,024

69,550
25,696
95,246

16,480
50,633
620
__477
68,210
3,295
41,792
350
45,437

Lesson Nine

469

Net current assets

22,773
118,019

Capital
Balance at 1 July
20X8
Add Profit for year

122,890
16,729
139,619
21,600
118,019

Less drawings
Balance at 30 June
20X9
Question 4
Frank Mercer
20X8
Dec 31

Balance b/f

Dec 31

Dividend

Cash book
20X8
1,793 Dec 31 Bank
charges
26 Dec 31 Standing
order
Dec 31 Direct
debit
____
Balance
c/d
1,819

Bank reconciliation as at 31 December 20X8

Balance per bank


statement
Add unrecorded
lodgments:
V Owen
K Walters

1,557

98
134
232

Less unpresented
cheques:
B Oliver (869)
L Philips (872)
Balance per cash book
(corrected)

71
37
(108)
1,681

18
32
88
1,681
1,819

470

Revision Aid

LESSON FIVE
Question 1

Bal b/d
Sales (163,194 +
1,386)
Cash refund

Sales Ledger Control A/C

386,430 Bal b/d


190
164,580 Cash received
158,288
350 Discounts
allowed
Returns inwards
Contra
Bad debts
written off
______ Balance c/d
551,730

Bal b/d
Cash paid (103,040 350)
Discounts received
Returns outwards (1,370
+ 2,000)
Contra
Balance c/d

2,160
590
870
1,360
388,272
551,730

Sales Ledger Control A/C

520 Bal b/d


102,690 Purchases (98,192
+ 36)
990 Bad debts
3,370
870
175,048 Balance c/d
283,488

184,740
98,228
2,160

___100
283,488

Question 2
(a)
Uncorrected balance b/f
Sales omitted (a)
Bank cheque dishonored
(1)

Balance b/d

Sales Ledger Control A/C

12,550 Discounts omitted (d)


850 Contra entry omitted
(f)
300 Bad debt omitted (g)
Returns inwards
omitted (j)
_____ Amended balance c/d
13,700
12,500

100
400
500
200
12,500
13,700

Lesson Nine

471

Note: Items (b), (c), (e), (h), (i) and (k) are matters affecting the personal accounts
of customers. They have no effect on the control account.

(b)
Statement Of Adjustments To List Of Personal Account Balances

Original total of list of balances


12,802
Add: debit balance omitted (b)
300
debit balance understated (e)
200
500
13,302
Less: transposition error (c ): understatement
180
of cash received
cash debited instead of credited (2 x
500
250) (h)
discounts received wrongly debited to
50
Bell (i)
Understatement of cash received (i)
_72
__802
12,500
Question 3
(a)
Balance
Correction of error interest
Balance

George Cash book

4,890 Bank charges (3)


320 Plant (4)
11,890
Cheque
dishonored
Correction of
error in entering
cheque (6)
_____ Error in addition
(7)
17,100

(b)

320
10,000
980
4,800
1,000
17,100

472

Revision Aid

Bank reconciliation
Balance per bank statement
Less lodgments not credited (2)
Add: dishonored cheque
Add: outstanding cheque (1)
Balance per cash book - overdrawn

12,800
2,890
9,910
980
1,000
11,890

Lesson Nine

473

Statement of effect on profit


Profit per draft accounts
Bank charges (3)
Depreciation (4)
Bad debt (5)
Motor expenses (6)
Additional depreciation (6)
Purchases understated (7)
Interest adjustment (8)
Repairs to premises (9)

81,208

320
1,000
980
2,100
600
1,000

320
__870
82,398
_6,300
76,098

____
6,300

(d)
Journal
George drawings
Repairs to premises
Repairs to Georges house
mistakenly charged as a business
expense
*Paul accounts payable ledger
account
George drawings

870

870

540
540

Business account paid by personal


cheque
*Note: a debit to accounts payable ledger control account is also acceptable for
this entry.
Question 4
Four errors not disclosed by the Trial Balance:
Error of Omission: This is where a transaction is completely omitted from the
records i.e. not posted at all.
Error of Commission: A transaction is posted in the wrong account but of the
same class e.g. a credit sale posted in a wrong debtors account (e.g. to debtor 1
instead of debtor 2)
Error of Principle: A transaction is not only posted to the wrong account but also
the class e.g. an expense of plant repair posted to the plant account (an asset).
Error of Original Entry: A transaction is posted to the correct accounts but the
amount is incorrect e.g. a credit sale of 250 is posted to the debtor and sales
account as 520.
(Refer to the text for further details)

474

Revision Aid

Lesson Nine

475

(b)
(i)
DR (Sh)
10,000

Suspense
ABD Bank loan

CR (Sh)
10,000

Cashbook
P& L Rent received

4,000

P& L Trading account


Closing stock

1,500

4,000
1,500

P& L discount allowed


P&L discount received

500
500

P& L Trading a/c opening stock


Suspense
Prepayments (prepaid insurance))
P& L
Insurance receivable
P& L - income

3,200
3,200
220
220
12,000
12,000

(ii)
Statement of Corrected net profit
(Sh)
Net profit
Add: Rent received
Discount received
Prepaid insurance
Insurance receivable

4,000
500
220
12,000

Less: closing stock overvalued


Discount allowed
Opening stock
Adjusted net profit
(iii)
ABD Loan

1,500
500
3,200

(Sh)
64,000

16,720
80,720
(5,200)
75,520

Suspense A/c
Sh
10,000 Balance b/d
_____ Opening stock
10,000

Sh
6,800
3,200
10,000

476

Revision Aid

Question 5
ACCOUNTS

Pre-Adjusted
Trial Balance
Dr
Cr

Capital
Purchases

Furniture
Debtors
Other
expenses
Creditors
Commission

4,814
4,307

350

4,638
946
82,79
5

25,00
0
13,05
0
5,833
1,060
4,638
1,066

350

350

205
165

Bad debts
Closing stock
Net profit

800

4,638
1,066

350
205

205
165

165

120

120

1,45
0
307

1,450

1,450

307

307

83,26
5
5

4,063
43,12
0
7

2,59
7
3

2,59
7
4

40,00
0

615

25,00
0
13,05
0
5,833
1,060

120

Liab.

36,24
6

800
1,45
0
307

Asset
s

5,164
4,307

615

165

25,00
0
14,50
0
6,140
1,060

Salaries due
Prepaid
insurance
Rent recd in
adv.
Accrued
commission
Depreciation

Balance sheet

26,15
4

5,164
4,307
205

965

T&P&L
Account
Dr
Cr

36,24
6

820

40,00
0

26,15
4
36,24
6

82,79
5

Column
numbers

26,15
4

Sales
Salaries
Opening
stock
Insurance
Rent income
Buildings

40,00
0

WORKSHEET
Adjustmen
Adjusted
ts Trial Balance
Dr
Cr
Dr
Cr

120

83,26
5
6

5,008

5,008

43,12
0
8

49,21
6
9

49,21
6
10

Lesson Nine

477

Question 6
Purpose of Control Accounts
i. To provide for arithmetic check on the postings made in the individual
account i.e. either the sales ledger or the purchases ledger.
ii. To provide a quick total of the debtors and creditors balances to be shown in
the trial balance.
iii. To detect and prevent errors and frauds on the debtor and creditors
account.
iv. To facilitate delegation of duties especially where the debtors and creditors
are many.

478

Bal b/d
Sales
Bills received
dishonored
Charges payable
Bal c/d

Bal b/d
Returns outwards
Bills payable
Bank
Cash
Balance c/d

Revision Aid

Sales
Sh
6,185,000
8,452,000
88,500

Ledger Control A/C


Bal b/d
Returns inwards
Bank

10,000 Cash
Bad debt
Discounts allowed
44,000 Bal c/d
14,779,000

Purchases ledger control a/c


Sh
16,500 Bal b/d
284,000 Purchases
930,000 Bills payable
dishonored
473,200
88,500
_4,196,500 Balance c/d
10,396,000

Sh
52,500
203,500
7,985,000
153,000
64,500
302,000
5,404,000
14,779,00
0

Sh
4,285,000
5,687,500
400,000

___23,500
10,396,00
0

Lesson Nine

479

LESSON SIX
Question 1
(a)
Dare
Statement of Capital as at 1 January 1996
Assets

Stocks
Debtors
Rates prepaid
Fixtures
Liabilities
Bank overdraft (add
unpresented cheques)
Accrued expenses
Creditors
Loan
Accrued interest [4,000 x 3% x
3
/12]
Heating and lighting

4,500
2,800
40
2,500
10,140

1,172
240
1,800
4,000
30
80

(7,322)
2,818

(b)
Dare
Profit and loss account for the year ended 31 December 1996

Gross profit
9,000
Discounts received
480
9,480
Less expenses
Rent and rates
465
Fixtures and fittings
350
(depreciation)
Lighting and heating
200
General expenses
450
Loan interest
120
Wages
2,914
Sundry expenses
140
Discounts allowed
520
Bad debt
200
(5,659)
Net profit
3,821

480

Revision Aid

(c)
Dare
Balance Sheet as at 31 December 19X6
Non current assets

Fixtures and fittings


Current assets
Stocks
Debtors
Prepayments
Bank (less unpresented
cheques)
Cash
Current liabilities
Creditors
Accruals
Capital
Net profit
Less drawings
Non current liabilities
Loan 3%

2,200
_290

2,550

5,800
3,000
50
673
__20
9,543
(2,490)
7,053
9,603
2,818
3,821
6,639
(1,036)
5,603
4,000
9,603

Question 2
AB Sport and Social Club
Income and Expenditure Account for the year ended 31 December 20X5

Income
Subscriptions (W1)
10,690
Bar and caf profit (W2)
9,200
Sale of sportswear (W3)
1,400
Hire of sportswear (W5)
1,700
Deposit account interest
800
23,790
Expenditure
Rent of clubhouse
6,000
Groundsperson
10,000
Heating oil (W6)
4,500
Depreciation 5,000 x 10%
500
(21,000)
Surplus of income over expenditure for
2,790
the year

Lesson Nine

481

482

Revision Aid

AB Sport and Social Club


Balance Sheet as at 31 December 20X5

Non current assets


Equipment for grounds person: cost
Depreciation
(3,500 + 500)

5,000
(4,000)
1,000

Current assets
Heating oil
Bar and caf stocks
Sports equipment for sale (4,000 2,000)
Sports equipment for hire (1,000 + 500)
Subscriptions in arrears
Bank deposit account
Bank current account

700
5,000
2,000
1,500
90
16,000
1,300
26,590

Current liabilities
Creditors for bar and caf purchases
Creditors for sportswear
Creditors for heating oil
Subscriptions in advance

800
450
200
_200
1,650

Net current assets


Net assets
Accumulated fund b/f
Surplus for the year
Accumulated fund c/f

24,940
25,940
23,150
2,790
25,940

Workings:
Subscriptions
Arrears b/f 1.1.X5 (10 + 230)
Subscription income for year
(bal fig)
Advance c/f 31.12X5

SUBSCRIPTIONS

240 Advance b/f


40
1.1.X5
10,690 Cash received
11,000

__200 Arrears c/f


___90
31.12.X5
11,130
11,130
Note: The write off of the 20X3 arrears (10) is dealt with in the above working.
Bar and caf profit

Sales

20,000

Lesson Nine

Cost of sales
Opening stock
Purchases*

483

7,000
8,800
15,800
(5,000)

Closing stock
10,800
Profit
9,200
*Note: Purchases are 9,000 + 800 1,000 = 8,800

484

Revision Aid

Sale of sportswear

Sales
Opening stock
Purchases (W4)

5,000

3,000
3,100
6,100
(4,000)

Closing stock
Closing stock
Gross Profit
Sportswear written
down
Net profit

(2,100)
2,900
(1,500)
1,400

Purchases of sportswear

4,500
450
(300)
4,650
For hire 1/3: 1,550

Bank
Add closing creditors
Less opening creditors
For sale 2/3: 3,100
Hire of sportswear

Receipts
Costs*
Opening stock
Purchases (W4)

3,000

750
1,550
2,300
(1,000)

Closing stock

(1,300)
1,700

Profit

*Note: While there is a case for treating the sportswear for hire as non current
assets, in club accounts it is more usual to treat such items as stock in trade.
Heating Oil
Opening stock
Purchases (4,000 +
200)
Less closing stock

1,00
0
4,20
0
5,20
0
(700
)

Lesson Nine

Expense for year

485

4,50
0

486

Revision Aid

Question 3
Mr Cherono
Manufacturing Profit and loss Account for the year ended 30 June 1988
Raw materials
Opening stock
40,0000
Purchases
855,000
895,000
Less closing stock
(80,000)
Raw materials consumed
815,000
Wages
_50,000
865,000
Factory overheads
Wages
96,000
Rent and rates
22,500
Water and electricity
13,000
131,500
Cost of goods completed
996,500
Factory profit
___3,500
Transfer price
1,000,000
Sales
Less cost of sales
Opening stock
Purchases and cost of goods
produced
Less closing stock
Gross profit
Profit on disposal of motor
vehicle
Factory profit
Less expenses
Interest on loan
Depreciation fixtures and
fittings
Motor vehicles
Wages
Rent and rates
Water and electricity
Motor expenses
Bad debt
Repairs
Bank charges
Insurance
Sundry expenses
Commission to lampshade

4,100,000
348,000
3,400,000
3,748,000
(282,000)

(3,466,000)
634,000
4,000
3,500
641,500

36,000
90,000
38,000
108,000
67,500
39,000
60,800
14,000
12,000
4,000
13,500
25,200
318

Lesson Nine

employee
Less UPCS
Net profit

487

__120

(508,438)
133,062

488

Cherono
Balance Sheet as at 30 June 1998
Non current assets
Fixtures and fittings
Motor vehicles
Current assets
Stock: Raw materials
Lampshades
Less UPCS
Other goods
Debtors
Prepayments
Bank balance
Current liabilities
Creditors
Accruals
Capital
Add net profit
Less drawings
Add loan

Revision Aid

Cost
900,000
152,000
1,052,000
80,000
30,000
(120)
252,000

107,000
27,318

Depreciatio
n
(440,000)
(38,000)
(478,000)

NBV
460,000
114,000
574,000

361,880
108,000
10,500
_98,000
578,380
(134,318)

444,062
1,018,062
740,000
133,062
873,062
(95,000)
778,062
_240,000
1,018,062

Lesson Nine

Question 4
Olympiad Athletics Club
Income and Expenditure Account for year ended 31 October 1983

Income
Annual subscriptions (4,680 + 70 + 230
4,740
(140 + 100))
Entrance fees
250
Life membership fees credited (850 + 53)
903
5,893
Training ground fees (7,660 470 + 325)
7,515
Sales of sporting requisites
8,774
Investment interest received
626
Insurance commissions received (53 11
55
+ 13)
Advertising revenue
603
Profit on sale of furniture (370 350)
20
Total income
23,486
Expenditure:
Cost of sporting requisites sold
(5,270 + 202 163 = 5,309 (purchases)
5,309 + 811 1,064 = 5,056)
5,056
Damaged stock etc
137
Wages of grounds man (250 + 3,600
3,550
300)
Postages (692 4)
688
Stationery (55 + 629 36)
648
Rates (300 + 846 380)
766
Subscriptions in arrear written off
40
World-wide Athletics Club affiliation fee
50
Training ground upkeep
1,200
Depreciation: buildings
3,500
Furniture, equipment etc
(10% x (7,900 800)
__710
Total expenditure
16,345
Surplus of income over expenditure
7,141

489

490

Revision Aid

Olympiad Athletics Club


Balance Sheet as at 31 October 1983
Non current assets
Land
Buildings
Furniture, equipment, etc

Cost
4,000
35,000
7,100
46,100

Investments
Investments at cost (7,400 + 5,600)
(current valuation 13,150)
Current assets
Stocks sporting requisites
- stationery
- stamps
Debtors subscriptions
- insurance commissions
Prepayments (300 + 380)
Bank deposit account
- current account
Cash
Current Liabilities
Creditors prepaid subscriptions
- Prepaid training
- Ground fees
- Premiums
- Sporting requisites

Depreciati
on
12,900
4,410
17,310

Net
4,000
22,100
2,690
28,790
13,000

927
36
4
230
13
680
3,000
2,563
122
7,575
100
470
160
202
932

Working capital
Net assets employed
Financed by:
Accumulated fund: as at 31 October
1982
Add:
Surplus of income over expenditure
for the year
As at 31 October 1983
Life membership fund (4,720 + 530
(850 + 53)

6,643
48,433
36,945
7,141
44,086
4,347
48,433

Lesson Nine

491

Accumulated fund b/f


Assets: Land
Buildings
Furniture
Investment
Stocks
Debtors
Prepayments
cash
Liabilities
Creditors: Subscriptions
Training
Premiums
Sporting requisites
Bank overdraft
Membership fund

4,000
25,600
3,750
7,400
866
191
550
___73
42,430
70
325
102
163
105
4,720

(5,485)
36,945

492

Revision Aid

LESSON 7
Question 1
Kimeu & Mwangi
Manufacturing,Trading Profit and Loss account for the year to 31.3.x 2
Shs
Shs
Raw materials
Opening stock
100,700
Purchases
716,250
816,950
Less stock of raw materials
(79,500)
Raw materials consumed
737,450
Factory wages
382,500
1,119,950
Prime cost
Add opening w/p
85,000
Less closing w/p
(126,250)
(41,250)
1,078,700
Factory overheads
Depreciation on plant
84,375
Factory expenses
354,000
438,375
Factory cost of completed goods
1,517,075
Add factory profit ( missing figure)
192,925
Transfer price given in the question (par)
1,710,000
(38,000 x 45)
Sales
Cost of sales
Opening stock of finished goods
Transfer price
Less closing stock of finished goods

2,775,500
1,200,000
1,710,000
2,910,000
(10,125,000
)

878,000
192,925
1,070,925

Add factory profit


Expenses
Depreciation on delivery van
Sales department wages
Selling department expenses
Increase for provision for bad debts
Provision for unrealized profits
Net profit
Share of factory profits
K
M
Share of remaining profit

(1,897,500)

80,250
150,750
277,500
5,000
112,500
154,340
38,585

(626,000)
444,925
(192,925)
252,000

Lesson Nine

493

K
M

100,800
151,200

252,000

494

Revision Aid

Workings for closing stock of completed units


Completed units b/f
Units manufactured
Less units sold
Closing stock

Drawings
Bal c/d

K
15,00
0
105,1
40
255,1
40

30,000
38,000
(45,500)
22,500 x 45 =
10,125,000
M
125,00
0
54,785

Share of factory
profit
Balance of profit

189,78
5

Kimeu & Maingi


Balance Sheet as at 31 March 1992
Non current assets
Property, plant and equipment
Freehold factory
Factory plant ( 843,750 151,250
84,375 = 608,125)
Delivery van ( 401,250 80,250 86,250
= 234,750)

K
154,34
0
100,80
0
255,14
0

Shs

M
38,585
51,200
189,785

Shs
1,053,750
608,125
234,750
1,896,625

Current Assets:
Stock: Raw materials
W.I.P
Finished goods
Debtors
Current Liabilities
Bank overdraft
Trade creditors
Accrued expenses and deferred income

79,500
126,250
900,000
405,000
1,510,750
(176,200)
(150,000)
(86,250)
(412,450)

Net current assets

1,098,300
2,994,925
1,400,000
1,425,000
2,825,000

Capitals: K
M
Current A/c: K
M

105,140
64,785

169,925
2,994,925

Lesson Nine

The finished good is net of the unrealized profit on closing stock.

495

496

Revision Aid

Question 2
Amis Lodge and Pym
Trading, Profit and loss appropriation account for year ended 31 March 198

Sales
404,500
Less
Opening stock
30,000
Purchases
225,000
Carriage inwards
4,000
229,000
Plant depreciation
259,000
Closing stock
(35,000)
Cost of sales
(224,000)
Gross profit
180,500
Discount received
4,530
Interest received
____750
185,780
Expenses
Carriage outwards
12,000
Vehicle depreciation
15,000
[25% x (80,000 20,000)]
Depreciation of plant
20,000
[20% x 100,000]
Discounts allowed
10,000
Office expenses [30,000 + 405]
30,805
Rent, rates , heat and light
7,300
[8,800 1,500]
Provision for bad debts increase
[(5% x 14,300) 420]
295
(95,400)
Net profit for year
90,380
Interest charged on drawings
etc
Amis
1,000
Lodge
900
Pym
720
2,620
93,000
Less
Salary Pym
13,000
Interest on capital accounts
Amis
8,000
Lodge
1,500
Pym
500
23,000
Residual profit
70,000

Lesson Nine

497

Less
Share of residual profit
Amis ( 5/10)
Lodge (3/10)
Pym (2/10)

35,000
21,000
14,000
70,000

(b)

Balances
Drawings
Appropn
interest
Bal c/d

1,00
0
25,0
00
1,00
0
16,0
00
43,0
00

500
22,00
0
900
_____
23,40
0

Current Accounts
P
A

400 Appropn
salary
15,0
8,00
00 Interest
0
720
35,0
Residue
00
11,3 Bal c/d
_____
80
27,5
43,0
00
00

1,500

13,00
0
500

21,00
0
900

14,00
0
____-

23,40
0

27,50
0

Question 3
Amber, Beryl and Coral
Trading, Profit and Loss Account for the year to 31 December 1996
000
000
Sales
2,000
Cost of sales
Opening stock
180
Purchases
1,400
1,580
Closing stock
(200)
1,380
Gross profit
620
Expenses
Wages and salaries (228
240
+ 12)
Sundry expenses
120
Bad and doubtful debts
26
Depreciation:
Building
5
Plant and
24
equipment
Interest on loan
__5
Amber
420
Net profit
200
Assume profit is earned proportionately throughout the year

498

Revision Aid

Profit and Loss Appropriation Account

1/1X6 to 30.6.X6
Salaries
Share of profit:
80,000 (60:40)
1.7.X6 to 31.12.X6
100,000 (40:40:20)

Amber
000

Beryl
000

Coral
000

10

10

20

48

32

80

40
98

40
82

20
20

100
200

Amber, Beryl and Coral


Balance Sheet as at 31 December 1996
Cost or
Aggregate
valuation depreciatio
n
Non current assets
Land at valuation
280
Nil
Buildings
250
35
Plant, equipment and
240
74
vehicles
770
109
Current assets
Stock
200
Debtors (420 16)
404
Less: provision for doubtful
30
debts
374
Cash at bank
38
612
Current liabilities
Trade creditor
350
Bonus
12
362
Net current assets
Long term loan Amber
Represented by:
Capital accounts: Amber
Beryl

Total
000

368
242

Net book
value
280
215
166
661

250
911
50
861

Lesson Nine

499

Coral

100
710

Current accounts: Amber


Beryl
Coral

82
64
_5
151
861

Proprietor funds

Goodwill
Balances c/f

A
00
0

B
00
0

80
368

80
242

448

322

CAPITAL ACCOUNTS
C
A
00
000
0
Balances b/f
280
40 Cash
100 Goodwill
120
(W1)
Revaluation
48
140
448
Balances b/f
368

B
000

C
000

210
140
80
32
322
242

140
100

500

Drawings
Balances c/f

Revision Aid

A
00
0
28
82

B
00
0
24
64

__

__

110

88

CURRENT ACCOUNTS
C
A
00
000
0
15 Balances b/f
7
5 Profit for
98
year
__ Loan
_5
interest
20
110

B
000

C
000

6
82

20

__

__

88

20

Question 4
The solution provided has the workings shown beside the accounts to make the
comparison easier. Remember to adhere to previous partnerships and
departmental formats.
(a) Aristocratic Autos
Trading and Profit and Loss Account for year ended 30 September 1986
Working Worksho Petrol/oil Showroo
Total
s
p
m

Sales and charges:


32,125
32,964
8,500 Cash
73,589
65,892
41,252
81,914 Credit
189,05
8
98,017
74,216
90,414 Total turnover
262,64
7

(2)

(1)

1,932
23,860

3,018
41,805

Less materials:
20,720 Opening stock
52,100 Purchases

25,792

44,823

72,820

(2,752)

(2,976)

23,040

41,847

47,510 Usage

34,163
57,203

5,685
47,532

____- Direct wages


47,510 Cost of sales

40,814

26,684

42,904 Gross profit

1,333

42,147

26,684

(25,310) Closing stock

- Profit on sale of
plant
42,904
Less

25,670
117,76
5
143,43
5
(31,038
)
112,39
7
39,848
152,24
5
110,40
2
1,333
111,73
5

Lesson Nine

(3)
(4)
(5)
(6)
(7)

501

7,024
2,613
1,939
4,477
16,898
32,951
9,196

2,945
2,185
3,389
8,324
16,843
9,841

4,391
10,200
7,880
5,846
4,130
11,302
43,749
(845)

Indirect wages
Salaries
Rates
Electricity
General expenses
Depreciation
Total
Net profit/loss for
year
Less appropriations
Interest on capitals
Duke (5% x
50,0000
Earl (5% x 40,000)
Residual profit*
Duke
Earl

11,415
10,200
13,438
9,970
11,996
36,524
93,543
18,192

(2,500)

(2,000)
13,692
(6,846)
(6,846)
*The equal division stipulated by the Partnership Act applies in the absence of
agreement to the contrary.

502

Revision Aid

Aristocratic Autos
Balance Sheet as at 30 September 1986
Working
Worksho Petrol/oil Showroo
s
p
m

(8)
(8)

(4)

(9)

5,020
24,891

4,260
4,859

29,911

9,119

2,752
1,365
2,586
316
7,019

2,976
537
2,915
1,605
8,033

4,225
915
5,140
1,879
31,790

5,602
564
6,166
1,867
10,986

Total

Non current assets at


written down value:
11,010 Freehold buildings
5,357 Plant, equipment and
vehicles
16,367
Current assets:
25,310 Stocks
- Debtors
7,799 Prepayments
30,470 Bank and cash
63,579
Current liabilities:
15,250 Creditors
983 Accruals
16,233
47,346 Working capital
63,713 Net assets employed

20,290
35,107
55,397
31,038
1,902
13,300
32,391
78,631
25,077
2,462
27,539
51,092
106,48
9

Financed by
Capital accounts
Duke
50,000
Earl
40,000
90,000
Current accounts:
Duke
6,906
Earl
9,853

(10)
(10)

16,489
106,48
9

Working
s
(1)

Plant disposal:

Cost
Accumulated depreciation

19,500
(15,633
)

Lesson Nine

503

Written down value


Proceeds
Profit on sale
Worksh
op

34,050
113
34,163
6,810
214
7,024

5,199
(2,586)

Petrol/oil

showroo
m

5,602
83
5,685

______-

4,160
231
4,391

5,860
(2,915)

15,679
(7,799)

2,613

2,945

7,880

1,838
101
1,939

2,072
113
2,185

5,543
303
5,846

3,990
487
4,477

3,021
368
3,389

3,681
449
4,130

2,520
14,378
16,898

2,840
5,484
8,324

7,600
3,702
11,302

Petrol/oil

Showroo
m

14,200
_____14,200

38,000
_____38,000

Worksh
op

12,600
_____12,600

3,867
5,200
1,333
Total

(2) Direct wages:


Per list
Accrual
Total
(3) Indirect wages:
Per list
Accrual
(4) Rates (apportioned on basis
of freehold buildings at (8)
below:
Per list
Prepayment
Total
(5) Electricity apportioned on
same basis as (4) above:
Per list
Accruals
Total
(6) General expenses
(apportioned on basis of
turnover:
Per list
Accruals
Total
(7) Depreciation:
Charge for year per (8)
below:
Freehold buildings
Plant, equipment etc
Total

(8) Freehold buildings (cost):


At 1 October 1985
Additions during year
Disposals during year
At 30 September 1986
Provision for depreciation

39,652
___196
39,848
10,970
445
11,415

26,738
(13,300
)
13,438
9,453
517
9,970

10,692
1,304
11,996

12,960
23,564
36,524

64,800
_____64,800

504

Revision Aid

7,100

19,390

on freehold buildings:
At 1 October 1985

31,550

5,060
-

Disposals during year

2,520

2,840

7,600

7,580

9,940

4,260

5,020

Charge for year

12,960

26,990

At 30 September 1986

44,510

11,010

Written down value at 30


September 1986

20,290

Plant, equipment etc. (cost):


65,180

22,900

17,450

26,210

4,520

1,060

Additions during year

(19,500)

_____-

____-

Disposals during year

27,420

18,510

71,890

48,254

17,077

9,451

14,378

5,484

3,702

46,999

22,561

13,153

(15,633)

At 1 October 1985

At 30 September 1986

Provision for depreciation


on plant, equipment etc
At 1 October 1985
Disposals during year
Charge for year
At 30 September 1986

105,53
0
31,790
(19,500
)
117,82
0

74,782
(15,633
)
23,564
82,713

Written down value


24,891

4,859

5,357

At 30 September 1986
(9) Accruals (per workings
above):
(2)

35,107

113

83

196

214

231

(3)

445

101

113

303

(5)

517

487

368

449

(6)

1,304

Lesson Nine

505

915

564

983

2,462
(10) Current accounts:
Earl
Duke
Opening balance
9,750
Interest on capital
2,500
Residual profit
6,846
Drawings
(12,190)
Closing balance
6,906

10,477
2,000
6,846
(9,740)
9,583

Question 5
WORKINGS
The first step is to derive the profit for the period:
Closing

Opening

Goodwill w/o

Closing
balances
Exors of
R decd

Assets minus external liabilities


(17,000 + 3,480 + 1,100 + 2,230 +
3,370 980)
Add back drawings (2,000 + 1,600 +
1,800)

26,200
5,400
31,600
25,240

Less assets minus external liabilities


(26,060 820)
Profit for period (1st July to 31st October)

3,00
0

3,00
0

7,50
0

7,50
0

11,5
00
11,5
00

10,5
00

10,5
00

6,360

CAPITAL ACCOUNTS
A
R
S

1,5 Opening
9,000 8,00
00 balances
0
Goodwill
2,500 2,50
raised
0
4,0 Bank
00
Capital
- Premium (1/5
x 7,500)
5,5
00

11,50
0

10,5
00

CURRENT ACCOUNTS

8,00
0
2,50
0
4,00
0
1,50
0

4,00
0
1,50
0

10,5
00

5,50
0

506

Balance b/d
Drawings
Closing
balance
Exors of R
(decd)

Revision Aid

T A

100

- Balances b/d

140

200

2,00
0
-

1,60
0
720

1,80
0
720

- Appropriation
a/c
- (profit)

2,12
0

2,12
0

2,12
0

260

2,26
0

2,32
0

2,12
0

2,26
0

2,32
0

2,12
0

The Capital and Current Accounts are given as workings for the Balance sheet
figures.

LESSON 8
Question 1 (a)
AZ Ltd
Manufacturing , Trading Profit and Loss Account for the year ended 31
October 1999
Sh000
Sh000
Raw Materials
Opening stock Raw material
380
Purchases Raw material
9,500
9,880
Less closing stock Raw material
(465)
Cost of raw materials consumed
9,415
Add: direct wages
1,350
direct expenses
_395
1,745
Prime Cost
11,160
Factory overheads
Factory expenses
290
Indirect materials
350
Factory insurance
150
Depreciation plant and
5,160
5,950
machinery
Total cost of production
17,110
Add opening W.I.P
560
17,670
Less closing W.I.P Finished
(695)
goods
Factory cost of production
16,975

Lesson Nine

finished goods
Sales
Less cost of sales
Opening stock finished goods
Factory cost of production
finished goods
Less closing stock finished
goods
Gross profit
Less expenses
Sales room expenses
Administration expenses
Office salaries and wages
Vehicle running expenses
Bad debts w/o
Overdraft interest
Debenture interest
Depreciation: furniture and
equipment
Motor vehicles
Less dividend
Net profit for the year
Add retained profit b/d
Less transfer to general reserve
Retained profit c/d

507

28,550
420
16,975
17,395
(610)

(16,785)
11,765

485
620
898
656
64
725
800
89
4,125

(8,462)
3,303
(4,000)
(697)
5,500
4,803
(2,000)
2,803

508

Revision Aid

AZ Ltd
Balance Sheet as at 31 October 1999
Non current assets
Land & Buildings
Plant and machinery
Furniture and equipment
Motor vehicles

Sh000
30,000
25,800
890
16,500
73,190

Current Assets
Stock Finished goods
Raw materials
WIP
Current Liabilities
Bank overdraft
Creditors
Accruals
Debenture interest
Dividends accrued
Financed by:
Authorized and issued
capital
Capital reserve
Share premium
Revenue reserve
General reserve
Retained profit
Non current liability
8% debenture

Sh000
(11,460)
(274)
(7,525)
19,259

Sh000
30,000
14,340
616
8,975
53,931

610
465
695
7,360
9,130
1,175
1,000
783
800
4,000

(7,758)

1,372
55,303
40,000
500
2,000
2,803
45,303
10,000
55,303

Lesson Nine

509

STA
Balance Sheet as at 1 November 19-6

Buildings
17,000
Equipment
3,480
20,480
Current assets
Stock
1,100
Debtors
2,230
Bank
4,950
8,280
Current liabilities
Creditors
(980)
7,300
27,780
Capital: Sam
7,500
Ted
7,500
Abe
4,000
19,000
Current accounts:
720
Sam
Ted
220
Abe
__940
19,940
Estate of Reg.
7,840
27,780

510

Revision Aid

KK Ltd
Balance Sheet as at 31 October 1998
Shs 000
Shs 000
Non Current Assets
Freehold property
Furniture and fittings
Motor vehicles

44,500
1,540
3,500
49,540

(292)
(965)
(1,257)

Goodwill
Current Assets
Stock
Debtors
Less provision for doubtful debts
Rent receivable
Cash at bank
Current Liabilities
Creditors
Accrued expenses
Debenture interest
Tax payable
Proposed dividends

4,398
1,540
(77)

332
189
350
8,960
4,500

1,463
35
10,492
16,388

(14,331)

2,057
50,840
30,000

Capital reserves
Share premium

10% debenture

44,500
1,248
2,535
48,283
500

Authorized and issued share capital


1,500,000 ordinary shares of Sh. 20
each fully paid

Revenue reserves
General reserve
Profit and loss account

Shs
000

350
4,500
12,490

16,990
47,340
3,500
50,840

Lesson Nine

511

512

Revision Aid

Question 3
(a)
1995

1996

Gross profit
percentage
Gross profit
Sales

24,000
64,000

Current ratio
Current assets
Current liabilities

23,900 = 1.68:1
14,200

31,000 = 1.52:1
20,400

Quick ratio
Current assets
less stock
Current liabilities

23,900 12,000 =
0.84:1
14,200

31,000 15,000 =
0.78:1
20,400

10,500 x 365 = 60
days
64,000

14,000 x 365 = 47
days
108,000

6,800 x 365 = 59 days


42,000

9,400 x 365 = 44 days


78,600

60,000____ = 70%
60,000 + 26,000

60,000____ = 55%
60,000 + 49,000

Debtors collection
period
Debtors x 365
Sales
Creditors payment
period
Trade creditors
x 365
Purchases (W)
Gearing ratio
Loan capital
Total capital

Cost of sales
Add: closing stock
Deduct: opening stock
Purchases
(b)

1995
000
40,000
12,000
52,000
10,000
42,000

= 37.5%

32,400 = 30%
108,000

1996
000
75,600
15,000
90,600
12,000
78,600

The gross profit margin has fallen when compared with last year, although in
absolute terms, both profit and sales are higher. Possibly the firm has
lowered the price of goods to increase sales, although there may be other
explanations (see part (c) below).

Lesson Nine

513

There has been a reduction in liquidity as evidenced by a fall in both the


current and the quick ratios. However, this is no immediate cause for
concern as the company appears to be paying its creditors more promptly
than last year.
The debtors collection period, already satisfactory, has decreased still
further from 60 to 47 days. There is not enough information to say whether
this is all due to good credit control, or whether some sales are being made
on shorter credit terms or for cash.
The creditors payment period has shortened. Possibly the company has
become more efficient at paying creditors, or perhaps it is purchasing goods
on shorter credit terms.
The gearing ratio has reduced but it is still too high. The reduction is mainly
due to an increase in retained profits and in the revaluation reserve. High
gearing involves greater risk for the shareholders.

Any two of the following:

An error in counting closing stock


An increase in prices from suppliers not passed on to customers
Deliberate reduction in margin in an attempt to increase sales volume

(d)
The position is not quite as clear-cut as this statement would suggest. Liquidity is
important, and a company ought to be able to pay its debts as they fall due.
However, an excessively high current ratio means that resources are tied up in
stock, debtors and cash instead of producing profits. Current assets should
generally be kept as low as is compatible with efficient production and paying
creditors as they fall due.
There is some truth in this statement. High gearing means greater risk, but also,
in good times, greater returns. It is important that the percentage return to
shareholders is greater than the percentage rate of interest being paid on the
borrowings.
Question 4
19X1
11 Feb
11 Feb

Cash
Share capital
(50,000 @
70p)
Share
premium
(50,000 @
20p)

4,000

Application and allotment account


19X1

10 Feb
Cash

35,000

16 Feb

Cash

10,000

______

49,000

49,000

514

19X1
29 Sep

1 Nov

Revision Aid

Ordinary Share capital account


19X1
500
11 Feb Application and
allotment
account
(50,000 @ 70p)
1 May
Call account
50,000
(50,000 @ 30p)

Forfeited
shares
Balance
carried
down

1 Nov
______
50,50
0

Bal c/d

19X1
1 May

Share
capital

Forfeited
shares reissued

Ordinary Share Premium Account


19X1
11 Feb Application and
allotment
account
(50,000 @ 20p)
1 Nov
Forfeited
shares reissued
10,250
account
10,25
0

1 Nov

35,000
15,000
___500
50,500

10,000
___250
10,250

15,000

Call account
19X1
1 May
Cash

_____

29 Sep

___150

Forfeited
shares

15,00
0

19X1
29 Sep

Forfeited Shares
19X1
29
Share capital (500
Sep
@ 1)

14,850

15,000

500

Call account

150

Forfeited
shares
reissued

350

____

500

500

Lesson Nine

19X1
1 Nov
1 Nov

Share
capital
Share
premium

515

500
250
750

Forfeited Shares
19X1
1 Nov
Cash
1 Nov

Forfeited shares

400
350
750

516

Revision Aid

STRATHMORE UNIVERSITY
MOCK EXAMINATION
CPA PART I\ CPS PART I
FINANCIAL ACCOUNTING I
The following should be done under examination condition.
Answer ALL questions. Marks allocated to each question are shown at the end of
the question. Show ALL your workings.
QUESTION ONE
Nafuu Foods Ltd. is a company in the hospitality industry. The following trial
balance has been extracted from its books on 31 October 2001.
Sh000
Revenue
Cost of sales
401,000
Wages and salaries
186,440
Operating expenses
95,860
Insurance
1,180
Directors fees
960
Ordinary share capital; 5,200,000 shares of Sh.20
each fully paid
Profit and loss account, 1 November 2000
9% debenture stock secured
Share premium
Capital redemption reserve
Trade debtors and creditors
63,860
Bad debts written off
240
Audit fees
400
Interest on loan and overdraft
13,900
Depreciation expense
17,300
Accruals
Interim dividends paid
Freehold land and buildings
164,600
Leasehold land and buildings (over 50 years)125,600
Leasehold land and buildings (over 50 years) 51,900
Furniture and equipment
85,600
Stock
46,400
Prepayments
720
Bank balance
2,820
Investments
9,800
1,294,580

Sh.000
816,160

104,000
77,600
160,000
18,000
56,400
61,520

900
26,000

1,294,580

Lesson Nine

517

You are provided with the following additional information:


1. The balances of fixed asset accounts as at the beginning of the year and
additions during the year were as follows:
Cost or valuation
Accumulated depreciationAdditions
1 November 2000 during the year
Sh. 000
Sh.000
Sh.000
Freehold land and buildings157,000
7,600
Leasehold land and buildings
(over 50 years)
121,800
3,800
Leasehold land and buildings
(under 50 years)
60,200
5,800
Furniture and equipment 150,800
52,200
1,800
The company does not provide for depreciation on freehold properties or
properties held on lease with 50 years or more to run at the balance sheet
date. Properties held on lease with less than 50 years to run are depreciated
over the un-expired term. Items of equipment are depreciated over their
estimated useful life.
2. Some of the leasehold property in the books costing Sh.7,500,000 had just 50
years remaining on the lease in October 2000 and has not yet been transferred
to the under 50 years category.
3. Disposals during the year included the following:
Cost
Sh. 000
Freehold land and buildings3,600
Leasehold under 50 years 2,300

Accumulated depreciationSale proceeds


Sh.000
Sh 000
1,700

4,800
960

All the sale proceeds have been included in the revenue: no other adjustment
has been made.
The determination of depreciation expense for the year included in the trial
balance above has correctly been done for those properties not disposed and
include in the under 50 years category at the beginning of the year.
1)
2)
3)
4)
5)

Freehold land was revalued on an existing basis by a professional valuer but


the surplus of Sh.6,000,000 has not yet been brought into account.
The investments in the trial balance are temporary quoted securities. As at 31
October 2001 their market value was Sh.10,500,000. Income from the
investments of Sh.450,000 is included in revenue.
Additional audit fees of Sh.600,000 need to be provided for.
The total balance of cash at bank includes Sh.1,500,000 overdraft on one of
the accounts.
The corporation tax on the years profit has been estimated at Sh.27,000,000.
Corporation tax on the previous years profit was finally agreed with the tax
authorities to be Sh.310,000 more than had been provided for in the profit
and loss account of the year.

518

6)

Revision Aid

The directors have decided to recommend a final dividend of Sh.5 per


ordinary share

Required:
a) A schedule showing fixed assets movements for the year ended 31 October
2001.
(10 marks)
b) Profit and loss account for the year ended 31 October 2001.
(10 marks)
c) Balance sheet at 31 October 2001.
(5 marks)

Lesson Nine

519

QUESTION TWO
Rotich and Sinei have been in partnership for several years, sharing profits and
losses in the ratio 2:1. Interest on fixed capitals was allowed at the rate of 10%
per annum, but no interest was charged or allowed on current accounts.
The following was the partnership trial balance as at 30 April 2001:
Sh.

Sh.

Fixed capital accounts


Rotich
Sinei

750,000
500,000

Current accounts
Rotich
Sinei

400,000
300,000

Leasehold premises (purchased 1 May 2000) 2,250,000


Purchases
4,100,000
Motor vehicle (cost)
1,600,000
Balance at bank
820,000
Salaries (including partners drawings)
1,300,000
Stocks: 30 April 2000
1,200,000
Furniture and fittings (cost)
300,000
Debtors
225,000
Accountancy and audit fees
105,000
Wages
550,000
Rent, rates and electricity
310,000
General expenses (Sh.352,400 for the six months
To 31 October 2000)
660,000
Cash introduced Tonui
1,250,000
Sh.
Sales (Sh.3,500,000 to 31 October 2000)
Accumulated depreciation: 1 May 2000
Motor vehicle
Furniture and fittings
Creditors

Sh.
8,750,000

300,000
100,000
1,970,000
13,420,000 13,420,000

Additional information:
1.

On 1 November 2000, Tonui was admitted as a partner and from that date,
profits and losses were to be dated in the ratio 2:2:1. For the purpose of this
admission, the value of goodwill was agreed at Sh.3,000,000. No account for
goodwill was to be maintained in the books, adjusting entries for transactions
between the partners being made in their current accounts. On that date,
Tonui introduced Sh.1,250,000 into the firm of which Sh.375,000 comprised
his fixed capital and the balance was credited to his current account.

520

Revision Aid

2.

Interest on fixed capitals was still to be allowed at the rate of 10% per annum
after Tonuis admission. In addition, after Tonuis admission, no interest was
to be charged or allowed on current accounts.

3.

Any apportionment of gross profit was to be made on the basis of sales.


Expenses, unless otherwise indicated, were to be apportioned on a time basis.
A charge was to be made for depreciation on motor vehicle and furniture and
fittings at 20% and 10% per annum respectively, calculated on cost.
On 30 April 2001, the stock was valued at Sh.1,275,000.
Salaries included the following partners drawings:
Rotich Sh.150,000, Sinei Sh.120,000 and Tonui Sh.62,500.
A difference in the books of Sh.48,000 had been written off at 30 April 2001 to
general expense, which was later found to be due to the following clerical
errors:
Sales returns of Sh.32,000 had been debited to sales returns but had not been
posted to the account of the customer concerned :
The purchases journal had been undercast by S.80,000.
Doubtful debts (for which full provision was required) amounted to Sh.30,000
and Sh.40,000 as at 31 October 2000 and 30 April 2001 respectively.
On 30 April 2001, rates and rent paid in advance amounted to Sh.50,000 and
a provision of Sh.15,000 for electricity consumed was required.

4.
5.
6.
7.

8.
9.

Required:
a) Trading and profit and loss account for the year ended 30 April 2001.
(9 marks)
b) Partners current accounts for the year ended 30April 2001.
(4 marks)
c) Balance sheet as at 30 April 2001.
(7 marks)
(Total: 20marks)
QUESTION THREE
a) State and briefly explain any three distinguishing features between (i) a
receipts and payments account and (ii) an income and expenditure account. (6
marks)
b) The accountant of Mamba Sports Club has extracted the following information
from the books of account for the year ended 31 March 2001:
Receipts
Balance brought forward
Subscriptions:
Year 1999 2000
Maintenance
2000 2001
2001 2002
Dinner dance

Sh.
288,000
249,000
124,000
2,050,000
194,000
723,000

Payments
Salaries and wages
New equipment
Repairs and

Sh.
254,000
565,000

Office expenses
415,000
Printing and stationery168,000
Purchase ofBeverages197,000

Lesson Nine

Beverage sales
Investments income

521

657,000
400,000

4,561,000

Dinner dance expenses315,000


Refund of subscriptions 45,000
Sports prizes
25,000
Transport
218,000
Investments
1,500,000
Balance carried forward 405,000
4,561,000

522

Revision Aid

Balances as at

31 March 2000
Sh.

31 March 2001
Sh.

Furniture and fittings (net)


Equipment (net)
Investments at cost
Subscriptions in arrears
Salaries accrued
Stock of beverages
Subscriptions in advance

240,000
690,000
3,500,000
300,000
68,000
162,000
85,000

375,000
72,000
184,000
-

Additional information:
1. Subscriptions in arrears are written-off after twelve months.
2. Depreciation is provided for on reducing balance method at 10% and 20% per
annum on furniture and fittings and equipment respectively.
3. Investments which had cost Sh.500,000 were sold on 30 March 2001 for
Sh.625,000. No entries have been made in the books in this respect.
Required:
a) Income and expenditure account for the year ended 31 March 2001. (8 marks)
b) Balance sheet as at 31 March 2001.
(6 marks)
(Total: 20 marks)
QUESTION FOUR
a)
b)

Explain the term bank reconciliation and state the reasons for its
preparation.
(6 marks)
Ssemakula, a sole trader received his bank statement for the month of June
2001. At that date the bank balance was Sh.706,500 whereas his cash book
balance was Sh.2,366,500. His accountant investigated the matter and
discovered the following discrepancies:
1) Bank charges of Sh.3,000 had not been entered in the cash book.
2) Cheques drawn by Ssemakula totalling Sh.22,500 had not yet been
presented to the bank.
3) He had not entered receipts of Sh.26,500 in his cash book.
4) The bank had not credited Mr Ssemakula with receipts of Sh.98,500 paid
into the bank on 30mJune 2001.
5) Standing order payments amounting to Sh.62.000 had not been entered
into the cash book.
6) In the cash book Ssemakula had entered a payment of Sh.74,900 as
Sh.79,400.
7) A cheque for Sh.15,000 from a debtor had been returned by the bank
marked refer to drawer but had not been written back into the cash book.
8) Ssemakula had brought forward the opening cash balance of Sh.329,250 as
a debit balance instead of a credit balance.
9) An old cheque payment amounting to Sh.44,000 had been written back in
the cash book but the bank had already honoured it.

Lesson Nine

523

10) Some of Ssemakulas customers had agreed to settle their debts by paying
directly into his bank account. Unfortunately, the bank had credited some
deposits amounting to Sh.832,500 to another customers account.
However, acting on information from his customers, Ssemakula had
actually entered the expected receipts from the debtors in his cash book.

524

Revision Aid

Required:
i)
ii)

A statement showing Ssemakulas adjusted cash book balance as at 30 June


2001.
(9 marks)
A bank reconciliation statement as at 30 June 2001.
(5 marks)
(Total: 20 marks)

QUESTION FIVE
The accounting profession has for a long time relied on certain accounting
conventions to guide accounting practice.
Yet the application of the same
conventions has been the source of criticism of the quality and relevance of
information contained in financial reports.
Some of these conventions include:
a)
b)
c)
d)
e)

The
The
The
The
The

business entity principle.


historical cost principle.
monetary principle.
matching principle.
conservatism principle.

Required:
For each of the principles listed above:
a) Explain its meaning
b) Justify its use.
c) Explain any weaknesses associated with its use.

(5 marks)
(5 marks)
(5 marks)

END OF MOCK EXAMINATION


NOW SEND YOUR ANSWERS TO THE DISTANCE LEARNING CENTRE FOR
MARKING

Das könnte Ihnen auch gefallen